Site Loader

Π‘ΠΎΠ΄Π΅Ρ€ΠΆΠ°Π½ΠΈΠ΅

Π—Π°ΠΊΠΎΠ½ Ома для элСктричСской Ρ†Π΅ΠΏΠΈ

ЭлСктричСский Ρ‚ΠΎΠΊ, ΠΊΠ°ΠΊ ΠΈ любоС Π΄Ρ€ΡƒΠ³ΠΎΠ΅ физичСскоС явлСниС подчиняСтся ΠΎΠΏΡ€Π΅Π΄Π΅Π»Π΅Π½Π½Ρ‹ΠΌ Π·Π°ΠΊΠΎΠ½Π°ΠΌ. Π’Π°ΠΊ, Π² 1826 Π³ΠΎΠ΄Ρƒ, Π“Π΅ΠΎΡ€Π³ Ом Π²Ρ‹Π²Π΅Π» эмпиричСский Π·Π°ΠΊΠΎΠ½, ΠΊΠΎΡ‚ΠΎΡ€Ρ‹ΠΉ способСн ΠΎΠ±ΡŠΡΡΠ½ΠΈΡ‚ΡŒ Π·Π°Π²ΠΈΡΠΈΠΌΠΎΡΡ‚ΡŒ силы Ρ‚ΠΎΠΊΠ°, напряТСния, Π° Ρ‚Π°ΠΊΠΆΠ΅ особСнностСй ΠΏΡ€ΠΎΠ²ΠΎΠ΄Π½ΠΈΠΊΠ° Π² элСктроцСпи. Π’ дальнСйшСм вносились ΠΎΠΏΡ€Π΅Π΄Π΅Π»Π΅Π½Π½Ρ‹Π΅ измСнСния, сам Π·Π°ΠΊΠΎΠ½ Ома для элСктричСской Ρ†Π΅ΠΏΠΈ модифицировался, ΠΈ Π½Π° Π΄Π°Π½Π½Ρ‹ΠΉ ΠΌΠΎΠΌΠ΅Π½Ρ‚ ΡƒΡ‡Π΅Π½Ρ‹Π΅ Π΅Π³ΠΎ ΠΈΠ½Ρ‚Π΅Ρ€ΠΏΡ€Π΅Ρ‚ΠΈΡ€ΡƒΡŽΡ‚ Π² Ρ‡Π΅Ρ‚Ρ‹Ρ€Π΅Ρ… Π²Π°Ρ€ΠΈΠ°Π½Ρ‚Π°Ρ…, ΠΊΠΎΡ‚ΠΎΡ€Ρ‹Π΅ ΠΌΡ‹ ΠΈ рассмотрим.

Π’ Ρ…ΠΎΠ΄Π΅ практичСских исслСдований, Π½Π° ΠΈΡ… Π±Π°Π·Π΅, ΡƒΡ‡Π΅Π½Ρ‹ΠΉ смог ΠΎΠΏΡ€Π΅Π΄Π΅Π»ΠΈΡ‚ΡŒ Π·Π°Π²ΠΈΡΠΈΠΌΠΎΡΡ‚ΡŒ силы Ρ‚ΠΎΠΊΠ° ΠΈ напряТСния ΠΎΡ‚ спСцифики ΠΏΡ€ΠΎΠ²ΠΎΠ΄Π½ΠΈΠΊΠ°, ΠΏΠΎ ΠΊΠΎΡ‚ΠΎΡ€ΠΎΠΌΡƒ ΠΏΡ€ΠΎΡ‚Π΅ΠΊΠ°Π΅Ρ‚ Ρ‚ΠΎΠΊ. Если Π±Ρ‹Ρ‚ΡŒ Ρ‚ΠΎΡ‡Π½Π΅Π΅, Ρ‚ΠΎ ΠΊΠ°ΠΆΠ΄Ρ‹ΠΉ ΠΌΠ°Ρ‚Π΅Ρ€ΠΈΠ°Π» ΠΈΠΌΠ΅Π΅Ρ‚ ΠΎΠΏΡ€Π΅Π΄Π΅Π»Π΅Π½Π½ΠΎΠ΅ сопротивлСниС ΠΈ Π½Π° ΠΎΠΏΡ€Π΅Π΄Π΅Π»Π΅Π½Π½ΠΎΠΌ участкС Ρ†Π΅ΠΏΠΈ, сила Ρ‚ΠΎΠΊΠ° вычисляСтся ΠΎΡ‚Π½ΠΎΡˆΠ΅Π½ΠΈΠ΅ΠΌ напряТСния ΠΈ сопротивлСния.

IΒ =Β U/Β R,

Π³Π΄Π΅ I – сила Ρ‚ΠΎΠΊΠ°, U – напряТСниС, R – сопротивлСниС ΠΏΡ€ΠΎΠ²ΠΎΠ΄Π½ΠΈΠΊΠ°.

ЀактичСски, этот Π·Π°ΠΊΠΎΠ½ Π°Π½Π°Π»ΠΎΠ³ΠΈΡ‡Π΅Π½ ΠΏΡ€ΠΎΡ…ΠΎΠΆΠ΄Π΅Π½ΠΈΡŽ Π²ΠΎΠ΄Ρ‹ ΠΏΠΎ Ρ‚Ρ€ΡƒΠ±Π°ΠΌ: Ρ‡Π΅ΠΌ большС Π΄ΠΈΠ°ΠΌΠ΅Ρ‚Ρ€ Ρ‚Ρ€ΡƒΠ±Ρ‹ ΠΈ Π½Π°ΠΏΠΎΡ€, Ρ‚Π΅ΠΌ большС Π΅Π΅ Π²Ρ‹Π»ΡŒΠ΅Ρ‚ΡΡ Π² ΠΊΠΎΠ½Π΅Ρ‡Π½ΠΎΠΉ Ρ‚ΠΎΡ‡ΠΊΠ΅.

Π—Π°ΠΊΠΎΠ½ Ома для Π·Π°ΠΌΠΊΠ½ΡƒΡ‚ΠΎΠΉ Ρ†Π΅ΠΏΠΈ

Подобная интСрпрСтация ΠΏΠΎΠ΄Ρ€Π°Π·ΡƒΠΌΠ΅Π²Π°Π΅Ρ‚ Π½Π°Π»ΠΈΡ‡ΠΈΠ΅ источника питания, Π° Ρ‚Π°ΠΊΠΆΠ΅ ΠΏΡ€ΠΎΠ²ΠΎΠ΄Π½ΠΈΠΊΠ°, ΠΏΠΎ ΠΊΠΎΡ‚ΠΎΡ€ΠΎΠΌΡƒ ΠΏΡ€ΠΎΡ‚Π΅ΠΊΠ°Π΅Ρ‚ Ρ‚ΠΎΠΊ. Π’ этом случаС, ΠΏΠΎΠΌΠΈΠΌΠΎ сопротивлСния Π½Π° ΠΎΡ‚Π΄Π΅Π»ΡŒΠ½ΠΎ взятом участкС слСдуСт ΡƒΡ‡ΠΈΡ‚Ρ‹Π²Π°Ρ‚ΡŒ ΠΈ Ρ‚ΠΎ, ΠΊΠΎΡ‚ΠΎΡ€ΠΎΠ΅ Π²ΠΎΠ·Π½ΠΈΠΊΠ°Π΅Ρ‚ Π² ИП. Учитывая эти Ρ„Π°ΠΊΡ‚ΠΎΡ€Ρ‹, ΠΌΠΎΠΆΠ½ΠΎ ΡΠΊΠ°Π·Π°Ρ‚ΡŒ, Ρ‡Ρ‚ΠΎ сила Ρ‚ΠΎΠΊΠ° Π±ΡƒΠ΄Π΅Ρ‚ Ρ€Π°Π²Π½Π° ΠΎΡ‚Π½ΠΎΡˆΠ΅Π½ΠΈΡŽ элСктродвиТущСй силы ΠΊ суммС сопротивлСний.

IΒ =Β E/Β RΠ²Π½+r,

Π³Π΄Π΅ Π• – Π­Π”Π‘, RΠ²Π½ – внСшнСС сопротивлСниС, Π° r соотвСтствСнно Π²Π½ΡƒΡ‚Ρ€Π΅Π½Π½Π΅Π΅.

Π—Π°ΠΊΠΎΠ½ Ома для Π·Π°ΠΌΠΊΠ½ΡƒΡ‚ΠΎΠΉ Ρ†Π΅ΠΏΠΈ ΠΌΠΎΠΆΠ½ΠΎ ΠΎΠ±ΡŠΡΡΠ½ΠΈΡ‚ΡŒ доступным языком. ЭлСктродвиТущая сила ΠΏΠΎ ΠΎΠΏΡ€Π΅Π΄Π΅Π»Π΅Π½ΠΈΡŽ Π΄ΠΎΠ»ΠΆΠ½Π° ΠΏΠΎΠ»Π½ΠΎΡ†Π΅Π½Π½ΠΎ ΠΎΠ±Π΅ΡΠΏΠ΅Ρ‡ΠΈΠ²Π°Ρ‚ΡŒ ΠΏΠΎΡΡ‚ΠΎΡΠ½Π½ΡƒΡŽ Ρ€Π°Π·Π½ΠΈΡ†Ρƒ ΠΏΠΎΡ‚Π΅Π½Ρ†ΠΈΠ°Π»ΠΎΠ², ΠΈ эта сила ΠΌΠΎΠΆΠ΅Ρ‚ ΠΈΠΌΠ΅Ρ‚ΡŒ Π½Π΅ΠΏΡ€ΠΈΡ€ΠΎΠ΄Π½ΠΎΠ΅ происхоТдСниС: химичСскоС, Ссли Π² качСствС источника ΠΈΡΠΏΠΎΠ»ΡŒΠ·ΡƒΠ΅Ρ‚ΡΡ Π±Π°Ρ‚Π°Ρ€Π΅ΠΉΠΊΠ° ΠΈΠ»ΠΈ мСханичСская, Π² случаС ΠΏΠΎΠ΄ΠΊΠ»ΡŽΡ‡Π΅Π½ΠΈΡ ΠΊ элСктричСской Ρ†Π΅ΠΏΠΈ Π³Π΅Π½Π΅Ρ€Π°Ρ‚ΠΎΡ€Π°. ΠŸΡ€ΠΈ ΠΏΠΎΠ΄ΠΊΠ»ΡŽΡ‡Π΅Π½ΠΈΠΈ ΠΌΠ΅Π΄Π½ΠΎΠΉ ΠΏΡ€ΠΎΠ²ΠΎΠ»ΠΎΠΊΠΈ с ΠΈΠ΄Π΅Π½Ρ‚ΠΈΡ‡Π½Ρ‹ΠΌ сСчСниСм ΠΊ Π±Π°Ρ‚Π°Ρ€Π΅ΠΉΠΊΠ΅ ΠΈ аккумулятору. Π­Ρ„Ρ„Π΅ΠΊΡ‚ Π΄ΠΎΠ»ΠΆΠ΅Π½ Π±Ρ‹Ρ‚ΡŒ Ρ‚Π°ΠΊΠΈΠΌ, Ρ‡Ρ‚ΠΎ ΠΏΠΎ этому ΠΏΡ€ΠΎΠ²ΠΎΠ΄Π½ΠΈΠΊΡƒ, Π² ΠΊΠΎΡ‚ΠΎΡ€ΠΎΠΌ сопротивлСниС практичСски отсутствуСт, Π΄ΠΎΠ»ΠΆΠ΅Π½ ΠΏΠΎΠΉΡ‚ΠΈ Ρ‚ΠΎΠΊ с Π²Π΅Π»ΠΈΡ‡ΠΈΠ½ΠΎΠΉ, стрСмящСйся ΠΊ бСсконСчности. Однако этого Π½Π΅ происходит ΠΈ Ρ€Π°Π·Π½ΠΈΡ†Π° Π² показатСлях Π±ΡƒΠ΄Π΅Ρ‚ сущСствСнной, Π° Π²ΠΎ Π²Ρ‚ΠΎΡ€ΠΎΠΌ случаС, ΠΏΡ€ΠΎΠ²ΠΎΠ»ΠΎΠΊΠ° ΠΈ вовсС ΠΌΠΎΠΆΠ΅Ρ‚ ΠΏΠ΅Ρ€Π΅Π³ΠΎΡ€Π΅Ρ‚ΡŒ. ИмСнно поэтому Π² расчСт бСрСтся Π²Π½ΡƒΡ‚Ρ€Π΅Π½Π½Π΅Π΅ сопротивлСниС источника питания, Ρ‡Ρ‚ΠΎΠ±Ρ‹ ΠΎΠΏΠΈΡΠ°Ρ‚ΡŒ ΠΏΠΎΠ΄ΠΎΠ±Π½ΠΎΠ΅ явлСниС.

Π—Π°ΠΊΠΎΠ½ ΠΎΠΌΠ° для Π½Π΅ΠΎΠ΄Π½ΠΎΡ€ΠΎΠ΄Π½ΠΎΠ³ΠΎ участка Ρ†Π΅ΠΏΠΈ

ΠŸΠ΅Ρ€Π΅Π΄ Ρ‚Π΅ΠΌ, ΠΊΠ°ΠΊ Π·Π°ΠΏΠΈΡΠ°Ρ‚ΡŒ Ρ„ΠΎΡ€ΠΌΡƒΠ»Ρƒ для ΠΏΠΎΠ΄ΠΎΠ±Π½ΠΎΠΉ ΠΈΠ½Ρ‚Π΅Ρ€ΠΏΡ€Π΅Ρ‚Π°Ρ†ΠΈΠΈ Π·Π°ΠΊΠΎΠ½Π°, слСдуСт Ρ€Π°Π·ΠΎΠ±Ρ€Π°Ρ‚ΡŒΡΡ Π² Ρ‚Π°ΠΊΠΈΡ… понятиях, ΠΊΠ°ΠΊ Π»ΠΈΠ½Π΅ΠΉΠ½Ρ‹Π΅ ΠΈ Π½Π΅Π»ΠΈΠ½Π΅ΠΉΠ½Ρ‹Π΅ участки Ρ†Π΅ΠΏΠΈ.

Если сопротивлСниС Π½ΠΈΠΊΠ°ΠΊΠΈΠΌ ΠΎΠ±Ρ€Π°Π·ΠΎΠΌ Π½Π΅ зависит ΠΎΡ‚ Ρ‚ΠΎΠΊΠ° ΠΈ ΠΏΠΎΠ΄Π°Π²Π°Π΅ΠΌΠΎΠ³ΠΎ напряТСния, Ρ‚ΠΎ с ростом Π²Ρ‚ΠΎΡ€ΠΎΠ³ΠΎ ΠΏΠ°Ρ€Π°ΠΌΠ΅Ρ‚Ρ€Π°, ΠΏΠ΅Ρ€Π²Ρ‹ΠΉ Π±ΡƒΠ΄Π΅Ρ‚ прямо ΠΏΡ€ΠΎΠΏΠΎΡ€Ρ†ΠΈΠΎΠ½Π°Π»ΡŒΠ½ΠΎ Π²ΠΎΠ·Ρ€Π°ΡΡ‚Π°Ρ‚ΡŒ ΠΈ Π½Π°ΠΎΠ±ΠΎΡ€ΠΎΡ‚, Ρ‚ΠΎ Π΅ΡΡ‚ΡŒ Π·Π°Π²ΠΈΡΠΈΠΌΠΎΡΡ‚ΡŒ ΠΌΠΎΠΆΠ½ΠΎ ΠΎΠΏΠΈΡΠ°Ρ‚ΡŒ прямой Π»ΠΈΠ½ΠΈΠ΅ΠΉ. Подобная Π·Π°Π²ΠΈΡΠΈΠΌΠΎΡΡ‚ΡŒ относится ΠΊ Π»ΠΈΠ½Π΅ΠΉΠ½Ρ‹ΠΌ участкам Ρ†Π΅ΠΏΠΈ ΠΈ сопротивлСниС ΠΈΠΌΠ΅Π΅Ρ‚ Π°Π½Π°Π»ΠΎΠ³ΠΈΡ‡Π½ΠΎΠ΅ Π½Π°Π·Π²Π°Π½ΠΈΠ΅.

Однако Π²Ρ‹ΡˆΠ΅ΠΈΠ·Π»ΠΎΠΆΠ΅Π½Π½Ρ‹ΠΉ Π²Π°Ρ€ΠΈΠ°Π½Ρ‚ считаСтся ΠΈΠ΄Π΅Π°Π»ΡŒΠ½Ρ‹ΠΌ ΠΈ Π΅Π³ΠΎ ΠΌΠΎΠΆΠ½ΠΎ ΡΠΌΠΎΠ΄Π΅Π»ΠΈΡ€ΠΎΠ²Π°Ρ‚ΡŒ лишь Π² ΠΈΠ΄Π΅Π°Π»ΡŒΠ½Ρ‹Ρ… условиях, Ρ‡Ρ‚ΠΎ фактичСски Π½Π΅Π²ΠΎΠ·ΠΌΠΎΠΆΠ½ΠΎ, вСдь, ΠΊΠ°ΠΊ ΠΌΠΈΠ½ΠΈΠΌΡƒΠΌ, ΠΎΠΊΡ€ΡƒΠΆΠ°ΡŽΡ‰Π°Ρ срСда вносит свои ΠΊΠΎΡ€Ρ€Π΅ΠΊΡ‚ΠΈΠ²Ρ‹. Π’ этом случаС, рост напряТСния Π½Π΅ Π±ΡƒΠ΄Π΅Ρ‚ прямо ΠΏΡ€ΠΎΠΏΠΎΡ€Ρ†ΠΈΠΎΠ½Π°Π»ΡŒΠ½Ρ‹ΠΌ силС Ρ‚ΠΎΠΊΠ° ΠΈ Π½Π° Π³Ρ€Π°Ρ„ΠΈΠΊΠ΅ Π·Π°Π²ΠΈΡΠΈΠΌΠΎΡΡ‚ΡŒ Π±ΡƒΠ΄Π΅Ρ‚ ΠΈΠ·ΠΎΠ±Ρ€Π°ΠΆΠ°Ρ‚ΡŒΡΡ Π² Π²ΠΈΠ΄Π΅ ΠΊΡ€ΠΈΠ²ΠΎΠΉ.

На рисункС ΠΈΠ·ΠΎΠ±Ρ€Π°ΠΆΠ΅Π½ΠΎ Π΄Π²Π° Π³Ρ€Π°Ρ„ΠΈΠΊΠ°, ΠΏΠ΅Ρ€Π²Ρ‹ΠΉ ΠΈΠ· ΠΊΠΎΡ‚ΠΎΡ€Ρ‹Ρ… описываСт Π»ΠΈΠ½Π΅ΠΉΠ½ΡƒΡŽ Π·Π°Π²ΠΈΡΠΈΠΌΠΎΡΡ‚ΡŒ, Π° Π²Ρ‚ΠΎΡ€ΠΎΠΉ Π½Π΅Π»ΠΈΠ½Π΅ΠΉΠ½ΡƒΡŽ.

Π§Ρ‚ΠΎΠ±Ρ‹ ΠΎΡ‚Ρ‡Π΅Ρ‚Π»ΠΈΠ²ΠΎ ΠΏΠΎΠ½ΠΈΠΌΠ°Ρ‚ΡŒ Ρ€Π°Π·Π½ΠΈΡ†Ρƒ ΠΌΠ΅ΠΆΠ΄Ρƒ этими понятиями, рассмотрим ΠΏΡ€ΠΈΠ½Ρ†ΠΈΠΏ Ρ€Π°Π±ΠΎΡ‚Ρ‹ ΠΎΠ±Ρ‹Ρ‡Π½ΠΎΠΉ элСктричСской Π»Π°ΠΌΠΏΡ‹ накаливания. ΠŸΡ€ΠΈ ΠΏΡ€ΠΎΡ…ΠΎΠΆΠ΄Π΅Π½ΠΈΠΈ Ρ‚ΠΎΠΊΠ° ΠΏΠΎ Π½ΠΈΡ‚ΠΈ, Ρ‚Π΅ΠΌΠΏΠ΅Ρ€Π°Ρ‚ΡƒΡ€Π° Π² Π·Π½Π°Ρ‡ΠΈΡ‚Π΅Π»ΡŒΠ½ΠΎΠΉ стСпСни ΠΏΠΎΠ²Ρ‹ΡˆΠ°Π΅Ρ‚ΡΡ, Ρ‡Ρ‚ΠΎ ΠΏΡ€ΠΈΠ²ΠΎΠ΄ΠΈΡ‚ ΠΊ Π·Π°ΠΌΠ΅Ρ‚Π½ΠΎΠΌΡƒ росту сопротивлСния. БоотвСтствСнно, ΠΏΡ€ΠΈ возрастании напряТСния, сила Ρ‚ΠΎΠΊΠ° Π±ΡƒΠ΄Π΅Ρ‚ ΡƒΠ²Π΅Π»ΠΈΡ‡ΠΈΠ²Π°Ρ‚ΡŒΡΡ ΠΌΠ΅Π΄Π»Π΅Π½Π½Π΅Π΅, Ρ‚ΠΎ Π΅ΡΡ‚ΡŒ Π½Π΅ Π»ΠΈΠ½Π΅ΠΉΠ½ΠΎ.

ΠŸΡ€ΠΈΠΌΠ΅Ρ‡Π°Π½ΠΈΠ΅: Π² Π½Π΅ΠΊΠΎΡ‚ΠΎΡ€Ρ‹Ρ… ситуациях, Π½Π΅ΠΊΠΎΡ‚ΠΎΡ€Ρ‹ΠΌΠΈ внСшними Ρ„Π°ΠΊΡ‚ΠΎΡ€Π°ΠΌΠΈ ΠΏΡ€Π΅Π½Π΅Π±Ρ€Π΅Π³Π°ΡŽΡ‚ ΠΏΠΎ ΠΏΡ€ΠΈΡ‡ΠΈΠ½Π΅ Ρ‚ΠΎΠ³ΠΎ, Ρ‡Ρ‚ΠΎ ΠΎΠ½ΠΈ ΠΎΡ‡Π΅Π½ΡŒ Π½Π΅Π·Π½Π°Ρ‡ΠΈΡ‚Π΅Π»ΡŒΠ½Ρ‹ ΠΈ Π² числовом эквивалСнтС Π½ΠΈΠΊΠΎΠΈΠΌ ΠΎΠ±Ρ€Π°Π·ΠΎΠΌ Π½Π΅ ΠΌΠΎΠ³ΡƒΡ‚ ΠΏΠΎΠ²Π»ΠΈΡΡ‚ΡŒ Π½Π° ΠΎΠ±Ρ‰ΡƒΡŽ ΠΊΠ°Ρ€Ρ‚ΠΈΠ½Ρƒ. Π­Ρ‚ΠΎ Π·Π½Π°Ρ‡ΠΈΡ‚, Ρ‡Ρ‚ΠΎ нСлинСйная Π·Π°Π²ΠΈΡΠΈΠΌΠΎΡΡ‚ΡŒ Π½Π° Π³Ρ€Π°Ρ„ΠΈΠΊΠ΅ фактичСски совпадаСт с Π»ΠΈΠ½Π΅ΠΉΠ½ΠΎΠΉ.

Учитывая Π²Ρ‹ΡˆΠ΅ΡΠΊΠ°Π·Π°Π½Π½ΠΎΠ΅, ΠΌΠΎΠΆΠ½ΠΎ ΡƒΡΡ‚Π°Π½ΠΎΠ²ΠΈΡ‚ΡŒ ΡΠ»Π΅Π΄ΡƒΡŽΡ‰ΡƒΡŽ Π·Π°Π²ΠΈΡΠΈΠΌΠΎΡΡ‚ΡŒ:

IΒ =Β U/Β RΒ =Β (f1 – f2) +Β E/Β R,

Π“Π΄Π΅ f1 ΠΈ f2 – ΠΏΠΎΡ‚Π΅Π½Ρ†ΠΈΠ°Π»Ρ‹ (соотвСтствСнно f1 – f2 называСтся Ρ€Π°Π·Π½ΠΈΡ†Π΅ΠΉ ΠΏΠΎΡ‚Π΅Π½Ρ†ΠΈΠ°Π»ΠΎΠ²), E – Π­Π”Π‘ Π½Π΅ΠΎΠ΄Π½ΠΎΡ€ΠΎΠ΄Π½ΠΎΠ³ΠΎ участка Ρ†Π΅ΠΏΠΈ, Π° R – суммарноС сопротивлСниС Π½Π° этом ΠΆΠ΅ участкС.

НуТно ΡƒΠΏΠΎΠΌΡΠ½ΡƒΡ‚ΡŒ ΠΈ ΠΎ Ρ‚ΠΎΠΌ, Ρ‡Ρ‚ΠΎ элСктродвиТущая сила Π½Π΅ всСгда Π² этом случаС Π±ΡƒΠ΄Π΅Ρ‚ ΠΈΠΌΠ΅Ρ‚ΡŒ ΠΏΠΎΠ»ΠΎΠΆΠΈΡ‚Π΅Π»ΡŒΠ½ΠΎΠ΅ Π·Π½Π°Ρ‡Π΅Π½ΠΈΠ΅. Если Π½Π°ΠΏΡ€Π°Π²Π»Π΅Π½ΠΈΠ΅ Ρ‚ΠΎΠΊΠ° источника Π±ΡƒΠ΄Π΅Ρ‚ Π°Π½Π°Π»ΠΎΠ³ΠΈΡ‡Π½Ρ‹ΠΌ с Π½Π°ΠΏΡ€Π°Π²Π»Π΅Π½ΠΈΠ΅ΠΌ Π² элСктричСской сСти, ΠΏΡ€ΠΎΡ‚ΠΎΠ½ΠΎΠ² Π±ΡƒΠ΄Π΅Ρ‚ большС, Ρ‡Π΅ΠΌ элСктронов (ΠΏΠΎΠ»ΠΎΠΆΠΈΡ‚Π΅Π»ΡŒΠ½Ρ‹Ρ… ΠΈ ΠΎΡ‚Ρ€ΠΈΡ†Π°Ρ‚Π΅Π»ΡŒΠ½Ρ‹Ρ… частиц), Ρ‚ΠΎ Π² этом случаС Π²Π΅Π»ΠΈΡ‡ΠΈΠ½Π° E Π±ΡƒΠ΄Π΅Ρ‚ ΠΈΠΌΠ΅Ρ‚ΡŒ Π·Π½Π°Ρ‡Π΅Π½ΠΈΠ΅ со Π·Π½Π°ΠΊΠΎΠΌ Β«+Β», Π² ΠΈΠ½ΠΎΠΉ ситуации, этот ΠΏΠ°Ρ€Π°ΠΌΠ΅Ρ‚Ρ€ Π±ΡƒΠ΄Π΅Ρ‚ со Π·Π½Π°ΠΊΠΎΠΌ Β«-Β».

Π—Π°ΠΊΠΎΠ½ Ома для ΠΏΠ΅Ρ€Π΅ΠΌΠ΅Π½Π½ΠΎΠ³ΠΎ Ρ‚ΠΎΠΊΠ°

Если Π² элСктроцСпи имССтся Π΅ΠΌΠΊΠΎΡΡ‚ΡŒ ΠΈΠ»ΠΈ ΠΈΠ½Π΅Ρ€Ρ‚Π½ΠΎΡΡ‚ΡŒ, Ρ‚ΠΎ этот Ρ„Π°ΠΊΡ‚ слСдуСт ΠΎΠ΄Π½ΠΎΠ·Π½Π°Ρ‡Π½ΠΎ ΡƒΡ‡ΠΈΡ‚Ρ‹Π²Π°Ρ‚ΡŒ ΠΏΡ€ΠΈ расчётах силы Ρ‚ΠΎΠΊΠ°. Они ΠΈΠΌΠ΅ΡŽΡ‚ собствСнныС ΠΏΠΎΠΊΠ°Π·Π°Ρ‚Π΅Π»ΠΈ сопротивлСния, Ρ‡Ρ‚ΠΎ ΠΏΡ€ΠΈΠ²ΠΎΠ΄ΠΈΡ‚ ΠΊ ситуации, которая Π±ΡƒΠ΄Π΅Ρ‚ ΠΈΠΌΠ΅Ρ‚ΡŒ ΠΏΠ΅Ρ€Π΅ΠΌΠ΅Π½Π½Ρ‹ΠΉ Ρ…Π°Ρ€Π°ΠΊΡ‚Π΅Ρ€. Π’ случаС Π—Π°ΠΊΠΎΠ½Π° Ома для ΠΏΠ΅Ρ€Π΅ΠΌΠ΅Π½Π½ΠΎΠ³ΠΎ Ρ‚ΠΎΠΊΠ°Β Ρ„ΠΎΡ€ΠΌΡƒΠ»Π° записываСтся ΡΠ»Π΅Π΄ΡƒΡŽΡ‰ΠΈΠΌ ΠΎΠ±Ρ€Π°Π·ΠΎΠΌ:

IΒ =Β U/Β Z, Π³Π΄Π΅

I – сила Ρ‚ΠΎΠΊΠ°, U – напряТСниС, Π° Z – суммарноС Π·Π½Π°Ρ‡Π΅Π½ΠΈΠ΅ сопротивлСния Π½Π° всСх участках элСктричСской Ρ†Π΅ΠΏΠΈ (этот ΠΏΠ°Ρ€Π°ΠΌΠ΅Ρ‚Ρ€ имСнуСтся Π΅Ρ‰Π΅, ΠΊΠ°ΠΊ импСданс).

Как Π³ΠΎΠ²ΠΎΡ€ΠΈΠ»ΠΎΡΡŒ ΠΈΠ·Π½Π°Ρ‡Π°Π»ΡŒΠ½ΠΎ, Π·Π°ΠΊΠΎΠ½ Ома считаСтся эмпиричСским. Π­Ρ‚ΠΎ ΠΎΠ±ΠΎΠ·Π½Π°Ρ‡Π°Π΅Ρ‚ Ρ‚ΠΎ, Ρ‡Ρ‚ΠΎ ΠΎΠ½ ΠΌΠΎΠΆΠ΅Ρ‚ Π½Π΅ всСгда Ρ€Π°Π±ΠΎΡ‚Π°Ρ‚ΡŒ ΠΈ Π²Ρ‹ΠΏΠΎΠ»Π½ΡΡ‚ΡŒ вычислСния Π½Π° Π΅Π³ΠΎ основС Π½Π΅ прСдставляСтся Π²ΠΎΠ·ΠΌΠΎΠΆΠ½Ρ‹ΠΌ. Подобная ситуация ΠΌΠΎΠΆΠ΅Ρ‚ ΡΠ»ΠΎΠΆΠΈΡ‚ΡŒΡΡ Π² Π½Π΅ΡΠΊΠΎΠ»ΡŒΠΊΠΈΡ… случаях:

  • Π² ситуации, ΠΊΠΎΠ³Π΄Π° ΡΠ»Π΅ΠΊΡ‚Ρ€ΠΎΡΠ΅Ρ‚ΡŒ ΠΈΠΌΠ΅Π΅Ρ‚ Π²Ρ‹ΡΠΎΠΊΡƒΡŽ частоту ΠΈ элСктромагнитноС ΠΏΠΎΠ»Π΅ ΠΌΠΎΠΆΠ΅Ρ‚ сильно ΠΈΠ·ΠΌΠ΅Π½ΡΡ‚ΡŒΡΡ Π·Π° ΠΊΠΎΡ€ΠΎΡ‚ΠΊΠΈΠ΅ ΠΏΡ€ΠΎΠΌΠ΅ΠΆΡƒΡ‚ΠΊΠΈ Π²Ρ€Π΅ΠΌΠ΅Π½ΠΈ;
  • ΠΏΡ€ΠΈ Π½Π°Π»ΠΈΡ‡ΠΈΠΈ ΠΏΡ€ΠΎΠ²ΠΎΠ΄Π½ΠΈΠΊΠΎΠ², ΠΊΠΎΡ‚ΠΎΡ€Ρ‹Π΅ ΠΎΠ±Π»Π°Π΄Π°ΡŽΡ‚ свойствами свСрхпроводимости, располоТСнных Π² условиях Π½ΠΈΠ·ΠΊΠΈΡ… Ρ‚Π΅ΠΌΠΏΠ΅Ρ€Π°Ρ‚ΡƒΡ€Π½Ρ‹Ρ… ΠΏΠΎΠΊΠ°Π·Π°Ρ‚Π΅Π»Π΅ΠΉ;
  • ΠΏΡ€ΠΈ ΠΏΠ΅Ρ€Π΅Π³Ρ€Π΅Π²Π΅ ΠΏΡ€ΠΎΠ²ΠΎΠ΄Π½ΠΈΠΊΠ° ΠΏΠΎΠ΄ воздСйствиСм проходящСго ΠΏΠΎ Π½Π΅ΠΌΡƒ Ρ‚ΠΎΠΊΠ°, ΠΎΡ‚Π½ΠΎΡˆΠ΅Π½ΠΈΠ΅ напряТСния ΠΈ сопротивлСния ΠΌΠΎΠΆΠ΅Ρ‚ Π½ΠΎΡΠΈΡ‚ΡŒ ΠΏΠ΅Ρ€Π΅ΠΌΠ΅Π½Π½Ρ‹ΠΉ, Π½Π΅ΠΎΠ΄Π½ΠΎΡ€ΠΎΠ΄Π½Ρ‹ΠΉ Ρ…Π°Ρ€Π°ΠΊΡ‚Π΅Ρ€;
  • Ссли ΠΏΡ€ΠΎΠ²ΠΎΠ΄Π½ΠΈΠΊ (диэлСктрик) находится ΠΏΠΎΠ΄ высоким напряТСниСм;
  • свСтодиодных Π»Π°ΠΌΠΏΠ°Ρ…;
  • Π² ΠΏΠΎΠ»ΡƒΠΏΡ€ΠΎΠ²ΠΎΠ΄Π½ΠΈΠΊΠ°Ρ… ΠΈ Π°Π½Π°Π»ΠΎΠ³ΠΈΡ‡Π½Ρ‹Ρ… устройствах.

На основС этого Π·Π°ΠΊΠΎΠ½Π°, ΠΌΠΎΠΆΠ½ΠΎ произвСсти Π²Ρ‹Π²ΠΎΠ΄ Π½Π΅ΠΊΠΎΡ‚ΠΎΡ€Ρ‹Ρ… Ρ„ΠΎΡ€ΠΌΡƒΠ» матСматичСским ΠΏΡƒΡ‚Π΅ΠΌ. Π‘ ΠΈΡ… ΠΏΠΎΠΌΠΎΡ‰ΡŒΡŽ ΠΌΠΎΠΆΠ½ΠΎ ΠΏΡ€ΠΎΠΈΠ·Π²ΠΎΠ΄ΠΈΡ‚ΡŒ Ρ€Π°Π·Π½ΠΎΠΎΠ±Ρ€Π°Π·Π½Ρ‹Π΅ расчСты.

ΠŸΠΎΠ΄Π΅Π»ΠΈΡ‚ΡŒΡΡ ссылкой:

ΠŸΠΎΡ…ΠΎΠΆΠ΅Π΅

1.5. Π—Π°ΠΊΠΎΠ½ Ома для Π·Π°ΠΌΠΊΠ½ΡƒΡ‚ΠΎΠΉ Ρ†Π΅ΠΏΠΈ

Если Π² ΠΏΡ€ΠΎΠ²ΠΎΠ΄Π½ΠΈΠΊΠ΅ ΡΠΎΠ·Π΄Π°Ρ‚ΡŒ элСктричСскоС ΠΏΠΎΠ»Π΅ ΠΈ Π½Π΅ ΠΏΡ€ΠΈΠ½ΡΡ‚ΡŒ ΠΌΠ΅Ρ€ для Π΅Π³ΠΎ поддСрТания, Ρ‚ΠΎ ΠΏΠ΅Ρ€Π΅ΠΌΠ΅Ρ‰Π΅Π½ΠΈΠ΅ зарядов ΠΎΡ‡Π΅Π½ΡŒ быстро ΠΏΡ€ΠΈΠ²Π΅Π΄Π΅Ρ‚ ΠΊ Ρ‚ΠΎΠΌΡƒ, Ρ‡Ρ‚ΠΎ ΠΏΠΎΠ»Π΅ Π²Π½ΡƒΡ‚Ρ€ΠΈ ΠΏΡ€ΠΎΠ²ΠΎΠ΄Π½ΠΈΠΊΠ° исчСзнСт ΠΈ Ρ‚ΠΎΠΊ прСкратится, поэтому для поддСрТания постоянного Ρ‚ΠΎΠΊΠ° Π² Ρ‚Π΅Ρ‡Π΅Π½ΠΈΠ΅ Π΄Π»ΠΈΡ‚Π΅Π»ΡŒΠ½ΠΎΠ³ΠΎ Π²Ρ€Π΅ΠΌΠ΅Π½ΠΈ Π½Π΅ΠΎΠ±Ρ…ΠΎΠ΄ΠΈΠΌΠΎ Π²Ρ‹ΠΏΠΎΠ»Π½Π΅Π½ΠΈΠ΅ Π΄Π²ΡƒΡ… условий: элСктричСская Ρ†Π΅ΠΏΡŒ Π΄ΠΎΠ»ΠΆΠ½Π° Π±Ρ‹Ρ‚ΡŒ Π·Π°ΠΌΠΊΠ½ΡƒΡ‚ΠΎΠΉ; Π² элСктричСской Ρ†Π΅ΠΏΠΈ наряду с участками, Π½Π° ΠΊΠΎΡ‚ΠΎΡ€Ρ‹Ρ… ΠΏΠΎΠ»ΠΎΠΆΠΈΡ‚Π΅Π»ΡŒ-

Π½Ρ‹Π΅ заряды двиТутся Π² сторону убывания ΠΏΠΎΡ‚Π΅Π½Ρ†ΠΈΠ°Π»Π°, Π΄ΠΎΠ»ΠΆΠ½Ρ‹ Π±Ρ‹Ρ‚ΡŒ участки, Π½Π° ΠΊΠΎΡ‚ΠΎΡ€Ρ‹Ρ… эти заряды двиТутся Π² сторону возрастания ΠΏΠΎΡ‚Π΅Π½Ρ†ΠΈΠ°Π»Π°, Ρ‚.Β Π΅. ΠΏΡ€ΠΎΡ‚ΠΈΠ² сил элСктростатичСского поля (см. ΠΈΠ·ΠΎΠ±Ρ€Π°ΠΆΠ΅Π½Π½ΡƒΡŽ ΡˆΡ‚Ρ€ΠΈΡ…ΠΎΠ²ΠΎΠΉ Π»ΠΈΠ½ΠΈΠ΅ΠΉ Ρ‡Π°ΡΡ‚ΡŒ Ρ†Π΅ΠΏΠΈ Π½Π° рис. 5).

ΠŸΠ΅Ρ€Π΅ΠΌΠ΅Ρ‰Π°Ρ‚ΡŒ ΠΏΠΎΠ»ΠΎΠΆΠΈΡ‚Π΅Π»ΡŒΠ½Ρ‹Π΅ заряды ΠΏΡ€ΠΎΡ‚ΠΈΠ² сил элСктростатичСского поля ΠΌΠΎΠ³ΡƒΡ‚ Ρ‚ΠΎΠ»ΡŒΠΊΠΎ силы нСэлСктростатичСского происхоТдСния, Π½Π°Π·Ρ‹Π²Π°Π΅ΠΌΡ‹Π΅ сторонними силами. Π’Π΅Π»ΠΈΡ‡ΠΈΠ½Π°, равная Ρ€Π°Π±ΠΎΡ‚Π΅ сторонних сил ΠΏΠΎ ΠΏΠ΅Ρ€Π΅ΠΌΠ΅Ρ‰Π΅Π½ΠΈΡŽ Π΅Π΄ΠΈΠ½ΠΈΡ‡Π½ΠΎΠ³ΠΎ ΠΏΠΎΠ»ΠΎΠΆΠΈΡ‚Π΅Π»ΡŒΠ½ΠΎΠ³ΠΎ заряда, называСтся элСктродвиТущСй силой (Π­Π”Π‘) ο₯, Π΄Π΅ΠΉΡΡ‚Π²ΡƒΡŽΡ‰Π΅ΠΉ Π² Ρ†Π΅ΠΏΠΈ ΠΈΠ»ΠΈ Π½Π° Π΅Π΅ участкС. Π­Π”Π‘ ο₯ измСряСтся Π² Π²ΠΎΠ»ΡŒΡ‚Π°Ρ… (Π’). Π˜ΡΡ‚ΠΎΡ‡Π½ΠΈΠΊ Π­Π”Π‘ ΠΈΠΌΠ΅Π΅Ρ‚ Π½Π΅ΠΊΠΎΡ‚ΠΎΡ€ΠΎΠ΅ Π²Π½ΡƒΡ‚Ρ€Π΅Π½Π½Π΅Π΅ сопротивлСниС , зависящСС ΠΎΡ‚ Π΅Π³ΠΎ устройства. Π­Ρ‚ΠΎ сопротивлСниС оказываСтся Π²ΠΊΠ»ΡŽΡ‡Π΅Π½Π½Ρ‹ΠΌ ΠΏΠΎΡΠ»Π΅Π΄ΠΎΠ²Π°Ρ‚Π΅Π»ΡŒΠ½ΠΎ с источником Π² ΠΎΠ±Ρ‰ΡƒΡŽ ΡΠ»Π΅ΠΊΡ‚Ρ€ΠΈΡ‡Π΅ΡΠΊΡƒΡŽ Ρ†Π΅ΠΏΡŒ. Π’ качСствС источников Π­Π”Π‘ ΠΈΡΠΏΠΎΠ»ΡŒΠ·ΡƒΡŽΡ‚ Π³Π°Π»ΡŒΠ²Π°Π½ΠΈΡ‡Π΅ΡΠΊΠΈΠ΅ элСмСнты ΠΈ Π³Π΅Π½Π΅Ρ€Π°Ρ‚ΠΎΡ€Ρ‹ постоянного Ρ‚ΠΎΠΊΠ° (рис. 6).

Если нСразвСтвлСнная замкнутая элСктричСская Ρ†Π΅ΠΏΡŒ (рис. 7) содСрТит нСсколько ΠΏΠΎΡΠ»Π΅Π΄ΠΎΠ²Π°Ρ‚Π΅Π»ΡŒΠ½ΠΎ соСдинСнных элСмСнтов с сопротивлСниСм ΠΈ источников Π­Π”Π‘ο₯ΠΊ, ΠΈΠΌΠ΅ΡŽΡ‰ΠΈΡ… Π²Π½ΡƒΡ‚Ρ€Π΅Π½Π½Π΅Π΅ сопротивлСниС Ρ‚ΠΎ Π΅Π΅ ΠΌΠΎΠΆΠ½ΠΎ Π·Π°ΠΌΠ΅Π½ΠΈΡ‚ΡŒ эквивалСнтной Ρ†Π΅ΠΏΡŒΡŽ, ΠΈΠ·ΠΎΠ±Ρ€Π°ΠΆΠ΅Π½Π½ΠΎΠΉ Π½Π° рис. 6. Π‘ΠΈΠ»Π° Ρ‚ΠΎΠΊΠ° Π² эквивалСнтной Ρ†Π΅ΠΏΠΈ опрСдСляСтся Π·Π°ΠΊΠΎΠ½ΠΎΠΌ Ома для Π·Π°ΠΌΠΊΠ½ΡƒΡ‚ΠΎΠΉ Ρ†Π΅ΠΏΠΈ:

, (14)

;

Π­Π”Π‘, ΠΊΠ°ΠΊ ΠΈ сила Ρ‚ΠΎΠΊΠ°, Π΅ΡΡ‚ΡŒ Π²Π΅Π»ΠΈΡ‡ΠΈΠ½Π° алгСбраичСская. Если Π­Π”Π‘ способствуСт двиТСнию ΠΏΠΎΠ»ΠΎΠΆΠΈΡ‚Π΅Π»ΡŒΠ½Ρ‹Ρ… зарядов Π² Π²Ρ‹Π±Ρ€Π°Π½Π½ΠΎΠΌ Π½Π°ΠΏΡ€Π°Π²Π»Π΅Π½ΠΈΠΈ, Ρ‚ΠΎ

ο₯ > 0, Ссли Π­Π”Π‘ прСпятствуСт двиТСнию ΠΏΠΎΠ»ΠΎΠΆΠΈΡ‚Π΅Π»ΡŒΠ½Ρ‹Ρ… зарядов Π² Π΄Π°Π½Π½ΠΎΠΌ Π½Π°ΠΏΡ€Π°Π²Π»Π΅Π½ΠΈΠΈ, Ρ‚ΠΎ ο₯Β < 0. Π§Ρ‚ΠΎΠ±Ρ‹ ΠΎΠΏΡ€Π΅Π΄Π΅Π»ΠΈΡ‚ΡŒ Π·Π½Π°ΠΊ Π­Π”Π‘, Π½Π΅ΠΎΠ±Ρ…ΠΎΠ΄ΠΈΠΌΠΎ ΠΏΠΎΠΊΠ°Π·Π°Ρ‚ΡŒ Π² элСктричСской Ρ†Π΅ΠΏΠΈ Π½Π°ΠΏΡ€Π°Π²Π»Π΅Π½ΠΈΠ΅ двиТСния ΠΏΠΎΠ»ΠΎΠΆΠΈΡ‚Π΅Π»ΡŒΠ½Ρ‹Ρ… зарядов. ΠŸΠΎΠ»ΠΎΠΆΠΈΡ‚Π΅Π»ΡŒΠ½Ρ‹Π΅ заряды Π² элСктричСской Ρ†Π΅ΠΏΠΈ двиТутся ΠΎΡ‚ ΠΏΠΎΠ»ΠΎΠΆΠΈΡ‚Π΅Π»ΡŒΠ½ΠΎΠ³ΠΎ полюса источника ΠΊ ΠΎΡ‚Ρ€ΠΈΡ†Π°Ρ‚Π΅Π»ΡŒΠ½ΠΎΠΌΡƒ ΠΏΠΎΠ»ΡŽΡΡƒ. Если ΠΏΠΎ Ρ…ΠΎΠ΄Ρƒ этого направлСния ΠΏΠ΅Ρ€Π΅ΠΉΡ‚ΠΈ Π²Π½ΡƒΡ‚Ρ€ΠΈ источника ΠΎΡ‚ ΠΎΡ‚Ρ€ΠΈΡ†Π°Ρ‚Π΅Π»ΡŒΠ½ΠΎΠ³ΠΎ полюса ΠΊ ΠΏΠΎΠ»ΠΎΠΆΠΈΡ‚Π΅Π»ΡŒΠ½ΠΎΠΌΡƒ, Ρ‚ΠΎ ο₯ > 0, Ссли ΠΏΠ΅Ρ€Π΅ΠΉΡ‚ΠΈ Π²Π½ΡƒΡ‚Ρ€ΠΈ источника ΠΎΡ‚ ΠΏΠΎΠ»ΠΎΠΆΠΈΡ‚Π΅Π»ΡŒΠ½ΠΎΠ³ΠΎ полюса ΠΊ ΠΎΡ‚Ρ€ΠΈΡ†Π°Ρ‚Π΅Π»ΡŒΠ½ΠΎΠΌΡƒ, Ρ‚ΠΎ ο₯Β <Β 0.

Рис. 6 Рис. 7

Из Π·Π°ΠΊΠΎΠ½Π° Ома для Π·Π°ΠΌΠΊΠ½ΡƒΡ‚ΠΎΠΉ Ρ†Π΅ΠΏΠΈ слСдуСт, Ρ‡Ρ‚ΠΎ ΠΏΠ°Π΄Π΅Π½ΠΈΠ΅ напряТСния

U Π½Π° Π·Π°ΠΆΠΈΠΌΠ°Ρ… источника мСньшС, Ρ‡Π΅ΠΌ Π­Π”Π‘. Π”Π΅ΠΉΡΡ‚Π²ΠΈΡ‚Π΅Π»ΡŒΠ½ΠΎ, ο₯, ΠΈΠ»ΠΈ ο₯. Π’Π°ΠΊ ΠΊΠ°ΠΊ ΠΏΠΎ Π·Π°ΠΊΠΎΠ½Ρƒ Ома для ΠΎΠ΄Π½ΠΎΡ€ΠΎΠ΄Π½ΠΎΠ³ΠΎ участка Ρ†Π΅ΠΏΠΈ напряТСниС Π½Π° Π·Π°ΠΆΠΈΠΌΠ°Ρ… источника , Ρ‚ΠΎ

ο₯ –Ir. (15)

Π’ Ρ€Π΅ΠΆΠΈΠΌΠ΅ ΠΊΠΎΡ€ΠΎΡ‚ΠΊΠΎΠ³ΠΎ замыкания (R << r) Π·Π°ΠΊΠΎΠ½ Ома для Π·Π°ΠΌΠΊΠ½ΡƒΡ‚ΠΎΠΉ Ρ†Π΅ΠΏΠΈ записываСтся Π² Π²ΠΈΠ΄Π΅: .

ΠŸΡ€ΠΈ Ρ€Π΅ΡˆΠ΅Π½ΠΈΠΈ Π·Π°Π΄Π°Ρ‡ Π½Π° ΠΎΠΏΡ€Π΅Π΄Π΅Π»Π΅Π½ΠΈΠ΅ силы Ρ‚ΠΎΠΊΠ°, Π­Π”Π‘ ΠΈ сопротивлСния Π² Π·Π°ΠΌΠΊΠ½ΡƒΡ‚ΠΎΠΉ элСктричСской Ρ†Π΅ΠΏΠΈ Π½Π΅ΠΎΠ±Ρ…ΠΎΠ΄ΠΈΠΌΠΎ:

1) Π½Π°Ρ‡Π΅Ρ€Ρ‚ΠΈΡ‚ΡŒ схСму ΠΈ ΡƒΠΊΠ°Π·Π°Ρ‚ΡŒ Π½Π° Π½Π΅ΠΉ всС элСмСнты Ρ†Π΅ΠΏΠΈ ΠΈ Π½Π°ΠΏΡ€Π°Π²Π»Π΅Π½ΠΈΠ΅ Ρ‚ΠΎΠΊΠ° Π² Ρ†Π΅ΠΏΠΈ;

2) Π·Π°ΠΌΠ΅Π½ΠΈΡ‚ΡŒ ΠΏΠΎΠ»ΡƒΡ‡Π΅Π½Π½ΡƒΡŽ схСму эквивалСнтной, Π² ΠΊΠΎΡ‚ΠΎΡ€ΠΎΠΉ (Π­Π”Π‘ ΡΠΊΠ»Π°Π΄Ρ‹Π²Π°ΡŽΡ‚ΡΡ алгСбраичСски),

3) ΠΈΡΠΏΠΎΠ»ΡŒΠ·ΡƒΡ Π·Π°ΠΊΠΎΠ½ Ома для Π·Π°ΠΌΠΊΠ½ΡƒΡ‚ΠΎΠΉ Ρ†Π΅ΠΏΠΈ, ΡƒΡΡ‚Π°Π½ΠΎΠ²ΠΈΡ‚ΡŒ связь ΠΌΠ΅ΠΆΠ΄Ρƒ силой Ρ‚ΠΎΠΊΠ° ΠΈ Π­Π”Π‘.

Π—Π°ΠΊΠΎΠ½ Ома ΠΈ элСктричСская Ρ†Π΅ΠΏΡŒ элСктровоза β€” EduTranslator

ΠŸΠ΅Ρ€Π²ΠΎΠ½Π°Ρ‡Π°Π»ΡŒΠ½ΠΎ элСктричСская тяга ΠΏΡ€ΠΈΠΌΠ΅Π½ΡΠ»Π°ΡΡŒΒ Π½Π° городских Ρ‚Ρ€Π°ΠΌΠ²Π°ΠΉΠ½Ρ‹Ρ… линиях ΠΈ ΠΏΡ€ΠΎΠΌΡ‹ΡˆΠ»Π΅Π½Π½Ρ‹Ρ… прСдприятиях, особСнно Π½Π° Ρ€ΡƒΠ΄Π½ΠΈΠΊΠ°Ρ… ΠΈ Π² ΡƒΠ³ΠΎΠ»ΡŒΠ½Ρ‹Ρ… копях. Но ΠΎΡ‡Π΅Π½ΡŒ скоро оказалось, Ρ‡Ρ‚ΠΎ ΠΎΠ½Π° Π²Ρ‹Π³ΠΎΠ΄Π½Π° Π½Π° ΠΏΠ΅Ρ€Π΅Π²Π°Π»ΡŒΠ½Ρ‹Ρ… ΠΈ Ρ‚ΠΎΠ½Π½Π΅Π»ΡŒΠ½Ρ‹Ρ… участках ΠΆΠ΅Π»Π΅Π·Π½Ρ‹Ρ… Π΄ΠΎΡ€ΠΎΠ³, Π° Ρ‚Π°ΠΊΠΆΠ΅ Π² ΠΏΡ€ΠΈΠ³ΠΎΡ€ΠΎΠ΄Π½ΠΎΠΌ Π΄Π²ΠΈΠΆΠ΅Π½ΠΈΠΈ. Π’ 1895 Π³. Π² БША Π±Ρ‹Π»ΠΈ элСктрифицированы Ρ‚ΠΎΠ½Π½Π΅Π»ΡŒ Π² Π‘Π°Π»Ρ‚ΠΈΠΌΠΎΡ€Π΅ ΠΈ Ρ‚ΠΎΠ½Π½Π΅Π»ΡŒΠ½Ρ‹Π΅ ΠΏΠΎΠ΄Ρ…ΠΎΠ΄Ρ‹ ΠΊ Нью-Π™ΠΎΡ€ΠΊΡƒ. Для этих Π»ΠΈΠ½ΠΈΠΉ построСны элСктровозы ΠΌΠΎΡ‰Π½ΠΎΡΡ‚ΡŒΡŽ185ΠΊΠ’Ρ‚(50ΠΊΠΌ/Ρ‡).[1]


ПослС ΠΏΠ΅Ρ€Π²ΠΎΠΉ ΠΌΠΈΡ€ΠΎΠ²ΠΎΠΉΒ Π²ΠΎΠΉΠ½Ρ‹Β Π½Π° ΠΏΡƒΡ‚ΡŒ элСктрификации ΠΆΠ΅Π»Π΅Π·Π½Ρ‹Ρ… Π΄ΠΎΡ€ΠΎΠ³Β Π²ΡΡ‚ΡƒΠΏΠ°ΡŽΡ‚ ΠΌΠ½ΠΎΠ³ΠΈΠ΅ страны. Π’Β  Π‘Π‘Π‘Π  был построСн ΠΏΠ΅Ρ€Π²Ρ‹ΠΉ отСчСствСнный элСктровоз сСрии Бс Π² 1932Π³. Π£ΠΆΠ΅ ΠΊ 1935 Π³. Π² Π‘Π‘Π‘Π  Π±Ρ‹Π»ΠΎ элСктрифицировано 1907 ΠΊΠΌ ΠΏΡƒΡ‚Π΅ΠΉ ΠΈ Π½Π°Ρ…ΠΎΠ΄ΠΈΠ»ΠΎΡΡŒ Π² эксплуатации 84 элСктровоза.
Π’ Π½Π°ΡΡ‚ΠΎΡΡ‰Π΅Π΅Β Π²Ρ€Π΅ΠΌΡΒ ΠΎΠ±Ρ‰Π°ΡΒ ΠΏΡ€ΠΎΡ‚ΡΠΆΠ΅Π½Π½ΠΎΡΡ‚ΡŒΒ  элСктричСских ТСлСзных Π΄ΠΎΡ€ΠΎΠ³Β Π²ΠΎ всСм ΠΌΠΈΡ€Π΅ достигла 200 тыс. ΠΊΠΌ, что составляСт ΠΏΡ€ΠΈΠΌΠ΅Ρ€Π½ΠΎ 20% ΠΎΠ±Ρ‰Π΅ΠΉ ΠΈΡ… Π΄Π»ΠΈΠ½Ρ‹. Π­Ρ‚ΠΎ, ΠΊΠ°ΠΊ ΠΏΡ€Π°Π²ΠΈΠ»ΠΎ, Π½Π°ΠΈΠ±ΠΎΠ»Π΅Π΅ грузонапряТСнныС Π»ΠΈΠ½ΠΈΠΈ, Π³ΠΎΡ€Π½Ρ‹Π΅ участки с ΠΊΡ€ΡƒΡ‚Ρ‹ΠΌΠΈ подъСмами ΠΈ многочислСнными ΠΊΡ€ΠΈΠ²Ρ‹ΠΌΠΈ участками ΠΏΡƒΡ‚ΠΈ, ΠΏΡ€ΠΈΠ³ΠΎΡ€ΠΎΠ΄Π½Ρ‹Π΅ ΡƒΠ·Π»Ρ‹ Π±ΠΎΠ»ΡŒΡˆΠΈΡ… Π³ΠΎΡ€ΠΎΠ΄ΠΎΠ² с интСнсивным Π΄Π²ΠΈΠΆΠ΅Π½ΠΈΠ΅ΠΌ элСктропоСздов.
Π’Π΅Ρ…Π½ΠΈΠΊΠ° элСктричСских ТСлСзных Π΄ΠΎΡ€ΠΎΠ³ Π·Π° врСмя ΠΈΡ… сущСствования измСнилась ΠΊΠΎΡ€Π΅Π½Π½Ρ‹ΠΌ ΠΎΠ±Ρ€Π°Π·ΠΎΠΌ, сохранился Ρ‚ΠΎΠ»ΡŒΠΊΠΎ ΠΏΡ€ΠΈΠ½Ρ†ΠΈΠΏ дСйствия. ΠŸΡ€ΠΈΠΌΠ΅Π½ΡΠ΅Ρ‚ΡΡ ΠΏΡ€ΠΈΠ²ΠΎΠ΄ осСй Π»ΠΎΠΊΠΎΠΌΠΎΡ‚ΠΈΠ²Π° ΠΎΡ‚ элСктричСских тяговых Π΄Π²ΠΈΠ³Π°Ρ‚Π΅Π»Π΅ΠΉ, ΠΊΠΎΡ‚ΠΎΡ€Ρ‹Π΅ ΠΈΡΠΏΠΎΠ»ΡŒΠ·ΡƒΡŽΡ‚ ΡΠ½Π΅Ρ€Π³ΠΈΡŽ элСктростанций. Π­Ρ‚Π° энСргия подводится ΠΎΡ‚ элСктростанций ΠΊ ΠΆΠ΅Π»Π΅Π·Π½ΠΎΠΉ Π΄ΠΎΡ€ΠΎΠ³Π΅ ΠΏΠΎ Π²Ρ‹ΡΠΎΠΊΠΎΠ²ΠΎΠ»ΡŒΡ‚Π½Ρ‹ΠΌ линиям элСктропСрСдачи, Π° ΠΊ элСктроподвиТному составу β€” ΠΏΠΎ ΠΊΠΎΠ½Ρ‚Π°ΠΊΡ‚Π½ΠΎΠΉ сСти. ΠžΠ±Ρ€Π°Ρ‚Π½ΠΎΠΉ Ρ†Π΅ΠΏΡŒΡŽ слуТат Ρ€Π΅Π»ΡŒΡΡ‹ ΠΈ зСмля.
ΠŸΡ€ΠΈΠΌΠ΅Π½ΡΡŽΡ‚ΡΡ три различныС систСмы элСктричСской  тяги β€” постоянного тока, ΠΏΠ΅Ρ€Π΅ΠΌΠ΅Π½Π½ΠΎΠ³ΠΎΒ  Ρ‚ΠΎΠΊΠ°, Π΄Π²ΠΎΠΉΠ½ΠΎΠ³ΠΎ питания, многосистСмныС.
Π’ Π΄Π°Π½Π½ΠΎΠΉ Ρ€Π°Π±ΠΎΡ‚Π΅ рассмотрим элСктровозы  с элСктричСской тягой основанной Π½Π° постоянном Ρ‚ΠΎΠΊΠ΅.[2]

ЦСль Ρ€Π°Π±ΠΎΡ‚Ρ‹: ΠΈΠ·ΡƒΡ‡Π΅Π½ΠΈΠ΅ практичСского примСнСния Π·Π°ΠΊΠΎΠ½Π° Ома Π² Ρ€Π°Π±ΠΎΡ‚Π΅ элСктричСской Ρ†Π΅ΠΏΠΈ элСктровоза

Для достиТСния поставлСнной Ρ†Π΅Π»ΠΈ Π²Ρ‹Π΄Π²ΠΈΠ½ΡƒΡ‚Ρ‹ ΡΠ»Π΅Π΄ΡƒΡŽΡ‰ΠΈΠ΅ Π·Π°Π΄Π°Ρ‡ΠΈ:

  1. Π˜Π·ΡƒΡ‡ΠΈΡ‚ΡŒ источники ΠΈΠ½Ρ„ΠΎΡ€ΠΌΠ°Ρ†ΠΈΠΈ ΠΏΠΎ Π΄Π°Π½Π½ΠΎΠΉ Ρ‚Π΅ΠΌΠ΅
  2. ΠŸΡ€ΠΎΠ°Π½Π°Π»ΠΈΠ·ΠΈΡ€ΠΎΠ²Π°Ρ‚ΡŒ Π·Π°ΠΊΠΎΠ½ Ома для участка Ρ†Π΅ΠΏΠΈ
  3. ΠŸΡ€ΠΎΠ°Π½Π°Π»ΠΈΠ·ΠΈΡ€ΠΎΠ²Π°Ρ‚ΡŒ Π·Π°ΠΊΠΎΠ½ Ома для ΠΏΠΎΠ»Π½ΠΎΠΉ Ρ†Π΅ΠΏΠΈ
  4. Π Π°ΡΡΠΌΠΎΡ‚Ρ€Π΅Ρ‚ΡŒ ΠΏΡ€ΠΈΠΌΠ΅Π½Π΅Π½ΠΈΠ΅ Π·Π°ΠΊΠΎΠ½Π° Ома Π² Ρ€Π°Π±ΠΎΡ‚Π΅ элСктричСской Ρ†Π΅ΠΏΠΈ элСктровоза

Π—ΠΠšΠžΠ ОМА

Π—Π°ΠΊΠΎΠ½ Ома для участка Ρ†Π΅ΠΏΠΈ

Π’ 1826 Π²Π΅Π»ΠΈΡ‡Π°ΠΉΡˆΠΈΠΉ Π½Π΅ΠΌΠ΅Ρ†ΠΊΠΈΠΉ Ρ„ΠΈΠ·ΠΈΠΊ Π“Π΅ΠΎΡ€Π³ Π‘ΠΈΠΌΠΎΠ½ Ом (Π² соотвСтствии с рисунком 1) ΠΏΡƒΠ±Π»ΠΈΠΊΡƒΠ΅Ρ‚ свою Ρ€Π°Π±ΠΎΡ‚Ρƒ Β«ΠžΠΏΡ€Π΅Π΄Π΅Π»Π΅Π½ΠΈΠ΅ Π·Π°ΠΊΠΎΠ½Π°, ΠΏΠΎ ΠΊΠΎΡ‚ΠΎΡ€ΠΎΠΌΡƒ ΠΌΠ΅Ρ‚Π°Π»Π»Ρ‹ проводят ΠΊΠΎΠ½Ρ‚Π°ΠΊΡ‚Π½ΠΎΠ΅ элСктричСство», Π³Π΄Π΅ Π΄Π°Π΅Ρ‚ Ρ„ΠΎΡ€ΠΌΡƒΠ»ΠΈΡ€ΠΎΠ²ΠΊΡƒ Π·Π½Π°ΠΌΠ΅Π½ΠΈΡ‚ΠΎΠΌΡƒ Π·Π°ΠΊΠΎΠ½Ρƒ. Π£Ρ‡Π΅Π½Ρ‹Π΅ Ρ‚ΠΎΠ³ΠΎ Π²Ρ€Π΅ΠΌΠ΅Π½ΠΈ встрСтили Π²Ρ€Π°ΠΆΠ΄Π΅Π±Π½ΠΎ ΠΏΡƒΠ±Π»ΠΈΠΊΠ°Ρ†ΠΈΠΈ Π²Π΅Π»ΠΈΠΊΠΎΠ³ΠΎ Ρ„ΠΈΠ·ΠΈΠΊΠ°. И лишь послС Ρ‚ΠΎΠ³ΠΎ, ΠΊΠ°ΠΊ Π΄Ρ€ΡƒΠ³ΠΎΠΉ ΡƒΡ‡Π΅Π½Ρ‹ΠΉ – Клод ΠŸΡƒΠ»ΡŒΠ΅, ΠΏΡ€ΠΈΡˆΠ΅Π» ΠΊ Ρ‚Π΅ΠΌ ΠΆΠ΅ Π²Ρ‹Π²ΠΎΠ΄Π°ΠΌ ΠΎΠΏΡ‹Ρ‚Π½Ρ‹ΠΌ ΠΏΡƒΡ‚Π΅ΠΌ, Π·Π°ΠΊΠΎΠ½ Ома ΠΏΡ€ΠΈΠ·Π½Π°Π»ΠΈ Π²ΠΎ всСм ΠΌΠΈΡ€Π΅. Β Β Β [3]  НСмСцкий Ρ„ΠΈΠ·ΠΈΠΊ Π“Π΅ΠΎΡ€Π³ Ом (1787 -1854) ΡΠΊΡΠΏΠ΅Ρ€ΠΈΠΌΠ΅Π½Ρ‚Π°Π»ΡŒΠ½ΠΎ установил, Ρ‡Ρ‚ΠΎ сила Ρ‚ΠΎΠΊΠ° I, Ρ‚Π΅ΠΊΡƒΡ‰Π΅Π³ΠΎ ΠΏΠΎ ΠΎΠ΄Π½ΠΎΡ€ΠΎΠ΄Π½ΠΎΠΌΡƒ мСталличСскому ΠΏΡ€ΠΎΠ²ΠΎΠ΄Π½ΠΈΠΊΡƒ (Ρ‚. Π΅. ΠΏΡ€ΠΎΠ²ΠΎΠ΄Π½ΠΈΠΊΡƒ, Π² ΠΊΠΎΡ‚ΠΎΡ€ΠΎΠΌ Π½Π΅ Π΄Π΅ΠΉΡΡ‚Π²ΡƒΡŽΡ‚ сторонниС силы), ΠΏΡ€ΠΎΠΏΠΎΡ€Ρ†ΠΈΠΎΠ½Π°Π»ΡŒΠ½ΠΎ Π½Π°ΠΏΡ€ΡΠΆΠ΅Π½ΠΈΡŽ U Π½Π° ΠΊΠΎΠ½Ρ†Π°Ρ… ΠΏΡ€ΠΎΠ²ΠΎΠ΄Π½ΠΈΠΊΠ° (Π² соотвСтствии с рисунком 2).

Π€ΠΎΡ€ΠΌΡƒΠ»ΠΈΡ€ΠΎΠ²ΠΊΠ° Π·Π°ΠΊΠΎΠ½Π° Ома для участка цСпи – сила Ρ‚ΠΎΠΊΠ° прямо ΠΏΡ€ΠΎΠΏΠΎΡ€Ρ†ΠΈΠΎΠ½Π°Π»ΡŒΠ½Π° Π½Π°ΠΏΡ€ΡΠΆΠ΅Π½ΠΈΡŽ, ΠΈ ΠΎΠ±Ρ€Π°Ρ‚Π½ΠΎ ΠΏΡ€ΠΎΠΏΠΎΡ€Ρ†ΠΈΠΎΠ½Π°Π»ΡŒΠ½Π°Β ΡΠΎΠΏΡ€ΠΎΡ‚ΠΈΠ²Π»Π΅Π½ΠΈΡŽ.
Π³Π΄Π΅ R — элСктричСскоС сопротивлСниС ΠΏΡ€ΠΎΠ²ΠΎΠ΄Π½ΠΈΠΊΠ°.
Π£Ρ€Π°Π²Π½Π΅Π½ΠΈΠ΅ I = U/R Π²Ρ‹Ρ€Π°ΠΆΠ°Π΅Ρ‚Β Π·Π°ΠΊΠΎΠ½ Ома для участка Ρ†Π΅ΠΏΠΈΒ (Π½Π΅ содСрТащСго источника Ρ‚ΠΎΠΊΠ°): сила Ρ‚ΠΎΠΊΠ° Π² ΠΏΡ€ΠΎΠ²ΠΎΠ΄Π½ΠΈΠΊΠ΅ прямо ΠΏΡ€ΠΎΠΏΠΎΡ€Ρ†ΠΈΠΎΠ½Π°Π»ΡŒΠ½Π° ΠΏΡ€ΠΈΠ»ΠΎΠΆΠ΅Π½Π½ΠΎΠΌΡƒ Π½Π°ΠΏΡ€ΡΠΆΠ΅Π½ΠΈΡŽ ΠΈ ΠΎΠ±Ρ€Π°Ρ‚Π½ΠΎ ΠΏΡ€ΠΎΠΏΠΎΡ€Ρ†ΠΈΠΎΠ½Π°Π»ΡŒΠ½ΠΎ ΡΠΎΠΏΡ€ΠΎΡ‚ΠΈΠ²Π»Π΅Π½ΠΈΡŽ ΠΏΡ€ΠΎΠ²ΠΎΠ΄Π½ΠΈΠΊΠ°. Участок Ρ†Π΅ΠΏΠΈ, Π² ΠΊΠΎΡ‚ΠΎΡ€ΠΎΠΌ Π½Π΅ Π΄Π΅ΠΉΡΡ‚Π²ΡƒΡŽΡ‚ э.Π΄.с. (сторонниС силы) Π½Π°Π·Ρ‹Π²Π°ΡŽΡ‚ ΠΎΠ΄Π½ΠΎΡ€ΠΎΠ΄Π½Ρ‹ΠΌ участком Ρ†Π΅ΠΏΠΈ, поэтому эта Ρ„ΠΎΡ€ΠΌΡƒΠ»ΠΈΡ€ΠΎΠ²ΠΊΠ° Π·Π°ΠΊΠΎΠ½Π° Ома справСдлива для ΠΎΠ΄Π½ΠΎΡ€ΠΎΠ΄Π½ΠΎΠ³ΠΎ участка Ρ†Π΅ΠΏΠΈ.
Π›ΡŽΠ±ΠΎΠΉ участок ΠΈΠ»ΠΈ элСмСнт элСктричСской Ρ†Π΅ΠΏΠΈ ΠΌΠΎΠΆΠ½ΠΎ ΠΎΡ…Π°Ρ€Π°ΠΊΡ‚Π΅Ρ€ΠΈΠ·ΠΎΠ²Π°Ρ‚ΡŒ ΠΏΡ€ΠΈ ΠΏΠΎΠΌΠΎΡ‰ΠΈ Ρ‚Ρ€Ρ‘Ρ… характСристик: Ρ‚ΠΎΠΊΠ°, напряТСния ΠΈ сопротивлСния. [4]

Как ΠΈΡΠΏΠΎΠ»ΡŒΠ·ΠΎΠ²Π°Ρ‚ΡŒ Ρ‚Ρ€Π΅ΡƒΠ³ΠΎΠ»ΡŒΠ½ΠΈΠΊ Ома: (Π² соотвСтствии с рисунком 3)Β Π·Π°ΠΊΡ€Ρ‹Π²Π°Π΅ΠΌ ΠΈΡΠΊΠΎΠΌΡƒΡŽ Π²Π΅Π»ΠΈΡ‡ΠΈΠ½Ρƒ β€” Π΄Π²Π° Π΄Ρ€ΡƒΠ³ΠΈΡ… символа Π΄Π°Π΄ΡƒΡ‚ Ρ„ΠΎΡ€ΠΌΡƒΠ»Ρƒ для Π΅Ρ‘ вычислСния. ΠšΡΡ‚Π°Ρ‚ΠΈ, Π·Π°ΠΊΠΎΠ½ΠΎΠΌ Ома называСтся Ρ‚ΠΎΠ»ΡŒΠΊΠΎ ΠΎΠ΄Π½Π° Ρ„ΠΎΡ€ΠΌΡƒΠ»Π° ΠΈΠ· Ρ‚Ρ€Π΅ΡƒΠ³ΠΎΠ»ΡŒΠ½ΠΈΠΊΠ° – Ρ‚Π°, которая ΠΎΡ‚Ρ€Π°ΠΆΠ°Π΅Ρ‚ Π·Π°Π²ΠΈΡΠΈΠΌΠΎΡΡ‚ΡŒ Ρ‚ΠΎΠΊΠ° ΠΎΡ‚ напряТСния ΠΈ сопротивлСния. Π”Π²Π΅ Π΄Ρ€ΡƒΠ³ΠΈΠ΅ Ρ„ΠΎΡ€ΠΌΡƒΠ»Ρ‹, хотя ΠΈ ΡΠ²Π»ΡΡŽΡ‚ΡΡ Π΅Ρ‘ слСдствиСм, физичСского смысла Π½Π΅ ΠΈΠΌΠ΅ΡŽΡ‚. РасчСты, выполняСмыС с ΠΏΠΎΠΌΠΎΡ‰ΡŒΡŽ Π·Π°ΠΊΠΎΠ½Π° Ома для участка Ρ†Π΅ΠΏΠΈ, Π±ΡƒΠ΄ΡƒΡ‚ ΠΏΡ€Π°Π²ΠΈΠ»ΡŒΠ½Ρ‹ Π² Ρ‚ΠΎΠΌ случаС, ΠΊΠΎΠ³Π΄Π° напряТСниС Π²Ρ‹Ρ€Π°ΠΆΠ΅Π½ΠΎ Π² Π²ΠΎΠ»ΡŒΡ‚Π°Ρ…, сопротивлСниС Π² ΠΎΠΌΠ°Ρ… ΠΈ Ρ‚ΠΎΠΊ Π² Π°ΠΌΠΏΠ΅Ρ€Π°Ρ…. Если ΠΈΡΠΏΠΎΠ»ΡŒΠ·ΡƒΡŽΡ‚ΡΡ ΠΊΡ€Π°Ρ‚Π½Ρ‹Π΅ Π΅Π΄ΠΈΠ½ΠΈΡ†Ρ‹ ΠΈΠ·ΠΌΠ΅Ρ€Π΅Π½ΠΈΠΉ этих Π²Π΅Π»ΠΈΡ‡ΠΈΠ½ (Π½Π°ΠΏΡ€ΠΈΠΌΠ΅Ρ€, ΠΌΠΈΠ»Π»ΠΈΠ°ΠΌΠΏΠ΅Ρ€, ΠΌΠΈΠ»Π»ΠΈΠ²ΠΎΠ»ΡŒΡ‚, ΠΌΠ΅Π³Π°ΠΎΠΌ ΠΈ Ρ‚. Π΄.), Ρ‚ΠΎ ΠΈΡ… слСдуСт пСрСвСсти соотвСтствСнно Π² Π°ΠΌΠΏΠ΅Ρ€Ρ‹, Π²ΠΎΠ»ΡŒΡ‚Ρ‹ ΠΈ ΠΎΠΌΡ‹. Π§Ρ‚ΠΎΠ±Ρ‹ ΠΏΠΎΠ΄Ρ‡Π΅Ρ€ΠΊΠ½ΡƒΡ‚ΡŒ это, ΠΈΠ½ΠΎΠ³Π΄Π° Ρ„ΠΎΡ€ΠΌΡƒΠ»Ρƒ Π·Π°ΠΊΠΎΠ½Π° Ома для участка Ρ†Π΅ΠΏΠΈ ΠΏΠΈΡˆΡƒΡ‚ Ρ‚Π°ΠΊ:

Π°ΠΌΠΏΠ΅Ρ€ = Π²ΠΎΠ»ΡŒΡ‚/ΠΎΠΌ. МоТно Ρ‚Π°ΠΊΠΆΠ΅ Ρ€Π°ΡΡΡ‡ΠΈΡ‚Ρ‹Π²Π°Ρ‚ΡŒ Ρ‚ΠΎΠΊ Π² ΠΌΠΈΠ»Π»ΠΈΠ°ΠΌΠΏΠ΅Ρ€Π°Ρ… ΠΈ ΠΌΠΈΠΊΡ€ΠΎΠ°ΠΌΠΏΠ΅Ρ€Π°Ρ…, ΠΏΡ€ΠΈ этом напряТСниС Π΄ΠΎΠ»ΠΆΠ½ΠΎ Π±Ρ‹Ρ‚ΡŒ Π²Ρ‹Ρ€Π°ΠΆΠ΅Π½ΠΎ Π² Π²ΠΎΠ»ΡŒΡ‚Π°Ρ…, Π° сопротивлСниС β€” Π² ΠΊΠΈΠ»ΠΎΠΎΠΌΠ°Ρ… ΠΈ ΠΌΠ΅Π³Π°ΠΎΠΌΠ°Ρ… соотвСтствСнно. [5]

1.Π—Π°ΠΊΠΎΠ½ Ома для ΠΏΠΎΠ»Π½ΠΎΠΉ Ρ†Π΅ΠΏΠΈ

Рассмотрим Π·Π°ΠΌΠΊΠ½ΡƒΡ‚ΡƒΡŽ ΡΠ»Π΅ΠΊΡ‚Ρ€ΠΈΡ‡Π΅ΡΠΊΡƒΡŽ Ρ†Π΅ΠΏΡŒ, ΡΠΎΡΡ‚ΠΎΡΡ‰ΡƒΡŽ ΠΈΠ· Π΄Π²ΡƒΡ… частСй: собствСнно источника с элСктродвиТущСй силой Ɛ и Π²Π½ΡƒΡ‚Ρ€Π΅Π½Π½ΠΈΠΌ сопротивлСниСм rΒ ΠΈ внСшнСй части Ρ†Π΅ΠΏΠΈ β€” ΠΏΡ€ΠΎΠ²ΠΎΠ΄Π½ΠΈΠΊΠ° с сопротивлСниСм R (Π² соотвСтствии с рисунком 4).

Π—Π°ΠΊΠΎΠ½ Ома для ΠΏΠΎΠ»Π½ΠΎΠΉ цСпи устанав­ливаСт Π·Π°Π²ΠΈΡΠΈΠΌΠΎΡΡ‚ΡŒ силы Ρ‚ΠΎΠΊΠ° Π² Π·Π°ΠΌΠΊΠ½ΡƒΡ‚ΠΎΠΉ Ρ†Π΅ΠΏΠΈΒ IΒ ΠΎΡ‚ элСктродвиТущСй силы источника Ɛ и ΠΏΠΎΠ»Π½ΠΎΠ³ΠΎ сопротивлСния Ρ†Π΅ΠΏΠΈΒ R + r.Β Π­Ρ‚Ρƒ Π·Π°Π²ΠΈΡΠΈΠΌΠΎΡΡ‚ΡŒ ΠΌΠΎΠΆΠ½ΠΎ ΡƒΡΡ‚Π°Π½ΠΎΠ²ΠΈΡ‚ΡŒ Π½Π° основании закона сохранСния энСргии и закона ДТоу­ля-Π›Π΅Π½Ρ†Π°. Если Ρ‡Π΅Ρ€Π΅Π· ΠΏΠΎΠΏΠ΅Ρ€Π΅Ρ‡Π½ΠΎΠ΅ сСчСниС ΠΏΡ€ΠΎΠ²ΠΎΠ΄Π½ΠΈΠΊΠ° Π·Π° врСмя Δt заряТСнными час­тицами пСрСносится заряд Δq, Ρ‚ΠΎΒ Ρ€Π°Π±ΠΎΡ‚Π° сторонних сил

Aст.Β = ƐΔq = ƐIΞ”t.

Если Π² Ρ†Π΅ΠΏΠΈ элСктричСская энСргия прСв­ращаСтся лишь Π² Ρ‚Π΅ΠΏΠ»ΠΎΠ²ΡƒΡŽ, Ρ‚ΠΎ ΠΏΠΎΒ Π·Π°ΠΊΠΎΠ½Ρƒ со­хранСния энСргии Аст.Β = QΒ ΠΈ ΠΎΠ±Ρ‰Π΅Π΅ коли­чСство Ρ‚Π΅ΠΏΠ»ΠΎΡ‚Ρ‹, Π²Ρ‹Π΄Π΅Π»ΡΡŽΡ‰Π΅Π΅ΡΡ Π² Π·Π°ΠΌΠΊΠ½ΡƒΡ‚ΠΎΠΉ Ρ†Π΅ΠΏΠΈ, Ρ€Π°Π²Π½ΠΎ суммС количСств Ρ‚Π΅ΠΏΠ»ΠΎΡ‚Ρ‹, Π²Ρ‹Β­Π΄Π΅Π»ΡΡŽΡ‰ΠΈΡ…ΡΡ Π²ΠΎ внСшнСй ΠΈ Π²Π½ΡƒΡ‚Ρ€Π΅Π½Π½Π΅ΠΉ час­тях Ρ†Π΅ΠΏΠΈ

Q = I2RΞ”t + I2rΞ”t.

Если

Aст.Β = Q = (Ɛ / R + r) β€’ IΞ”t,

Ρ‚ΠΎ

ƐIΞ”t = I2RΞ”t + I2rΞ”t.

Π˜Ρ‚Π°ΠΊ,

Ɛ = IR + Ir

ΠΈ

I = Ɛ / (R + r),

Ρ‡Ρ‚ΠΎ ΠΈ Π²Ρ‹Ρ€Π°ΠΆΠ°Π΅Ρ‚Β Π·Π°ΠΊΠΎΠ½ Ома для ΠΏΠΎΠ»Π½ΠΎΠΉ Ρ†Π΅ΠΏΠΈ.

Π—Π°ΠΊΠΎΠ½ Ома для ΠΏΠΎΠ»Π½ΠΎΠΉ Ρ†Π΅ΠΏΠΈ. Π‘ΠΈΠ»Π° Ρ‚ΠΎΠΊΠ° Π² Π·Π°ΠΌΠΊΠ½ΡƒΡ‚ΠΎΠΉ Ρ†Π΅ΠΏΠΈ измСряСтся ΠΎΡ‚Π½ΠΎΒ­ΡˆΠ΅Π½ΠΈΠ΅ΠΌ элСктродвиТущСй силы источника Ρ‚ΠΎΠΊΠ°, ΠΈΠΌΠ΅ΡŽΡ‰Π΅Π³ΠΎΡΡ Π² этой Ρ†Π΅ΠΏΠΈ, ΠΊ ΠΏΠΎΠ»Π½ΠΎΠΌΡƒ Π΅Π΅ ΡΠΎΠΏΡ€ΠΎΡ‚ΠΈΠ²Π»Π΅Π½ΠΈΡŽ.

Из сказанного Π²Ρ‹ΡˆΠ΅ ΠΌΠΎΠΆΠ½ΠΎ ΡΠ΄Π΅Π»Π°Ρ‚ΡŒ Π²Ρ‹Β­Π²ΠΎΠ΄, Ρ‡Ρ‚ΠΎ Π·Π°ΠΊΠΎΠ½ Ома для ΠΏΠΎΠ»Π½ΠΎΠΉ цСпи являСт­ся ΠΎΠ΄Π½ΠΈΠΌ ΠΈΠ· Π²Ρ‹Ρ€Π°ΠΆΠ΅Π½ΠΈΠΉ Π·Π°ΠΊΠΎΠ½Π° сохранСния энСргии.

Π’ΠΎ ΠΌΠ½ΠΎΠ³ΠΈΡ… случаях для характСристики источников Ρ‚ΠΎΠΊΠ° нСдостаточно ΠΈΡΠΏΠΎΠ»ΡŒΠ·ΠΎΠ²Π°Ρ‚ΡŒ Π»ΠΈΡˆΡŒΒ Π­Π”Π‘. ΠŸΡƒΡΡ‚ΡŒ, Π½Π°ΠΏΡ€ΠΈΠΌΠ΅Ρ€, Π½Π΅ΠΎΠ±Ρ…ΠΎΠ΄ΠΈΠΌΠΎ ΡƒΡΡ‚Π°Π½ΠΎΠ²ΠΈΡ‚ΡŒ, Ρ‚ΠΎΠΊ ΠΊΠ°ΠΊΠΎΠΉ максимальной силы ΠΌΠΎΠΆΠ΅Ρ‚ Π΄Π°Ρ‚ΡŒ ΠΎΠΏΡ€Π΅Π΄Π΅Π»Π΅Π½Π½Ρ‹ΠΉ источник Ρ‚ΠΎΠΊΠ°. Если ΠΈΡΡ…ΠΎΠ΄ΠΈΡ‚ΡŒ ΠΈΠ·Β Π·Π°ΠΊΠΎΠ½Π° Ома для ΠΏΠΎΠ»Π½ΠΎΠΉ Ρ†Π΅ΠΏΠΈ

I = Ɛ / (R + r),

Ρ‚ΠΎ ΠΎΡ‡Π΅Π²ΠΈΠ΄Π½ΠΎ, Ρ‡Ρ‚ΠΎ максимальной сила Ρ‚ΠΎΠΊΠ° Π² Ρ†Π΅ΠΏΠΈ Π±ΡƒΠ΄Π΅Ρ‚ Ρ‚ΠΎΠ³Π΄Π°, ΠΊΠΎΠ³Π΄Π° внСшнСС сопротивлСниС Ρ†Π΅ΠΏΠΈΒ R стрСмится ΠΊ Π½ΡƒΠ»ΡŽ β€” это короткоС Π·Π°ΠΌΡ‹ΠΊΠ°Π½ΠΈΠ΅Β Π² Ρ†Π΅ΠΏΠΈ. ΠŸΡ€ΠΈ этом Ρ‚ΠΎΠΊΒ ΠΊΠΎΡ€ΠΎΡ‚ΠΊΠΎΠ³ΠΎ замыкания имССт силу ImaxΒ = Ɛ / r, ΠΏΠΎΡΠΊΠΎΠ»ΡŒΠΊΡƒΒ ΖΒ ΠΈΒ rΒ ΠΈΠ·ΠΌΠ΅Π½ΠΈΡ‚ΡŒ для Π΄Π°Π½Π½ΠΎΠ³ΠΎ источника ΠΌΡ‹ Π½Π΅ ΠΌΠΎΠΆΠ΅ΠΌ, ΠΎΠ½ΠΈ ΡΠ²Β­Π»ΡΡŽΡ‚ΡΡ характСристиками источника.

Если ΠΏΡ€Π΅Π΄ΡΡ‚Π°Π²ΠΈΡ‚ΡŒ, Ρ‡Ρ‚ΠΎ сопротивлСниС внС­шнСй части Ρ†Π΅ΠΏΠΈ стрСмится ΠΊ бСсконСч­ности (Ρ†Π΅ΠΏΡŒ становится Ρ€Π°Π·ΠΎΠΌΠΊΠ½ΡƒΡ‚ΠΎΠΉ), Ρ‚ΠΎ напряТСниС Π½Π° ΠΏΠΎΠ»ΡŽΡΠ°Ρ… источника Ρ‚ΠΎΠΊΠ°Β IR стрСмится ΠΊ элСктродвиТущСй силС, Ρ‚ΠΎ Π΅ΡΡ‚ΡŒ: элСктродвиТущая сила источника Ρ‚ΠΎΠΊΠ° Ρ€Π°Π²Π½Π° Π½Π°ΠΏΡ€ΡΠΆΠ΅Π½ΠΈΡŽ Π½Π° ΠΏΠΎΠ»ΡŽΡΠ°Ρ… Ρ€Π°Π·ΠΎΠΌΠΊΠ½ΡƒΡ‚ΠΎΠ³ΠΎ источ­ника. [6]

2. ΠŸΡ€ΠΈΠΌΠ΅Π½Π΅Π½ΠΈΠ΅ Π·Π°ΠΊΠΎΠ½Π° Ома Π² Ρ€Π°Π±ΠΎΡ‚Π΅ элСктричСской Ρ†Π΅ΠΏΠΈ элСктровоза

Π—Π°ΠΊΠΎΠ½ Ома ΠΌΠΎΠΆΠ΅Ρ‚ Π±Ρ‹Ρ‚ΡŒ ΠΏΡ€ΠΈΠΌΠ΅Π½Ρ‘Π½ ΠΊ любой физичСской систСмС, Π² ΠΊΠΎΡ‚ΠΎΡ€ΠΎΠΉ Π΄Π΅ΠΉΡΡ‚Π²ΡƒΡŽΡ‚ ΠΏΠΎΡ‚ΠΎΠΊΠΈ частиц ΠΈΠ»ΠΈ ΠΏΠΎΠ»Π΅ΠΉ ΠΏΡ€Π΅ΠΎΠ΄ΠΎΠ»Π΅Π²Π°ΡŽΡ‰ΠΈΠ΅ сопротивлСниС. Π•Π³ΠΎ ΠΌΠΎΠΆΠ½ΠΎ ΠΏΡ€ΠΈΠΌΠ΅Π½ΡΡ‚ΡŒ для расчёта гидравличСских, пнСвматичСских, ΠΌΠ°Π³Π½ΠΈΡ‚Π½Ρ‹Ρ…, элСктричСских, свСтовых, Ρ‚Π΅ΠΏΠ»ΠΎΠ²Ρ‹Ρ… ΠΏΠΎΡ‚ΠΎΠΊΠΎΠ² ΠΈ Ρ‚. Π΄. Рассмотрим ΠΏΡ€ΠΈΠΌΠ΅Π½Π΅Π½ΠΈΠ΅ Π·Π°ΠΊΠΎΠ½Π° Ома Π² элСктричСской Ρ†Π΅ΠΏΠΈ элСктровоза (Π² соотвСтствии с рисунком 5). ΠžΡ‚Β Π²Π½Π΅ΡˆΠ½Π΅ΠΉ элСктричСской сСти (элСктростанции), которая вырабатываСт ΠΏΠ΅Ρ€Π΅ΠΌΠ΅Π½Π½Ρ‹ΠΉ Ρ‚Ρ€Π΅Ρ…Ρ„Π°Π·Π½Ρ‹ΠΉ Ρ‚ΠΎΠΊΒ ΠΏΡ€ΠΎΠΌΡ‹ΡˆΠ»Π΅Π½Π½ΠΎΠΉ частоты (50 Π“Π¦), Ρ‚ΠΎΠΊ поступаСт Π½Π° ΠΏΠΎΠ²Ρ‹ΡˆΠ°ΡŽΡ‰ΠΈΠ΅ трансформаторы, ΠΊΠΎΡ‚ΠΎΡ€Ρ‹Π΅ ΠΏΠΎΠ²Ρ‹ΡˆΠ°ΡŽΡ‚ напряТСниС ΠΎΡ‚ 200 тыс. Π΄ΠΎ Π±ΠΎΠ»Π΅Π΅ 1 ΠΌΠ»Π½. Π’. Π”Π°Π»Π΅Π΅ этот ΠΏΠ΅Ρ€Π΅ΠΌΠ΅Π½Π½Ρ‹ΠΉ Ρ‚Ρ€Π΅Ρ…Ρ„Π°Π·Π½Ρ‹ΠΉ Ρ‚ΠΎΠΊ ΠΏΠΎ линиям элСктропСрСдач поступаСт Π½Π° тяговыС подстанции, располоТСнныС вдоль ΠΆΠ΅Π»Π΅Π·Π½ΠΎΠ΄ΠΎΡ€ΠΎΠΆΠ½ΠΎΠ³ΠΎ ΠΏΡƒΡ‚ΠΈ Π½Π° расстоянии 50 β€” 100 ΠΊΠΌ.
Π’ тяговых подстанциях это высокоС напряТСниС поступаСт на тяговый  ΠΏΠΎΠ½ΠΈΠΆΠ°ΡŽΡ‰ΠΈΠΉ трансформатор, ΠΊΠΎΡ‚ΠΎΡ€Ρ‹ΠΉ ΠΏΠΎΠ½ΠΈΠΆΠ°Π΅Ρ‚ напряТСниС Π΄ΠΎ 3000 Π’ ΠΈ ΠΏΠΎΠ΄Π°Π΅Ρ‚ Π΅Π³ΠΎ Π½Π° Π²Ρ‹ΠΏΡ€ΡΠΌΠΈΡ‚Π΅Π»ΡŒΠ½ΠΎΠ΅ устройство, Π³Π΄Π΅ ΠΏΠΎ Π΄Π²ΡƒΡ…ΠΏΠΎΠ»ΡƒΠΏΠ΅Ρ€ΠΈΠΎΠ΄Π½ΠΎΠΉ схСмС ΠΏΠ΅Ρ€Π΅ΠΌΠ΅Π½Π½Ρ‹ΠΉ Ρ‚Ρ€Π΅Ρ…Ρ„Π°Π·Π½Ρ‹ΠΉ Ρ‚ΠΎΠΊ прСобразуСтся Π² постоянный Ρ‚ΠΎΠΊ напряТСниСм 3000 Π’. Π­Ρ‚ΠΎΡ‚ Ρ‚ΠΎΠΊ ΠΏΠΎ Π΄Π²ΡƒΡ…ΠΏΡ€ΠΎΠ²ΠΎΠ΄Π½ΠΎΠΉ схСмС подводится ΠΎΠ΄Π½ΠΎΠΉ ΠΏΠΎΠ»ΡΡ€Π½ΠΎΡΡ‚ΡŒΡŽ ΠΊ Ρ€Π΅Π»ΡŒΡΠ°ΠΌ, Π° Π΄Ρ€ΡƒΠ³ΠΎΠΉ β€” ΠΊ ΠΊΠΎΠ½Ρ‚Π°ΠΊΡ‚Π½ΠΎΠΌΡƒ ΠΏΡ€ΠΎΠ²ΠΎΠ΄Ρƒ, располоТСнному Π²Ρ‹ΡˆΠ΅ элСктровоза посСрСдинС Ρ€Π΅Π»ΡŒΡΠΎΠ² вдоль всСго ΠΆΠ΅Π»Π΅Π·Π½ΠΎΠ΄ΠΎΡ€ΠΎΠΆΠ½ΠΎΠ³ΠΎ ΠΏΡƒΡ‚ΠΈ.
ΠŸΡ€ΠΈ поднятом ΠΏΠ°Π½Ρ‚ΠΎΠ³Ρ€Π°Ρ„Π΅ постоянноС высокоС  напряТСниС ΠΏΠΎΡΡ‚ΡƒΠΏΠ°Π΅Ρ‚Β Π²Β Π²Ρ‹ΡΠΎΠΊΠΎΠ²ΠΎΠ»ΡŒΡ‚Π½Ρ‹Π΅ ΠΊΠ°ΠΌΠ΅Ρ€Ρ‹, гдС располоТСны контакторы ΠΈ пусковыС рСостаты. ΠœΠ°ΡˆΠΈΠ½ΠΈΡΡ‚ с ΠΏΠΎΠΌΠΎΡ‰ΡŒΡŽ ΠΊΠΎΠ½Ρ‚Ρ€ΠΎΠ»Π»Π΅Ρ€Π°, располоТСнного Π² ΠΊΠ°Π±ΠΈΠ½Π΅ ΠΌΠ°ΡˆΠΈΠ½ΠΈΡΡ‚Π°, ΠΏΠΎΠ΄ΠΊΠ»ΡŽΡ‡Π°Π΅Ρ‚ пусковыС рСостаты ΠΊ тяговым элСктродвигатСлям постоянного Ρ‚ΠΎΠΊΠ°, располоТСнным Π½Π° осях Ρ‚Π΅Π»Π΅ΠΆΠ΅ΠΊ. ΠžΡ‚ тяговых элСктродвигатСлСй Ρ‡Π΅Ρ€Π΅Π· Π·Π°Π·Π΅ΠΌΠ»ΡΡŽΡ‰ΠΈΠ΅ ΡˆΠΈΠ½Ρ‹ элСктричСский Ρ‚ΠΎΠΊ поступаСт Π½Π° колСсныС ΠΏΠ°Ρ€Ρ‹, Π° ΠΎΡ‚ Π½ΠΈΡ… β€” Π² Ρ€Π΅Π»ΡŒΡΡ‹, Π° ΠΏΠΎ Ρ€Π΅Π»ΡŒΡΠ°ΠΌ β€” возвращаСтся Π½Π° Ρ‚ΡΠ³ΠΎΠ²ΡƒΡŽ ΠΏΠΎΠ΄ΡΡ‚Π°Π½Ρ†ΠΈΡŽ. ЭлСктричСская Ρ†Π΅ΠΏΡŒ оказываСтся Π·Π°ΠΌΠΊΠ½ΡƒΡ‚ΠΎΠΉ ΠΈ ΠΏΠΎ тяговым элСктродвигатСлям Π½Π°Ρ‡ΠΈΠ½Π°Π΅Ρ‚ ΠΏΡ€ΠΎΡ‚Π΅ΠΊΠ°Ρ‚ΡŒ постоянный Ρ‚ΠΎΠΊ. Якоря элСктродвигатСлСй Π½Π°Ρ‡ΠΈΠ½Π°ΡŽΡ‚ Π²Ρ€Π°Ρ‰Π°Ρ‚ΡŒΡΡ, прСобразуя ΡΠ»Π΅ΠΊΡ‚Ρ€ΠΎΡΠ½Π΅Ρ€Π³ΠΈΡŽ постоянного Ρ‚ΠΎΠΊΠ° Π² ΠΌΠ΅Ρ…Π°Π½ΠΈΡ‡Π΅ΡΠΊΡƒΡŽ Ρ€Π°Π±ΠΎΡ‚Ρƒ вращСния якорСй. На Π²Π°Π»Ρƒ якоря Π·Π°ΠΊΡ€Π΅ΠΏΠ»Π΅Π½Π° вСдущая ΡˆΠ΅ΡΡ‚Π΅Ρ€Π½Ρ, которая находится Π² постоянном Π·Π°Ρ†Π΅ΠΏΠ»Π΅Π½ΠΈΠΈ с Π²Π΅Π΄ΠΎΠΌΠΎΠΉ ΡˆΠ΅ΡΡ‚Π΅Ρ€Π½Π΅ΠΉ, Π·Π°ΠΊΡ€Π΅ΠΏΠ»Π΅Π½Π½ΠΎΠΉ Π½Π° оси колСсной ΠΏΠ°Ρ€Ρ‹. ВСдомая ΡˆΠ΅ΡΡ‚Π΅Ρ€Π½Ρ вращаСтся ΠΈ Π²Ρ€Π°Ρ‰Π°Π΅Ρ‚ ось колСсной ΠΏΠ°Ρ€Ρ‹ ΠΈ колСса элСктровоза Π½Π°Ρ‡ΠΈΠ½Π°ΡŽΡ‚ Π²Ρ€Π°Ρ‰Π°Ρ‚ΡŒΡΡ.
Благодаря Π½Π°Π»ΠΈΡ‡ΠΈΡŽΒ ΡΠΈΠ»Β Ρ‚Ρ€Π΅Π½ΠΈΡ, мСТду колСсами ΠΈΒ Ρ€Π΅Π»ΡŒΡΠ°ΠΌΠΈ Π²ΠΎΠ·Π½ΠΈΠΊΠ°Π΅Ρ‚Β ΠΊΠ°ΡΠ°Ρ‚Π΅Π»ΡŒΠ½Π°ΡΒ  сила тяги:
FΠΊΒ = NΠ΄Β  * Fkg = NΠ΄Β (3,6 * Π‘Β * Π€Β * Ig β€” ”F), Н
Π³Π΄Π΅:
Fkg β€” ΠΊΠ°ΡΠ°Ρ‚Π΅Π»ΡŒΠ½Π°ΡΒ  сила тяги одного тягового  элСктродвигатСля, Н
NΠ΄Β β€” число двиТущихся осСй или  тяговых элСктродвигатСлСй локомотива
Π‘ β€” постоянная элСктроподвиТного состава, которая  зависит от ΠΏΠ΅Ρ€Π΅Π΄Π°Ρ‚ΠΎΡ‡Π½ΠΎΠ³ΠΎ ΠΎΡ‚Π½ΠΎΡˆΠ΅Π½ΠΈΡ Π·ΡƒΠ±Ρ‡Π°Ρ‚ΠΎΠΉ ΠΏΠ΅Ρ€Π΅Π΄Π°Ρ‡ΠΈ, Π΄ΠΈΠ°ΠΌΠ΅Ρ‚Ρ€Π° Π΄Π²ΠΈΠΆΡƒΡ‰ΠΈΡ… колСс Π»ΠΎΠΊΠΎΠΌΠΎΡ‚ΠΈΠ²Π°, конструктивной постоянной тягового элСктродвигатСля, Π²ΠΊΠ»ΡŽΡ‡Π°ΡŽΡ‰Π°Ρ Π² сСбя число ΠΏΠ°Ρ€ полюсов, число ΠΏΠ°Ρ€Π°Π»Π»Π΅Π»ΡŒΠ½Ρ‹Ρ… Π²Π΅Ρ‚Π²Π΅ΠΉ ΠΈ Π°ΠΊΡ‚ΠΈΠ²Π½Ρ‹Ρ… ΠΏΡ€ΠΎΠ²ΠΎΠ΄Π½ΠΈΠΊΠΎΠ² ΠΎΠ±ΠΌΠΎΡ‚ΠΊΠΈ якоря
Π€ β€” ΠΌΠ°Π³Π½ΠΈΡ‚Π½Ρ‹ΠΉΒ  поток тягового элСктродвигатСля, Π’Π±
Ig β€” ΠΏΠ΅Ρ€Π΅ΠΌΠ΅Π½Π½Ρ‹ΠΉΒ  ток тягового элСктродвигатСля, А
”F β€” сила, Π²ΠΎΠ·Π½ΠΈΠΊΠ°ΡŽΡ‰Π°ΡΒ ΠΈΠ·-Π·Π° мСханичСских ΠΈΒ  ΠΌΠ°Π³Π½ΠΈΡ‚Π½Ρ‹Ρ…Β ΠΏΠΎΡ‚Π΅Ρ€ΡŒ в тяговом элСктродвигатСлС ΠΈΒ ΠΏΠΎΡ‚Π΅Ρ€ΡŒ Π²Β Π·ΡƒΠ±Ρ‡Π°Ρ‚ΠΎΠΉ ΠΏΠ΅Ρ€Π΅Π΄Π°Ρ‡Π΅.
Благодаря Π½Π°Π»ΠΈΡ‡ΠΈΡŽΒ ΠΊΠ°ΡΠ°Ρ‚Π΅Π»ΡŒΠ½ΠΎΠΉΒ ΡΠΈΠ»Ρ‹ тяги элСктровоз двиТСтся Π²ΠΏΠ΅Ρ€Π΅Π΄. Π‘ΠΊΠΎΡ€ΠΎΡΡ‚ΡŒ двиТСния элСктровоза рСгулируСтся ΠΌΠ°ΡˆΠΈΠ½ΠΈΡΡ‚ΠΎΠΌ с ΠΏΠΎΠΌΠΎΡ‰ΡŒΡŽ ΠΊΠΎΠ½Ρ‚Ρ€ΠΎΠ»Π»Π΅Ρ€Π°, ΠΊΠΎΡ‚ΠΎΡ€Ρ‹ΠΉ располоТСн Π² ΠΊΠ°Π±ΠΈΠ½Π΅ ΠΌΠ°ΡˆΠΈΠ½ΠΈΡΡ‚Π°. ΠšΠΎΠ½Ρ‚Ρ€ΠΎΠ»Π»Π΅Ρ€ ΠΏΡ€ΠΈ ΠΈΠ·ΠΌΠ΅Π½Π΅Π½ΠΈΠΈ ΠΌΠ°ΡˆΠΈΠ½ΠΈΡΡ‚ΠΎΠΌ полоТСния Π΅Π³ΠΎ Ρ€ΡƒΡ‡ΠΊΠΈ измСняСт Π²Π΅Π»ΠΈΡ‡ΠΈΠ½Ρƒ сопротивлСния пусковых рСостатов. Π§Π΅ΠΌ мСньшС ΠΈΡ… сопротивлСниС, Ρ‚Π΅ΠΌ большС Π²Π΅Π»ΠΈΡ‡ΠΈΠ½Π° Ρ‚ΠΎΠΊΠ° Ig (ΠΏΠΎ Π·Π°ΠΊΠΎΠ½Ρƒ Ома), ΠΏΡ€ΠΎΡ‚Π΅ΠΊΠ°ΡŽΡ‰Π΅Π³ΠΎ ΠΏΠΎ тяговым элСктродвигатСлям, Ρ‚Π΅ΠΌ большС частота вращСния якорСй тягового элСктродвигатСля ΠΈ Ρ‚Π΅ΠΌ большС ΡΠΊΠΎΡ€ΠΎΡΡ‚ΡŒ элСктровоза.
НаправлСниС двиТСния ΠΌΠ°ΡˆΠΈΠ½ΠΈΡΡ‚ измСняСт ΡΒ ΠΏΠΎΠΌΠΎΡ‰ΡŒΡŽΒ  ΡΠΏΠ΅Ρ†ΠΈΠ°Π»ΡŒΠ½ΠΎΠ³ΠΎ ΠΏΠ΅Ρ€Π΅ΠΊΠ»ΡŽΡ‡Π°Ρ‚Π΅Π»Ρ, ΠΊΠΎΡ‚ΠΎΡ€Ρ‹ΠΉΒ  измСняСт ΠΏΠΎΠ»ΡΡ€Π½ΠΎΡΡ‚ΡŒ Ρ‚ΠΎΠΊΠ° ΠΎΠ΄Π½ΠΎΠ²Ρ€Π΅ΠΌΠ΅Π½Π½ΠΎ Ρƒ всСх элСктродвигатСлСй ΠΎΠ΄Π½ΠΎΠΉ ΠΈΠ· Π΄Π²ΡƒΡ… ΠΎΠ±ΠΌΠΎΡ‚ΠΎΠΊ Π½Π° ΠΎΠ±Ρ€Π°Ρ‚Π½ΠΎΠ΅ ΠΈ якоря Π½Π°Ρ‡ΠΈΠ½Π°ΡŽΡ‚ Π²Ρ€Π°Ρ‰Π°Ρ‚ΡŒΡΡ Π² ΠΎΠ±Ρ€Π°Ρ‚Π½ΡƒΡŽ сторону ΠΈ элСктровоз двиТСтся Π½Π°Π·Π°Π΄.

Π—Π°ΠΊΠ»ΡŽΡ‡Π΅Π½ΠΈΠ΅Β 

ЦСль Π΄Π°Π½Π½ΠΎΠΉ Ρ€Π°Π±ΠΎΡ‚Ρ‹ β€” ΠΈΠ·ΡƒΡ‡Π΅Π½ΠΈΠ΅ примСнСния Π·Π°ΠΊΠΎΠ½Π° Ома Π² Ρ€Π°Π±ΠΎΡ‚Π΅ элСктричСской Ρ†Π΅ΠΏΠΈ элСктровоза. Π’ процСссС Ρ€Π°Π±ΠΎΡ‚Ρ‹ я ΠΈΠ·ΡƒΡ‡ΠΈΠ»Β  источники ΠΈΠ½Ρ„ΠΎΡ€ΠΌΠ°Ρ†ΠΈΠΈ ΠΏΠΎ Π΄Π°Π½Π½ΠΎΠΉ Ρ‚Π΅ΠΌΠ΅, ΠΏΡ€ΠΎΠ°Π½Π°Π»ΠΈΠ·ΠΈΡ€ΠΎΠ²Π°Π» Π·Π°ΠΊΠΎΠ½ Ома для участка Ρ†Π΅ΠΏΠΈ, для ΠΏΠΎΠ»Π½ΠΎΠΉ Ρ†Π΅ΠΏΠΈ. На основС ΠΈΠ·ΡƒΡ‡Π΅Π½Π½Ρ‹Ρ… ΠΌΠ΅Ρ‚ΠΎΠ΄ΠΈΠΊ рассмотрСл ΠΏΡ€ΠΈΠΌΠ΅Π½Π΅Π½ΠΈΠ΅ Π·Π°ΠΊΠΎΠ½Π° Ома Π² Ρ€Π°Π±ΠΎΡ‚Π΅ элСктричСской Ρ†Π΅ΠΏΠΈ элСктровоза.
Π’ дальнСйшСм я ΠΏΠ»Π°Π½ΠΈΡ€ΡƒΡŽ, Ρ€Π°ΡΡΡ‡ΠΈΡ‚Π°Ρ‚ΡŒ ΠΏΠ°Ρ€Π°ΠΌΠ΅Ρ‚Ρ€Ρ‹ элСктричСской Ρ†Π΅ΠΏΠΈ элСктровоза.

Ρ€Π°Π·Π½ΠΈΡ†Π° с Π²Ρ‹Ρ€Π°ΠΆΠ΅Π½ΠΈΠ΅ΠΌ для участка ΠΊΠΎΠ½Ρ‚ΡƒΡ€Π°, ΠΎΠΏΡ€Π΅Π΄Π΅Π»Π΅Π½ΠΈΠ΅, Ρ„ΠΎΡ€ΠΌΡƒΠ»Π°

Π‘Ρ€Π΅Π΄ΠΈ извСстных ΡˆΠΈΡ€ΠΎΠΊΠΎΠΉ общСствСнности физичСских Ρ„ΠΎΡ€ΠΌΡƒΠ» Π»ΠΈΠ΄ΠΈΡ€ΡƒΠ΅Ρ‚ E=mc2. По популярности с Π½Π΅ΠΉ ΠΌΠΎΠΆΠ΅Ρ‚ ΡΠΎΠΏΠ΅Ρ€Π½ΠΈΡ‡Π°Ρ‚ΡŒ Ρ‚ΠΎΠ»ΡŒΠΊΠΎ U=IR. Π­Ρ‚ΠΎ простоС Π²Ρ‹Ρ€Π°ΠΆΠ΅Π½ΠΈΠ΅ ΠΈΠΌΠ΅Π΅Ρ‚ Ρ„ΡƒΠ½Π΄Π°ΠΌΠ΅Π½Ρ‚Π°Π»ΡŒΠ½ΠΎΠ΅ Π·Π½Π°Ρ‡Π΅Π½ΠΈΠ΅ для элСктротСхники ΠΈ описываСт матСматичСски ΡΠΎΠΎΡ‚Π½ΠΎΡˆΠ΅Π½ΠΈΠ΅ ΠΌΠ΅ΠΆΠ΄Ρƒ ΠΏΠ°Ρ€Π°ΠΌΠ΅Ρ‚Ρ€Π°ΠΌΠΈ участка элСктричСской Ρ†Π΅ΠΏΠΈ. МСнСС извСстСн Π·Π°ΠΊΠΎΠ½ Ома для ΠΏΠΎΠ»Π½ΠΎΠΉ Ρ†Π΅ΠΏΠΈ, ΠΊΠΎΡ‚ΠΎΡ€Ρ‹ΠΉ рассматриваСт Π½Π°Π³Ρ€ΡƒΠ·ΠΊΡƒ Π½Π΅ΠΎΡ‚Π΄Π΅Π»ΠΈΠΌΠΎ ΠΎΡ‚ источника напряТСния.

ΠžΡΠ½ΠΎΠ²Π½Ρ‹Π΅ понятия

ЭлСктричСский Ρ‚ΠΎΠΊ Ρ‚Π΅Ρ‡Ρ‘Ρ‚, ΠΊΠΎΠ³Π΄Π° Π·Π°ΠΌΠΊΠ½ΡƒΡ‚Ρ‹ΠΉ ΠΊΠΎΠ½Ρ‚ΡƒΡ€ позволяСт элСктронам ΠΏΠ΅Ρ€Π΅ΠΌΠ΅Ρ‰Π°Ρ‚ΡŒΡΡ ΠΎΡ‚ высокого ΠΏΠΎΡ‚Π΅Π½Ρ†ΠΈΠ°Π»Π° ΠΊ Π±ΠΎΠ»Π΅Π΅ Π½ΠΈΠ·ΠΊΠΎΠΌΡƒ Π² Ρ†Π΅ΠΏΠΈ. Π˜Π½Π°Ρ‡Π΅ говоря, Ρ‚ΠΎΠΊ Ρ‚Ρ€Π΅Π±ΡƒΠ΅Ρ‚ источника элСктронов, ΠΎΠ±Π»Π°Π΄Π°ΡŽΡ‰Π΅Π³ΠΎ энСргиСй для привСдСния ΠΈΡ… Π² Π΄Π²ΠΈΠΆΠ΅Π½ΠΈΠ΅, Π° Ρ‚Π°ΠΊΠΆΠ΅ Ρ‚ΠΎΡ‡ΠΊΠΈ ΠΈΡ… возвращСния ΠΎΡ‚Ρ€ΠΈΡ†Π°Ρ‚Π΅Π»ΡŒΠ½Ρ‹Ρ… зарядов, для ΠΊΠΎΡ‚ΠΎΡ€ΠΎΠΉ Ρ…Π°Ρ€Π°ΠΊΡ‚Π΅Ρ€Π΅Π½ ΠΈΡ… Π΄Π΅Ρ„ΠΈΡ†ΠΈΡ‚. Как физичСскоС явлСниС Ρ‚ΠΎΠΊ Π² Ρ†Π΅ΠΏΠΈ характСризуСтся трСмя Ρ„ΡƒΠ½Π΄Π°ΠΌΠ΅Π½Ρ‚Π°Π»ΡŒΠ½Ρ‹ΠΌΠΈ Π²Π΅Π»ΠΈΡ‡ΠΈΠ½Π°ΠΌΠΈ:

  • напряТСниС;
  • сила Ρ‚ΠΎΠΊΠ°;
  • сопротивлСниС ΠΏΡ€ΠΎΠ²ΠΎΠ΄Π½ΠΈΠΊΠ°, ΠΏΠΎ ΠΊΠΎΡ‚ΠΎΡ€ΠΎΠΌΡƒ двиТутся элСктроны.

Π‘ΠΈΠ»Π° ΠΈ напряТСниС

Π‘ΠΈΠ»Π° Ρ‚ΠΎΠΊΠ° (I, измСряСтся Π² АмпСрах) Π΅ΡΡ‚ΡŒ ΠΎΠ±ΡŠΡ‘ΠΌ элСктронов (заряд), ΠΏΠ΅Ρ€Π΅ΠΌΠ΅Ρ‰Π°ΡŽΡ‰ΠΈΡ…ΡΡ Ρ‡Π΅Ρ€Π΅Π· мСсто Π² Ρ†Π΅ΠΏΠΈ Π·Π° Π΅Π΄ΠΈΠ½ΠΈΡ†Ρƒ Π²Ρ€Π΅ΠΌΠ΅Π½ΠΈ. Π˜Π½Ρ‹ΠΌΠΈ словами, ΠΈΠ·ΠΌΠ΅Ρ€Π΅Π½ΠΈΠ΅ I β€” это ΠΎΠΏΡ€Π΅Π΄Π΅Π»Π΅Π½ΠΈΠ΅ количСства элСктронов, находящихся Π² Π΄Π²ΠΈΠΆΠ΅Π½ΠΈΠΈ. Π’Π°ΠΆΠ½ΠΎ ΠΏΠΎΠ½ΠΈΠΌΠ°Ρ‚ΡŒ, Ρ‡Ρ‚ΠΎ Ρ‚Π΅Ρ€ΠΌΠΈΠ½ относится Ρ‚ΠΎΠ»ΡŒΠΊΠΎ ΠΊ двиТСнию: статичСскиС заряды, Π½Π°ΠΏΡ€ΠΈΠΌΠ΅Ρ€, Π½Π° ΠΊΠ»Π΅ΠΌΠΌΠ°Ρ… нСподсоСдинённой Π±Π°Ρ‚Π°Ρ€Π΅ΠΈ, Π½Π΅ ΠΈΠΌΠ΅ΡŽΡ‚ измСряСмого значСния I. Π’ΠΎΠΊ, ΠΊΠΎΡ‚ΠΎΡ€Ρ‹ΠΉ ΠΏΡ€ΠΎΡ‚Π΅ΠΊΠ°Π΅Ρ‚ Π² ΠΎΠ΄Π½ΠΎΠΌ Π½Π°ΠΏΡ€Π°Π²Π»Π΅Π½ΠΈΠΈ, называСтся постоянным (DC), Π° пСриодичСски ΠΈΠ·ΠΌΠ΅Π½ΡΡŽΡ‰ΠΈΠΉ Π½Π°ΠΏΡ€Π°Π²Π»Π΅Π½ΠΈΠ΅ β€” ΠΏΠ΅Ρ€Π΅ΠΌΠ΅Π½Π½Ρ‹ΠΌ (AC).

Π’ΠΎΠ»ΡŒΡ‚ β€” Π΅Π΄ΠΈΠ½ΠΈΡ†Π° измСрСния, примСняСмая для элСктричСской Ρ€Π°Π·Π½ΠΈΡ†Ρ‹ ΠΏΠΎΡ‚Π΅Π½Ρ†ΠΈΠ°Π»ΠΎΠ², самого ΠΏΠΎΡ‚Π΅Π½Ρ†ΠΈΠ°Π»Π° ΠΈ элСктродвиТущСй силы. Π’Π΅Ρ€ΠΌΠΈΠ½ напряТСниС (U) относится ΠΊ элСктричСской разности ΠΏΠΎΡ‚Π΅Π½Ρ†ΠΈΠ°Π»ΠΎΠ² ΠΌΠ΅ΠΆΠ΄Ρƒ Ρ‚ΠΎΡ‡ΠΊΠ°ΠΌΠΈ. Π›ΡŽΠ±Ρ‹Π΅ статичСскиС заряды ΠΈΠΌΠ΅ΡŽΡ‚ Π·Π½Π°Ρ‡Π΅Π½ΠΈΠ΅ Π² Π’ΠΎΠ»ΡŒΡ‚Π°Ρ…, Π° Π²Π΅Π»ΠΈΡ‡ΠΈΠ½Π° ΠΈΡ… разности опрСдСляСтся ΠΊΠ°ΠΊ U.

НапряТСниС ΠΌΠΎΠΆΠ½ΠΎ ΠΏΡ€ΠΎΠΈΠ»Π»ΡŽΡΡ‚Ρ€ΠΈΡ€ΠΎΠ²Π°Ρ‚ΡŒ Ρ‚Π°ΠΊΠΈΠΌ явлСниСм, ΠΊΠ°ΠΊ Π΄Π°Π²Π»Π΅Π½ΠΈΠ΅, ΠΈΠ»ΠΈ ΠΊΠ°ΠΊ Ρ€Π°Π·Π½ΠΎΡΡ‚ΡŒ ΠΏΠΎΡ‚Π΅Π½Ρ†ΠΈΠ°Π»ΡŒΠ½ΠΎΠΉ энСргии ΠΏΡ€Π΅Π΄ΠΌΠ΅Ρ‚ΠΎΠ² ΠΏΠΎΠ΄ воздСйствиСм Π³Ρ€Π°Π²ΠΈΡ‚Π°Ρ†ΠΈΠΈ. Для Ρ‚ΠΎΠ³ΠΎ Ρ‡Ρ‚ΠΎΠ±Ρ‹ ΡΠΎΠ·Π΄Π°Ρ‚ΡŒ этот дисбаланс, Π½ΡƒΠΆΠ½ΠΎ Π·Π°Ρ‚Ρ€Π°Ρ‚ΠΈΡ‚ΡŒ ΠΏΡ€Π΅Π΄Π²Π°Ρ€ΠΈΡ‚Π΅Π»ΡŒΠ½ΠΎ ΡΠ½Π΅Ρ€Π³ΠΈΡŽ, которая ΠΈ Π±ΡƒΠ΄Π΅Ρ‚ Ρ€Π΅Π°Π»ΠΈΠ·ΠΎΠ²Π°Π½Π° Π² Π΄Π²ΠΈΠΆΠ΅Π½ΠΈΠΈ ΠΏΡ€ΠΈ ΡΠΎΠΎΡ‚Π²Π΅Ρ‚ΡΡ‚Π²ΡƒΡŽΡ‰ΠΈΡ… ΠΎΠ±ΡΡ‚ΠΎΡΡ‚Π΅Π»ΡŒΡΡ‚Π²Π°Ρ…. НапримСр, Π² ΠΏΠ°Π΄Π΅Π½ΠΈΠΈ Π³Ρ€ΡƒΠ·Π° с высоты рСализуСтся Ρ€Π°Π±ΠΎΡ‚Π° ΠΏΠΎ Π΅Π³ΠΎ ΠΏΠΎΠ΄ΡŠΡ‘ΠΌΡƒ, Π² Π³Π°Π»ΡŒΠ²Π°Π½ΠΈΡ‡Π΅ΡΠΊΠΈΡ… батарСях Ρ€Π°Π·Π½ΠΎΡΡ‚ΡŒ ΠΏΠΎΡ‚Π΅Π½Ρ†ΠΈΠ°Π»ΠΎΠ² Π½Π° ΠΊΠ»Π΅ΠΌΠΌΠ°Ρ… образуСтся Π·Π° счёт прСобразования химичСской энСргии, Π² Π³Π΅Π½Π΅Ρ€Π°Ρ‚ΠΎΡ€Π°Ρ… β€” Π² Ρ€Π΅Π·ΡƒΠ»ΡŒΡ‚Π°Ρ‚Π΅ воздСйствия элСктромагнитного поля.

Π‘ΠΎΠΏΡ€ΠΎΡ‚ΠΈΠ²Π»Π΅Π½ΠΈΠ΅ ΠΏΡ€ΠΎΠ²ΠΎΠ΄Π½ΠΈΠΊΠΎΠ²

НСзависимо ΠΎΡ‚ Ρ‚ΠΎΠ³ΠΎ, насколько Ρ…ΠΎΡ€ΠΎΡˆ ΠΎΠ±Ρ‹Ρ‡Π½Ρ‹ΠΉ ΠΏΡ€ΠΎΠ²ΠΎΠ΄Π½ΠΈΠΊ, ΠΎΠ½ Π½ΠΈΠΊΠΎΠ³Π΄Π° Π½Π΅ Π±ΡƒΠ΄Π΅Ρ‚ ΠΏΡ€ΠΎΠΏΡƒΡΠΊΠ°Ρ‚ΡŒ сквозь сСбя элСктроны Π±Π΅Π· ΠΊΠ°ΠΊΠΎΠ³ΠΎ-Π»ΠΈΠ±ΠΎ сопротивлСния ΠΈΡ… двиТСнию. МоТно Ρ€Π°ΡΡΠΌΠ°Ρ‚Ρ€ΠΈΠ²Π°Ρ‚ΡŒ сопротивлСниС ΠΊΠ°ΠΊ Π°Π½Π°Π»ΠΎΠ³ мСханичСского трСния, хотя это сравнСниС Π½Π΅ Π±ΡƒΠ΄Π΅Ρ‚ ΡΠΎΠ²Π΅Ρ€ΡˆΠ΅Π½Π½Ρ‹ΠΌ. Когда Ρ‚ΠΎΠΊ ΠΏΡ€ΠΎΡ‚Π΅ΠΊΠ°Π΅Ρ‚ Ρ‡Π΅Ρ€Π΅Π· ΠΏΡ€ΠΎΠ²ΠΎΠ΄Π½ΠΈΠΊ, нСкоторая Ρ€Π°Π·Π½ΠΎΡΡ‚ΡŒ ΠΏΠΎΡ‚Π΅Π½Ρ†ΠΈΠ°Π»ΠΎΠ² прСобразуСтся Π² Ρ‚Π΅ΠΏΠ»ΠΎ, поэтому всСгда Π±ΡƒΠ΄Π΅Ρ‚ ΠΏΠ°Π΄Π΅Π½ΠΈΠ΅ напряТСния Π½Π° рСзисторС. ЭлСктричСскиС ΠΎΠ±ΠΎΠ³Ρ€Π΅Π²Π°Ρ‚Π΅Π»ΠΈ, Ρ„Π΅Π½Ρ‹ ΠΈ Π΄Ρ€ΡƒΠ³ΠΈΠ΅ ΠΏΠΎΠ΄ΠΎΠ±Π½Ρ‹Π΅ устройства ΠΏΡ€Π΅Π΄Π½Π°Π·Π½Π°Ρ‡Π΅Π½Ρ‹ ΠΈΡΠΊΠ»ΡŽΡ‡ΠΈΡ‚Π΅Π»ΡŒΠ½ΠΎ для рассСивания элСктричСской энСргии Π² Π²ΠΈΠ΄Π΅ Ρ‚Π΅ΠΏΠ»Π°.

Π£ΠΏΡ€ΠΎΡ‰Ρ‘Π½Π½ΠΎ сопротивлСниС (обозначаСтся ΠΊΠ°ΠΊ R) являСтся ΠΌΠ΅Ρ€ΠΎΠΉ Ρ‚ΠΎΠ³ΠΎ, насколько ΠΏΠΎΡ‚ΠΎΠΊ элСктронов тормозится Π² Ρ†Π΅ΠΏΠΈ. Оно измСряСтся Π² ΠžΠΌΠ°Ρ…. ΠŸΡ€ΠΎΠ²ΠΎΠ΄ΠΈΠΌΠΎΡΡ‚ΡŒ рСзистора ΠΈΠ»ΠΈ Π΄Ρ€ΡƒΠ³ΠΎΠ³ΠΎ элСмСнта опрСдСляСтся двумя свойствами:

  • Π³Π΅ΠΎΠΌΠ΅Ρ‚Ρ€ΠΈΠ΅ΠΉ;
  • ΠΌΠ°Ρ‚Π΅Ρ€ΠΈΠ°Π»ΠΎΠΌ.

Π€ΠΎΡ€ΠΌΠ° ΠΈΠΌΠ΅Π΅Ρ‚ ваТнСйшСС Π·Π½Π°Ρ‡Π΅Π½ΠΈΠ΅, это ΠΎΡ‡Π΅Π²ΠΈΠ΄Π½ΠΎ Π½Π° гидравличСской Π°Π½Π°Π»ΠΎΠ³ΠΈΠΈ: ΠΏΡ€ΠΎΡ‚ΠΎΠ»ΠΊΠ½ΡƒΡ‚ΡŒ Π²ΠΎΠ΄Ρƒ Ρ‡Π΅Ρ€Π΅Π· Π΄Π»ΠΈΠ½Π½ΡƒΡŽ ΠΈ ΡƒΠ·ΠΊΡƒΡŽ Ρ‚Ρ€ΡƒΠ±Ρƒ Π³ΠΎΡ€Π°Π·Π΄ΠΎ тяТСлСС, Ρ‡Π΅ΠΌ Ρ‡Π΅Ρ€Π΅Π· ΠΊΠΎΡ€ΠΎΡ‚ΠΊΡƒΡŽ ΠΈ ΡˆΠΈΡ€ΠΎΠΊΡƒΡŽ. ΠœΠ°Ρ‚Π΅Ρ€ΠΈΠ°Π»Ρ‹ ΠΈΠ³Ρ€Π°ΡŽΡ‚ ΠΎΠΏΡ€Π΅Π΄Π΅Π»ΡΡŽΡ‰ΡƒΡŽ Ρ€ΠΎΠ»ΡŒ. НапримСр, элСктроны ΠΌΠΎΠ³ΡƒΡ‚ свободно ΠΏΠ΅Ρ€Π΅ΠΌΠ΅Ρ‰Π°Ρ‚ΡŒΡΡ Π² ΠΌΠ΅Π΄Π½ΠΎΠΌ ΠΏΡ€ΠΎΠ²ΠΎΠ΄Π΅, Π½ΠΎ Π½Π΅ способны ΠΏΡ€ΠΎΡ‚Π΅ΠΊΠ°Ρ‚ΡŒ Π²ΠΎΠΎΠ±Ρ‰Π΅ Ρ‡Π΅Ρ€Π΅Π· Ρ‚Π°ΠΊΠΈΠ΅ изоляторы, ΠΊΠ°ΠΊ ΠΊΠ°ΡƒΡ‡ΡƒΠΊ, нСзависимо ΠΎΡ‚ ΠΈΡ… Ρ„ΠΎΡ€ΠΌΡ‹. ΠšΡ€ΠΎΠΌΠ΅ Π³Π΅ΠΎΠΌΠ΅Ρ‚Ρ€ΠΈΠΈ ΠΈ ΠΌΠ°Ρ‚Π΅Ρ€ΠΈΠ°Π»Π°, ΡΡƒΡ‰Π΅ΡΡ‚Π²ΡƒΡŽΡ‚ ΠΈ Π΄Ρ€ΡƒΠ³ΠΈΠ΅ Ρ„Π°ΠΊΡ‚ΠΎΡ€Ρ‹, Π²Π»ΠΈΡΡŽΡ‰ΠΈΠ΅ Π½Π° ΠΏΡ€ΠΎΠ²ΠΎΠ΄ΠΈΠΌΠΎΡΡ‚ΡŒ.

Π—Π°ΠΊΠΎΠ½ для участка Ρ†Π΅ΠΏΠΈ

БущСствуСт Ρ„ΡƒΠ½Π΄Π°ΠΌΠ΅Π½Ρ‚Π°Π»ΡŒΠ½Π°Ρ связь ΠΌΠ΅ΠΆΠ΄Ρƒ напряТСниСм, Ρ‚ΠΎΠΊΠΎΠΌ ΠΈ ΠΏΡ€ΠΎΠ²ΠΎΠ΄ΠΈΠΌΠΎΡΡ‚ΡŒΡŽ. Π­Ρ‚ΠΎ Π·Π½Π°ΠΌΠ΅Π½ΠΈΡ‚ΠΎΠ΅ ΡƒΡ€Π°Π²Π½Π΅Π½ΠΈΠ΅ называСтся Π·Π°ΠΊΠΎΠ½ΠΎΠΌ Ома, ΠΈ Π΅Π³ΠΎ ΠΌΠΎΠΆΠ½ΠΎ ΠΎΡ‚ΠΎΠ±Ρ€Π°Π·ΠΈΡ‚ΡŒ трСмя эквивалСнтными способами:

Π’Ρ‹Ρ€Π°ΠΆΠ΅Π½Π½Ρ‹ΠΉ Π² словах ΠΎΠ½ Π·Π²ΡƒΡ‡ΠΈΡ‚ Ρ‚Π°ΠΊ: Ρ‚ΠΎΠΊ, ΠΏΡ€ΠΎΡ‚Π΅ΠΊΠ°ΡŽΡ‰ΠΈΠΉ Ρ‡Π΅Ρ€Π΅Π· ΠΏΡ€ΠΎΠ²ΠΎΠ΄Π½ΠΈΠΊ ΠΌΠ΅ΠΆΠ΄Ρƒ двумя ΠΊΠΎΠ½Ρ‚Π°ΠΊΡ‚Π°ΠΌΠΈ, прямо ΠΏΡ€ΠΎΠΏΠΎΡ€Ρ†ΠΈΠΎΠ½Π°Π»Π΅Π½ Π½Π°ΠΏΡ€ΡΠΆΠ΅Π½ΠΈΡŽ Π½Π° этих ΠΊΠΎΠ½Ρ‚Π°ΠΊΡ‚Π°Ρ…. ΠŸΠ΅Ρ€Π²Ρ‹Π΅ Π΄Π²Π° выраТСния Ρ„ΠΈΠΊΡΠΈΡ€ΡƒΡŽΡ‚ константу ΠΏΡ€ΠΎΠΏΠΎΡ€Ρ†ΠΈΠΎΠ½Π°Π»ΡŒΠ½ΠΎΡΡ‚ΠΈ ΠΌΠ΅ΠΆΠ΄Ρƒ Ρ‚ΠΎΠΊΠΎΠΌ ΠΈ напряТСниСм. ПослСднСС ΠΌΠΎΠΆΠ½ΠΎ Ρ€Π°ΡΡΠΌΠ°Ρ‚Ρ€ΠΈΠ²Π°Ρ‚ΡŒ ΠΊΠ°ΠΊ ΠΎΠΏΡ€Π΅Π΄Π΅Π»Π΅Π½ΠΈΠ΅ для Π΅Π΄ΠΈΠ½ΠΈΡ‡Π½ΠΎΠ³ΠΎ рСзистора β€” элСмСнта, ΠΏΠΎΠ·Π²ΠΎΠ»ΡΡŽΡ‰Π΅Π³ΠΎ ΠΏΡ€ΠΎΡ‚Π΅ΠΊΠ°Ρ‚ΡŒ Π΅Π΄ΠΈΠ½ΠΈΡ†Π΅ Ρ‚ΠΎΠΊΠ° ΠΏΠΎΠ΄ Π΅Π΄ΠΈΠ½ΠΈΡ‡Π½Ρ‹ΠΌ напряТСниСм.

ΠŸΡ€ΠΈΠ²Π΅Π΄Ρ‘Π½Π½Ρ‹Π΅ матСматичСскиС ΡΠΎΠΎΡ‚Π½ΠΎΡˆΠ΅Π½ΠΈΡ β€” основа для элСктротСхники ΠΈ элСктроники. Π—Π°ΠΊΠΎΠ½ Π±Ρ‹Π» Π½Π°Π·Π²Π°Π½ Π² Ρ‡Π΅ΡΡ‚ΡŒ Π½Π΅ΠΌΠ΅Ρ†ΠΊΠΎΠ³ΠΎ Ρ„ΠΈΠ·ΠΈΠΊΠ° Π“Π΅ΠΎΡ€Π³Π° Π‘ΠΈΠΌΠΎΠ½Π° Ома, ΠΊΠΎΡ‚ΠΎΡ€Ρ‹ΠΉ Π² ΠΌΠΎΠ½ΠΎΠ³Ρ€Π°Ρ„ΠΈΠΈ, ΠΎΠΏΡƒΠ±Π»ΠΈΠΊΠΎΠ²Π°Π½Π½ΠΎΠΉ Π² 1827 Π³., описал измСрСния ΠΏΡ€ΠΈΠ»ΠΎΠΆΠ΅Π½Π½ΠΎΠ³ΠΎ напряТСния ΠΈ Ρ‚ΠΎΠΊΠ° с ΠΏΠΎΠΌΠΎΡ‰ΡŒΡŽ простых элСктричСских Ρ†Π΅ΠΏΠ΅ΠΉ, состоящих ΠΈΠ· ΠΏΡ€ΠΎΠ²ΠΎΠ΄ΠΎΠ² Ρ€Π°Π·Π»ΠΈΡ‡Π½ΠΎΠΉ Π΄Π»ΠΈΠ½Ρ‹.

Π˜ΡΡΠ»Π΅Π΄ΠΎΠ²Π°Ρ‚Π΅Π»ΡŒ объяснил свои ΡΠΊΡΠΏΠ΅Ρ€ΠΈΠΌΠ΅Π½Ρ‚Π°Π»ΡŒΠ½Ρ‹Π΅ Ρ€Π΅Π·ΡƒΠ»ΡŒΡ‚Π°Ρ‚Ρ‹ нСсколько слоТнСС, Ρ‡Π΅ΠΌ ΠΎΡ‚Ρ€Π°ΠΆΠ΅Π½ΠΎ Π² ΠΏΡ€ΠΈΠ²Π΅Π΄Ρ‘Π½Π½Ρ‹Ρ… уравнСниях, извСстных Π² соврСмСнной Ρ„ΠΈΠ·ΠΈΠΊΠ΅ ΠΊΠ°ΠΊ Π½Π΅ΠΏΠΎΠ»Π½Ρ‹ΠΉ Π·Π°ΠΊΠΎΠ½ Ома. Для Ρ‚ΠΎΠ³ΠΎ Ρ‡Ρ‚ΠΎΠ±Ρ‹ ΡΡ„ΠΎΡ€ΠΌΡƒΠ»ΠΈΡ€ΠΎΠ²Π°Ρ‚ΡŒ Π·Π°ΠΊΠΎΠ½ Ома для ΠΏΠΎΠ»Π½ΠΎΠΉ элСктричСской Ρ†Π΅ΠΏΠΈ, Π½Π΅ΠΎΠ±Ρ…ΠΎΠ΄ΠΈΠΌΠΎ ΠΎΠΏΠ΅Ρ€ΠΈΡ€ΠΎΠ²Π°Ρ‚ΡŒ понятиями Π²Π½ΡƒΡ‚Ρ€Π΅Π½Π½Π΅Π³ΠΎ сопротивлСния источника Ρ‚ΠΎΠΊΠ° ΠΈ элСктродвиТущСй силы.

ЭлСктродвиТущая сила

ΠŸΠ΅Ρ€Π΅ΠΌΠ΅Ρ‰Π΅Π½ΠΈΠ΅ элСктронов Π² любом источникС создаётся с ΠΏΠΎΠΌΠΎΡ‰ΡŒΡŽ сторонних сил. Π˜Ρ… ΠΏΡ€ΠΈΡ€ΠΎΠ΄Π° ΠΌΠΎΠΆΠ΅Ρ‚ Π±Ρ‹Ρ‚ΡŒ Ρ€Π°Π·Π»ΠΈΡ‡Π½ΠΎΠΉ. Π’ Π³Π°Π»ΡŒΠ²Π°Π½ΠΈΡ‡Π΅ΡΠΊΠΈΡ… элСмСнтах ΠΈΠ»ΠΈ аккумуляторах ΠΎΠ½ΠΈ Π²ΠΎΠ·Π½ΠΈΠΊΠ°ΡŽΡ‚ Π² Ρ€Π΅Π·ΡƒΠ»ΡŒΡ‚Π°Ρ‚Π΅ элСктрохимичСских процСссов. Π’ Π³Π΅Π½Π΅Ρ€Π°Ρ‚ΠΎΡ€Π°Ρ… Ρ‚ΠΎΠΊΠ° ΠΎΠ½ΠΈ ΠΏΠΎΡΠ²Π»ΡΡŽΡ‚ΡΡ ΠΊΠ°ΠΊ Ρ€Π΅Π·ΡƒΠ»ΡŒΡ‚Π°Ρ‚ двиТСния ΠΏΡ€ΠΎΠ²ΠΎΠ΄Π½ΠΈΠΊΠΎΠ² Π² ΠΌΠ°Π³Π½ΠΈΡ‚Π½ΠΎΠΌ ΠΏΠΎΠ»Π΅. Π˜ΡΡ‚ΠΎΡ‡Π½ΠΈΠΊ Ρ‚ΠΎΠΊΠ° Π² элСктричСской схСмС ΠΈΠ³Ρ€Π°Π΅Ρ‚ Ρ‚Ρƒ ΠΆΠ΅ Ρ€ΠΎΠ»ΡŒ, Ρ‡Ρ‚ΠΎ ΠΈ насос, ΠΏΠ΅Ρ€Π΅ΠΊΠ°Ρ‡ΠΈΠ²Π°ΡŽΡ‰ΠΈΠΉ ΠΆΠΈΠ΄ΠΊΠΎΡΡ‚ΡŒ Π² Π·Π°ΠΌΠΊΠ½ΡƒΡ‚ΠΎΠΉ гидравличСской систСмС.

Под воздСйствиСм Π²Π½Π΅ΡˆΠ½ΠΈΡ… сил заряды Π΄Π²ΠΈΠ³Π°ΡŽΡ‚ΡΡ Π²Π½ΡƒΡ‚Ρ€ΠΈ источника Ρ‚ΠΎΠΊΠ° ΠΏΡ€ΠΎΡ‚ΠΈΠ² сил элСктростатичСского поля. Π­Ρ‚ΠΎ позволяСт ΠΏΠΎΠ΄Π΄Π΅Ρ€ΠΆΠΈΠ²Π°Ρ‚ΡŒ постоянный Ρ‚ΠΎΠΊ Π² Π·Π°ΠΌΠΊΠ½ΡƒΡ‚ΠΎΠΌ ΠΊΠΎΠ½Ρ‚ΡƒΡ€Π΅ Π΄ΠΎ Ρ‚Π΅Ρ… ΠΏΠΎΡ€, ΠΏΠΎΠΊΠ° Ρ€Π°Π±ΠΎΡ‚Π°ΡŽΡ‚ внСшниС силы. ЀизичСская Π²Π΅Π»ΠΈΡ‡ΠΈΠ½Π°, равная ΠΎΡ‚Π½ΠΎΡˆΠ΅Π½ΠΈΡŽ Π·Π°Ρ‚Ρ€Π°Ρ‡Π΅Π½Π½ΠΎΠΉ энСргии сторонних сил Π½Π° ΠΏΠ΅Ρ€Π΅ΠΌΠ΅Ρ‰Π΅Π½ΠΈΠ΅ заряда, называСтся элСктродвиТущСй силой источника Ρ‚ΠΎΠΊΠ°. Она ΠΌΠΎΠΆΠ΅Ρ‚ Π±Ρ‹Ρ‚ΡŒ прСдставлСна Ρ„ΠΎΡ€ΠΌΡƒΠ»ΠΎΠΉ β„° = A/q. Π’ этом Π²Ρ‹Ρ€Π°ΠΆΠ΅Π½ΠΈΠΈ:

  • β„° β€” Π­Π”Π‘ Π² Π²ΠΎΠ»ΡŒΡ‚Π°Ρ…;
  • A β€” Ρ€Π°Π±ΠΎΡ‚Π° Π² дТоулях;
  • q β€” заряд Π² ΠΊΡƒΠ»ΠΎΠ½Π°Ρ….

По Π°Π½Π°Π»ΠΎΠ³ΠΈΠΈ с Π·Π°ΠΌΠΊΠ½ΡƒΡ‚ΠΎΠΉ гидравличСской систСмой ΠΈ насосом, элСктричСскиС заряды ΠΏΡ€ΠΎΡ‚Π΅ΠΊΠ°ΡŽΡ‚ Π½Π΅ΠΏΡ€Π΅Ρ€Ρ‹Π²Π½ΠΎ ΠΏΠΎ всСму ΠΊΠΎΠ½Ρ‚ΡƒΡ€Ρƒ, ΠΈ привСсти ΠΈΡ… Π² Π΄Π²ΠΈΠΆΠ΅Π½ΠΈΠ΅ ΠΌΠΎΠ³ΡƒΡ‚ Ρ‚ΠΎΠ»ΡŒΠΊΠΎ внСшниС силы. Π­Ρ‚ΠΎ ΠΎΠ·Π½Π°Ρ‡Π°Π΅Ρ‚, Ρ‡Ρ‚ΠΎ Ρ€Π°Π±ΠΎΡ‚Ρƒ ΠΏΠΎ ΠΏΠ΅Ρ€Π΅ΠΌΠ΅Ρ‰Π΅Π½ΠΈΡŽ заряда Π»ΡŽΠ±Ρ‹ΠΌ источником ΠΌΠΎΠΆΠ½ΠΎ Ρ€Π°ΡΡΠΌΠ°Ρ‚Ρ€ΠΈΠ²Π°Ρ‚ΡŒ ΠΊΠ°ΠΊ Π­Π”Π‘ ΠΈ ΠΈΠ·ΠΌΠ΅Ρ€ΡΡ‚ΡŒ Π² Π²ΠΎΠ»ΡŒΡ‚Π°Ρ…. Π’Ρ‹Π²ΠΎΠ΄ ΠΎ ΠΌΠΎΠ΄Π΅Π»ΠΈ Ρ†Π΅ΠΏΠΈ с источником, Π² ΠΊΠΎΡ‚ΠΎΡ€ΠΎΠΉ ΠΏΡ€ΠΎΡ‚Π΅ΠΊΠ°Π΅Ρ‚ Ρ‚ΠΎΠΊ, ΠΊΠ°ΠΊ ΠΎ Π·Π°ΠΌΠΊΠ½ΡƒΡ‚ΠΎΠΌ ΠΊΠΎΠ½Ρ‚ΡƒΡ€Π΅ ΠΊΡ€Π°ΠΉΠ½Π΅ Π²Π°ΠΆΠ΅Π½ для понимания Π·Π°ΠΊΠΎΠ½Π° Ома для ΠΏΠΎΠ»Π½ΠΎΠ³ΠΎ участка Ρ†Π΅ΠΏΠΈ.

Π’Π½Π΅ΡˆΠ½Π΅Π΅ ΠΈ Π²Π½ΡƒΡ‚Ρ€Π΅Π½Π½Π΅Π΅ сопротивлСниС

ВсС Π±Π°Ρ‚Π°Ρ€Π΅ΠΈ ΠΈ Π³Π΅Π½Π΅Ρ€Π°Ρ‚ΠΎΡ€Ρ‹ ΠΎΠ±Π»Π°Π΄Π°ΡŽΡ‚ Π²Π½ΡƒΡ‚Ρ€Π΅Π½Π½ΠΈΠΌ сопротивлСниСм: элСктроды ΠΈ элСктролиты Π½Π΅Π°Π±ΡΠΎΠ»ΡŽΡ‚Π½Ρ‹Π΅ ΠΏΡ€ΠΎΠ²ΠΎΠ΄Π½ΠΈΠΊΠΈ, ΠΊΠ°ΠΊ ΠΈ ΠΏΡ€ΠΎΠ²ΠΎΠ΄Π° ΠΎΠ±ΠΌΠΎΡ‚ΠΎΠΊ элСктричСских машин. Оно ΠΌΠΎΠΆΠ΅Ρ‚ Π²Π°Ρ€ΡŒΠΈΡ€ΠΎΠ²Π°Ρ‚ΡŒΡΡ ΠΎΡ‚ тысячных Π΄ΠΎΠ»Π΅ΠΉ ΠΎΠΌΠ° Π΄ΠΎ Π½Π΅ΡΠΊΠΎΠ»ΡŒΠΊΠΈΡ… ΠΎΠΌ. Π­Ρ‚ΠΎΡ‚ физичСский ΠΏΠ°Ρ€Π°ΠΌΠ΅Ρ‚Ρ€ являСтся ΠΊΠ»ΡŽΡ‡Π΅Π²Ρ‹ΠΌ Π² Π·Π°ΠΊΠΎΠ½Π΅ Ома для всСй Ρ†Π΅ΠΏΠΈ. Π’ качСствС матСматичСских ΠΌΠΎΠ΄Π΅Π»Π΅ΠΉ для рассмотрСния ΠΈ ΠΈΠ»Π»ΡŽΡΡ‚Ρ€Π°Ρ†ΠΈΠΈ элСктричСских процСссов Ρ€Π°Π·Π»ΠΈΡ‡Π°ΡŽΡ‚:

  • Π˜Π΄Π΅Π°Π»ΡŒΠ½Ρ‹ΠΉ источник Ρ‚ΠΎΠΊΠ° (ИИВ). Π“Π΅Π½Π΅Ρ€ΠΈΡ€ΡƒΠ΅Ρ‚ элСктричСский Ρ‚ΠΎΠΊ, Π½Π΅ зависящий ΠΎΡ‚ ΠΈΠ·ΠΌΠ΅Π½Π΅Π½ΠΈΠΉ напряТСния. Π’Π½ΡƒΡ‚Ρ€Π΅Π½Π½Π΅Π΅ сопротивлСниС ИИВ бСсконСчно, напряТСниС ΠΏΠΎΠ»Π½ΠΎΡΡ‚ΡŒΡŽ опрСдСляСтся ΠΏΠΎΠ΄ΠΊΠ»ΡŽΡ‡Ρ‘Π½Π½ΠΎΠΉ схСмой. Ни ΠΎΠ΄ΠΈΠ½ физичСский источник Ρ‚ΠΎΠΊΠ° Π½Π΅ ΠΌΠΎΠΆΠ΅Ρ‚ Ρ€Π°Π±ΠΎΡ‚Π°Ρ‚ΡŒ Π² условиях Ρ€Π°Π·Ρ€Ρ‹Π²Π° Ρ†Π΅ΠΏΠΈ, поэтому ИИВ Π²ΠΎΠ·ΠΌΠΎΠΆΠ΅Π½ Ρ‚ΠΎΠ»ΡŒΠΊΠΎ Π² качСствС абстрактной ΠΌΠΎΠ΄Π΅Π»ΠΈ.
  • Π˜Π΄Π΅Π°Π»ΡŒΠ½Ρ‹ΠΉ источник напряТСния (ИИН). ΠŸΡ€Π΅Π΄ΡΡ‚Π°Π²Π»ΡΠ΅Ρ‚ собой устройство, ΠΏΠΎΠ΄Π΄Π΅Ρ€ΠΆΠΈΠ²Π°ΡŽΡ‰Π΅Π΅ постоянноС Π²Ρ‹Ρ…ΠΎΠ΄Π½ΠΎΠ΅ напряТСниС нСзависимо ΠΎΡ‚ Ρ‚ΠΎΠΊΠ°, ΠΏΡ€ΠΎΡ‚Π΅ΠΊΠ°ΡŽΡ‰Π΅Π³ΠΎ ΠΏΠΎ ΠΊΠΎΠ½Ρ‚ΡƒΡ€Ρƒ. ΠžΠ±Π»Π°Π΄Π°Π΅Ρ‚ Π½ΡƒΠ»Π΅Π²Ρ‹ΠΌ Π²Π½ΡƒΡ‚Ρ€Π΅Π½Π½ΠΈΠΌ сопротивлСниСм. ИИН ΡƒΠ΄ΠΎΠ±Π΅Π½ для модСлирования практичСских источников, ΠΊΠΎΡ‚ΠΎΡ€Ρ‹Π΅ ΠΌΠΎΠΆΠ½ΠΎ ΠΏΡ€Π΅Π΄ΡΡ‚Π°Π²ΠΈΡ‚ΡŒ ΠΊΠ°ΠΊ ИНН с ΠΏΠΎΠ΄ΠΊΠ»ΡŽΡ‡Ρ‘Π½Π½Ρ‹ΠΌ рСзистором.

Π’Π½ΡƒΡ‚Ρ€Π΅Π½Π½Π΅ сопротивлСниС источника элСктричСской энСргии являСтся Ρ„Π°ΠΊΡ‚ΠΎΡ€ΠΎΠΌ обСспСчСния максимальной мощности для ΠΏΠΎΠ΄ΠΊΠ»ΡŽΡ‡Ρ‘Π½Π½ΠΎΠΉ ΠΊ Π½Π΅ΠΌΡƒ Π½Π°Π³Ρ€ΡƒΠ·ΠΊΠΈ. НаиболСС эффСктивный пСрСнос энСргии происходит, ΠΊΠΎΠ³Π΄Π° внСшнСС сопротивлСниС Π·Π½Π°Ρ‡ΠΈΡ‚Π΅Π»ΡŒΠ½ΠΎ ΠΏΡ€Π΅Π²Ρ‹ΡˆΠ°Π΅Ρ‚ Π²Π½ΡƒΡ‚Ρ€Π΅Π½Π½Π΅Π΅ Ρƒ источника.

НапримСр, свинцово-кислотныС аккумуляторы автомобиля, благодаря Π½ΠΈΠ·ΠΊΠΎΠΌΡƒ Π²Π½ΡƒΡ‚Ρ€Π΅Π½Π½Π΅ΠΌΡƒ ΡΠΎΠΏΡ€ΠΎΡ‚ΠΈΠ²Π»Π΅Π½ΠΈΡŽ, способны ΡΠΎΠ·Π΄Π°Π²Π°Ρ‚ΡŒ ΠΎΡ‚Π½ΠΎΡΠΈΡ‚Π΅Π»ΡŒΠ½ΠΎ высокиС Ρ‚ΠΎΠΊΠΈ ΠΏΡ€ΠΈ ΡΡ€Π°Π²Π½ΠΈΡ‚Π΅Π»ΡŒΠ½ΠΎ Π½ΠΈΠ·ΠΊΠΎΠΌ напряТСнии. Однако, с Π΄Ρ€ΡƒΠ³ΠΎΠΉ стороны, Π²Ρ‹ΡΠΎΠΊΠΎΠ²ΠΎΠ»ΡŒΡ‚Π½Ρ‹Π΅ источники Π΄ΠΎΠ»ΠΆΠ½Ρ‹ ΠΈΠΌΠ΅Ρ‚ΡŒ высокоС Π²Π½ΡƒΡ‚Ρ€Π΅Π½Π½Π΅ сопротивлСниС, Ρ‡Ρ‚ΠΎΠ±Ρ‹ ΠΎΠ³Ρ€Π°Π½ΠΈΡ‡ΠΈΡ‚ΡŒ количСство Ρ‚ΠΎΠΊΠ°, ΠΏΡ€ΠΎΡ‚Π΅ΠΊΠ°ΡŽΡ‰Π΅Π³ΠΎ Π² Ρ€Π΅Π·ΡƒΠ»ΡŒΡ‚Π°Ρ‚Π΅ случайного ΠΊΠΎΡ€ΠΎΡ‚ΠΊΠΎΠ³ΠΎ замыкания.

ΠŸΠΎΠ»Π½Ρ‹ΠΉ Π·Π°ΠΊΠΎΠ½

Π’Ρ‹Ρ€Π°ΠΆΠ΅Π½ΠΈΠ΅ U=IR описываСт явлСния Π²ΠΎ Ρ„Ρ€Π°Π³ΠΌΠ΅Π½Ρ‚Π΅ элСктричСской Ρ†Π΅ΠΏΠΈ, Ρ‡Π΅Ρ€Π΅Π· ΠΊΠΎΡ‚ΠΎΡ€ΡƒΡŽ ΠΏΡ€ΠΎΡ‚Π΅ΠΊΠ°Π΅Ρ‚ Ρ‚ΠΎΠΊ. Π’ этом ΡƒΡ€Π°Π²Π½Π΅Π½ΠΈΠΈ Π½Π΅ принимаСтся Π²ΠΎ Π²Π½ΠΈΠΌΠ°Π½ΠΈΠ΅ Π½Π°Π»ΠΈΡ‡ΠΈΠ΅ источников. Если ΠΈΡΠΏΡ€Π°Π²ΠΈΡ‚ΡŒ Ρ‚Π°ΠΊΠΎΠ΅ ΡƒΠΏΡ€ΠΎΡ‰Π΅Π½ΠΈΠ΅, Ρ‚ΠΎ ΠΌΠΎΠΆΠ½ΠΎ ΠΏΠΎΠ»ΡƒΡ‡ΠΈΡ‚ΡŒ Ρ„ΠΎΡ€ΠΌΡƒΠ»Ρƒ Π·Π°ΠΊΠΎΠ½Π° Ома для ΠΏΠΎΠ»Π½ΠΎΠΉ Ρ†Π΅ΠΏΠΈ: β„° =I (R+r).

Π’ этом ΡƒΡ€Π°Π²Π½Π΅Π½ΠΈΠΈ прСдусмотрСно Π½Π°Π»ΠΈΡ‡ΠΈΠ΅ Π² ΠΊΠΎΠ½Ρ‚ΡƒΡ€Π΅ источника питания элСктродвиТущСй силы β„° c Π²Π½ΡƒΡ‚Ρ€Π΅Π½Π½ΠΈΠΌ сопротивлСниСм r. ΠŸΠΎΡΠΊΠΎΠ»ΡŒΠΊΡƒ Π­Π”Π‘ β€” практичСски Π²Π΅Π»ΠΈΡ‡ΠΈΠ½Π°, зависящая ΠΎΡ‚ Π²Π½Π΅ΡˆΠ½ΠΈΡ… сил, Ρ‚ΠΎ физичСский смысл ΠΈΠΌΠ΅Π΅Ρ‚ расчёт силы Ρ‚ΠΎΠΊΠ° для ΠΏΠΎΠ»Π½ΠΎΠΉ Ρ†Π΅ΠΏΠΈ ΠΏΡ€ΠΈ ΠΏΠΎΠΌΠΎΡ‰ΠΈ выраТСния: I=β„°/(R+r).

Π’Π°ΠΊΠΈΠΌ ΠΎΠ±Ρ€Π°Π·ΠΎΠΌ, ΠΏΠΎΠ»Π½Ρ‹ΠΉ постулат Ома гласит ΠΎ зависимости силы Ρ‚ΠΎΠΊΠ° Π² Π·Π°ΠΌΠΊΠ½ΡƒΡ‚ΠΎΠΌ ΠΊΠΎΠ½Ρ‚ΡƒΡ€Π΅ ΠΎΡ‚ Π²Π½ΡƒΡ‚Ρ€Π΅Π½Π½Π΅Π³ΠΎ сопротивлСния Π΅Π³ΠΎ источника, Ρ‚ΠΎ Π΅ΡΡ‚ΡŒ ΡƒΡ‡ΠΈΡ‚Ρ‹Π²Π°Π΅Ρ‚ сопротивлСниС элСктролита ΠΈ элСктродов для Π³Π°Π»ΡŒΠ²Π°Π½ΠΈΡ‡Π΅ΡΠΊΠΈΡ… элСмСнтов ΠΈ ΠΏΡ€ΠΎΠ²ΠΎΠ΄ΠΈΠΌΠΎΡΡ‚ΡŒ ΠΎΠ±ΠΌΠΎΡ‚ΠΎΠΊ Π³Π΅Π½Π΅Ρ€Π°Ρ‚ΠΎΡ€ΠΎΠ². ОсновноС практичСскоС ΠΏΡ€ΠΈΠΌΠ΅Π½Π΅Π½ΠΈΠ΅ β€” расчёт силы Ρ‚ΠΎΠΊΠ° Π² Π»ΠΈΠ½Π΅ΠΉΠ½Ρ‹Ρ… элСктричСских цСпях DC, ΠΎΠΏΡ€Π΅Π΄Π΅Π»Π΅Π½ΠΈΠ΅ мощности ΠΈ импСданса Π»ΡŽΠ±Ρ‹Ρ… элСмСнтов Ρ†Π΅ΠΏΠΈ.

Π€ΠΎΡ€ΠΌΡƒΠ»ΠΈΡ€ΠΎΠ²ΠΊΠ° ΠΈ физичСский смысл Π·Π°ΠΊΠΎΠ½Π° ΡƒΡ‚Π΅Ρ‡ΠΊΠΈ энСргии Π² ΠΏΡ€Π΅Π΄Π΅Π»Π°Ρ… Π·Π°ΠΌΠΊΠ½ΡƒΡ‚ΠΎΠΉ Ρ†Π΅ΠΏΠΈ

Π‘ΠΎΠΎΠ±Ρ‰Π΅Π½ΠΈΠ΅ Π½Π° Ρ‚Π΅ΠΌΡƒ: Π—Π°ΠΊΠΎΠ½ Ома для Π·Π°ΠΌΠΊΠ½ΡƒΡ‚ΠΎΠΉ Ρ†Π΅ΠΏΠΈ. ΠžΡΠ½ΠΎΠ²Π½Ρ‹Π΅ понятия.

Π—Π°ΠΊΠΎΠ½ Ома для Π·Π°ΠΌΠΊΠ½ΡƒΡ‚ΠΎΠΉ Ρ†Π΅ΠΏΠΈ

Замкнутая (полная) элСктричСская Ρ†Π΅ΠΏΡŒ состоит из источника Ρ‚ΠΎΠΊΠ°Β ΠΈ сопротивлСния.

Π˜ΡΡ‚ΠΎΡ‡Π½ΠΈΠΊ Ρ‚ΠΎΠΊΠ° ΠΈΠΌΠ΅Π΅Ρ‚ Π­Π”Π‘ () ΠΈ сопротивлСниС (r), ΠΊΠΎΡ‚ΠΎΡ€ΠΎΠ΅ Π½Π°Π·Ρ‹Π²Π°ΡŽΡ‚Β Π²Π½ΡƒΡ‚Ρ€Π΅Π½Π½ΠΈΠΌ. Π­Π”Π‘ (элСктродвиТущая сила) β€” Ρ€Π°Π±ΠΎΡ‚Π° сторонних сил ΠΏΠΎ ΠΏΠ΅Ρ€Π΅ΠΌΠ΅Ρ‰Π΅Π½ΠΈΡŽ ΠΏΠΎΠ»ΠΎΠΆΠΈΡ‚Π΅Π»ΡŒΠ½ΠΎΠ³ΠΎ заряда ΠΏΠΎ Π·Π°ΠΌΠΊΠ½ΡƒΡ‚ΠΎΠΉ Ρ†Π΅ΠΏΠΈ (физичСский смысл Π°Π½Π°Π»ΠΎΠ³ΠΈΡ‡Π΅Π½Β Π½Π°ΠΏΡ€ΡΠΆΠ΅Π½ΠΈΡŽ,Β ΠΏΠΎΡ‚Π΅Π½Ρ†ΠΈΠ°Π»Ρƒ). ПолноС сопротивлСниС Ρ†Π΅ΠΏΠΈ β€” R+r.

  • 1) НапряТСниС Π½Π° Π·Π°ΠΆΠΈΠΌΠ°Ρ… источника, Π° соотвСтствСнно ΠΈ Π²ΠΎ внСшнСй Ρ†Π΅ΠΏΠΈ
  • ,Π³Π΄Π΅ Π²Π΅Π»ΠΈΡ‡ΠΈΠ½Π°Β Β β€”Β ΠΏΠ°Π΄Π΅Π½ΠΈΠ΅ напряТСния внутри источника Ρ‚ΠΎΠΊΠ°.
  • 2) Если внСшнСС сопротивлСниС Π·Π°ΠΌΠΊΠ½ΡƒΡ‚ΠΎΠΉ Ρ†Π΅ΠΏΠΈ Ρ€Π°Π²Π½ΠΎ Π½ΡƒΠ»ΡŽ, Ρ‚ΠΎ Ρ‚Π°ΠΊΠΎΠΉ Ρ€Π΅ΠΆΠΈΠΌ источника Ρ‚ΠΎΠΊΠ° называСтся коротким Π·Π°ΠΌΡ‹ΠΊΠ°Π½ΠΈΠ΅ΠΌ.
  • 3) Для ΠΏΠΎΠ»Π½ΠΎΠΉ Ρ†Π΅ΠΏΠΈΒ Π·Π°ΠΊΠΎΠ½ ДТоуля-Π›Π΅Π½Ρ†Π°

  1. ΠšΠΎΡΡ„Ρ„ΠΈΡ†ΠΈΠ΅Π½Ρ‚ ΠΏΠΎΠ»Π΅Π·Π½ΠΎΠ³ΠΎ дСйствия
  2. ΠœΠΎΡ‰Π½ΠΎΡΡ‚ΡŒ, выдСляСмая Π½Π° внСшнСм участкС Ρ†Π΅ΠΏΠΈ, называСтся полСзной
  3. ΠŸΡ€ΠΈ условии R=r ΠΌΠΎΡ‰Π½ΠΎΡΡ‚ΡŒ, выдСляСмая Π²ΠΎ внСшнСй Ρ†Π΅ΠΏΠΈ, максимальная для Π΄Π°Π½Π½ΠΎΠ³ΠΎ источника ΠΈ Ρ€Π°Π²Π½Π°
  4. Полная ΠΌΠΎΡ‰Π½ΠΎΡΡ‚ΡŒ β€” сумма ΠΏΠΎΠ»Π΅Π·Π½ΠΎΠΉ ΠΈ тСряСмой мощности

ΠšΠΎΡΡ„Ρ„ΠΈΡ†ΠΈΠ΅Π½Ρ‚ ΠΏΠΎΠ»Π΅Π·Π½ΠΎΠ³ΠΎ дСйствия источника Ρ‚ΠΎΠΊΠ° β€” ΠΎΡ‚Π½ΠΎΡˆΠ΅Π½ΠΈΠ΅ ΠΏΠΎΠ»Π΅Π·Π½ΠΎΠΉ мощности ΠΊ ΠΏΠΎΠ»Π½ΠΎΠΉ

Π˜ΡΡ‚ΠΎΡ‡Π½ΠΈΠΊ Π­Π”Π‘

Для сущСствования постоянного Ρ‚ΠΎΠΊΠ° Π² Ρ†Π΅ΠΏΠΈ Π½Π΅ΠΎΠ±Ρ…ΠΎΠ΄ΠΈΠΌΠΎ Π½Π΅ΠΏΡ€Π΅Ρ€Ρ‹Π²Π½ΠΎ Ρ€Π°Π·Π΄Π΅Π»ΡΡ‚ΡŒ элСктричСскиС заряды, ΠΊΠΎΡ‚ΠΎΡ€Ρ‹Π΅ ΠΏΠΎΠ΄ дСйствиСм сил ΠšΡƒΠ»ΠΎΠ½Π°Β ΡΡ‚Ρ€Π΅ΠΌΡΡ‚ΡΡ ΡΠΎΠ΅Π΄ΠΈΠ½ΠΈΡ‚ΡŒΡΡ. Для этого Π½Π΅ΠΎΠ±Ρ…ΠΎΠ΄ΠΈΠΌΡ‹ сторонниС силы.

Π­Π”Π‘ Ρ…Π°Ρ€Π°ΠΊΡ‚Π΅Ρ€ΠΈΠ·ΡƒΠ΅Ρ‚ дСйствиС этих сторонних сил. А сама эта Ρ€Π°Π±ΠΎΡ‚Π° осущСствляСтся Π²Π½ΡƒΡ‚Ρ€ΠΈ источников Π­Π”Π‘.

ЭлСктричСскиС заряды Π²Π½ΡƒΡ‚Ρ€ΠΈ источников Π­Π”Π‘ двиТутся ΠΏΡ€ΠΎΡ‚ΠΈΠ² кулоновских сил ΠΏΠΎΠ΄ воздСйствиСм сторонних сил.

  • Бравнивая элСктричСский Ρ‚ΠΎΠΊ с Ρ‚Π΅Ρ‡Π΅Π½ΠΈΠ΅ΠΌ Тидкости Π² Ρ‚Ρ€ΡƒΠ±Π°Ρ…, ΠΌΠΎΠΆΠ½ΠΎ ΡΠΊΠ°Π·Π°Ρ‚ΡŒ, Ρ‡Ρ‚ΠΎ источник Ρ€Π°Π±ΠΎΡ‚Π°Π΅Ρ‚, ΠΊΠ°ΠΊ насос, ΠΊΠΎΡ‚ΠΎΡ€Ρ‹ΠΉ ΠΏΠΎΠ΄Π°Π΅Ρ‚ Π²ΠΎΠ΄Ρƒ ΠΈΠ· Π½ΠΈΠΆΠ½Π΅Π³ΠΎ Ρ€Π΅Π·Π΅Ρ€Π²ΡƒΠ°Ρ€Π° Π² Π²Π΅Ρ€Ρ…Π½ΠΈΠΉ, ΠΈΠ· ΠΊΠΎΡ‚ΠΎΡ€ΠΎΠ³ΠΎ ΠΎΠ½Π° ΠΏΠΎΠ΄ дСйствиСм силы тяТСсти стСкаСт Π² Π½ΠΈΠΆΠ½ΠΈΠΉ Ρ€Π΅Π·Π΅Ρ€Π²ΡƒΠ°Ρ€.
  • Π’ Π±Ρ‹Ρ‚Ρƒ «источником Ρ‚ΠΎΠΊΠ°Β» часто Π½Π΅Ρ‚ΠΎΡ‡Π½ΠΎ Π½Π°Π·Ρ‹Π²Π°ΡŽΡ‚ любой источник элСктричСского напряТСния (Π±Π°Ρ‚Π°Ρ€Π΅ΡŽ, Π³Π΅Π½Π΅Ρ€Π°Ρ‚ΠΎΡ€, Ρ€ΠΎΠ·Π΅Ρ‚ΠΊΡƒ), Π½ΠΎ Π² строго физичСском смыслС это Π½Π΅ Ρ‚Π°ΠΊ, Π±ΠΎΠ»Π΅Π΅ Ρ‚ΠΎΠ³ΠΎ, ΠΎΠ±Ρ‹Ρ‡Π½ΠΎ ΠΈΡΠΏΠΎΠ»ΡŒΠ·ΡƒΠ΅ΠΌΡ‹Π΅ Π² Π±Ρ‹Ρ‚Ρƒ источники напряТСния ΠΏΠΎ своим характСристикам Π³ΠΎΡ€Π°Π·Π΄ΠΎ Π±Π»ΠΈΠΆΠ΅ ΠΊ источнику Π­Π”Π‘, Ρ‡Π΅ΠΌ ΠΊ источнику Ρ‚ΠΎΠΊΠ° ΠΈΠ·-Π·Π° наличия Π²Π½ΡƒΡ‚Ρ€Π΅Π½Π½Π΅Π³ΠΎ сопротивлСния.
  • Π’ настоящСС врСмя Π²Ρ‹ΠΏΡƒΡΠΊΠ°ΡŽΡ‚ мноТСство Ρ€Π°Π·Π»ΠΈΡ‡Π½Ρ‹Ρ… источников Π­Π”Π‘ β€” ΠΎΡ‚ ΠΌΠ°Π»Π΅Π½ΡŒΠΊΠΈΡ… Π±Π°Ρ‚Π°Ρ€Π΅Π΅ΠΊ для часов Π΄ΠΎ Π³Π΅Π½Π΅Ρ€Π°Ρ‚ΠΎΡ€ΠΎΠ².
  • Π’Π½ΡƒΡ‚Ρ€ΠΈ источника Ρ‚ΠΎΠΊΠ° происходит Ρ€Π°Π·Π΄Π΅Π»Π΅Π½ΠΈΠ΅ зарядов ΠΈΠ·-Π·Π° процСссов, происходящих Π²Π½ΡƒΡ‚Ρ€ΠΈ источника, Π½Π°ΠΏΡ€ΠΈΠΌΠ΅Ρ€, химичСских процСссов.
  • Π“Π°Π»ΡŒΠ²Π°Π½ΠΈΡ‡Π΅ΡΠΊΠΈΠΉ элСмСнт — химичСский источник Ρ‚ΠΎΠΊΠ°, основанный Π½Π° взаимодСйствии Π΄Π²ΡƒΡ… ΠΌΠ΅Ρ‚Π°Π»Π»ΠΎΠ² ΠΈ (ΠΈΠ»ΠΈ) ΠΈΡ… оксидов Π² элСктролитС (Π±Π°Ρ‚Π°Ρ€Π΅ΠΉΠΊΠΈ, аккумуляторы).

  1. Π“Π΅Π½Π΅Ρ€Π°Ρ‚ΠΎΡ€Ρ‹Β β€” ΡΠΎΠ·Π΄Π°ΡŽΡ‚ Ρ‚ΠΎΠΊ Π·Π° счСт расходования мСханичСской энСргии.
  2. ВСрмоэлСмСнты — ΠΈΡΠΏΠΎΠ»ΡŒΠ·ΡƒΡŽΡ‚ ΡΠ½Π΅Ρ€Π³ΠΈΡŽ Ρ‚Π΅ΠΏΠ»ΠΎΠ²ΠΎΠ³ΠΎ двиТСния заряТСнных частиц.
  3. ЀотоэлСмСнты — ΡΠΎΠ·Π΄Π°ΡŽΡ‚ Ρ‚ΠΎΠΊ Π·Π° счСт энСргии свСта.
  4. Π‘ΠΎΠ΅Π΄ΠΈΠ½Π΅Π½ΠΈΠ΅ источников Ρ‚ΠΎΠΊΠ°*
  5. Рассмотрим n ΠΎΠ΄ΠΈΠ½Π°ΠΊΠΎΠ²Ρ‹Ρ… источников Π­Π”Π‘
  6. ΠŸΡ€Π°Π²ΠΈΠ»Π° ΠšΠΈΡ€Ρ…Π³ΠΎΡ„Π°**
  7. Для расчСта слоТных Ρ€Π°Π·Π²Π΅Ρ‚Π²Π»Π΅Π½Π½Ρ‹Ρ… Ρ†Π΅ΠΏΠ΅ΠΉ, ΠΊΠΎΡ‚ΠΎΡ€Ρ‹Π΅ нСльзя свСсти ΠΊ эквивалСнтной Ρ†Π΅ΠΏΠΈ, ΠΈΡΠΏΠΎΠ»ΡŒΠ·ΡƒΡŽΡ‚ ΠΏΡ€Π°Π²ΠΈΠ»Π° ΠšΠΈΡ€Ρ…Π³ΠΎΡ„Π°:
  8. 1) АлгСбраичСская сумма сил Ρ‚ΠΎΠΊΠΎΠ², сходящихся Π² ΡƒΠ·Π»Π΅ Ρ€Π°Π²Π½Π° Π½ΡƒΠ»ΡŽ.
  9. 2) АлгСбраичСская сумма ΠΏΠ°Π΄Π΅Π½ΠΈΠΉ напряТСний Π² любом простом Π·Π°ΠΌΠΊΠ½ΡƒΡ‚ΠΎΠΌ ΠΊΠΎΠ½Ρ‚ΡƒΡ€Π΅ Ρ€Π°Π²Π½Π° алгСбраичСской суммС Π­Π”Π‘, ΠΊΠΎΡ‚ΠΎΡ€Ρ‹Π΅ Π΅ΡΡ‚ΡŒ Π² этом ΠΊΠΎΠ½Ρ‚ΡƒΡ€Π΅.

Π˜ΡΡ‚ΠΎΡ‡Π½ΠΈΠΊ: https://infourok.ru/soobschenie-na-temu-zakon-oma-dlya-zamknutoy-cepi-osnovnie-ponyatiya-3036846.html

Π€ΠΈΠ·ΠœΠ°Ρ‚

ЭлСктродвиТущая сила.Β 

ЭДБ — энСргСтичСская  характСристика источника.Β Π­Ρ‚ΠΎ физичСская Π²Π΅Π»ΠΈΡ‡ΠΈΠ½Π°, равная ΠΎΡ‚Π½ΠΎΡˆΠ΅Π½ΠΈΡŽ Ρ€Π°Π±ΠΎΡ‚Ρ‹, ΡΠΎΠ²Π΅Ρ€ΡˆΠ΅Π½Π½ΠΎΠΉ сторонни­ми силами ΠΏΡ€ΠΈ ΠΏΠ΅Ρ€Π΅ΠΌΠ΅Ρ‰Π΅Π½ΠΈΠΈ элСктричСского заряда ΠΏΠΎ Π·Π°ΠΌΠΊΠ½ΡƒΡ‚ΠΎΠΉ Ρ†Π΅ΠΏΠΈ, ΠΊ этому заряду:

Π˜Π·ΠΌΠ΅Ρ€ΡΠ΅Ρ‚ΡΡ Π² Π²ΠΎΠ»ΡŒΡ‚Π°Ρ… (Π’).

Π—Π°ΠΊΠΎΠ½ Ома для Π·Π°ΠΌΠΊΠ½ΡƒΡ‚ΠΎΠΉ Ρ†Π΅ΠΏΠΈ ΠΈ Π½Π΅ΠΎΠ΄Π½ΠΎΡ€ΠΎΠ΄Π½ΠΎΠ³ΠΎ участка Ρ†Π΅ΠΏΠΈ.

Π—Π°ΠΊΠΎΠ½ ΠΎΠΌΠ° для Π·Π°ΠΌΠΊΠ½ΡƒΡ‚ΠΎΠΉ Ρ†Π΅ΠΏΠΈ Π³ΠΎΠ²ΠΎΡ€ΠΈΡ‚ ΠΎ Ρ‚ΠΎΠΌ Ρ‡Ρ‚ΠΎ. Π’Π΅Π»ΠΈΡ‡ΠΈΠ½Π° Ρ‚ΠΎΠΊΠ° Π² Π·Π°ΠΌΠΊΠ½ΡƒΡ‚ΠΎΠΉ Ρ†Π΅ΠΏΠΈ, которая состоит ΠΈΠ· источника Ρ‚ΠΎΠΊΠ° ΠΎΠ±Π»Π°Π΄Π°ΡŽΡ‰Π΅Π³ΠΎ Π²Π½ΡƒΡ‚Ρ€Π΅Π½Π½ΠΈΠΌ сопротивлСниСм, Π° Ρ‚Π°ΠΊΠΆΠ΅ внСшним Π½Π°Π³Ρ€ΡƒΠ·ΠΎΡ‡Π½Ρ‹ΠΌ сопротивлСниСм. Π‘ΡƒΠ΄Π΅Ρ‚ Ρ€Π°Π²Π½Π° ΠΎΡ‚Π½ΠΎΡˆΠ΅Π½ΠΈΡŽ элСктродвиТущСй силы источника ΠΊ суммС внСшнСго ΠΈ Π²Π½ΡƒΡ‚Ρ€Π΅Π½Π½Π΅Π³ΠΎ сопротивлСний.Β 

ΠŸΡ€ΠΈ ΠΏΡ€ΠΎΡ…ΠΎΠΆΠ΄Π΅Π½ΠΈΠΈ элСктричСского Ρ‚ΠΎΠΊΠ° Π² Π·Π°ΠΌΠΊΠ½ΡƒΡ‚ΠΎΠΉ Ρ†Π΅ΠΏΠΈ Π½Π° свободныС заряды Π΄Π΅ΠΉΡΡ‚Π²ΡƒΡŽΡ‚ силы со стороны стационарного элСктричСского поля ΠΈ сторонниС силы.

ΠŸΡ€ΠΈ этом Π½Π° ΠΎΡ‚Π΄Π΅Π»ΡŒΠ½Ρ‹Ρ… участках этой Ρ†Π΅ΠΏΠΈ Ρ‚ΠΎΠΊ создаСтся Ρ‚ΠΎΠ»ΡŒΠΊΠΎ стационарным элСктричСским ΠΏΠΎΠ»Π΅ΠΌ. Π’Π°ΠΊΠΈΠ΅ участки Ρ†Π΅ΠΏΠΈ Π½Π°Π·Ρ‹Π²Π°ΡŽΡ‚ΡΡ ΠΎΠ΄Π½ΠΎΡ€ΠΎΠ΄Π½Ρ‹ΠΌΠΈ. На Π½Π΅ΠΊΠΎΡ‚ΠΎΡ€Ρ‹Ρ… участках этой Ρ†Π΅ΠΏΠΈ, ΠΊΡ€ΠΎΠΌΠ΅ сил стационарного элСктричСского поля, Π΄Π΅ΠΉΡΡ‚Π²ΡƒΡŽΡ‚ ΠΈ сторонниС силы.

Участок Ρ†Π΅ΠΏΠΈ, Π½Π° ΠΊΠΎΡ‚ΠΎΡ€ΠΎΠΌ Π΄Π΅ΠΉΡΡ‚Π²ΡƒΡŽΡ‚ сторонниС силы, Π½Π°Π·Ρ‹Π²Π°ΡŽΡ‚ Π½Π΅ΠΎΠ΄Π½ΠΎΡ€ΠΎΠ΄Π½Ρ‹ΠΌ участком Ρ†Π΅ΠΏΠΈ.

Для Ρ‚ΠΎΠ³ΠΎ Ρ‡Ρ‚ΠΎΠ±Ρ‹ Π²Ρ‹ΡΡΠ½ΠΈΡ‚ΡŒ, ΠΎΡ‚ Ρ‡Π΅Π³ΠΎ зависит сила Ρ‚ΠΎΠΊΠ° Π½Π° этих участках, Π½Π΅ΠΎΠ±Ρ…ΠΎΠ΄ΠΈΠΌΠΎ ΡƒΡ‚ΠΎΡ‡Π½ΠΈΡ‚ΡŒ понятиС напряТСния.

Рис. 1
Рассмотрим Π²Π½Π°Ρ‡Π°Π»Π΅ ΠΎΠ΄Π½ΠΎΡ€ΠΎΠ΄Π½Ρ‹ΠΉ участок Ρ†Π΅ΠΏΠΈ (рис. 1, Π°). Π’ этом случаС Ρ€Π°Π±ΠΎΡ‚Ρƒ ΠΏΠΎ ΠΏΠ΅Ρ€Π΅ΠΌΠ΅Ρ‰Π΅Π½ΠΈΡŽ заряда ΡΠΎΠ²Π΅Ρ€ΡˆΠ°ΡŽΡ‚ Ρ‚ΠΎΠ»ΡŒΠΊΠΎ силы стационарного элСктричСского поля, ΠΈ этот участок Ρ…Π°Ρ€Π°ΠΊΡ‚Π΅Ρ€ΠΈΠ·ΡƒΡŽΡ‚ Ρ€Π°Π·Π½ΠΎΡΡ‚ΡŒΡŽ ΠΏΠΎΡ‚Π΅Π½Ρ†ΠΈΠ°Π»ΠΎΠ² Δφ. Π Π°Π·Π½ΠΎΡΡ‚ΡŒ ΠΏΠΎΡ‚Π΅Π½Ρ†ΠΈΠ°Π»ΠΎΠ² Π½Π° ΠΊΠΎΠ½Ρ†Π°Ρ… участка , Π³Π΄Π΅ AK β€” Ρ€Π°Π±ΠΎΡ‚Π° сил стационарного элСктричСского поля. НСоднородный участок Ρ†Π΅ΠΏΠΈ (рис. 1, Π±) содСрТит Π² ΠΎΡ‚Π»ΠΈΡ‡ΠΈΠ΅ ΠΎΡ‚ ΠΎΠ΄Π½ΠΎΡ€ΠΎΠ΄Π½ΠΎΠ³ΠΎ участка источник Π­Π”Π‘, ΠΈ ΠΊ Ρ€Π°Π±ΠΎΡ‚Π΅ сил элСктростатичСского поля Π½Π° этом участкС добавляСтся Ρ€Π°Π±ΠΎΡ‚Π° сторонних сил. По ΠΎΠΏΡ€Π΅Π΄Π΅Π»Π΅Π½ΠΈΡŽ,Β , Π³Π΄Π΅ q β€” ΠΏΠΎΠ»ΠΎΠΆΠΈΡ‚Π΅Π»ΡŒΠ½Ρ‹ΠΉ заряд, ΠΊΠΎΡ‚ΠΎΡ€Ρ‹ΠΉ пСрСмСщаСтся ΠΌΠ΅ΠΆΠ΄Ρƒ Π»ΡŽΠ±Ρ‹ΠΌΠΈ двумя Ρ‚ΠΎΡ‡ΠΊΠ°ΠΌΠΈ Ρ†Π΅ΠΏΠΈ;Β Β β€” Ρ€Π°Π·Π½ΠΎΡΡ‚ΡŒ ΠΏΠΎΡ‚Π΅Π½Ρ†ΠΈΠ°Π»ΠΎΠ² Ρ‚ΠΎΡ‡Π΅ΠΊ Π² Π½Π°Ρ‡Π°Π»Π΅ ΠΈ ΠΊΠΎΠ½Ρ†Π΅ рассматриваСмого участка;Β . Π’ΠΎΠ³Π΄Π° говорят ΠΎ напряТСнии для напряТСнности: Eстац. э. ΠΏ. = Eэ/стат. ΠΏ. + Eстор. НапряТСниС U Π½Π° участкС Ρ†Π΅ΠΏΠΈ прСдставляСт собой Ρ„ΠΈΠ·ΠΈΡ‡Π΅ΡΠΊΡƒΡŽ ΡΠΊΠ°Π»ΡΡ€Π½ΡƒΡŽ Π²Π΅Π»ΠΈΡ‡ΠΈΠ½Ρƒ, Ρ€Π°Π²Π½ΡƒΡŽ суммарной Ρ€Π°Π±ΠΎΡ‚Π΅ сторонних сил ΠΈ сил элСктростатичСского поля ΠΏΠΎ ΠΏΠ΅Ρ€Π΅ΠΌΠ΅Ρ‰Π΅Π½ΠΈΡŽ Π΅Π΄ΠΈΠ½ΠΈΡ‡Π½ΠΎΠ³ΠΎ ΠΏΠΎΠ»ΠΎΠΆΠΈΡ‚Π΅Π»ΡŒΠ½ΠΎΠ³ΠΎ заряда Π½Π° этом участкС:

Из этой Ρ„ΠΎΡ€ΠΌΡƒΠ»Ρ‹ Π²ΠΈΠ΄Π½ΠΎ, Ρ‡Ρ‚ΠΎ Π² ΠΎΠ±Ρ‰Π΅ΠΌ случаС напряТСниС Π½Π° Π΄Π°Π½Π½ΠΎΠΌ участкС Ρ†Π΅ΠΏΠΈ Ρ€Π°Π²Π½ΠΎ алгСбраичСской суммС разности ΠΏΠΎΡ‚Π΅Π½Ρ†ΠΈΠ°Π»ΠΎΠ² ΠΈ Π­Π”Π‘ Π½Π° этом участкС. Если ΠΆΠ΅ Π½Π° участкС Π΄Π΅ΠΉΡΡ‚Π²ΡƒΡŽΡ‚ Ρ‚ΠΎΠ»ΡŒΠΊΠΎ элСктричСскиС силы (Ξ΅ = 0), Ρ‚ΠΎ . Π’Π°ΠΊΠΈΠΌ ΠΎΠ±Ρ€Π°Π·ΠΎΠΌ, Ρ‚ΠΎΠ»ΡŒΠΊΠΎ для ΠΎΠ΄Π½ΠΎΡ€ΠΎΠ΄Π½ΠΎΠ³ΠΎ участка Ρ†Π΅ΠΏΠΈ понятия напряТСния ΠΈ разности ΠΏΠΎΡ‚Π΅Π½Ρ†ΠΈΠ°Π»ΠΎΠ² ΡΠΎΠ²ΠΏΠ°Π΄Π°ΡŽΡ‚.

Π—Π°ΠΊΠΎΠ½ Ома для Π½Π΅ΠΎΠ΄Π½ΠΎΡ€ΠΎΠ΄Π½ΠΎΠ³ΠΎ участка Ρ†Π΅ΠΏΠΈ ΠΈΠΌΠ΅Π΅Ρ‚ Π²ΠΈΠ΄:

Π³Π΄Π΅ R β€” ΠΎΠ±Ρ‰Π΅Π΅ сопротивлСниС Π½Π΅ΠΎΠ΄Π½ΠΎΡ€ΠΎΠ΄Π½ΠΎΠ³ΠΎ участка.

Π­Π”Π‘ Ξ΅ ΠΌΠΎΠΆΠ΅Ρ‚ Π±Ρ‹Ρ‚ΡŒ ΠΊΠ°ΠΊ ΠΏΠΎΠ»ΠΎΠΆΠΈΡ‚Π΅Π»ΡŒΠ½ΠΎΠΉ, Ρ‚Π°ΠΊ ΠΈ ΠΎΡ‚Ρ€ΠΈΡ†Π°Ρ‚Π΅Π»ΡŒΠ½ΠΎΠΉ.

Π­Ρ‚ΠΎ связано с ΠΏΠΎΠ»ΡΡ€Π½ΠΎΡΡ‚ΡŒΡŽ Π²ΠΊΠ»ΡŽΡ‡Π΅Π½ΠΈΡ Π­Π”Π‘ Π² участок: Ссли Π½Π°ΠΏΡ€Π°Π²Π»Π΅Π½ΠΈΠ΅, создаваСмоС источником Ρ‚ΠΎΠΊΠ°, совпадаСт с Π½Π°ΠΏΡ€Π°Π²Π»Π΅Π½ΠΈΠ΅ΠΌ Ρ‚ΠΎΠΊΠ°, проходящСго Π² участкС (Π½Π°ΠΏΡ€Π°Π²Π»Π΅Π½ΠΈΠ΅ Ρ‚ΠΎΠΊΠ° Π½Π° участкС совпадаСт Π²Π½ΡƒΡ‚Ρ€ΠΈ источника с Π½Π°ΠΏΡ€Π°Π²Π»Π΅Π½ΠΈΠ΅ΠΌ ΠΎΡ‚ ΠΎΡ‚Ρ€ΠΈΡ†Π°Ρ‚Π΅Π»ΡŒΠ½ΠΎΠ³ΠΎ полюса ΠΊ ΠΏΠΎΠ»ΠΎΠΆΠΈΡ‚Π΅Π»ΡŒΠ½ΠΎΠΌΡƒ), Ρ‚.Π΅. Π­Π”Π‘ способствуСт двиТСнию ΠΏΠΎΠ»ΠΎΠΆΠΈΡ‚Π΅Π»ΡŒΠ½Ρ‹Ρ… зарядов Π² Π΄Π°Π½Π½ΠΎΠΌ Π½Π°ΠΏΡ€Π°Π²Π»Π΅Π½ΠΈΠΈ, Ρ‚ΠΎ Ξ΅ > 0, Π² ΠΏΡ€ΠΎΡ‚ΠΈΠ²Π½ΠΎΠΌ случаС, Ссли Π­Π”Π‘ прСпятствуСт двиТСнию ΠΏΠΎΠ»ΠΎΠΆΠΈΡ‚Π΅Π»ΡŒΠ½Ρ‹Ρ… зарядов Π² Π΄Π°Π½Π½ΠΎΠΌ Π½Π°ΠΏΡ€Π°Π²Π»Π΅Π½ΠΈΠΈ, Ρ‚ΠΎ Ξ΅

ΠŸΡ€Π°Π²ΠΈΠ»Π° ΠšΠΈΡ…Π³ΠΎΡ„Π°.Β 

Π Π°Π±ΠΎΡ‚Π° ΠΈ ΠΌΠΎΡ‰Π½ΠΎΡΡ‚ΡŒ Ρ‚ΠΎΠΊΠ°. Π’Π΅ΠΏΠ»ΠΎΠ²ΠΎΠ΅ дСйствиС Ρ‚ΠΎΠΊΠ°. Π—Π°ΠΊΠΎΠ½ ДТоуля-Π›Π΅Π½Ρ†Π°.

ΠŸΡ€ΠΈ ΠΏΡ€ΠΎΡ‚Π΅ΠΊΠ°Π½ΠΈΠΈ Ρ‚ΠΎΠΊΠ° ΠΏΠΎ ΠΎΠ΄Π½ΠΎΡ€ΠΎΠ΄Π½ΠΎΠΌΡƒ участку Ρ†Π΅ΠΏΠΈ элСктричСскоС ΠΏΠΎΠ»Π΅ ΡΠΎΠ²Π΅Ρ€ΡˆΠ°Π΅Ρ‚ Ρ€Π°Π±ΠΎΡ‚Ρƒ. Π—Π° врСмя Ξ”tΒ ΠΏΠΎ Ρ†Π΅ΠΏΠΈ ΠΏΡ€ΠΎΡ‚Π΅ΠΊΠ°Π΅Ρ‚ заряд Ξ”qΒ =Β IΒ Ξ”t. ЭлСктричСскоС ΠΏΠΎΠ»Π΅ Π½Π° Π²Ρ‹Π΄Π΅Π»Π΅Π½Π½ΠΎΠΌ учСсткС ΡΠΎΠ²Π΅Ρ€ΡˆΠ°Π΅Ρ‚ Ρ€Π°Π±ΠΎΡ‚ΡƒΒ 

Ξ”AΒ = (Ο†1 – Ο†2)Β Ξ”qΒ = Δφ12Β IΒ Ξ”tΒ =Β UΒ IΒ Ξ”t,

Π³Π΄Π΅Β UΒ = Δφ12 – напряТСниС. Π­Ρ‚Ρƒ Ρ€Π°Π±ΠΎΡ‚Ρƒ Π½Π°Π·Ρ‹Π²Π°ΡŽΡ‚Β Ρ€Π°Π±ΠΎΡ‚ΠΎΠΉ элСктричСского Ρ‚ΠΎΠΊΠ°.

  • Если ΠΎΠ±Π΅ части Ρ„ΠΎΡ€ΠΌΡƒΠ»Ρ‹Β 
  • Π­Ρ‚ΠΎ ΡΠΎΠΎΡ‚Π½ΠΎΡˆΠ΅Π½ΠΈΠ΅ Π²Ρ‹Ρ€Π°ΠΆΠ°Π΅Ρ‚ Π·Π°ΠΊΠΎΠ½ сохранСния энСргии для ΠΎΠ΄Π½ΠΎΡ€ΠΎΠ΄Π½ΠΎΠ³ΠΎ участка Ρ†Π΅ΠΏΠΈ.
  • Π Π°Π±ΠΎΡ‚Π° Ξ”A элСктричСского Ρ‚ΠΎΠΊΠ°Β I, ΠΏΡ€ΠΎΡ‚Π΅ΠΊΠ°ΡŽΡ‰Π΅Π³ΠΎ ΠΏΠΎ Π½Π΅ΠΏΠΎΠ΄Π²ΠΈΠΆΠ½ΠΎΠΌΡƒ ΠΏΡ€ΠΎΠ²ΠΎΠ΄Π½ΠΈΠΊΡƒ с сопротивлСниСм R, прСобразуСтся Π² Ρ‚Π΅ΠΏΠ»ΠΎ Ξ”Q, Π²Ρ‹Π΄Π΅Π»ΡΡŽΡ‰Π΅Π΅ΡΡ Π½Π° ΠΏΡ€ΠΎΠ²ΠΎΠ΄Π½ΠΈΠΊΠ΅.

Π—Π°ΠΊΠΎΠ½ прСобразования Ρ€Π°Π±ΠΎΡ‚Ρ‹ Ρ‚ΠΎΠΊΠ° Π² Ρ‚Π΅ΠΏΠ»ΠΎ Π±Ρ‹Π» ΡΠΊΡΠΏΠ΅Ρ€ΠΈΠΌΠ΅Π½Ρ‚Π°Π»ΡŒΠ½ΠΎ установлСн нСзависимо Π΄Ρ€ΡƒΠ³ ΠΎΡ‚ Π΄Ρ€ΡƒΠ³Π°Β Π”ΠΆ.Β Π”ΠΆΠΎΡƒΠ»Π΅ΠΌΒ ΠΈΒ Π­.Β Π›Π΅Π½Ρ†Π΅ΠΌΒ ΠΈ носит Π½Π°Π·Π²Π°Π½ΠΈΠ΅Β Π·Π°ΠΊΠΎΠ½Π° ДТоуля–ЛСнца.

ΠœΠΎΡ‰Π½ΠΎΡΡ‚ΡŒ элСктричСского Ρ‚ΠΎΠΊΠ° Ρ€Π°Π²Π½Π° ΠΎΡ‚Π½ΠΎΡˆΠ΅Π½ΠΈΡŽ Ρ€Π°Π±ΠΎΡ‚Ρ‹ Ρ‚ΠΎΠΊΠ° Ξ”AΒ ΠΊ ΠΈΠ½Ρ‚Π΅Ρ€Π²Π°Π»Ρƒ Π²Ρ€Π΅ΠΌΠ΅Π½ΠΈ Ξ”t, Π·Π° ΠΊΠΎΡ‚ΠΎΡ€ΠΎΠ΅ эта Ρ€Π°Π±ΠΎΡ‚Π° Π±Ρ‹Π»Π° ΡΠΎΠ²Π΅Ρ€ΡˆΠ΅Π½Π°:Β 

Π Π°Π±ΠΎΡ‚Π° элСктричСского Ρ‚ΠΎΠΊΠ° Π² БИ выраТаСтся в дТоулях (Π”ΠΆ), ΠΌΠΎΡ‰Π½ΠΎΡΡ‚ΡŒ – Π²Β Π²Π°Ρ‚Ρ‚Π°Ρ…Β (Π’Ρ‚).

Рассмотрим Ρ‚Π΅ΠΏΠ΅Ρ€ΡŒ ΠΏΠΎΠ»Π½ΡƒΡŽ Ρ†Π΅ΠΏΡŒ постоянного Ρ‚ΠΎΠΊΠ°, ΡΠΎΡΡ‚ΠΎΡΡ‰ΡƒΡŽ ΠΈΠ· источника с элСктродвиТущСй силой  и Π²Π½ΡƒΡ‚Ρ€Π΅Π½Π½ΠΈΠΌ сопротивлСниСм rΒ ΠΈ внСшнСго ΠΎΠ΄Π½ΠΎΡ€ΠΎΠ΄Π½ΠΎΠ³ΠΎ участка с сопротивлСниСм R.Β Π—Π°ΠΊΠΎΠ½ Ома для ΠΏΠΎΠ»Π½ΠΎΠΉ цСпи записываСтся Π² Π²ΠΈΠ΄Π΅Β 

Π£ΠΌΠ½ΠΎΠΆΠΈΠ² ΠΎΠ±Π΅ части этой Ρ„ΠΎΡ€ΠΌΡƒΠ»Ρ‹ Π½Π° Ξ”qΒ =Β IΞ”t, ΠΌΡ‹ ΠΏΠΎΠ»ΡƒΡ‡ΠΈΠΌ ΡΠΎΠΎΡ‚Π½ΠΎΡˆΠ΅Π½ΠΈΠ΅, Π²Ρ‹Ρ€Π°ΠΆΠ°ΡŽΡ‰Π΅Π΅ Π·Π°ΠΊΠΎΠ½ сохранСния энСргии для ΠΏΠΎΠ»Π½ΠΎΠΉ Ρ†Π΅ΠΏΠΈ постоянного Ρ‚ΠΎΠΊΠ°:Β 

RΒ I2Ξ”tΒ +Β rΒ I2Ξ”tΒ =Β Β IΞ”tΒ =Β Ξ”Aст.

ΠŸΠ΅Ρ€Π²Ρ‹ΠΉ Ρ‡Π»Π΅Π½ Π² Π»Π΅Π²ΠΎΠΉ части Ξ”QΒ =Β RΒ I2Ξ”t – Ρ‚Π΅ΠΏΠ»ΠΎ, Π²Ρ‹Π΄Π΅Π»ΡΡŽΡ‰Π΅Π΅ΡΡ Π½Π° внСшнСм участкС Ρ†Π΅ΠΏΠΈ Π·Π° врСмя Ξ”t, Π²Ρ‚ΠΎΡ€ΠΎΠΉ Ρ‡Π»Π΅Π½ Ξ”Qист =Β rΒ I2Ξ”t – Ρ‚Π΅ΠΏΠ»ΠΎ, Π²Ρ‹Π΄Π΅Π»ΡΡŽΡ‰Π΅Π΅ΡΡ Π²Π½ΡƒΡ‚Ρ€ΠΈ источника Π·Π° Ρ‚ΠΎ ΠΆΠ΅ врСмя.

Π’Ρ‹Ρ€Π°ΠΆΠ΅Π½ΠΈΠ΅Β Β IΞ”tΒ Ρ€Π°Π²Π½ΠΎ Ρ€Π°Π±ΠΎΡ‚Π΅ сторонних сил Ξ”Aст, Π΄Π΅ΠΉΡΡ‚Π²ΡƒΡŽΡ‰ΠΈΡ… Π²Π½ΡƒΡ‚Ρ€ΠΈ источника.

ΠŸΡ€ΠΈ ΠΏΡ€ΠΎΡ‚Π΅ΠΊΠ°Π½ΠΈΠΈ элСктричСского Ρ‚ΠΎΠΊΠ° ΠΏΠΎ Π·Π°ΠΌΠΊΠ½ΡƒΡ‚ΠΎΠΉ Ρ†Π΅ΠΏΠΈ Ρ€Π°Π±ΠΎΡ‚Π° сторонних сил Ξ”Aст прСобразуСтся Π² Ρ‚Π΅ΠΏΠ»ΠΎ, Π²Ρ‹Π΄Π΅Π»ΡΡŽΡ‰Π΅Π΅ΡΡ Π²ΠΎ внСшнСй Ρ†Π΅ΠΏΠΈ (Ξ”Q) ΠΈ Π²Π½ΡƒΡ‚Ρ€ΠΈ источника (Ξ”Qист).Β 

Π‘Π»Π΅Π΄ΡƒΠ΅Ρ‚ ΠΎΠ±Ρ€Π°Ρ‚ΠΈΡ‚ΡŒ Π²Π½ΠΈΠΌΠ°Π½ΠΈΠ΅, Ρ‡Ρ‚ΠΎ Π² это ΡΠΎΠΎΡ‚Π½ΠΎΡˆΠ΅Π½ΠΈΠ΅ Π½Π΅ Π²Ρ…ΠΎΠ΄ΠΈΡ‚ Ρ€Π°Π±ΠΎΡ‚Π° элСктричСского поля. ΠŸΡ€ΠΈ ΠΏΡ€ΠΎΡ‚Π΅ΠΊΠ°Π½ΠΈΠΈ Ρ‚ΠΎΠΊΠ° ΠΏΠΎ Π·Π°ΠΌΠΊΠ½ΡƒΡ‚ΠΎΠΉ Ρ†Π΅ΠΏΠΈ элСктричСскоС ΠΏΠΎΠ»Π΅ Ρ€Π°Π±ΠΎΡ‚Ρ‹ Π½Π΅ ΡΠΎΠ²Π΅Ρ€ΡˆΠ°Π΅Ρ‚; поэтому тСпло производится ΠΎΠ΄Π½ΠΈΠΌΠΈ Ρ‚ΠΎΠ»ΡŒΠΊΠΎ сторонними силами, Π΄Π΅ΠΉΡΡ‚Π²ΡƒΡŽΡ‰ΠΈΠΌΠΈ Π²Π½ΡƒΡ‚Ρ€ΠΈ источника. Роль элСктричСского поля сводится ΠΊ ΠΏΠ΅Ρ€Π΅Ρ€Π°ΡΠΏΡ€Π΅Π΄Π΅Π»Π΅Π½ΠΈΡŽ Ρ‚Π΅ΠΏΠ»Π° ΠΌΠ΅ΠΆΠ΄Ρƒ Ρ€Π°Π·Π»ΠΈΡ‡Π½Ρ‹ΠΌΠΈ участками Ρ†Π΅ΠΏΠΈ.

Π’Π½Π΅ΡˆΠ½ΡΡ Ρ†Π΅ΠΏΡŒ ΠΌΠΎΠΆΠ΅Ρ‚ ΠΏΡ€Π΅Π΄ΡΡ‚Π°Π²Π»ΡΡ‚ΡŒ собой Π½Π΅ Ρ‚ΠΎΠ»ΡŒΠΊΠΎ ΠΏΡ€ΠΎΠ²ΠΎΠ΄Π½ΠΈΠΊ с сопротивлСниСм R, Π½ΠΎ ΠΈ ΠΊΠ°ΠΊΠΎΠ΅-Π»ΠΈΠ±ΠΎ устройство, ΠΏΠΎΡ‚Ρ€Π΅Π±Π»ΡΡŽΡ‰Π΅Π΅ ΠΌΠΎΡ‰Π½ΠΎΡΡ‚ΡŒ, Π½Π°ΠΏΡ€ΠΈΠΌΠ΅Ρ€, ΡΠ»Π΅ΠΊΡ‚Ρ€ΠΎΠ΄Π²ΠΈΠ³Π°Ρ‚Π΅Π»ΡŒ постоянного Ρ‚ΠΎΠΊΠ°. Π’ этом случаС ΠΏΠΎΠ΄Β RΒ Π½ΡƒΠΆΠ½ΠΎ ΠΏΠΎΠ½ΠΈΠΌΠ°Ρ‚ΡŒΒ ΡΠΊΠ²ΠΈΠ²Π°Π»Π΅Π½Ρ‚Π½ΠΎΠ΅ сопротивлСниС Π½Π°Π³Ρ€ΡƒΠ·ΠΊΠΈ.

ЭнСргия, выдСляСмая Π²ΠΎ внСшнСй Ρ†Π΅ΠΏΠΈ, ΠΌΠΎΠΆΠ΅Ρ‚ частично ΠΈΠ»ΠΈ ΠΏΠΎΠ»Π½ΠΎΡΡ‚ΡŒΡŽ ΠΏΡ€Π΅ΠΎΠ±Ρ€Π°Π·ΠΎΠ²Ρ‹Π²Π°Ρ‚ΡŒΡΡ Π½Π΅ Ρ‚ΠΎΠ»ΡŒΠΊΠΎ Π² Ρ‚Π΅ΠΏΠ»ΠΎ, Π½ΠΎ ΠΈ Π² Π΄Ρ€ΡƒΠ³ΠΈΠ΅ Π²ΠΈΠ΄Ρ‹ энСргии, Π½Π°ΠΏΡ€ΠΈΠΌΠ΅Ρ€, Π² ΠΌΠ΅Ρ…Π°Π½ΠΈΡ‡Π΅ΡΠΊΡƒΡŽ Ρ€Π°Π±ΠΎΡ‚Ρƒ, ΡΠΎΠ²Π΅Ρ€ΡˆΠ°Π΅ΠΌΡƒΡŽ элСктродвигатСлСм.

ΠŸΠΎΡΡ‚ΠΎΠΌΡƒ вопрос ΠΎΠ± использовании энСргии источника Ρ‚ΠΎΠΊΠ° ΠΈΠΌΠ΅Π΅Ρ‚ большоС практичСскоС Π·Π½Π°Ρ‡Π΅Π½ΠΈΠ΅.

Полная ΠΌΠΎΡ‰Π½ΠΎΡΡ‚ΡŒ источника, Ρ‚ΠΎ Π΅ΡΡ‚ΡŒ Ρ€Π°Π±ΠΎΡ‚Π°, ΡΠΎΠ²Π΅Ρ€ΡˆΠ°Π΅ΠΌΠ°Ρ сторонними силами Π·Π° Π΅Π΄ΠΈΠ½ΠΈΡ†Ρƒ Π²Ρ€Π΅ΠΌΠ΅Π½ΠΈ, Ρ€Π°Π²Π½Π°Β 

Π’ΠΎ внСшнСй Ρ†Π΅ΠΏΠΈ выдСляСтся ΠΌΠΎΡ‰Π½ΠΎΡΡ‚ΡŒΒ 

ΠžΡ‚Π½ΠΎΡˆΠ΅Π½ΠΈΠ΅Β Β Ρ€Π°Π²Π½ΠΎΠ΅Β 

называСтся коэффициСнтом ΠΏΠΎΠ»Π΅Π·Π½ΠΎΠ³ΠΎ дСйствия источника.

На рис.Β 1.11.1 графичСски прСдставлСны зависимости мощности источника Pист, ΠΏΠΎΠ»Π΅Π·Π½ΠΎΠΉ мощности P, выдСляСмой Π²ΠΎ внСшнСй Ρ†Π΅ΠΏΠΈ, ΠΈ коэффициСнта ΠΏΠΎΠ»Π΅Π·Π½ΠΎΠ³ΠΎ дСйствия Ξ· ΠΎΡ‚ Ρ‚ΠΎΠΊΠ° Π² Ρ†Π΅ΠΏΠΈΒ I для источника с Π­Π”Π‘, Ρ€Π°Π²Π½ΠΎΠΉΒ , ΠΈ Π²Π½ΡƒΡ‚Ρ€Π΅Π½Π½ΠΈΠΌ сопротивлСниСм r. Π’ΠΎΠΊ Π² Ρ†Π΅ΠΏΠΈ ΠΌΠΎΠΆΠ΅Ρ‚ ΠΈΠ·ΠΌΠ΅Π½ΡΡ‚ΡŒΡΡ Π² ΠΏΡ€Π΅Π΄Π΅Π»Π°Ρ… ΠΎΡ‚Β IΒ =Β 0 (ΠΏΡ€ΠΈΒ ) Π΄ΠΎΒ Β (ΠΏΡ€ΠΈΒ RΒ =Β 0).

Рисунок 1.11.1.
Π—Π°Π²ΠΈΡΠΈΠΌΠΎΡΡ‚ΡŒ мощности источника Pист, мощности Π²ΠΎ внСшнСй Ρ†Π΅ΠΏΠΈΒ PΒ ΠΈ ΠšΠŸΠ” источника Ξ· ΠΎΡ‚ силы Ρ‚ΠΎΠΊΠ°

Из ΠΏΡ€ΠΈΠ²Π΅Π΄Π΅Π½Π½Ρ‹Ρ… Π³Ρ€Π°Ρ„ΠΈΠΊΠΎΠ² Π²ΠΈΠ΄Π½ΠΎ, Ρ‡Ρ‚ΠΎ максимальная ΠΌΠΎΡ‰Π½ΠΎΡΡ‚ΡŒ Π²ΠΎ внСшнСй Ρ†Π΅ΠΏΠΈΒ Pmax, равная 

достигаСтся ΠΏΡ€ΠΈΒ RΒ =Β r. ΠŸΡ€ΠΈ этом Ρ‚ΠΎΠΊ Π² Ρ†Π΅ΠΏΠΈΒ 

Π° ΠšΠŸΠ” источника Ρ€Π°Π²Π΅Π½ 50Β %. МаксимальноС Π·Π½Π°Ρ‡Π΅Π½ΠΈΠ΅ ΠšΠŸΠ” источника достигаСтся ΠΏΡ€ΠΈΒ IΒ β†’Β 0, Ρ‚.Β Π΅. ΠΏΡ€ΠΈΒ RΒ β†’Β βˆž. Π’ случаС ΠΊΠΎΡ€ΠΎΡ‚ΠΊΠΎΠ³ΠΎ замыкания полСзная ΠΌΠΎΡ‰Π½ΠΎΡΡ‚ΡŒΒ PΒ =Β 0 ΠΈ вся ΠΌΠΎΡ‰Π½ΠΎΡΡ‚ΡŒ выдСляСтся Π²Π½ΡƒΡ‚Ρ€ΠΈ источника, Ρ‡Ρ‚ΠΎ ΠΌΠΎΠΆΠ΅Ρ‚ привСсти ΠΊ Π΅Π³ΠΎ ΠΏΠ΅Ρ€Π΅Π³Ρ€Π΅Π²Ρƒ ΠΈ Ρ€Π°Π·Ρ€ΡƒΡˆΠ΅Π½ΠΈΡŽ. ΠšΠŸΠ” источника ΠΏΡ€ΠΈ этом обращаСтся Π² Π½ΡƒΠ»ΡŒ.

Π˜ΡΡ‚ΠΎΡ‡Π½ΠΈΠΊ: http://fizmatinf.blogspot.com/2013/05/33.html

Π—Π°ΠΊΠΎΠ½ Ома для ΠΏΠΎΠ»Π½ΠΎΠΉ элСктричСской Ρ†Π΅ΠΏΠΈ: Ρ€Π°Π·Π½ΠΈΡ†Π° с Π²Ρ‹Ρ€Π°ΠΆΠ΅Π½ΠΈΠ΅ΠΌ для участка ΠΊΠΎΠ½Ρ‚ΡƒΡ€Π°, ΠΎΠΏΡ€Π΅Π΄Π΅Π»Π΅Π½ΠΈΠ΅, Ρ„ΠΎΡ€ΠΌΡƒΠ»Π°

Π‘Ρ€Π΅Π΄ΠΈ извСстных ΡˆΠΈΡ€ΠΎΠΊΠΎΠΉ общСствСнности физичСских Ρ„ΠΎΡ€ΠΌΡƒΠ» Π»ΠΈΠ΄ΠΈΡ€ΡƒΠ΅Ρ‚ E=mc2. По популярности с Π½Π΅ΠΉ ΠΌΠΎΠΆΠ΅Ρ‚ ΡΠΎΠΏΠ΅Ρ€Π½ΠΈΡ‡Π°Ρ‚ΡŒ Ρ‚ΠΎΠ»ΡŒΠΊΠΎ U=IR. Π­Ρ‚ΠΎ простоС Π²Ρ‹Ρ€Π°ΠΆΠ΅Π½ΠΈΠ΅ ΠΈΠΌΠ΅Π΅Ρ‚ Ρ„ΡƒΠ½Π΄Π°ΠΌΠ΅Π½Ρ‚Π°Π»ΡŒΠ½ΠΎΠ΅ Π·Π½Π°Ρ‡Π΅Π½ΠΈΠ΅ для элСктротСхники ΠΈ описываСт матСматичСски ΡΠΎΠΎΡ‚Π½ΠΎΡˆΠ΅Π½ΠΈΠ΅ ΠΌΠ΅ΠΆΠ΄Ρƒ ΠΏΠ°Ρ€Π°ΠΌΠ΅Ρ‚Ρ€Π°ΠΌΠΈ участка элСктричСской Ρ†Π΅ΠΏΠΈ. МСнСС извСстСн Π·Π°ΠΊΠΎΠ½ Ома для ΠΏΠΎΠ»Π½ΠΎΠΉ Ρ†Π΅ΠΏΠΈ, ΠΊΠΎΡ‚ΠΎΡ€Ρ‹ΠΉ рассматриваСт Π½Π°Π³Ρ€ΡƒΠ·ΠΊΡƒ Π½Π΅ΠΎΡ‚Π΄Π΅Π»ΠΈΠΌΠΎ ΠΎΡ‚ источника напряТСния.

ΠžΡΠ½ΠΎΠ²Π½Ρ‹Π΅ понятия

ЭлСктричСский Ρ‚ΠΎΠΊ Ρ‚Π΅Ρ‡Ρ‘Ρ‚, ΠΊΠΎΠ³Π΄Π° Π·Π°ΠΌΠΊΠ½ΡƒΡ‚Ρ‹ΠΉ ΠΊΠΎΠ½Ρ‚ΡƒΡ€ позволяСт элСктронам ΠΏΠ΅Ρ€Π΅ΠΌΠ΅Ρ‰Π°Ρ‚ΡŒΡΡ ΠΎΡ‚ высокого ΠΏΠΎΡ‚Π΅Π½Ρ†ΠΈΠ°Π»Π° ΠΊ Π±ΠΎΠ»Π΅Π΅ Π½ΠΈΠ·ΠΊΠΎΠΌΡƒ Π² Ρ†Π΅ΠΏΠΈ.

Π˜Π½Π°Ρ‡Π΅ говоря, Ρ‚ΠΎΠΊ Ρ‚Ρ€Π΅Π±ΡƒΠ΅Ρ‚ источника элСктронов, ΠΎΠ±Π»Π°Π΄Π°ΡŽΡ‰Π΅Π³ΠΎ энСргиСй для привСдСния ΠΈΡ… Π² Π΄Π²ΠΈΠΆΠ΅Π½ΠΈΠ΅, Π° Ρ‚Π°ΠΊΠΆΠ΅ Ρ‚ΠΎΡ‡ΠΊΠΈ ΠΈΡ… возвращСния ΠΎΡ‚Ρ€ΠΈΡ†Π°Ρ‚Π΅Π»ΡŒΠ½Ρ‹Ρ… зарядов, для ΠΊΠΎΡ‚ΠΎΡ€ΠΎΠΉ Ρ…Π°Ρ€Π°ΠΊΡ‚Π΅Ρ€Π΅Π½ ΠΈΡ… Π΄Π΅Ρ„ΠΈΡ†ΠΈΡ‚.

Как физичСскоС явлСниС Ρ‚ΠΎΠΊ Π² Ρ†Π΅ΠΏΠΈ характСризуСтся трСмя Ρ„ΡƒΠ½Π΄Π°ΠΌΠ΅Π½Ρ‚Π°Π»ΡŒΠ½Ρ‹ΠΌΠΈ Π²Π΅Π»ΠΈΡ‡ΠΈΠ½Π°ΠΌΠΈ:

  • напряТСниС;
  • сила Ρ‚ΠΎΠΊΠ°;
  • сопротивлСниС ΠΏΡ€ΠΎΠ²ΠΎΠ΄Π½ΠΈΠΊΠ°, ΠΏΠΎ ΠΊΠΎΡ‚ΠΎΡ€ΠΎΠΌΡƒ двиТутся элСктроны.

Π‘ΠΈΠ»Π° ΠΈ напряТСниС

Π’ΠΎΠ»ΡŒΡ‚ β€” Π΅Π΄ΠΈΠ½ΠΈΡ†Π° измСрСния, примСняСмая для элСктричСской Ρ€Π°Π·Π½ΠΈΡ†Ρ‹ ΠΏΠΎΡ‚Π΅Π½Ρ†ΠΈΠ°Π»ΠΎΠ², самого ΠΏΠΎΡ‚Π΅Π½Ρ†ΠΈΠ°Π»Π° ΠΈ элСктродвиТущСй силы. Π’Π΅Ρ€ΠΌΠΈΠ½ напряТСниС (U) относится ΠΊ элСктричСской разности ΠΏΠΎΡ‚Π΅Π½Ρ†ΠΈΠ°Π»ΠΎΠ² ΠΌΠ΅ΠΆΠ΄Ρƒ Ρ‚ΠΎΡ‡ΠΊΠ°ΠΌΠΈ. Π›ΡŽΠ±Ρ‹Π΅ статичСскиС заряды ΠΈΠΌΠ΅ΡŽΡ‚ Π·Π½Π°Ρ‡Π΅Π½ΠΈΠ΅ Π² Π’ΠΎΠ»ΡŒΡ‚Π°Ρ…, Π° Π²Π΅Π»ΠΈΡ‡ΠΈΠ½Π° ΠΈΡ… разности опрСдСляСтся ΠΊΠ°ΠΊ U.

НапряТСниС ΠΌΠΎΠΆΠ½ΠΎ ΠΏΡ€ΠΎΠΈΠ»Π»ΡŽΡΡ‚Ρ€ΠΈΡ€ΠΎΠ²Π°Ρ‚ΡŒ Ρ‚Π°ΠΊΠΈΠΌ явлСниСм, ΠΊΠ°ΠΊ Π΄Π°Π²Π»Π΅Π½ΠΈΠ΅, ΠΈΠ»ΠΈ ΠΊΠ°ΠΊ Ρ€Π°Π·Π½ΠΎΡΡ‚ΡŒ ΠΏΠΎΡ‚Π΅Π½Ρ†ΠΈΠ°Π»ΡŒΠ½ΠΎΠΉ энСргии ΠΏΡ€Π΅Π΄ΠΌΠ΅Ρ‚ΠΎΠ² ΠΏΠΎΠ΄ воздСйствиСм Π³Ρ€Π°Π²ΠΈΡ‚Π°Ρ†ΠΈΠΈ.

Для Ρ‚ΠΎΠ³ΠΎ Ρ‡Ρ‚ΠΎΠ±Ρ‹ ΡΠΎΠ·Π΄Π°Ρ‚ΡŒ этот дисбаланс, Π½ΡƒΠΆΠ½ΠΎ Π·Π°Ρ‚Ρ€Π°Ρ‚ΠΈΡ‚ΡŒ ΠΏΡ€Π΅Π΄Π²Π°Ρ€ΠΈΡ‚Π΅Π»ΡŒΠ½ΠΎ ΡΠ½Π΅Ρ€Π³ΠΈΡŽ, которая ΠΈ Π±ΡƒΠ΄Π΅Ρ‚ Ρ€Π΅Π°Π»ΠΈΠ·ΠΎΠ²Π°Π½Π° Π² Π΄Π²ΠΈΠΆΠ΅Π½ΠΈΠΈ ΠΏΡ€ΠΈ ΡΠΎΠΎΡ‚Π²Π΅Ρ‚ΡΡ‚Π²ΡƒΡŽΡ‰ΠΈΡ… ΠΎΠ±ΡΡ‚ΠΎΡΡ‚Π΅Π»ΡŒΡΡ‚Π²Π°Ρ….

НапримСр, Π² ΠΏΠ°Π΄Π΅Π½ΠΈΠΈ Π³Ρ€ΡƒΠ·Π° с высоты рСализуСтся Ρ€Π°Π±ΠΎΡ‚Π° ΠΏΠΎ Π΅Π³ΠΎ ΠΏΠΎΠ΄ΡŠΡ‘ΠΌΡƒ, Π² Π³Π°Π»ΡŒΠ²Π°Π½ΠΈΡ‡Π΅ΡΠΊΠΈΡ… батарСях Ρ€Π°Π·Π½ΠΎΡΡ‚ΡŒ ΠΏΠΎΡ‚Π΅Π½Ρ†ΠΈΠ°Π»ΠΎΠ² Π½Π° ΠΊΠ»Π΅ΠΌΠΌΠ°Ρ… образуСтся Π·Π° счёт прСобразования химичСской энСргии, Π² Π³Π΅Π½Π΅Ρ€Π°Ρ‚ΠΎΡ€Π°Ρ… β€” Π² Ρ€Π΅Π·ΡƒΠ»ΡŒΡ‚Π°Ρ‚Π΅ воздСйствия элСктромагнитного поля.

Π‘ΠΎΠΏΡ€ΠΎΡ‚ΠΈΠ²Π»Π΅Π½ΠΈΠ΅ ΠΏΡ€ΠΎΠ²ΠΎΠ΄Π½ΠΈΠΊΠΎΠ²

НСзависимо ΠΎΡ‚ Ρ‚ΠΎΠ³ΠΎ, насколько Ρ…ΠΎΡ€ΠΎΡˆ ΠΎΠ±Ρ‹Ρ‡Π½Ρ‹ΠΉ ΠΏΡ€ΠΎΠ²ΠΎΠ΄Π½ΠΈΠΊ, ΠΎΠ½ Π½ΠΈΠΊΠΎΠ³Π΄Π° Π½Π΅ Π±ΡƒΠ΄Π΅Ρ‚ ΠΏΡ€ΠΎΠΏΡƒΡΠΊΠ°Ρ‚ΡŒ сквозь сСбя элСктроны Π±Π΅Π· ΠΊΠ°ΠΊΠΎΠ³ΠΎ-Π»ΠΈΠ±ΠΎ сопротивлСния ΠΈΡ… двиТСнию. МоТно Ρ€Π°ΡΡΠΌΠ°Ρ‚Ρ€ΠΈΠ²Π°Ρ‚ΡŒ сопротивлСниС ΠΊΠ°ΠΊ Π°Π½Π°Π»ΠΎΠ³ мСханичСского трСния, хотя это сравнСниС Π½Π΅ Π±ΡƒΠ΄Π΅Ρ‚ ΡΠΎΠ²Π΅Ρ€ΡˆΠ΅Π½Π½Ρ‹ΠΌ.

Когда Ρ‚ΠΎΠΊ ΠΏΡ€ΠΎΡ‚Π΅ΠΊΠ°Π΅Ρ‚ Ρ‡Π΅Ρ€Π΅Π· ΠΏΡ€ΠΎΠ²ΠΎΠ΄Π½ΠΈΠΊ, нСкоторая Ρ€Π°Π·Π½ΠΎΡΡ‚ΡŒ ΠΏΠΎΡ‚Π΅Π½Ρ†ΠΈΠ°Π»ΠΎΠ² прСобразуСтся Π² Ρ‚Π΅ΠΏΠ»ΠΎ, поэтому всСгда Π±ΡƒΠ΄Π΅Ρ‚ ΠΏΠ°Π΄Π΅Π½ΠΈΠ΅ напряТСния Π½Π° рСзисторС.

ЭлСктричСскиС ΠΎΠ±ΠΎΠ³Ρ€Π΅Π²Π°Ρ‚Π΅Π»ΠΈ, Ρ„Π΅Π½Ρ‹ ΠΈ Π΄Ρ€ΡƒΠ³ΠΈΠ΅ ΠΏΠΎΠ΄ΠΎΠ±Π½Ρ‹Π΅ устройства ΠΏΡ€Π΅Π΄Π½Π°Π·Π½Π°Ρ‡Π΅Π½Ρ‹ ΠΈΡΠΊΠ»ΡŽΡ‡ΠΈΡ‚Π΅Π»ΡŒΠ½ΠΎ для рассСивания элСктричСской энСргии Π² Π²ΠΈΠ΄Π΅ Ρ‚Π΅ΠΏΠ»Π°.

Π£ΠΏΡ€ΠΎΡ‰Ρ‘Π½Π½ΠΎ сопротивлСниС (обозначаСтся ΠΊΠ°ΠΊ R) являСтся ΠΌΠ΅Ρ€ΠΎΠΉ Ρ‚ΠΎΠ³ΠΎ, насколько ΠΏΠΎΡ‚ΠΎΠΊ элСктронов тормозится Π² Ρ†Π΅ΠΏΠΈ. Оно измСряСтся Π² ΠžΠΌΠ°Ρ…. ΠŸΡ€ΠΎΠ²ΠΎΠ΄ΠΈΠΌΠΎΡΡ‚ΡŒ рСзистора ΠΈΠ»ΠΈ Π΄Ρ€ΡƒΠ³ΠΎΠ³ΠΎ элСмСнта опрСдСляСтся двумя свойствами:

Π€ΠΎΡ€ΠΌΠ° ΠΈΠΌΠ΅Π΅Ρ‚ ваТнСйшСС Π·Π½Π°Ρ‡Π΅Π½ΠΈΠ΅, это ΠΎΡ‡Π΅Π²ΠΈΠ΄Π½ΠΎ Π½Π° гидравличСской Π°Π½Π°Π»ΠΎΠ³ΠΈΠΈ: ΠΏΡ€ΠΎΡ‚ΠΎΠ»ΠΊΠ½ΡƒΡ‚ΡŒ Π²ΠΎΠ΄Ρƒ Ρ‡Π΅Ρ€Π΅Π· Π΄Π»ΠΈΠ½Π½ΡƒΡŽ ΠΈ ΡƒΠ·ΠΊΡƒΡŽ Ρ‚Ρ€ΡƒΠ±Ρƒ Π³ΠΎΡ€Π°Π·Π΄ΠΎ тяТСлСС, Ρ‡Π΅ΠΌ Ρ‡Π΅Ρ€Π΅Π· ΠΊΠΎΡ€ΠΎΡ‚ΠΊΡƒΡŽ ΠΈ ΡˆΠΈΡ€ΠΎΠΊΡƒΡŽ. ΠœΠ°Ρ‚Π΅Ρ€ΠΈΠ°Π»Ρ‹ ΠΈΠ³Ρ€Π°ΡŽΡ‚ ΠΎΠΏΡ€Π΅Π΄Π΅Π»ΡΡŽΡ‰ΡƒΡŽ Ρ€ΠΎΠ»ΡŒ.

НапримСр, элСктроны ΠΌΠΎΠ³ΡƒΡ‚ свободно ΠΏΠ΅Ρ€Π΅ΠΌΠ΅Ρ‰Π°Ρ‚ΡŒΡΡ Π² ΠΌΠ΅Π΄Π½ΠΎΠΌ ΠΏΡ€ΠΎΠ²ΠΎΠ΄Π΅, Π½ΠΎ Π½Π΅ способны ΠΏΡ€ΠΎΡ‚Π΅ΠΊΠ°Ρ‚ΡŒ Π²ΠΎΠΎΠ±Ρ‰Π΅ Ρ‡Π΅Ρ€Π΅Π· Ρ‚Π°ΠΊΠΈΠ΅ изоляторы, ΠΊΠ°ΠΊ ΠΊΠ°ΡƒΡ‡ΡƒΠΊ, нСзависимо ΠΎΡ‚ ΠΈΡ… Ρ„ΠΎΡ€ΠΌΡ‹.

ΠšΡ€ΠΎΠΌΠ΅ Π³Π΅ΠΎΠΌΠ΅Ρ‚Ρ€ΠΈΠΈ ΠΈ ΠΌΠ°Ρ‚Π΅Ρ€ΠΈΠ°Π»Π°, ΡΡƒΡ‰Π΅ΡΡ‚Π²ΡƒΡŽΡ‚ ΠΈ Π΄Ρ€ΡƒΠ³ΠΈΠ΅ Ρ„Π°ΠΊΡ‚ΠΎΡ€Ρ‹, Π²Π»ΠΈΡΡŽΡ‰ΠΈΠ΅ Π½Π° ΠΏΡ€ΠΎΠ²ΠΎΠ΄ΠΈΠΌΠΎΡΡ‚ΡŒ.

Π—Π°ΠΊΠΎΠ½ для участка Ρ†Π΅ΠΏΠΈ

БущСствуСт Ρ„ΡƒΠ½Π΄Π°ΠΌΠ΅Π½Ρ‚Π°Π»ΡŒΠ½Π°Ρ связь ΠΌΠ΅ΠΆΠ΄Ρƒ напряТСниСм, Ρ‚ΠΎΠΊΠΎΠΌ ΠΈ ΠΏΡ€ΠΎΠ²ΠΎΠ΄ΠΈΠΌΠΎΡΡ‚ΡŒΡŽ. Π­Ρ‚ΠΎ Π·Π½Π°ΠΌΠ΅Π½ΠΈΡ‚ΠΎΠ΅ ΡƒΡ€Π°Π²Π½Π΅Π½ΠΈΠ΅ называСтся Π·Π°ΠΊΠΎΠ½ΠΎΠΌ Ома, ΠΈ Π΅Π³ΠΎ ΠΌΠΎΠΆΠ½ΠΎ ΠΎΡ‚ΠΎΠ±Ρ€Π°Π·ΠΈΡ‚ΡŒ трСмя эквивалСнтными способами:

ΠŸΡ€ΠΈΠ²Π΅Π΄Ρ‘Π½Π½Ρ‹Π΅ матСматичСскиС ΡΠΎΠΎΡ‚Π½ΠΎΡˆΠ΅Π½ΠΈΡ β€” основа для элСктротСхники ΠΈ элСктроники. Π—Π°ΠΊΠΎΠ½ Π±Ρ‹Π» Π½Π°Π·Π²Π°Π½ Π² Ρ‡Π΅ΡΡ‚ΡŒ Π½Π΅ΠΌΠ΅Ρ†ΠΊΠΎΠ³ΠΎ Ρ„ΠΈΠ·ΠΈΠΊΠ° Π“Π΅ΠΎΡ€Π³Π° Π‘ΠΈΠΌΠΎΠ½Π° Ома, ΠΊΠΎΡ‚ΠΎΡ€Ρ‹ΠΉ Π² ΠΌΠΎΠ½ΠΎΠ³Ρ€Π°Ρ„ΠΈΠΈ, ΠΎΠΏΡƒΠ±Π»ΠΈΠΊΠΎΠ²Π°Π½Π½ΠΎΠΉ Π² 1827 Π³., описал измСрСния ΠΏΡ€ΠΈΠ»ΠΎΠΆΠ΅Π½Π½ΠΎΠ³ΠΎ напряТСния ΠΈ Ρ‚ΠΎΠΊΠ° с ΠΏΠΎΠΌΠΎΡ‰ΡŒΡŽ простых элСктричСских Ρ†Π΅ΠΏΠ΅ΠΉ, состоящих ΠΈΠ· ΠΏΡ€ΠΎΠ²ΠΎΠ΄ΠΎΠ² Ρ€Π°Π·Π»ΠΈΡ‡Π½ΠΎΠΉ Π΄Π»ΠΈΠ½Ρ‹.

Π˜ΡΡΠ»Π΅Π΄ΠΎΠ²Π°Ρ‚Π΅Π»ΡŒ объяснил свои ΡΠΊΡΠΏΠ΅Ρ€ΠΈΠΌΠ΅Π½Ρ‚Π°Π»ΡŒΠ½Ρ‹Π΅ Ρ€Π΅Π·ΡƒΠ»ΡŒΡ‚Π°Ρ‚Ρ‹ нСсколько слоТнСС, Ρ‡Π΅ΠΌ ΠΎΡ‚Ρ€Π°ΠΆΠ΅Π½ΠΎ Π² ΠΏΡ€ΠΈΠ²Π΅Π΄Ρ‘Π½Π½Ρ‹Ρ… уравнСниях, извСстных Π² соврСмСнной Ρ„ΠΈΠ·ΠΈΠΊΠ΅ ΠΊΠ°ΠΊ Π½Π΅ΠΏΠΎΠ»Π½Ρ‹ΠΉ Π·Π°ΠΊΠΎΠ½ Ома. Для Ρ‚ΠΎΠ³ΠΎ Ρ‡Ρ‚ΠΎΠ±Ρ‹ ΡΡ„ΠΎΡ€ΠΌΡƒΠ»ΠΈΡ€ΠΎΠ²Π°Ρ‚ΡŒ Π·Π°ΠΊΠΎΠ½ Ома для ΠΏΠΎΠ»Π½ΠΎΠΉ элСктричСской Ρ†Π΅ΠΏΠΈ, Π½Π΅ΠΎΠ±Ρ…ΠΎΠ΄ΠΈΠΌΠΎ ΠΎΠΏΠ΅Ρ€ΠΈΡ€ΠΎΠ²Π°Ρ‚ΡŒ понятиями Π²Π½ΡƒΡ‚Ρ€Π΅Π½Π½Π΅Π³ΠΎ сопротивлСния источника Ρ‚ΠΎΠΊΠ° ΠΈ элСктродвиТущСй силы.

ЭлСктродвиТущая сила

Под воздСйствиСм Π²Π½Π΅ΡˆΠ½ΠΈΡ… сил заряды Π΄Π²ΠΈΠ³Π°ΡŽΡ‚ΡΡ Π²Π½ΡƒΡ‚Ρ€ΠΈ источника Ρ‚ΠΎΠΊΠ° ΠΏΡ€ΠΎΡ‚ΠΈΠ² сил элСктростатичСского поля. Π­Ρ‚ΠΎ позволяСт ΠΏΠΎΠ΄Π΄Π΅Ρ€ΠΆΠΈΠ²Π°Ρ‚ΡŒ постоянный Ρ‚ΠΎΠΊ Π² Π·Π°ΠΌΠΊΠ½ΡƒΡ‚ΠΎΠΌ ΠΊΠΎΠ½Ρ‚ΡƒΡ€Π΅ Π΄ΠΎ Ρ‚Π΅Ρ… ΠΏΠΎΡ€, ΠΏΠΎΠΊΠ° Ρ€Π°Π±ΠΎΡ‚Π°ΡŽΡ‚ внСшниС силы. ЀизичСская Π²Π΅Π»ΠΈΡ‡ΠΈΠ½Π°, равная ΠΎΡ‚Π½ΠΎΡˆΠ΅Π½ΠΈΡŽ Π·Π°Ρ‚Ρ€Π°Ρ‡Π΅Π½Π½ΠΎΠΉ энСргии сторонних сил Π½Π° ΠΏΠ΅Ρ€Π΅ΠΌΠ΅Ρ‰Π΅Π½ΠΈΠ΅ заряда, называСтся элСктродвиТущСй силой источника Ρ‚ΠΎΠΊΠ°. Она ΠΌΠΎΠΆΠ΅Ρ‚ Π±Ρ‹Ρ‚ΡŒ прСдставлСна Ρ„ΠΎΡ€ΠΌΡƒΠ»ΠΎΠΉ β„° = A/q. Π’ этом Π²Ρ‹Ρ€Π°ΠΆΠ΅Π½ΠΈΠΈ:

  • β„° β€” Π­Π”Π‘ Π² Π²ΠΎΠ»ΡŒΡ‚Π°Ρ…;
  • A β€” Ρ€Π°Π±ΠΎΡ‚Π° Π² дТоулях;
  • q β€” заряд Π² ΠΊΡƒΠ»ΠΎΠ½Π°Ρ….

По Π°Π½Π°Π»ΠΎΠ³ΠΈΠΈ с Π·Π°ΠΌΠΊΠ½ΡƒΡ‚ΠΎΠΉ гидравличСской систСмой ΠΈ насосом, элСктричСскиС заряды ΠΏΡ€ΠΎΡ‚Π΅ΠΊΠ°ΡŽΡ‚ Π½Π΅ΠΏΡ€Π΅Ρ€Ρ‹Π²Π½ΠΎ ΠΏΠΎ всСму ΠΊΠΎΠ½Ρ‚ΡƒΡ€Ρƒ, ΠΈ привСсти ΠΈΡ… Π² Π΄Π²ΠΈΠΆΠ΅Π½ΠΈΠ΅ ΠΌΠΎΠ³ΡƒΡ‚ Ρ‚ΠΎΠ»ΡŒΠΊΠΎ внСшниС силы.

Π­Ρ‚ΠΎ ΠΎΠ·Π½Π°Ρ‡Π°Π΅Ρ‚, Ρ‡Ρ‚ΠΎ Ρ€Π°Π±ΠΎΡ‚Ρƒ ΠΏΠΎ ΠΏΠ΅Ρ€Π΅ΠΌΠ΅Ρ‰Π΅Π½ΠΈΡŽ заряда Π»ΡŽΠ±Ρ‹ΠΌ источником ΠΌΠΎΠΆΠ½ΠΎ Ρ€Π°ΡΡΠΌΠ°Ρ‚Ρ€ΠΈΠ²Π°Ρ‚ΡŒ ΠΊΠ°ΠΊ Π­Π”Π‘ ΠΈ ΠΈΠ·ΠΌΠ΅Ρ€ΡΡ‚ΡŒ Π² Π²ΠΎΠ»ΡŒΡ‚Π°Ρ….

Π’Ρ‹Π²ΠΎΠ΄ ΠΎ ΠΌΠΎΠ΄Π΅Π»ΠΈ Ρ†Π΅ΠΏΠΈ с источником, Π² ΠΊΠΎΡ‚ΠΎΡ€ΠΎΠΉ ΠΏΡ€ΠΎΡ‚Π΅ΠΊΠ°Π΅Ρ‚ Ρ‚ΠΎΠΊ, ΠΊΠ°ΠΊ ΠΎ Π·Π°ΠΌΠΊΠ½ΡƒΡ‚ΠΎΠΌ ΠΊΠΎΠ½Ρ‚ΡƒΡ€Π΅ ΠΊΡ€Π°ΠΉΠ½Π΅ Π²Π°ΠΆΠ΅Π½ для понимания Π·Π°ΠΊΠΎΠ½Π° Ома для ΠΏΠΎΠ»Π½ΠΎΠ³ΠΎ участка Ρ†Π΅ΠΏΠΈ.

Π’Π½Π΅ΡˆΠ½Π΅Π΅ ΠΈ Π²Π½ΡƒΡ‚Ρ€Π΅Π½Π½Π΅Π΅ сопротивлСниС

ВсС Π±Π°Ρ‚Π°Ρ€Π΅ΠΈ ΠΈ Π³Π΅Π½Π΅Ρ€Π°Ρ‚ΠΎΡ€Ρ‹ ΠΎΠ±Π»Π°Π΄Π°ΡŽΡ‚ Π²Π½ΡƒΡ‚Ρ€Π΅Π½Π½ΠΈΠΌ сопротивлСниСм: элСктроды ΠΈ элСктролиты Π½Π΅Π°Π±ΡΠΎΠ»ΡŽΡ‚Π½Ρ‹Π΅ ΠΏΡ€ΠΎΠ²ΠΎΠ΄Π½ΠΈΠΊΠΈ, ΠΊΠ°ΠΊ ΠΈ ΠΏΡ€ΠΎΠ²ΠΎΠ΄Π° ΠΎΠ±ΠΌΠΎΡ‚ΠΎΠΊ элСктричСских машин.

Оно ΠΌΠΎΠΆΠ΅Ρ‚ Π²Π°Ρ€ΡŒΠΈΡ€ΠΎΠ²Π°Ρ‚ΡŒΡΡ ΠΎΡ‚ тысячных Π΄ΠΎΠ»Π΅ΠΉ ΠΎΠΌΠ° Π΄ΠΎ Π½Π΅ΡΠΊΠΎΠ»ΡŒΠΊΠΈΡ… ΠΎΠΌ. Π­Ρ‚ΠΎΡ‚ физичСский ΠΏΠ°Ρ€Π°ΠΌΠ΅Ρ‚Ρ€ являСтся ΠΊΠ»ΡŽΡ‡Π΅Π²Ρ‹ΠΌ Π² Π·Π°ΠΊΠΎΠ½Π΅ Ома для всСй Ρ†Π΅ΠΏΠΈ.

Π’ качСствС матСматичСских ΠΌΠΎΠ΄Π΅Π»Π΅ΠΉ для рассмотрСния ΠΈ ΠΈΠ»Π»ΡŽΡΡ‚Ρ€Π°Ρ†ΠΈΠΈ элСктричСских процСссов Ρ€Π°Π·Π»ΠΈΡ‡Π°ΡŽΡ‚:

  • Π˜Π΄Π΅Π°Π»ΡŒΠ½Ρ‹ΠΉ источник Ρ‚ΠΎΠΊΠ° (ИИВ). Π“Π΅Π½Π΅Ρ€ΠΈΡ€ΡƒΠ΅Ρ‚ элСктричСский Ρ‚ΠΎΠΊ, Π½Π΅ зависящий ΠΎΡ‚ ΠΈΠ·ΠΌΠ΅Π½Π΅Π½ΠΈΠΉ напряТСния. Π’Π½ΡƒΡ‚Ρ€Π΅Π½Π½Π΅Π΅ сопротивлСниС ИИВ бСсконСчно, напряТСниС ΠΏΠΎΠ»Π½ΠΎΡΡ‚ΡŒΡŽ опрСдСляСтся ΠΏΠΎΠ΄ΠΊΠ»ΡŽΡ‡Ρ‘Π½Π½ΠΎΠΉ схСмой. Ни ΠΎΠ΄ΠΈΠ½ физичСский источник Ρ‚ΠΎΠΊΠ° Π½Π΅ ΠΌΠΎΠΆΠ΅Ρ‚ Ρ€Π°Π±ΠΎΡ‚Π°Ρ‚ΡŒ Π² условиях Ρ€Π°Π·Ρ€Ρ‹Π²Π° Ρ†Π΅ΠΏΠΈ, поэтому ИИВ Π²ΠΎΠ·ΠΌΠΎΠΆΠ΅Π½ Ρ‚ΠΎΠ»ΡŒΠΊΠΎ Π² качСствС абстрактной ΠΌΠΎΠ΄Π΅Π»ΠΈ.
  • Π˜Π΄Π΅Π°Π»ΡŒΠ½Ρ‹ΠΉ источник напряТСния (ИИН). ΠŸΡ€Π΅Π΄ΡΡ‚Π°Π²Π»ΡΠ΅Ρ‚ собой устройство, ΠΏΠΎΠ΄Π΄Π΅Ρ€ΠΆΠΈΠ²Π°ΡŽΡ‰Π΅Π΅ постоянноС Π²Ρ‹Ρ…ΠΎΠ΄Π½ΠΎΠ΅ напряТСниС нСзависимо ΠΎΡ‚ Ρ‚ΠΎΠΊΠ°, ΠΏΡ€ΠΎΡ‚Π΅ΠΊΠ°ΡŽΡ‰Π΅Π³ΠΎ ΠΏΠΎ ΠΊΠΎΠ½Ρ‚ΡƒΡ€Ρƒ. ΠžΠ±Π»Π°Π΄Π°Π΅Ρ‚ Π½ΡƒΠ»Π΅Π²Ρ‹ΠΌ Π²Π½ΡƒΡ‚Ρ€Π΅Π½Π½ΠΈΠΌ сопротивлСниСм. ИИН ΡƒΠ΄ΠΎΠ±Π΅Π½ для модСлирования практичСских источников, ΠΊΠΎΡ‚ΠΎΡ€Ρ‹Π΅ ΠΌΠΎΠΆΠ½ΠΎ ΠΏΡ€Π΅Π΄ΡΡ‚Π°Π²ΠΈΡ‚ΡŒ ΠΊΠ°ΠΊ ИНН с ΠΏΠΎΠ΄ΠΊΠ»ΡŽΡ‡Ρ‘Π½Π½Ρ‹ΠΌ рСзистором.

Π’Π½ΡƒΡ‚Ρ€Π΅Π½Π½Π΅ сопротивлСниС источника элСктричСской энСргии являСтся Ρ„Π°ΠΊΡ‚ΠΎΡ€ΠΎΠΌ обСспСчСния максимальной мощности для ΠΏΠΎΠ΄ΠΊΠ»ΡŽΡ‡Ρ‘Π½Π½ΠΎΠΉ ΠΊ Π½Π΅ΠΌΡƒ Π½Π°Π³Ρ€ΡƒΠ·ΠΊΠΈ. НаиболСС эффСктивный пСрСнос энСргии происходит, ΠΊΠΎΠ³Π΄Π° внСшнСС сопротивлСниС Π·Π½Π°Ρ‡ΠΈΡ‚Π΅Π»ΡŒΠ½ΠΎ ΠΏΡ€Π΅Π²Ρ‹ΡˆΠ°Π΅Ρ‚ Π²Π½ΡƒΡ‚Ρ€Π΅Π½Π½Π΅Π΅ Ρƒ источника.

НапримСр, свинцово-кислотныС аккумуляторы автомобиля, благодаря Π½ΠΈΠ·ΠΊΠΎΠΌΡƒ Π²Π½ΡƒΡ‚Ρ€Π΅Π½Π½Π΅ΠΌΡƒ ΡΠΎΠΏΡ€ΠΎΡ‚ΠΈΠ²Π»Π΅Π½ΠΈΡŽ, способны ΡΠΎΠ·Π΄Π°Π²Π°Ρ‚ΡŒ ΠΎΡ‚Π½ΠΎΡΠΈΡ‚Π΅Π»ΡŒΠ½ΠΎ высокиС Ρ‚ΠΎΠΊΠΈ ΠΏΡ€ΠΈ ΡΡ€Π°Π²Π½ΠΈΡ‚Π΅Π»ΡŒΠ½ΠΎ Π½ΠΈΠ·ΠΊΠΎΠΌ напряТСнии. Однако, с Π΄Ρ€ΡƒΠ³ΠΎΠΉ стороны, Π²Ρ‹ΡΠΎΠΊΠΎΠ²ΠΎΠ»ΡŒΡ‚Π½Ρ‹Π΅ источники Π΄ΠΎΠ»ΠΆΠ½Ρ‹ ΠΈΠΌΠ΅Ρ‚ΡŒ высокоС Π²Π½ΡƒΡ‚Ρ€Π΅Π½Π½Π΅ сопротивлСниС, Ρ‡Ρ‚ΠΎΠ±Ρ‹ ΠΎΠ³Ρ€Π°Π½ΠΈΡ‡ΠΈΡ‚ΡŒ количСство Ρ‚ΠΎΠΊΠ°, ΠΏΡ€ΠΎΡ‚Π΅ΠΊΠ°ΡŽΡ‰Π΅Π³ΠΎ Π² Ρ€Π΅Π·ΡƒΠ»ΡŒΡ‚Π°Ρ‚Π΅ случайного ΠΊΠΎΡ€ΠΎΡ‚ΠΊΠΎΠ³ΠΎ замыкания.

ΠŸΠΎΠ»Π½Ρ‹ΠΉ Π·Π°ΠΊΠΎΠ½

Π’ этом ΡƒΡ€Π°Π²Π½Π΅Π½ΠΈΠΈ прСдусмотрСно Π½Π°Π»ΠΈΡ‡ΠΈΠ΅ Π² ΠΊΠΎΠ½Ρ‚ΡƒΡ€Π΅ источника питания элСктродвиТущСй силы β„° c Π²Π½ΡƒΡ‚Ρ€Π΅Π½Π½ΠΈΠΌ сопротивлСниСм r. ΠŸΠΎΡΠΊΠΎΠ»ΡŒΠΊΡƒ Π­Π”Π‘ β€” практичСски Π²Π΅Π»ΠΈΡ‡ΠΈΠ½Π°, зависящая ΠΎΡ‚ Π²Π½Π΅ΡˆΠ½ΠΈΡ… сил, Ρ‚ΠΎ физичСский смысл ΠΈΠΌΠ΅Π΅Ρ‚ расчёт силы Ρ‚ΠΎΠΊΠ° для ΠΏΠΎΠ»Π½ΠΎΠΉ Ρ†Π΅ΠΏΠΈ ΠΏΡ€ΠΈ ΠΏΠΎΠΌΠΎΡ‰ΠΈ выраТСния: I=β„°/(R+r).

Π’Π°ΠΊΠΈΠΌ ΠΎΠ±Ρ€Π°Π·ΠΎΠΌ, ΠΏΠΎΠ»Π½Ρ‹ΠΉ постулат Ома гласит ΠΎ зависимости силы Ρ‚ΠΎΠΊΠ° Π² Π·Π°ΠΌΠΊΠ½ΡƒΡ‚ΠΎΠΌ ΠΊΠΎΠ½Ρ‚ΡƒΡ€Π΅ ΠΎΡ‚ Π²Π½ΡƒΡ‚Ρ€Π΅Π½Π½Π΅Π³ΠΎ сопротивлСния Π΅Π³ΠΎ источника, Ρ‚ΠΎ Π΅ΡΡ‚ΡŒ ΡƒΡ‡ΠΈΡ‚Ρ‹Π²Π°Π΅Ρ‚ сопротивлСниС элСктролита ΠΈ элСктродов для Π³Π°Π»ΡŒΠ²Π°Π½ΠΈΡ‡Π΅ΡΠΊΠΈΡ… элСмСнтов ΠΈ ΠΏΡ€ΠΎΠ²ΠΎΠ΄ΠΈΠΌΠΎΡΡ‚ΡŒ ΠΎΠ±ΠΌΠΎΡ‚ΠΎΠΊ Π³Π΅Π½Π΅Ρ€Π°Ρ‚ΠΎΡ€ΠΎΠ². ОсновноС практичСскоС ΠΏΡ€ΠΈΠΌΠ΅Π½Π΅Π½ΠΈΠ΅ β€” расчёт силы Ρ‚ΠΎΠΊΠ° Π² Π»ΠΈΠ½Π΅ΠΉΠ½Ρ‹Ρ… элСктричСских цСпях DC, ΠΎΠΏΡ€Π΅Π΄Π΅Π»Π΅Π½ΠΈΠ΅ мощности ΠΈ импСданса Π»ΡŽΠ±Ρ‹Ρ… элСмСнтов Ρ†Π΅ΠΏΠΈ.

Π˜ΡΡ‚ΠΎΡ‡Π½ΠΈΠΊ: https://rusenergetics.ru/novichku/smysl-polnogo-zakona-oma

Π—Π°ΠΊΠΎΠ½ ДТоуля-Π›Π΅Π½Ρ†Π°: ΠΎΠΏΡ€Π΅Π΄Π΅Π»Π΅Π½ΠΈΠ΅, Ρ„ΠΎΡ€ΠΌΡƒΠ»Π°, ΠΏΡ€ΠΈΠΌΠ΅Π½Π΅Π½ΠΈΠ΅

ΠœΡ‹ Π΅ΠΆΠ΅Π΄Π½Π΅Π²Π½ΠΎ ΠΏΠΎΠ»ΡŒΠ·ΡƒΠ΅ΠΌΡΡ ΡΠ»Π΅ΠΊΡ‚Ρ€ΠΎΠ½Π°Π³Ρ€Π΅Π²Π°Ρ‚Π΅Π»ΡŒΠ½Ρ‹ΠΌΠΈ ΠΏΡ€ΠΈΠ±ΠΎΡ€Π°ΠΌΠΈ, Π½Π΅ Π·Π°Π΄ΡƒΠΌΡ‹Π²Π°ΡΡΡŒ, ΠΎΡ‚ΠΊΡƒΠ΄Π° бСрётся Ρ‚Π΅ΠΏΠ»ΠΎ. РазумССтся, Π²Ρ‹ Π·Π½Π°Π΅Ρ‚Π΅, Ρ‡Ρ‚ΠΎ Ρ‚Π΅ΠΏΠ»ΠΎΠ²ΡƒΡŽ ΡΠ½Π΅Ρ€Π³ΠΈΡŽ Π²Ρ‹Ρ€Π°Π±Π°Ρ‚Ρ‹Π²Π°Π΅Ρ‚ элСктричСство. Но ΠΊΠ°ΠΊ это происходит, Π° Ρ‚Π΅ΠΌ Π±ΠΎΠ»Π΅Π΅, ΠΊΠ°ΠΊ ΠΎΡ†Π΅Π½ΠΈΡ‚ΡŒ количСство выдСляСмого Ρ‚Π΅ΠΏΠ»Π°, Π·Π½Π°ΡŽΡ‚ Π½Π΅ всС. На Π΄Π°Π½Π½Ρ‹ΠΉ вопрос ΠΎΡ‚Π²Π΅Ρ‡Π°Π΅Ρ‚ Π·Π°ΠΊΠΎΠ½ ДТоуля-Π›Π΅Π½Ρ†Π°, ΠΎΠ±Π½Π°Ρ€ΠΎΠ΄ΠΎΠ²Π°Π½Π½Ρ‹ΠΉ Π² ΠΏΠΎΠ·Π°ΠΏΡ€ΠΎΡˆΠ»ΠΎΠΌ столСтии.

Π’ 1841 Π³ΠΎΠ΄Ρƒ усилия английского Ρ„ΠΈΠ·ΠΈΠΊΠ° ДТоуля, Π° Π² 1842 Π³. исслСдования русского ΡƒΡ‡Ρ‘Π½ΠΎΠ³ΠΎ Π›Π΅Π½Ρ†Π° ΡƒΠ²Π΅Π½Ρ‡Π°Π»ΠΈΡΡŒ ΠΎΡ‚ΠΊΡ€Ρ‹Ρ‚ΠΈΠ΅ΠΌ Π·Π°ΠΊΠΎΠ½Π°, ΠΏΡ€ΠΈΠΌΠ΅Π½Π΅Π½ΠΈΠ΅ ΠΊΠΎΡ‚ΠΎΡ€ΠΎΠ³ΠΎ позволяСт количСствСнно ΠΎΡ†Π΅Π½ΠΈΡ‚ΡŒ Ρ€Π΅Π·ΡƒΠ»ΡŒΡ‚Π°Ρ‚Ρ‹ Ρ‚Π΅ΠΏΠ»ΠΎΠ²ΠΎΠ³ΠΎ дСйствия элСктричСского Ρ‚ΠΎΠΊΠ° [ 1 ]. Π‘ Ρ‚Π΅Ρ… ΠΏΠΎΡ€ ΠΈΠ·ΠΎΠ±Ρ€Π΅Ρ‚Π΅Π½ΠΎ мноТСство ΠΏΡ€ΠΈΠ±ΠΎΡ€ΠΎΠ², Π² основС ΠΊΠΎΡ‚ΠΎΡ€Ρ‹Ρ… Π»Π΅ΠΆΠΈΡ‚ Ρ‚Π΅ΠΏΠ»ΠΎΠ²ΠΎΠ΅ дСйствиС Ρ‚ΠΎΠΊΠ°. НСкоторыС ΠΈΠ· Π½ΠΈΡ…, ΠΈΠ·ΠΎΠ±Ρ€Π°ΠΆΠ΅Π½Ρ‹ Π½Π° рис. 1.

Рис. 1. Π’Π΅ΠΏΠ»ΠΎΠ²Ρ‹Π΅ ΠΏΡ€ΠΈΠ±ΠΎΡ€Ρ‹

ΠžΠΏΡ€Π΅Π΄Π΅Π»Π΅Π½ΠΈΠ΅ ΠΈ Ρ„ΠΎΡ€ΠΌΡƒΠ»Π°

Π’Π΅ΠΏΠ»ΠΎΠ²ΠΎΠΉ Π·Π°ΠΊΠΎΠ½ ΠΌΠΎΠΆΠ½ΠΎ ΡΡ„ΠΎΡ€ΠΌΡƒΠ»ΠΈΡ€ΠΎΠ²Π°Ρ‚ΡŒ ΠΈ Π·Π°ΠΏΠΈΡΠ°Ρ‚ΡŒ Π² ΡΠ»Π΅Π΄ΡƒΡŽΡ‰Π΅ΠΉ Ρ€Π΅Π΄Π°ΠΊΡ†ΠΈΠΈ: Β«ΠšΠΎΠ»ΠΈΡ‡Π΅ΡΡ‚Π²ΠΎ Ρ‚Π΅ΠΏΠ»Π°, Π²Ρ‹Ρ€Π°Π±ΠΎΡ‚Π°Π½Π½ΠΎΠ³ΠΎ Ρ‚ΠΎΠΊΠΎΠΌ, прямо ΠΏΡ€ΠΎΠΏΠΎΡ€Ρ†ΠΈΠΎΠ½Π°Π»ΡŒΠ½ΠΎ ΠΊΠ²Π°Π΄Ρ€Π°Ρ‚Ρƒ ΠΏΡ€ΠΈΠ»ΠΎΠΆΠ΅Π½Π½ΠΎΠ³ΠΎ ΠΊ Π΄Π°Π½Π½ΠΎΠΌΡƒ участку Ρ†Π΅ΠΏΠΈ Ρ‚ΠΎΠΊΠ°, сопротивлСния ΠΏΡ€ΠΎΠ²ΠΎΠ΄Π½ΠΈΠΊΠ° ΠΈ ΠΏΡ€ΠΎΠΌΠ΅ΠΆΡƒΡ‚ΠΊΠ° Π²Ρ€Π΅ΠΌΠ΅Π½ΠΈ, Π² Ρ‚Π΅Ρ‡Π΅Π½ΠΈΠ΅ ΠΊΠΎΡ‚ΠΎΡ€ΠΎΠ³ΠΎ элСктричСство дСйствовало Π½Π° ΠΏΡ€ΠΎΠ²ΠΎΠ΄Π½ΠΈΠΊΒ».

ΠžΠ±ΠΎΠ·Π½Π°Ρ‡ΠΈΠΌ символом Q количСство выдСляСмого Ρ‚Π΅ΠΏΠ»Π°, Π° символами I, R ΠΈ Ξ”t – силу Ρ‚ΠΎΠΊΠ°, сопротивлСниС ΠΈ ΠΏΡ€ΠΎΠΌΠ΅ΠΆΡƒΡ‚ΠΎΠΊ Π²Ρ€Π΅ΠΌΠ΅Π½ΠΈ, соотвСтствСнно. Π’ΠΎΠ³Π΄Π° Ρ„ΠΎΡ€ΠΌΡƒΠ»Π° Π·Π°ΠΊΠΎΠ½Π° ДТоуля-Π›Π΅Π½Ρ†Π° Π±ΡƒΠ΄Π΅Ρ‚ ΠΈΠΌΠ΅Ρ‚ΡŒ Π²ΠΈΠ΄: Q = I2*R*Ξ”t

Богласно Π·Π°ΠΊΠΎΠ½Π°ΠΌ Ома I=U/R, ΠΎΡ‚ΠΊΡƒΠ΄Π° R = U/I. ΠŸΠΎΠ΄ΡΡ‚Π°Π²Π»ΡΡ выраТСния Π² Ρ„ΠΎΡ€ΠΌΡƒΠ»Ρƒ ДТоуля-Π›Π΅Π½Ρ†Π° ΠΏΠΎΠ»ΡƒΡ‡ΠΈΠΌ: Q = U2/R * Ξ”t β‡’ Q = U*I*Ξ”t.

Π’Ρ‹Π²Π΅Π΄Π΅Π½Π½Ρ‹Π΅ Π½Π°ΠΌΠΈ Ρ„ΠΎΡ€ΠΌΡƒΠ»Ρ‹ – Ρ€Π°Π·Π»ΠΈΡ‡Π½Ρ‹Π΅ Ρ„ΠΎΡ€ΠΌΡ‹ записи Π·Π°ΠΊΠΎΠ½Π° ДТоуля-Π›Π΅Π½Ρ†Π°. Зная Ρ‚Π°ΠΊΠΈΠ΅ ΠΏΠ°Ρ€Π°ΠΌΠ΅Ρ‚Ρ€Ρ‹ ΠΊΠ°ΠΊ напряТСниС ΠΈΠ»ΠΈ силу Ρ‚ΠΎΠΊΠ°, ΠΌΠΎΠΆΠ½ΠΎ Π»Π΅Π³ΠΊΠΎ Ρ€Π°ΡΡΡ‡ΠΈΡ‚Π°Ρ‚ΡŒ количСство Ρ‚Π΅ΠΏΠ»Π°, выдСляСмого Π½Π° участкС Ρ†Π΅ΠΏΠΈ, ΠΎΠ±Π»Π°Π΄Π°ΡŽΡ‰Π΅ΠΌ сопротивлСниСм R.

Π”ΠΈΡ„Ρ„Π΅Ρ€Π΅Π½Ρ†ΠΈΠ°Π»ΡŒΠ½Π°Ρ Ρ„ΠΎΡ€ΠΌΠ°

Π§Ρ‚ΠΎΠ±Ρ‹ ΠΏΠ΅Ρ€Π΅ΠΉΡ‚ΠΈ ΠΊ Π΄ΠΈΡ„Ρ„Π΅Ρ€Π΅Π½Ρ†ΠΈΠ°Π»ΡŒΠ½ΠΎΠΉ Ρ„ΠΎΡ€ΠΌΠ΅ Π·Π°ΠΊΠΎΠ½Π°, ΠΏΡ€ΠΎΠ°Π½Π°Π»ΠΈΠ·ΠΈΡ€ΡƒΠ΅ΠΌ ΡƒΡ‚Π²Π΅Ρ€ΠΆΠ΄Π΅Π½ΠΈΠ΅ ДТоуля-Π›Π΅Π½Ρ†Π° ΠΏΡ€ΠΈΠΌΠ΅Π½ΠΈΡ‚Π΅Π»ΡŒΠ½ΠΎ ΠΊ элСктронной Ρ‚Π΅ΠΎΡ€ΠΈΠΈ.

ΠŸΡ€ΠΈΡ€Π°Ρ‰Π΅Π½ΠΈΠ΅ энСргии элСктрона Ξ”W Π·Π° счёт Ρ€Π°Π±ΠΎΡ‚Ρ‹ элСктричСских сил поля Ρ€Π°Π²Π½ΠΎ разности энСргий элСктрона Π² ΠΊΠΎΠ½Ρ†Π΅ ΠΏΡ€ΠΎΠ±Π΅Π³Π° (m/2)*(u=Ο…max)2ΠΈ Π² Π½Π°Ρ‡Π°Π»Π΅ ΠΏΡ€ΠΎΠ±Π΅Π³Π° (mu2)/2 , Ρ‚ΠΎ Π΅ΡΡ‚ΡŒ

Π—Π΄Π΅ΡΡŒ u – ΡΠΊΠΎΡ€ΠΎΡΡ‚ΡŒ хаотичСского Π΄Π²ΠΈΠΆΠ΅Π½ΠΈΠ΅ (вСкторная Π²Π΅Π»ΠΈΡ‡ΠΈΠ½Π°), Π° Ο…max – максимальная ΡΠΊΠΎΡ€ΠΎΡΡ‚ΡŒ элСктричСского заряда Π² Π΄Π°Π½Π½Ρ‹ΠΉ ΠΌΠΎΠΌΠ΅Π½Ρ‚ Π²Ρ€Π΅ΠΌΠ΅Π½ΠΈ.

ΠŸΠΎΡΠΊΠΎΠ»ΡŒΠΊΡƒ установлСно, Ρ‡Ρ‚ΠΎ ΡΠΊΠΎΡ€ΠΎΡΡ‚ΡŒ хаотичСского двиТСния с ΠΎΠ΄ΠΈΠ½Π°ΠΊΠΎΠ²ΠΎΠΉ Π²Π΅Ρ€ΠΎΡΡ‚Π½ΠΎΡΡ‚ΡŒΡŽ совпадаСт с максимальной (ΠΏΠΎ Π½Π°ΠΏΡ€Π°Π²Π»Π΅Π½ΠΈΡŽ ΠΈ Π² ΠΏΡ€ΠΎΡ‚ΠΈΠ²ΠΎΠΏΠΎΠ»ΠΎΠΆΠ½ΠΎΠΌ Π½Π°ΠΏΡ€Π°Π²Π»Π΅Π½ΠΈΠΈ), Ρ‚ΠΎ Π²Ρ‹Ρ€Π°ΠΆΠ΅Π½ΠΈΠ΅ 2*u*Ο…max Π² срСднСм Ρ€Π°Π²Π½ΠΎ Π½ΡƒΠ»ΡŽ. Π’ΠΎΠ³Π΄Π° полная энСргия, Π²Ρ‹Π΄Π΅Π»ΡΡŽΡ‰Π°ΡΡΡ ΠΏΡ€ΠΈ столкновСниях элСктронов с Π°Ρ‚ΠΎΠΌΠ°ΠΌΠΈ, ΠΎΠ±Ρ€Π°Π·ΡƒΡŽΡ‰ΠΈΠΌΠΈ ΡƒΠ·Π»Ρ‹ кристалличСской Ρ€Π΅ΡˆΡ‘Ρ‚ΠΊΠΈ, составляСт:

Π­Ρ‚ΠΎ ΠΈ Π΅ΡΡ‚ΡŒ Π·Π°ΠΊΠΎΠ½ ДТоуля-Π›Π΅Π½Ρ†Π°, записанный Π² Π΄ΠΈΡ„Ρ„Π΅Ρ€Π΅Π½Ρ†ΠΈΠ°Π»ΡŒΠ½ΠΎΠΉ Ρ„ΠΎΡ€ΠΌΠ΅. Π—Π΄Π΅ΡΡŒ Ξ³ – ΡΠΎΠ³Π»Π°ΡΡƒΡŽΡ‰ΠΈΠΉ коэффициСнт,Β  E – Π½Π°ΠΏΡ€ΡΠΆΡ‘Π½Π½ΠΎΡΡ‚ΡŒ поля.

Π˜Π½Ρ‚Π΅Π³Ρ€Π°Π»ΡŒΠ½Π°Ρ Ρ„ΠΎΡ€ΠΌΠ°

ΠŸΡ€Π΅Π΄ΠΏΠΎΠ»ΠΎΠΆΠΈΠΌ, Ρ‡Ρ‚ΠΎ ΠΏΡ€ΠΎΠ²ΠΎΠ΄Π½ΠΈΠΊ ΠΈΠΌΠ΅Π΅Ρ‚ Ρ†ΠΈΠ»ΠΈΠ½Π΄Ρ€ΠΈΡ‡Π΅ΡΠΊΡƒΡŽ Ρ„ΠΎΡ€ΠΌΡƒ с сСчСниСм S. ΠŸΡƒΡΡ‚ΡŒ Π΄Π»ΠΈΠ½Π° этого ΠΏΡ€ΠΎΠ²ΠΎΠ΄Π½ΠΈΠΊΠ° составляСт l. Π’ΠΎΠ³Π΄Π° ΠΌΠΎΡ‰Π½ΠΎΡΡ‚ΡŒ P, выдСляСмая Π² ΠΎΠ±ΡŠΡ‘ΠΌΠ΅ V= lS составляСт:

Π³Π΄Π΅R – ΠΏΠΎΠ»Π½ΠΎΠ΅ сопротивлСниС ΠΏΡ€ΠΎΠ²ΠΎΠ΄Π½ΠΈΠΊΠ°.

Учитывая, Ρ‡Ρ‚ΠΎU = IΓ—R, ΠΈΠ· послСднСй Ρ„ΠΎΡ€ΠΌΡƒΠ»Ρ‹ ΠΈΠΌΠ΅Π΅ΠΌ:

  • P = UΓ—I;
  • P = I2R;
  • P = U2/R.

Если Π²Π΅Π»ΠΈΡ‡ΠΈΠ½Π° Ρ‚ΠΎΠΊΠ° со Π²Ρ€Π΅ΠΌΠ΅Π½Π΅ΠΌ мСняСтся, Ρ‚ΠΎ количСство Ρ‚Π΅ΠΏΠ»ΠΎΡ‚Ρ‹ вычисляСтся ΠΏΠΎ Ρ„ΠΎΡ€ΠΌΡƒΠ»Π΅:

Π”Π°Π½Π½ΠΎΠ΅ Π²Ρ‹Ρ€Π°ΠΆΠ΅Π½ΠΈΠ΅, Π° Ρ‚Π°ΠΊΠΆΠ΅ Π²Ρ‹ΡˆΠ΅ΠΏΠ΅Ρ€Π΅Ρ‡ΠΈΡΠ»Π΅Π½Π½Ρ‹Π΅ Ρ„ΠΎΡ€ΠΌΡƒΠ»Ρ‹, ΠΊΠΎΡ‚ΠΎΡ€Ρ‹Π΅ ΠΌΠΎΠΆΠ½ΠΎ ΠΏΠ΅Ρ€Π΅ΠΏΠΈΡΠ°Ρ‚ΡŒ Π² Ρ‚Π°ΠΊΠΎΠΌ ΠΆΠ΅ Π²ΠΈΠ΄Π΅, принято Π½Π°Π·Ρ‹Π²Π°Ρ‚ΡŒ ΠΈΠ½Ρ‚Π΅Π³Ρ€Π°Π»ΡŒΠ½ΠΎΠΉ Ρ„ΠΎΡ€ΠΌΠΎΠΉ Π·Π°ΠΊΠΎΠ½Π° ДТоуля-Π›Π΅Π½Ρ†Π°.

Π€ΠΎΡ€ΠΌΡƒΠ»Ρ‹ ΠΎΡ‡Π΅Π½ΡŒ ΡƒΠ΄ΠΎΠ±Π½Ρ‹ ΠΏΡ€ΠΈ вычислСнии мощности Ρ‚ΠΎΠΊΠ° Π² Π½Π°Π³Ρ€Π΅Π²Π°Ρ‚Π΅Π»ΡŒΠ½Ρ‹Ρ… элСмСнтах. Если извСстно сопротивлСниС Ρ‚Π°ΠΊΠΎΠ³ΠΎ элСмСнта, Ρ‚ΠΎ зная напряТСниС Π±Ρ‹Ρ‚ΠΎΠ²ΠΎΠΉ сСти Π»Π΅Π³ΠΊΠΎ ΠΎΠΏΡ€Π΅Π΄Π΅Π»ΠΈΡ‚ΡŒ ΠΌΠΎΡ‰Π½ΠΎΡΡ‚ΡŒ ΠΏΡ€ΠΈΠ±ΠΎΡ€Π°, Π½Π°ΠΏΡ€ΠΈΠΌΠ΅Ρ€, элСктрочайника ΠΈΠ»ΠΈ паяльника.

ЀизичСский смысл

Вспомним, ΠΊΠ°ΠΊ элСктричСский Ρ‚ΠΎΠΊ ΠΏΡ€ΠΎΡ‚Π΅ΠΊΠ°Π΅Ρ‚ ΠΏΠΎ мСталличСскому ΠΏΡ€ΠΎΠ²ΠΎΠ΄Π½ΠΈΠΊΡƒ.

Как Ρ‚ΠΎΠ»ΡŒΠΊΠΎ элСктричСская Ρ†Π΅ΠΏΡŒ замкнётся, Ρ‚ΠΎ ΠΏΠΎΠ΄ дСйствиСм Π­Π”Π‘ Π΄Π²ΠΈΠΆΠ΅Π½ΠΈΠ΅ свободных элСктронов упорядочиваСтся, ΠΈ ΠΎΠ½ΠΈ ΡƒΡΡ‚Ρ€Π΅ΠΌΠ»ΡΡŽΡ‚ΡΡ ΠΊ ΠΏΠΎΠ»ΠΎΠΆΠΈΡ‚Π΅Π»ΡŒΠ½ΠΎΠΌΡƒ ΠΏΠΎΠ»ΡŽΡΡƒ источника питания.

Однако Π½Π° ΠΈΡ… ΠΏΡƒΡ‚ΠΈ Π²ΡΡ‚Ρ€Π΅Ρ‡Π°ΡŽΡ‚ΡΡ стройныС ряды кристалличСских Ρ€Π΅ΡˆΡ‘Ρ‚ΠΎΠΊ, Π°Ρ‚ΠΎΠΌΡ‹ ΠΊΠΎΡ‚ΠΎΡ€Ρ‹Ρ… ΡΠΎΠ·Π΄Π°ΡŽΡ‚ прСпятствия упорядочСнному двиТСнию, Ρ‚ΠΎ Π΅ΡΡ‚ΡŒ ΠΎΠΊΠ°Π·Ρ‹Π²Π°ΡŽΡ‚ сопротивлСниС.

На ΠΏΡ€Π΅ΠΎΠ΄ΠΎΠ»Π΅Π½ΠΈΠ΅ сопротивлСния ΡƒΡ…ΠΎΠ΄ΠΈΡ‚ Ρ‡Π°ΡΡ‚ΡŒ энСргии двиТущихся элСктронов. Π’ соотвСтствии с Ρ„ΡƒΠ½Π΄Π°ΠΌΠ΅Π½Ρ‚Π°Π»ΡŒΠ½Ρ‹ΠΌ Π·Π°ΠΊΠΎΠ½ΠΎΠΌ сохранСния энСргии, ΠΎΠ½Π° Π½Π΅ ΠΌΠΎΠΆΠ΅Ρ‚ бСсслСдно ΠΈΡΡ‡Π΅Π·Π½ΡƒΡ‚ΡŒ. Она-Ρ‚ΠΎ ΠΈ прСвращаСтся Π² Ρ‚Π΅ΠΏΠ»ΠΎ, Π²Ρ‹Π·Ρ‹Π²Π°ΡŽΡ‰Π΅Π΅ Π½Π°Π³Ρ€Π΅Π²Π°Π½ΠΈΠ΅ ΠΏΡ€ΠΎΠ²ΠΎΠ΄Π½ΠΈΠΊΠ°. НакапливаСмая тСпловая энСргия излучаСтся Π² ΠΎΠΊΡ€ΡƒΠΆΠ°ΡŽΡ‰Π΅Π΅ пространство ΠΈΠ»ΠΈ Π½Π°Π³Ρ€Π΅Π²Π°Π΅Ρ‚ Π΄Ρ€ΡƒΠ³ΠΈΠ΅ ΠΏΡ€Π΅Π΄ΠΌΠ΅Ρ‚Ρ‹, ΡΠΎΠΏΡ€ΠΈΠΊΠ°ΡΠ°ΡŽΡ‰ΠΈΠ΅ΡΡ с ΠΏΡ€ΠΎΠ²ΠΎΠ΄Π½ΠΈΠΊΠΎΠΌ.

На рисункС 2 ΠΈΠ·ΠΎΠ±Ρ€Π°ΠΆΡ‘Π½Π° схСма ΠΎΠΏΡ‹Ρ‚Π°, Π΄Π΅ΠΌΠΎΠ½ΡΡ‚Ρ€ΠΈΡ€ΡƒΡŽΡ‰Π΅Π³ΠΎ Π·Π°ΠΊΠΎΠ½ Ρ‚Π΅ΠΏΠ»ΠΎΠ²ΠΎΠ³ΠΎ дСйствия Ρ‚ΠΎΠΊΠ°, Ρ€Π°Π·ΠΎΠ³Ρ€Π΅Π²Π°ΡŽΡ‰Π΅Π³ΠΎ участок ΠΏΡ€ΠΎΠ²ΠΎΠ΄Π° Π² элСктричСской Ρ†Π΅ΠΏΠΈ.

Рис. 2. Π’Π΅ΠΏΠ»ΠΎΠ²ΠΎΠ΅ дСйствиС Ρ‚ΠΎΠΊΠ°

Π―Π²Π»Π΅Π½ΠΈΠ΅ нагрСвания ΠΏΡ€ΠΎΠ²ΠΎΠ΄Π½ΠΈΠΊΠΎΠ² Π±Ρ‹Π»ΠΎ извСстно практичСски с ΠΌΠΎΠΌΠ΅Π½Ρ‚Π° получСния элСктротока, Π½ΠΎ исслСдоватСли Π½Π΅ ΠΌΠΎΠ³Π»ΠΈ Ρ‚ΠΎΠ³Π΄Π° ΠΎΠ±ΡŠΡΡΠ½ΠΈΡ‚ΡŒ Π΅Π³ΠΎ ΠΏΡ€ΠΈΡ€ΠΎΠ΄Ρƒ, ΠΈ Ρ‚Π΅ΠΌ Π±ΠΎΠ»Π΅Π΅, ΠΏΡ€Π΅Π΄Π»ΠΎΠΆΠΈΡ‚ΡŒ способ ΠΎΡ†Π΅Π½ΠΊΠΈ количСства выдСляСмого Ρ‚Π΅ΠΏΠ»Π°. Π­Ρ‚Ρƒ ΠΏΡ€ΠΎΠ±Π»Π΅ΠΌΡƒ Ρ€Π΅ΡˆΠ°Π΅Ρ‚ Π·Π°ΠΊΠΎΠ½Β  ДТоуля-Π›Π΅Π½Ρ†Π°, ΠΊΠΎΡ‚ΠΎΡ€Ρ‹ΠΌ ΠΌΡ‹ ΠΏΠΎΠ»ΡŒΠ·ΡƒΠ΅ΠΌΡΡ ΠΏΠΎ сСгодняшний дСнь.

ΠŸΡ€Π°ΠΊΡ‚ΠΈΡ‡Π΅ΡΠΊΠ°Ρ польза Π·Π°ΠΊΠΎΠ½Π° ДТоуля-Π›Π΅Π½Ρ†Π°

ΠŸΡ€ΠΈ
сильном Π½Π°Π³Ρ€Π΅Π²Π°Π½ΠΈΠΈ ΠΌΠΎΠΆΠ½ΠΎ Π½Π°Π±Π»ΡŽΠ΄Π°Ρ‚ΡŒ ΠΈΠ·Π»ΡƒΡ‡Π΅Π½ΠΈΠ΅ Π²ΠΈΠ΄ΠΈΠΌΠΎΠ³ΠΎ спСктра свСта, Ρ‡Ρ‚ΠΎ
происходит, Π½Π°ΠΏΡ€ΠΈΠΌΠ΅Ρ€, Π² Π»Π°ΠΌΠΏΠΎΡ‡ΠΊΠ΅ накаливания. Π‘Π»Π°Π±ΠΎ Π½Π°Π³Ρ€Π΅Ρ‚Ρ‹Π΅ Ρ‚Π΅Π»Π° Ρ‚ΠΎΠΆΠ΅ ΠΈΠ·Π»ΡƒΡ‡Π°ΡŽΡ‚
Ρ‚Π΅ΠΏΠ»ΠΎΠ²ΡƒΡŽ ΡΠ½Π΅Ρ€Π³ΠΈΡŽ, Π½ΠΎ Π² Π΄ΠΈΠ°ΠΏΠ°Π·ΠΎΠ½Π΅ инфракрасного излучСния, ΠΊΠΎΡ‚ΠΎΡ€ΠΎΠ³ΠΎ ΠΌΡ‹ Π½Π΅ Π²ΠΈΠ΄ΠΈΠΌ,
Π½ΠΎ ΠΌΠΎΠΆΠ΅ΠΌ ΠΎΡ‰ΡƒΡ‚ΠΈΡ‚ΡŒ своими Ρ‚Π΅ΠΏΠ»ΠΎΠ²Ρ‹ΠΌΠΈ Ρ€Π΅Ρ†Π΅ΠΏΡ‚ΠΎΡ€Π°ΠΌΠΈ.

Π”ΠΎΠΏΡƒΡΠΊΠ°Ρ‚ΡŒ сильноС Π½Π°Π³Ρ€Π΅Π²Π°Π½ΠΈΠ΅ ΠΏΡ€ΠΎΠ²ΠΎΠ΄Π½ΠΈΠΊΠΎΠ² нСльзя, Ρ‚Π°ΠΊ ΠΊΠ°ΠΊ чрСзмСрная Ρ‚Π΅ΠΌΠΏΠ΅Ρ€Π°Ρ‚ΡƒΡ€Π° Ρ€Π°Π·Ρ€ΡƒΡˆΠ°Π΅Ρ‚ структуру ΠΌΠ΅Ρ‚Π°Π»Π»Π°, ΠΏΡ€ΠΎΡ‰Π΅ говоря – ΠΏΠ»Π°Π²ΠΈΡ‚ Π΅Π³ΠΎ. Π­Ρ‚ΠΎ ΠΌΠΎΠΆΠ΅Ρ‚ привСсти ΠΊ Π²Ρ‹Π²ΠΎΠ΄Ρƒ ΠΈΠ· строя элСктрооборудования, Π° Ρ‚Π°ΠΊΠΆΠ΅ ΡΡ‚Π°Ρ‚ΡŒ ΠΏΡ€ΠΈΡ‡ΠΈΠ½ΠΎΠΉ ΠΏΠΎΠΆΠ°Ρ€Π°. Для Ρ‚ΠΎΠ³ΠΎ, Ρ‡Ρ‚ΠΎΠ±Ρ‹ Π½Π΅ Π΄ΠΎΠΏΡƒΡΡ‚ΠΈΡ‚ΡŒ критичСских ΠΏΠ°Ρ€Π°ΠΌΠ΅Ρ‚Ρ€ΠΎΠ² нагрСвания Π½Π΅ΠΎΠ±Ρ…ΠΎΠ΄ΠΈΠΌΠΎ Π΄Π΅Π»Π°Ρ‚ΡŒ расчёты Ρ‚Π΅ΠΏΠ»ΠΎΠ²Ρ‹Ρ… элСмСнтов, ΠΏΠΎΠ»ΡŒΠ·ΡƒΡΡΡŒ Ρ„ΠΎΡ€ΠΌΡƒΠ»Π°ΠΌΠΈ, ΠΎΠΏΠΈΡΡ‹Π²Π°ΡŽΡ‰ΠΈΠΌΠΈ Π·Π°ΠΊΠΎΠ½ ДТоуля-Π›Π΅Π½Ρ†Π°.

ΠŸΡ€ΠΎΠ°Π½Π°Π»ΠΈΠ·ΠΈΡ€ΠΎΠ²Π°Π² Π²Ρ‹Ρ€Π°ΠΆΠ΅Π½ΠΈΠ΅ U2/R убСТдаСмся, Ρ‡Ρ‚ΠΎ ΠΊΠΎΠ³Π΄Π° сопротивлСниС стрСмится ΠΊ Π½ΡƒΠ»ΡŽ, Ρ‚ΠΎ количСство Π²Ρ‹Π΄Π΅Π»Π΅Π½Π½ΠΎΠ³ΠΎ Ρ‚Π΅ΠΏΠ»Π° стрСмится ΠΊ бСсконСчности. Вакая ситуация Π²ΠΎΠ·Π½ΠΈΠΊΠ°Π΅Ρ‚ ΠΏΡ€ΠΈ ΠΊΠΎΡ€ΠΎΡ‚ΠΊΠΈΡ… замыканиях. Π’ это основная ΠΎΠΏΠ°ΡΠ½ΠΎΡΡ‚ΡŒ ΠšΠ—.

Π’ Π±ΠΎΡ€ΡŒΠ±Π΅ с ΠΊΠΎΡ€ΠΎΡ‚ΠΊΠΈΠΌΠΈ замыканиями ΠΈΡΠΏΠΎΠ»ΡŒΠ·ΡƒΡŽΡ‚:

  • автоматичСскиС Π²Ρ‹ΠΊΠ»ΡŽΡ‡Π°Ρ‚Π΅Π»ΠΈ:
  • элСктронныС Π·Π°Ρ‰ΠΈΡ‚Π½Ρ‹Π΅ Π±Π»ΠΎΠΊΠΈ;
  • ΠΏΠ»Π°Π²ΠΊΠΈΠ΅ ΠΏΡ€Π΅Π΄ΠΎΡ…Ρ€Π°Π½ΠΈΡ‚Π΅Π»ΠΈ;
  • Π΄Ρ€ΡƒΠ³ΠΈΠ΅ Π·Π°Ρ‰ΠΈΡ‚Π½Ρ‹Π΅ устройства.

ΠŸΡ€ΠΈΠΌΠ΅Π½Π΅Π½ΠΈΠ΅ ΠΈ практичСский смысл

НСпосрСдствСнноС
ΠΏΡ€Π΅Π²Ρ€Π°Ρ‰Π΅Π½ΠΈΠ΅ элСктричСства Π² Ρ‚Π΅ΠΏΠ»ΠΎΠ²ΡƒΡŽ ΡΠ½Π΅Ρ€Π³ΠΈΡŽ нСльзя Π½Π°Π·Π²Π°Ρ‚ΡŒ экономичСски
Π²Ρ‹Π³ΠΎΠ΄Π½Ρ‹ΠΌ. Однако, с Ρ‚ΠΎΡ‡ΠΊΠΈ зрСния удобства ΠΈ доступности соврСмСнного
чСловСчСства ΠΊ источникам элСктроэнСргии Ρ€Π°Π·Π»ΠΈΡ‡Π½Ρ‹Π΅ Π½Π°Π³Ρ€Π΅Π²Π°Ρ‚Π΅Π»ΡŒΠ½Ρ‹Π΅ ΠΏΡ€ΠΈΠ±ΠΎΡ€Ρ‹
ΠΏΡ€ΠΎΠ΄ΠΎΠ»ΠΆΠ°ΡŽΡ‚ массово ΠΏΡ€ΠΈΠΌΠ΅Π½ΡΡ‚ΡŒΡΡ ΠΊΠ°ΠΊ Π² Π±Ρ‹Ρ‚Ρƒ, Ρ‚Π°ΠΊ ΠΈ Π½Π° производствС.

ΠŸΠ΅Ρ€Π΅Ρ‡ΠΈΡΠ»ΠΈΠΌ Π½Π΅ΠΊΠΎΡ‚ΠΎΡ€Ρ‹Π΅ ΠΈΠ· Π½ΠΈΡ…:

  • элСктрочайники;
  • ΡƒΡ‚ΡŽΠ³ΠΈ;
  • Ρ„Π΅Π½Ρ‹;
  • Π²Π°Ρ€ΠΎΡ‡Π½Ρ‹Π΅ ΠΏΠ»ΠΈΡ‚Ρ‹;
  • паяльники;
  • сварочныС
    Π°ΠΏΠΏΠ°Ρ€Π°Ρ‚Ρ‹ ΠΈ ΠΌΠ½ΠΎΠ³ΠΎΠ΅ Π΄Ρ€ΡƒΠ³ΠΎΠ΅.

На рисункС 3 ΠΈΠ·ΠΎΠ±Ρ€Π°ΠΆΠ΅Π½Ρ‹ Π±Ρ‹Ρ‚ΠΎΠ²Ρ‹Π΅ Π½Π°Π³Ρ€Π΅Π²Π°Ρ‚Π΅Π»ΡŒΠ½Ρ‹Π΅ ΠΏΡ€ΠΈΠ±ΠΎΡ€Ρ‹, ΠΊΠΎΡ‚ΠΎΡ€Ρ‹ΠΌΠΈ ΠΌΡ‹ часто ΠΏΠΎΠ»ΡŒΠ·ΡƒΠ΅ΠΌΡΡ.

Рис. 3. Π‘Ρ‹Ρ‚ΠΎΠ²Ρ‹Π΅ Π½Π°Π³Ρ€Π΅Π²Π°Ρ‚Π΅Π»ΡŒΠ½Ρ‹Π΅ ΠΏΡ€ΠΈΠ±ΠΎΡ€Ρ‹

ИспользованиС Ρ‚Π΅ΠΏΠ»ΠΎΠ²Ρ‹Ρ… мощностСй Π² химичСской, мСталлургичСской ΠΈ Π² Π΄Ρ€ΡƒΠ³ΠΈΡ… ΠΏΡ€ΠΎΠΌΡ‹ΡˆΠ»Π΅Π½Π½Ρ‹Ρ… отраслях тСсно связно с использованиСм элСктричСской энСргии.

Π‘Π΅Π· знания физичСского Π·Π°ΠΊΠΎΠ½Π° ДТоуля-Π›Π΅Π½Ρ†Π° Π±Ρ‹Π»ΠΎ Π±Ρ‹ Π½Π΅Π²ΠΎΠ·ΠΌΠΎΠΆΠ½ΠΎ ΡΠΊΠΎΠ½ΡΡ‚Ρ€ΡƒΠΈΡ€ΠΎΠ²Π°Ρ‚ΡŒ бСзопасный Π½Π°Π³Ρ€Π΅Π²Π°Ρ‚Π΅Π»ΡŒΠ½Ρ‹ΠΉ ΠΏΡ€ΠΈΠ±ΠΎΡ€. Для этого Π½ΡƒΠΆΠ½Ρ‹ расчёты, ΠΊΠΎΡ‚ΠΎΡ€Ρ‹Π΅ Π½Π΅Π²ΠΎΠ·ΠΌΠΎΠΆΠ½ΠΎ ΡΠ΄Π΅Π»Π°Ρ‚ΡŒ Π±Π΅Π· примСнСния рассмотрСнных Π½Π°ΠΌΠΈ Ρ„ΠΎΡ€ΠΌΡƒΠ». На основС расчётов происходит Π²Ρ‹Π±ΠΎΡ€ ΠΌΠ°Ρ‚Π΅Ρ€ΠΈΠ°Π»ΠΎΠ² с Π½ΡƒΠΆΠ½Ρ‹ΠΌ ΡƒΠ΄Π΅Π»ΡŒΠ½Ρ‹ΠΌ сопротивлСниСм, Π²Π»ΠΈΡΡŽΡ‰ΠΈΠΌ Π½Π° Π½Π°Π³Ρ€Π΅Π²Π°Ρ‚Π΅Π»ΡŒΠ½ΡƒΡŽ ΡΠΏΠΎΡΠΎΠ±Π½ΠΎΡΡ‚ΡŒ устройств.

Π—Π°ΠΊΠΎΠ½ ДТоуля-Π›Π΅Π½Ρ†Π° Π±Π΅Π· прСувСличСния ΠΌΠΎΠΆΠ½ΠΎ Π½Π°Π·Π²Π°Ρ‚ΡŒ Π³Π΅Π½ΠΈΠ°Π»ΡŒΠ½Ρ‹ΠΌ. Π­Ρ‚ΠΎ ΠΎΠ΄ΠΈΠ½ ΠΈΠ· Ρ‚Π΅Ρ… Π·Π°ΠΊΠΎΠ½ΠΎΠ², ΠΊΠΎΡ‚ΠΎΡ€Ρ‹Π΅ повлияли Π½Π° Ρ€Π°Π·Π²ΠΈΡ‚ΠΈΠ΅ элСктротСхники.

Π˜ΡΡ‚ΠΎΡ‡Π½ΠΈΠΊ: https://www.asutpp.ru/zakon-dzhoulya-lentsa.html

Замкнутая ΠΈ разомкнутая элСктричСская Ρ†Π΅ΠΏΡŒ

ΠžΠΏΡ€Π΅Π΄Π΅Π»Π΅Π½ΠΈΠ΅ 1

ЭлСктричСской Ρ†Π΅ΠΏΡŒΡŽ Π½Π°Π·Ρ‹Π²Π°ΡŽΡ‚ ΡΠΎΠ²ΠΎΠΊΡƒΠΏΠ½ΠΎΡΡ‚ΡŒ Ρ€Π°Π·Π»ΠΈΡ‡Π½Ρ‹Ρ… устройств, ΠΊΠΎΡ‚ΠΎΡ€Ρ‹Π΅ соСдинСны ΠΊΠΎΠ½ΠΊΡ€Π΅Ρ‚Π½Ρ‹ΠΌ способом. Устройства Π΄ΠΎΠ»ΠΆΠ½Ρ‹ ΠΎΠ±Π΅ΡΠΏΠ΅Ρ‡ΠΈΠ²Π°Ρ‚ΡŒ ΠΏΡƒΡ‚ΡŒ для протСкания элСктричСского Ρ‚ΠΎΠΊΠ°. Π‘ΡƒΡ‰Π΅ΡΡ‚Π²ΡƒΡŽΡ‚ Ρ€Π°Π·Π»ΠΈΡ‡Π½Ρ‹Π΅ элСмСнты Ρ†Π΅ΠΏΠ΅ΠΉ, слуТащиС для мноТСства Ρ†Π΅Π»Π΅ΠΉ. Для описания Ρ†Π΅ΠΏΠ΅ΠΉ ΠΈΡΠΏΠΎΠ»ΡŒΠ·ΡƒΡŽΡ‚ ΡΠΏΠ΅Ρ†ΠΈΠ°Π»ΡŒΠ½Ρ‹Π΅ элСктричСскиС схСмы.

Π’ состав любой элСктричСской Ρ†Π΅ΠΏΠΈ входят Ρ€Π°Π·Π»ΠΈΡ‡Π½Ρ‹Π΅ элСмСнты:

  • Π˜ΡΡ‚ΠΎΡ‡Π½ΠΈΠΊ Ρ‚ΠΎΠΊΠ°. Им, Π½Π°ΠΏΡ€ΠΈΠΌΠ΅Ρ€, ΠΌΠΎΠΆΠ΅Ρ‚ Π±Ρ‹Ρ‚ΡŒ ΠΊΠ°Ρ‚ΡƒΡˆΠΊΠ° индуктивности, ΠΏΠΎ ΠΊΠΎΡ‚ΠΎΡ€ΠΎΠΉ ΠΊΠ°ΠΊΠΎΠ΅-Ρ‚ΠΎ врСмя ΡˆΡ‘Π» Ρ‚ΠΎΠΊ внСшнСго источника.
  • ΠŸΡ€ΠΎΠ²ΠΎΠ΄Π½ΠΈΠΊΠΈ;
  • Нагрузка (Π² случаС, ΠΊΠΎΠ³Π΄Π° ΠΎΠ½Π° постоянна, Π²ΠΎΠ»ΡŒΡ‚Π°ΠΌΠΏΠ΅Ρ€Π½Π°Ρ характСристичСская кривая прСдставляСт собой ΠΏΡ€ΡΠΌΡƒΡŽ линию, Π° такая Π½Π°Π³Ρ€ΡƒΠ·ΠΊΠ° зовётся Π»ΠΈΠ½Π΅ΠΉΠ½ΠΎΠΉ;
  • Устройства Π·Π°Ρ‰ΠΈΡ‚Ρ‹;
  • Устройства ΠΊΠΎΠΌΠΌΡƒΡ‚Π°Ρ†ΠΈΠΈ.

Π Π°Π·Π»ΠΈΡ‡Π°ΡŽΡ‚ Π΄Π²Π° Π²ΠΈΠ΄Π° элСмСнтов Ρ†Π΅ΠΏΠ΅ΠΉ: пассивныС ΠΈ Π°ΠΊΡ‚ΠΈΠ²Π½Ρ‹Π΅. ΠŸΠ°ΡΡΠΈΠ²Π½Ρ‹Π΅ ΠΏΡ€Π΅Π΄ΡΡ‚Π°Π²Π»ΡΡŽΡ‚ собой ΡΠΎΠ΅Π΄ΠΈΠ½ΠΈΡ‚Π΅Π»ΡŒΠ½Ρ‹Π΅ элСмСнты ΠΈ ΠΏΡ€ΠΈΠ±ΠΎΡ€Ρ‹-ΠΏΠΎΡ‚Ρ€Π΅Π±ΠΈΡ‚Π΅Π»ΠΈ элСктроэнСргии, Ρ‚Π°ΠΊΠΆΠ΅ ΠΊ пассивным элСмСнтам относятся кондСнсаторы. АктивныС элСмСнты β€” это элСктродвигатСли, Π·Π°Ρ€ΡΠΆΠ°ΡŽΡ‰ΠΈΠ΅ΡΡ аккумуляторы ΠΈ Ρ€Π°Π·Π»ΠΈΡ‡Π½Ρ‹Π΅ источники Π­Π”Π‘.

НичСго нСпонятно?

ΠŸΠΎΠΏΡ€ΠΎΠ±ΡƒΠΉ ΠΎΠ±Ρ€Π°Ρ‚ΠΈΡ‚ΡŒΡΡ Π·Π° ΠΏΠΎΠΌΠΎΡ‰ΡŒΡŽ ΠΊ прСподаватСлям

ΠžΡΠ½ΠΎΠ²Π½Ρ‹ΠΌΠΈ Π²ΠΈΠ΄Π°ΠΌΠΈ элСктричСской Ρ†Π΅ΠΏΠΈ ΡΠ²Π»ΡΡŽΡ‚ΡΡ:

  • замкнутая Ρ†Π΅ΠΏΡŒ;
  • разомкнутая Ρ†Π΅ΠΏΡŒ.

Замкнутая элСктричСская Ρ†Π΅ΠΏΡŒ

Замкнутая элСктричСская Ρ†Π΅ΠΏΡŒ прСдставляСт собой Π½Π°ΠΈΠ±ΠΎΠ»Π΅Π΅ простой Π²Π°Ρ€ΠΈΠ°Π½Ρ‚ соСдинСния. Она состоит ΠΈΠ· источника элСктроэнСргии, потрСбитСля энСргии ΠΈ ΡΠΎΠ΅Π΄ΠΈΠ½ΠΈΡ‚Π΅Π»ΡŒΠ½Ρ‹Ρ… элСмСнтов Π² Π²ΠΈΠ΄Π΅ ΠΎΠ±Ρ‹Ρ‡Π½Ρ‹Ρ… ΠΏΡ€ΠΎΠ²ΠΎΠ΄ΠΎΠ². ΠŸΡ€ΠΎΠ²ΠΎΠ΄Π° Π² Ρ†Π΅ΠΏΠΈ ΠΎΠ±ΡΠ·Π°Ρ‚Π΅Π»ΡŒΠ½ΠΎ Π΄ΠΎΠ»ΠΆΠ½Ρ‹ ΠΈΠΌΠ΅Ρ‚ΡŒ ΡΠΎΠΎΡ‚Π²Π΅Ρ‚ΡΡ‚Π²ΡƒΡŽΡ‰ΡƒΡŽ ΠΈΠ·ΠΎΠ»ΡΡ†ΠΈΡŽ.

Для обСспСчСния ΡΡ‚Π°Π±ΠΈΠ»ΡŒΠ½ΠΎΠΉ ΠΈ бСзопасной Ρ€Π°Π±ΠΎΡ‚Ρ‹ элСктричСской Ρ†Π΅ΠΏΠΈ Π΅Π΅ ΡΠ½Π°Π±ΠΆΠ°ΡŽΡ‚ Π΄ΠΎΠΏΠΎΠ»Π½ΠΈΡ‚Π΅Π»ΡŒΠ½Ρ‹ΠΌΠΈ элСмСнтами. ΠžΠ±Ρ‹Ρ‡Π½ΠΎ это Ρ€Π°Π·Π»ΠΈΡ‡Π½Ρ‹Π΅ ΡΠ»Π΅ΠΊΡ‚Ρ€ΠΎΠΈΠ·ΠΌΠ΅Ρ€ΠΈΡ‚Π΅Π»ΡŒΠ½Ρ‹Π΅ ΠΏΡ€ΠΈΠ±ΠΎΡ€Ρ‹, с ΠΏΠΎΠΌΠΎΡ‰ΡŒΡŽ ΠΊΠΎΡ‚ΠΎΡ€Ρ‹Ρ… ΠΌΠΎΠΆΠ½ΠΎ ΡƒΠ·Π½Π°Ρ‚ΡŒ Π²Π΅Π»ΠΈΡ‡ΠΈΠ½Ρƒ Ρ‚ΠΎΠΊΠΎΠ² ΠΈ напряТСния Π² систСмС, Π° Ρ‚Π°ΠΊΠΆΠ΅ ΠΎΠ±ΠΎΡ€ΡƒΠ΄ΠΎΠ²Π°Π½ΠΈΠ΅, ΠΏΡ€Π΅Π΄Π½Π°Π·Π½Π°Ρ‡Π΅Π½Π½ΠΎΠ΅ для замыкания ΠΈ размыкания Ρ†Π΅ΠΏΠΈ.

ВсС Π·Π°ΠΌΠΊΠ½ΡƒΡ‚Ρ‹Π΅ элСктричСскиС Ρ†Π΅ΠΏΠΈ дСлят Π½Π° Π΄Π²Π΅ основныС части:

  • внСшний участок Ρ†Π΅ΠΏΠΈ;
  • Π²Π½ΡƒΡ‚Ρ€Π΅Π½Π½ΠΈΠΉ участок Ρ†Π΅ΠΏΠΈ.

ΠžΠΏΡ€Π΅Π΄Π΅Π»Π΅Π½ΠΈΠ΅ 2

Π’Π½ΡƒΡ‚Ρ€Π΅Π½Π½ΠΈΠΉ участок Ρ†Π΅ΠΏΠΈ – нСпосрСдствСнно источник элСктроэнСргии Ρƒ потрСбитСля.

Π’Π½Π΅ΡˆΠ½ΠΈΠΉ участок Ρ†Π΅ΠΏΠΈ – систСма, которая состоит ΠΈΠ· ΠΎΠ΄Π½ΠΎΠ³ΠΎ ΠΈΠ»ΠΈ ΠΌΠ½ΠΎΠ³ΠΈΡ… ΠΏΠΎΡ‚Ρ€Π΅Π±ΠΈΡ‚Π΅Π»Π΅ΠΉ элСктроэнСргии, Π° Ρ‚Π°ΠΊΠΆΠ΅ ΡΠΎΠ΅Π΄ΠΈΠ½ΠΈΡ‚Π΅Π»ΡŒΠ½Ρ‹Ρ… ΠΏΡ€ΠΎΠ²ΠΎΠ΄ΠΎΠ² ΠΈ ΠΏΡ€ΠΈΠ±ΠΎΡ€ΠΎΠ². ВсС ΠΎΠ½ΠΈ Π΄ΠΎΠ»ΠΆΠ½Ρ‹ ΠΈΠΌΠ΅Ρ‚ΡŒ ΠΎΡ‚Π½ΠΎΡˆΠ΅Π½ΠΈΠ΅ ΠΊ Ρ„ΡƒΠ½ΠΊΡ†ΠΈΠΎΠ½ΠΈΡ€ΠΎΠ²Π°Π½ΠΈΡŽ Π·Π°ΠΌΠΊΠ½ΡƒΡ‚ΠΎΠΉ элСктричСской Ρ†Π΅ΠΏΠΈ.

Π—Π°ΠΊΠΎΠ½ Ома для Π·Π°ΠΌΠΊΠ½ΡƒΡ‚ΠΎΠΉ Ρ†Π΅ΠΏΠΈ

Π—Π°ΠΊΠΎΠ½ Ома для Π·Π°ΠΌΠΊΠ½ΡƒΡ‚ΠΎΠΉ Ρ†Π΅ΠΏΠΈ ΠΏΠΎΠΊΠ°Π·Ρ‹Π²Π°Π΅Ρ‚ ΠΎΠΏΡ€Π΅Π΄Π΅Π»Π΅Π½Π½ΠΎΠ΅ Π·Π½Π°Ρ‡Π΅Π½ΠΈΠ΅ Ρ‚ΠΎΠΊΠ°. Оно зависит ΠΎΡ‚ сопротивлСния источника, Π° Ρ‚Π°ΠΊΠΆΠ΅ ΠΎΡ‚ сопротивлСния Π½Π°Π³Ρ€ΡƒΠ·ΠΊΠΈ.

Π’Π΅Π»ΠΈΡ‡ΠΈΠ½Π° Ρ‚ΠΎΠΊΠ° Π² Π·Π°ΠΌΠΊΠ½ΡƒΡ‚ΠΎΠΉ Ρ†Π΅ΠΏΠΈ, которая состоит ΠΈΠ· источника Ρ†Π΅ΠΏΠΈ, Π±ΡƒΠ΄Π΅Ρ‚ Ρ€Π°Π²Π½ΡΡ‚ΡŒΡΡ ΠΎΡ‚Π½ΠΎΡˆΠ΅Π½ΠΈΡŽ элСктродвиТущСй силы источника ΠΊ суммС внСшнСго ΠΈ Π²Π½ΡƒΡ‚Ρ€Π΅Π½Π½Π΅Π³ΠΎ сопротивлСний. ΠŸΡ€ΠΈ этом источник Ρ‚ΠΎΠΊΠ° Π΄ΠΎΠ»ΠΆΠ΅Π½ ΠΎΠ±Π»Π°Π΄Π°Ρ‚ΡŒ внСшним ΠΈ Π²Π½ΡƒΡ‚Ρ€Π΅Π½Π½ΠΈΠΌ Π½Π°Π³Ρ€ΡƒΠ·ΠΎΡ‡Π½Ρ‹ΠΌ сопротивлСниСм.

Вакая Π·Π°Π²ΠΈΡΠΈΠΌΠΎΡΡ‚ΡŒ Π±Ρ‹Π»Π° установлСна ΡΠΊΡΠΏΠ΅Ρ€ΠΈΠΌΠ΅Π½Ρ‚Π°Π»ΡŒΠ½Ρ‹ΠΌ ΠΏΡƒΡ‚Π΅ΠΌ Π² Π½Π°Ρ‡Π°Π»Π΅ 19 Π²Π΅ΠΊΠ° извСстным ΡƒΡ‡Π΅Π½Ρ‹ΠΌ Π“Π΅ΠΎΡ€Π³ΠΎΠΌ Омом. Он смог ΠΎΠΏΠΈΡΠ°Ρ‚ΡŒ Ρ€Π΅Π·ΡƒΠ»ΡŒΡ‚Π°Ρ‚Ρ‹ собствСнных ΠΎΠΏΡ‹Ρ‚ΠΎΠ² Π½Π° матСматичСском ΡƒΡ€ΠΎΠ²Π½Π΅.

Π—Π°ΠΊΠΎΠ½ Ома для Π·Π°ΠΌΠΊΠ½ΡƒΡ‚ΠΎΠΉ Ρ†Π΅ΠΏΠΈ ΠΌΠΎΠΆΠ½ΠΎ Π·Π°ΠΏΠΈΡΠ°Ρ‚ΡŒ ΡΠ»Π΅Π΄ΡƒΡŽΡ‰ΠΈΠΌ ΠΎΠ±Ρ€Π°Π·ΠΎΠΌ:

$I=frac{varepsilon}{R+r}$, Π³Π΄Π΅:

  • $varepsilon$ β€” элСктродвиТущая сила источника напряТСния;
  • $R$ β€” сопротивлСниС всСх Π²Π½Π΅ΡˆΠ½ΠΈΡ… элСмСнтов Ρ†Π΅ΠΏΠΈ, Π½Π°ΠΏΡ€ΠΈΠΌΠ΅Ρ€, ΠΏΡ€ΠΎΠ²ΠΎΠ΄Π½ΠΈΠΊΠΎΠ²;
  • $r$ β€” Π²Π½ΡƒΡ‚Ρ€Π΅Π½Π½Π΅Π΅ сопротивлСниС источника напряТСния;
  • $I$ – сила Ρ‚ΠΎΠΊΠ° Π² Ρ†Π΅ΠΏΠΈ.
  • РасчСт для ΠΎΠΏΡ€Π΅Π΄Π΅Π»Π΅Π½Π½ΠΎΠ³ΠΎ сопротивлСния:
  • $varepsilon =I_1 R_1+I_1 r$
  • $varepsilon=I_2 R_2+I_2 r$
  • ПослС подстановки ΠΏΠΎΠ»ΡƒΡ‡Π΅Π½Π½Ρ‹Ρ… Π·Π½Π°Ρ‡Π΅Π½ΠΈΠΉ, Ρ„ΠΎΡ€ΠΌΡƒΠ»Π° ΠΏΡ€ΠΈΠΎΠ±Ρ€Π΅Ρ‚Π°Π΅Ρ‚ Ρ‚Π°ΠΊΠΎΠΉ Π²ΠΈΠ΄:
  • $varepsilon=frac{I_1 I_2 (R_2-R_1)}{I_2-I_1}$

ЀизичСский смысл Π·Π°ΠΊΠΎΠ½Π° Ома для Π·Π°ΠΌΠΊΠ½ΡƒΡ‚ΠΎΠΉ Ρ†Π΅ΠΏΠΈ

Π—Π°ΠΌΠΊΠ½ΡƒΡ‚ΡƒΡŽ ΡΠ»Π΅ΠΊΡ‚Ρ€ΠΈΡ‡Π΅ΡΠΊΡƒΡŽ Ρ†Π΅ΠΏΡŒ ΠΎΠ±Ρ€Π°Π·ΡƒΡŽΡ‚ ΠΏΠΎΡ‚Ρ€Π΅Π±ΠΈΡ‚Π΅Π»ΠΈ энСргии Ρ‚ΠΎΠ»ΡŒΠΊΠΎ Π² совокупности с источником Ρ‚ΠΎΠΊΠ°. ΠŸΡ€ΠΎΡ…ΠΎΠ΄ΡΡ‰ΠΈΠΉ Ρ‡Π΅Ρ€Π΅Π· потрСбитСля Ρ‚ΠΎΠΊ Ρ‚Π΅Ρ‡Π΅Ρ‚ ΠΎΠ±Ρ€Π°Ρ‚Π½ΠΎ Π½Π° Π΅Π³ΠΎ источник. ΠŸΠΎΡΡ‚ΠΎΠΌΡƒ Ρ‚ΠΎΠΊΡƒ достаСтся сопротивлСниС ΠΏΡ€ΠΎΠ²ΠΎΠ΄Π½ΠΈΠΊΠ° ΠΈ источника. Из этого складываСтся ΠΎΠ±Ρ‰Π΅Π΅ сопротивлСниС Π·Π°ΠΌΠΊΠ½ΡƒΡ‚ΠΎΠΉ Ρ†Π΅ΠΏΠΈ, ΠΏΡ€Π΅Π΄ΠΏΠΎΠ»Π°Π³Π°ΡŽΡ‰Π΅Π΅ Π½Π°Π»ΠΈΡ‡ΠΈΠ΅ Π΄Π²ΡƒΡ… основных ΠΊΠΎΠΌΠΏΠΎΠ½Π΅Π½Ρ‚ΠΎΠ²: сопротивлСния источника ΠΈ сопротивлСния потрСбитСля.

Π—Π°Π²ΠΈΡΠΈΠΌΠΎΡΡ‚ΡŒ Ρ‚ΠΎΠΊΠ° ΠΎΡ‚ элСктродвиТущСй силы источника ΠΈ сопротивлСния Ρ†Π΅ΠΏΠΈ состоит Π² ΡΠ»Π΅Π΄ΡƒΡŽΡ‰Π΅ΠΌ: ΠΏΡ€ΠΈ ΡƒΠ²Π΅Π»ΠΈΡ‡Π΅Π½ΠΈΠΈ элСктродвиТущСй силы увСличиваСтся энСргия носитСлСй зарядов. Π­Ρ‚ΠΎ ΠΎΠ·Π½Π°Ρ‡Π°Π΅Ρ‚, Ρ‡Ρ‚ΠΎ становится большС ΡΠΊΠΎΡ€ΠΎΡΡ‚ΡŒ двиТСния зарядов Π² упорядочСнном Π²ΠΈΠ΄Π΅. Если ΡƒΠ²Π΅Π»ΠΈΡ‡ΠΈΠ²Π°Ρ‚ΡŒ Ρ€Π°Π·ΠΌΠ΅Ρ€ сопротивлСния Ρ†Π΅ΠΏΠΈ, Ρ‚ΠΎ Π²Π΅Π»ΠΈΡ‡ΠΈΠ½Π° Ρ‚ΠΎΠΊΠ° Π±ΡƒΠ΄Π΅Ρ‚ ΡƒΠΌΠ΅Π½ΡŒΡˆΠ°Ρ‚ΡŒΡΡ.

ЭлСктричСский Ρ‚ΠΎΠΊ ΠΏΡ€ΠΎΡ…ΠΎΠ΄ΠΈΡ‚ нСпосрСдствСнно ΠΏΠΎ Π·Π°ΠΌΠΊΠ½ΡƒΡ‚ΠΎΠΉ Ρ†Π΅ΠΏΠΈ. НСобходимым условиСм присутствия элСктричСского Ρ‚ΠΎΠΊΠ° Π² Ρ†Π΅ΠΏΠΈ являСтся Π½Π°Π΄Π΅ΠΆΠ½ΠΎΠ΅ соСдинСниС ΠΏΡ€ΠΎΠ²ΠΎΠ΄Π½ΠΈΠΊΠ°ΠΌΠΈ источника элСктричСской энСргии с Π΅Π΅ потрСбитСлями.

Π˜ΡΡ‚ΠΎΡ‡Π½ΠΈΠΊΠΈ элСктроэнСргии для Ρ€Π°Π·Π»ΠΈΡ‡Π½ΠΎΠΉ Π°ΠΏΠΏΠ°Ρ€Π°Ρ‚ΡƒΡ€Ρ‹: Π³Π΅Π½Π΅Ρ€Π°Ρ‚ΠΎΡ€Ρ‹, аккумуляторы, Π³Π°Π»ΡŒΠ²Π°Π½ΠΈΡ‡Π΅ΡΠΊΠΈΠ΅ элСмСнты.

Π’ Ρ€Π°Π·Π»ΠΈΡ‡Π½Ρ‹Ρ… устройствах ΠΌΠΎΠ³ΡƒΡ‚ Π±Ρ‹Ρ‚ΡŒ ΠΎΠΏΡ€Π΅Π΄Π΅Π»Π΅Π½Π½Ρ‹Π΅ ΠΏΠΎΡ‚Ρ€Π΅Π±ΠΈΡ‚Π΅Π»ΠΈ элСктричСской энСргии. Π§Π°Ρ‰Π΅ всСго ΠΈΡ… ΠΏΡ€Π΅Π΄ΡΡ‚Π°Π²Π»ΡΡŽΡ‚ Π² Π²ΠΈΠ΄Π΅ Π»Π°ΠΌΠΏ ΠΈΠ»ΠΈ элСктродвигатСлСй.

Для соСдинСния источников ΠΈ ΠΏΠΎΡ‚Ρ€Π΅Π±ΠΈΡ‚Π΅Π»Π΅ΠΉ Π² Π΅Π΄ΠΈΠ½ΡƒΡŽ Ρ†Π΅ΠΏΡŒ ΠΏΡ€ΠΈΠΌΠ΅Π½ΡΡŽΡ‚ ΠΏΡ€ΠΎΠ²ΠΎΠ΄Π½ΠΈΠΊΠΈ ΠΈΠ· мСталличСских ΠΌΠ°Ρ‚Π΅Ρ€ΠΈΠ°Π»ΠΎΠ². Они ΠΌΠΎΠ³ΡƒΡ‚ Π±Ρ‹Ρ‚ΡŒ Ρ€Π°Π·Π»ΠΈΡ‡Π½ΠΎΠΉ Ρ„ΠΎΡ€ΠΌΡ‹, Π΄Π»ΠΈΠ½Ρ‹, Ρ‚ΠΎΠ»Ρ‰ΠΈΠ½Ρ‹, ΠΎΠ±Π»Π°Π΄Π°Ρ‚ΡŒ ΠΎΠΏΡ€Π΅Π΄Π΅Π»Π΅Π½Π½Ρ‹ΠΌΠΈ тСхничСскими характСристиками. Часто ΠΏΡ€ΠΈΠΌΠ΅Π½ΡΡŽΡ‚ΡΡ ΠΏΡ€ΠΎΠ²ΠΎΠ΄Π½ΠΈΠΊΠΈ, ΠΊΠΎΡ‚ΠΎΡ€Ρ‹Π΅ ΠΈΠ·ΠΎΠ»ΠΈΡ€ΠΎΠ²Π°Π½Ρ‹ Π΄Ρ€ΡƒΠ³ ΠΎΡ‚ Π΄Ρ€ΡƒΠ³Π°.

Для возникновСния Ρ‚ΠΎΠΊΠ° Π½ΡƒΠΆΠ½ΠΎ ΡΠΎΠ΅Π΄ΠΈΠ½ΠΈΡ‚ΡŒ Π΄Π²Π΅ Ρ‚ΠΎΡ‡ΠΊΠΈ. Одна ΠΈΠ· Ρ‚ΠΎΡ‡Π΅ΠΊ Π΄ΠΎΠ»ΠΆΠ½Π° ΠΈΠΌΠ΅Ρ‚ΡŒ ΠΈΠ·Π±Ρ‹Ρ‚ΠΎΠΊ элСктронов ΠΏΠΎ ΠΎΡ‚Π½ΠΎΡˆΠ΅Π½ΠΈΡŽ ΠΊΠΎ Π²Ρ‚ΠΎΡ€ΠΎΠΉ Ρ‚ΠΎΡ‡ΠΊΠ΅. БпСциалисты Π½Π°Π·Ρ‹Π²Π°ΡŽΡ‚ это дСйствиС созданиСм разности ΠΏΠΎΡ‚Π΅Π½Ρ†ΠΈΠ°Π»ΠΎΠ² ΠΌΠ΅ΠΆΠ΄Ρƒ Ρ‚ΠΎΡ‡ΠΊΠ°ΠΌΠΈ. Π˜ΡΡ‚ΠΎΡ‡Π½ΠΈΠΊ Ρ‚ΠΎΠΊΠ° слуТит основным элСмСнтом для создания разности ΠΏΠΎΡ‚Π΅Π½Ρ†ΠΈΠ°Π»ΠΎΠ² Π² элСктричСской Ρ†Π΅ΠΏΠΈ.

Π›ΡŽΠ±ΠΎΠΉ ΠΏΠΎΡ‚Ρ€Π΅Π±ΠΈΡ‚Π΅Π»ΡŒ элСктричСской энСргии ΠΌΠΎΠΆΠ΅Ρ‚ ΡΠ²Π»ΡΡ‚ΡŒΡΡ Π½Π°Π³Ρ€ΡƒΠ·ΠΊΠΎΠΉ Π² Ρ†Π΅ΠΏΠΈ. Нагрузка создаСт сопротивлСниС элСктричСскому Ρ‚ΠΎΠΊΡƒ.

ЭлСктричСский Ρ‚ΠΎΠΊ Π°ΠΊΡ‚ΠΈΠ²Π½ΠΎ ΠΈΡΠΏΠΎΠ»ΡŒΠ·ΡƒΡŽΡ‚ ΠΏΡ€ΠΈ создании искусствСнного освСщСния. ЭлСктричСскиС простыС Π»Π°ΠΌΠΏΡ‹ слуТат ΠΏΡ€ΠΈΠΌΠ΅Ρ€ΠΎΠΌ Π·Π°ΠΌΠΊΠ½ΡƒΡ‚ΠΎΠΉ Ρ†Π΅ΠΏΠΈ.

Разомкнутая элСктричСская Ρ†Π΅ΠΏΡŒ

ΠŸΡ€ΠΈ отсутствии ΠΏΠΎΡ‚ΠΎΠΊΠ° элСктронов Π½Π΅ΠΎΠ±Ρ…ΠΎΠ΄ΠΈΠΌΠΎΠ΅ напряТСниС источника Ρ†Π΅ΠΏΠΈ проявляСтся Π½Π° ΠΊΠΎΠ½Ρ†Π°Ρ… Ρ‚ΠΎΡ‡Π΅ΠΊ. Π’ этом случаС происходит процСсс оТидания ΠΌΠΎΠΌΠ΅Π½Ρ‚Π° соСдинСния ΠΊΠΎΠ½Ρ†ΠΎΠ² Ρ‚ΠΎΡ‡Π΅ΠΊ, Ρ‡Ρ‚ΠΎΠ±Ρ‹ возобновился ΠΏΠΎΡ‚ΠΎΠΊ элСктронов. ΠŸΠΎΠ΄ΠΎΠ±Π½ΡƒΡŽ Ρ†Π΅ΠΏΡŒ принято Π½Π°Π·Ρ‹Π²Π°Ρ‚ΡŒ Ρ€Π°Π·ΠΎΠΌΠΊΠ½ΡƒΡ‚ΠΎΠΉ.

Π—Π°ΠΌΠ΅Ρ‡Π°Π½ΠΈΠ΅ 1

ΠŸΡ€ΠΈ связывании ΠΊΠΎΠ½Ρ†ΠΎΠ² ΠΏΡ€ΠΎΠ²ΠΎΠ΄ΠΎΠ², Π³Π΄Π΅ сущСствуСт Ρ€Π°Π·Ρ€Ρ‹Π², Π½Π΅ΠΏΡ€Π΅Ρ€Ρ‹Π²Π½ΠΎΡΡ‚ΡŒ всСй Ρ†Π΅ΠΏΠΈ восстановится. Π­Ρ‚ΠΎ основная Ρ€Π°Π·Π½ΠΈΡ†Π° ΠΌΠ΅ΠΆΠ΄Ρƒ Π·Π°ΠΌΠΊΠ½ΡƒΡ‚ΠΎΠΉ ΠΈ Ρ€Π°Π·ΠΎΠΌΠΊΠ½ΡƒΡ‚ΠΎΠΉ Ρ†Π΅ΠΏΡŒΡŽ.

ΠŸΡ€ΠΈ Π²ΠΊΠ»ΡŽΡ‡Π΅Π½ΠΈΠΈ ΠΈ Π²Ρ‹ΠΊΠ»ΡŽΡ‡Π΅Π½ΠΈΠΈ элСктричСского освСщСния (Π»Π°ΠΌΠΏΡ‹) трСбуСтся постоянно ΠΎΡΡƒΡ‰Π΅ΡΡ‚Π²Π»ΡΡ‚ΡŒ ΠΏΠΎΡ…ΠΎΠΆΠΈΠ΅ процСссы. Для удобства Π±Ρ‹Π»ΠΈ созданы ΡΠΏΠ΅Ρ†ΠΈΠ°Π»ΡŒΠ½Ρ‹Π΅ устройства. Π˜Ρ… Π½Π°Π·Ρ‹Π²Π°ΡŽΡ‚ Π²Ρ‹ΠΊΠ»ΡŽΡ‡Π°Ρ‚Π΅Π»ΡΠΌΠΈ ΠΈΠ»ΠΈ Ρ€ΡƒΠ±ΠΈΠ»ΡŒΠ½ΠΈΠΊΠ°ΠΌΠΈ. Они Π² автоматичСском Ρ€Π΅ΠΆΠΈΠΌΠ΅ ΠΏΠΎ сигналу ΡƒΠΏΡ€Π°Π²Π»ΡΡŽΡ‚ ΠΏΠΎΡ‚ΠΎΠΊΠ°ΠΌΠΈ элСктронов Π² Ρ†Π΅ΠΏΠΈ, контролируя Π½Π°Ρ‡Π°Π»ΠΎ ΠΈ Π·Π°Π²Π΅Ρ€ΡˆΠ΅Π½ΠΈΠ΅ Ρ€Π°Π±ΠΎΡ‚Ρ‹ элСктрооборудования.

Π ΡƒΠ±ΠΈΠ»ΡŒΠ½ΠΈΠΊΠΈ практичСски идСально подходят для дСмонстрации ΠΏΡ€ΠΈΠ½Ρ†ΠΈΠΏΠΎΠ² Ρ€Π°Π±ΠΎΡ‚Ρ‹ Π²Ρ‹ΠΊΠ»ΡŽΡ‡Π°Ρ‚Π΅Π»Π΅ΠΉ ΠΈ ΠΏΠ΅Ρ€Π΅ΠΊΠ»ΡŽΡ‡Π°Ρ‚Π΅Π»Π΅ΠΉ.

Однако ΠΏΡ€ΠΈ использовании ΠΈΡ… Π² Π±ΠΎΠ»ΡŒΡˆΠΈΡ… элСктричСских цСпях сущСствуСт Π½Π΅ΠΌΠ°Π»ΠΎ ΠΏΡ€ΠΎΠ±Π»Π΅ΠΌ, связанных с бСзопасной эксплуатациСй.

Π’Π°ΠΊ ΠΊΠ°ΠΊ Π½Π΅ΠΊΠΎΡ‚ΠΎΡ€Ρ‹Π΅ части Ρ€ΡƒΠ±ΠΈΠ»ΡŒΠ½ΠΈΠΊΠΎΠ² ΠΎΡ‚ΠΊΡ€Ρ‹Ρ‚Ρ‹, Ρ‚ΠΎ сущСствуСт Π²Π΅Ρ€ΠΎΡΡ‚Π½ΠΎΡΡ‚ΡŒ воспламСнСния Π³ΠΎΡ€ΡŽΡ‡ΠΈΡ… ΠΌΠ°Ρ‚Π΅Ρ€ΠΈΠ°Π»ΠΎΠ². Π’ соврСмСнных Π²Ρ‹ΠΊΠ»ΡŽΡ‡Π°Ρ‚Π΅Π»ΡΡ… ΠΏΡ€ΠΈΠΌΠ΅Π½ΡΡŽΡ‚ΡΡ ΠΏΠΎΠ΄Π²ΠΈΠΆΠ½Ρ‹Π΅ ΠΈ Π½Π΅ΠΏΠΎΠ΄Π²ΠΈΠΆΠ½Ρ‹Π΅ ΠΊΠΎΠ½Ρ‚Π°ΠΊΡ‚Ρ‹, ΠΊΠΎΡ‚ΠΎΡ€Ρ‹Π΅ Π·Π°Ρ‰ΠΈΡ‰Π΅Π½Ρ‹ изоляционным корпусом.

Π˜ΡΡ‚ΠΎΡ‡Π½ΠΈΠΊ: https://spravochnick.ru/fizika/elektricheskie_cepi_-_chto_eto/zamknutaya_i_razomknutaya_elektricheskaya_cep/

Π—Π°ΠΊΠΎΠ½ сохранСния. ЀизичСский смысл Π·Π°ΠΊΠΎΠ½Π° сохранСния энСргии ΠΈ заряда. Π—Π°ΠΊΠΎΠ½ сохранСния заряда ΠΊΠ°ΠΊ основа элСктроники

  • Π—Π°ΠΊΠΎΠ½Ρ‹ сохранСния – это Ρ„ΡƒΠ½Π΄Π°ΠΌΠ΅Π½Ρ‚Π°Π»ΡŒΠ½Ρ‹Π΅ физичСскиС Π·Π°ΠΊΠΎΠ½Ρ‹, согласно ΠΊΠΎΡ‚ΠΎΡ€Ρ‹ΠΌ ΠΏΡ€ΠΈ ΠΎΠΏΡ€Π΅Π΄Π΅Π»Π΅Π½Π½Ρ‹Ρ… условиях Π½Π΅ΠΊΠΎΡ‚ΠΎΡ€Ρ‹Π΅ ΠΈΠ·ΠΌΠ΅Ρ€ΠΈΠΌΡ‹Π΅ физичСскиС Π²Π΅Π»ΠΈΡ‡ΠΈΠ½Ρ‹ , Ρ…Π°Ρ€Π°ΠΊΡ‚Π΅Ρ€ΠΈΠ·ΡƒΡŽΡ‰ΠΈΠ΅ Π·Π°ΠΌΠΊΠ½ΡƒΡ‚ΡƒΡŽ Ρ„ΠΈΠ·ΠΈΡ‡Π΅ΡΠΊΡƒΡŽ систСму, Π½Π΅ ΠΈΠ·ΠΌΠ΅Π½ΡΡŽΡ‚ΡΡ с Ρ‚Π΅Ρ‡Π΅Π½ΠΈΠ΅ΠΌ Π²Ρ€Π΅ΠΌΠ΅Π½ΠΈ.
  • E=Q+A E=const
  • Π‘ΡƒΡ‚ΡŒ Π·Π°ΠΊΠΎΠ½Π° сохранСния энСргии состоит Π² Ρ‚ΠΎΠΌ, Ρ‡Ρ‚ΠΎ энСргия Π½ΠΈΠΊΡƒΠ΄Π° Π½Π΅ исчСзаСт, Π° ΠΏΠ΅Ρ€Π΅Ρ…ΠΎΠ΄ΠΈΡ‚ ΠΈΠ· ΠΎΠ΄Π½ΠΎΠ³ΠΎ Π²ΠΈΠ΄Π° энСргии Π² Π΄Ρ€ΡƒΠ³ΡƒΡŽ.

Π—Π°ΠΊΠΎΠ½ сохранСния заряда ΡƒΡ‚Π²Π΅Ρ€ΠΆΠ΄Π°Π΅Ρ‚, Ρ‡Ρ‚ΠΎ Π²ΠΎ врСмя взаимодСйствия Π½Π΅ΠΊΠΎΡ‚ΠΎΡ€ΠΎΠΉ Π·Π°ΠΌΠΊΠ½ΡƒΡ‚ΠΎΠΉ систСмы с ΠΎΠΊΡ€ΡƒΠΆΠ°ΡŽΡ‰ΠΈΠΌ пространством количСство заряда ΠΊΠΎΡ‚ΠΎΡ€ΠΎΠ΅ Π²Ρ‹Ρ…ΠΎΠ΄ΠΈΡ‚ ΠΈΠ· систСмы Ρ‡Π΅Ρ€Π΅Π· Π΅Π΅ ΠΏΠΎΠ²Π΅Ρ€Ρ…Π½ΠΎΡΡ‚ΡŒ Ρ€Π°Π²Π½ΠΎ количСству заряда ΠΏΠΎΡΡ‚ΡƒΠΏΠΈΠ²ΡˆΠ΅Π³ΠΎ Π²Π½ΡƒΡ‚Ρ€ΡŒ систСмы. Π”Ρ€ΡƒΠ³ΠΈΠΌΠΈ словами алгСбраичСская сумма всСх зарядов систСмы Ρ€Π°Π²Π½Π° Π½ΡƒΠ»ΡŽ.

Как извСстно Π² ΠΏΡ€ΠΈΡ€ΠΎΠ΄Π΅ сущСствуСт Π΄Π²Π° Π²ΠΈΠ΄Π° зарядов. Π­Ρ‚ΠΎ ΠΏΠΎΠ»ΠΎΠΆΠΈΡ‚Π΅Π»ΡŒΠ½Ρ‹Π΅ ΠΈ ΠΎΡ‚Ρ€ΠΈΡ†Π°Ρ‚Π΅Π»ΡŒΠ½Ρ‹Π΅. Π’Π°ΠΊΠΆΠ΅ Π²Π΅Π»ΠΈΡ‡ΠΈΠ½Π° заряда дискрСтна, Ρ‚ΠΎ Π΅ΡΡ‚ΡŒ ΠΎΠ½ ΠΌΠΎΠΆΠ΅Ρ‚ ΠΌΠ΅Π½ΡΡ‚ΡŒΡΡ Ρ‚ΠΎΠ»ΡŒΠΊΠΎ порциями. Π­Π»Π΅ΠΌΠ΅Π½Ρ‚Π°Ρ€Π½Ρ‹ΠΌ зарядом считаСтся заряд элСктрона. Если ΠΊ Π°Ρ‚ΠΎΠΌΡƒ Π΄ΠΎΠ±Π°Π²ΠΈΡ‚ΡŒ ΠΎΠ΄ΠΈΠ½ элСктрон, Ρ‚ΠΎ ΠΎΠ½ становится ΠΎΡ‚Ρ€ΠΈΡ†Π°Ρ‚Π΅Π»ΡŒΠ½ΠΎ заряТСнным ΠΈΠΎΠ½ΠΎΠΌ. А Ссли Π΅Π³ΠΎ ΠΎΡ‚Π½ΡΡ‚ΡŒ Ρ‚ΠΎ ΠΏΠΎΠ»ΠΎΠΆΠΈΡ‚Π΅Π»ΡŒΠ½Ρ‹ΠΌ.

Основная идСя Π·Π°ΠΊΠΎΠ½Π° сохранСния заряда состоит Π² Ρ‚ΠΎΠΌ, Ρ‡Ρ‚ΠΎ заряд Π½Π΅ Π²ΠΎΠ·Π½ΠΈΠΊΠ°Π΅Ρ‚ ΠΈΠ· Π½Π΅ΠΎΡ‚ΠΊΡƒΠ΄Π° ΠΈ Π½Π΅ исчСзаСт Π² Π½ΠΈΠΊΡƒΠ΄Π°. ΠŸΡ€ΠΈ Π²ΠΎΠ·Π½ΠΈΠΊΠ½ΠΎΠ²Π΅Π½ΠΈΠΈ заряда ΠΎΠ΄Π½ΠΎΠ³ΠΎ Π·Π½Π°ΠΊΠ° Ρ‚ΡƒΡ‚ ΠΆΠ΅ появляСтся заряд ΠΏΡ€ΠΎΡ‚ΠΈΠ²ΠΎΠΏΠΎΠ»ΠΎΠΆΠ½ΠΎΠ³ΠΎ Π·Π½Π°ΠΊΠ° Ρ‚ΠΎΠΉ ΠΆΠ΅ Π²Π΅Π»ΠΈΡ‡ΠΈΠ½Ρ‹.

2. Вакуумная элСктроника, основныС понятия ΠΈ устройства.

Вакуумная элСктроника – это Ρ€Π°Π·Π΄Π΅Π» элСктроники, Π²ΠΊΠ»ΡŽΡ‡Π°ΡŽΡ‰ΠΈΠΉ Π² сСбя исслСдования взаимодСйствия ΠΏΠΎΡ‚ΠΎΠΊΠ° свободных элСктронов с элСктричСскими ΠΈ ΠΌΠ°Π³Π½ΠΈΡ‚Π½Ρ‹ΠΌΠΈ полями Π² Π²Π°ΠΊΡƒΡƒΠΌΠ΅, Π° Ρ‚Π°ΠΊ ΠΆΠ΅ ΠΌΠ΅Ρ‚ΠΎΠ΄Ρ‹ создания элСктронных ΠΏΡ€ΠΈΠ±ΠΎΡ€ΠΎΠ² ΠΈ устройств, Π² ΠΊΠΎΡ‚ΠΎΡ€Ρ‹Ρ… это взаимодСйствиС ΠΈΡΠΏΠΎΠ»ΡŒΠ·ΡƒΠ΅Ρ‚ΡΡ.

К элСктровакуумным ΠΏΡ€ΠΈΠ±ΠΎΡ€Π°ΠΌ относятся элСктронныС Π»Π°ΠΌΠΏΡ‹, элСктронно-Π»ΡƒΡ‡Π΅Β­Π²Ρ‹Π΅ Ρ‚Ρ€ΡƒΠ±ΠΊΠΈ ΠΈ фотоэлСктронныС ΠΏΡ€ΠΈΠ±ΠΎΡ€Ρ‹. Π­Π»Π΅ΠΊΡ‚Ρ€ΠΎΠ½Π½Ρ‹Π΅ Π»Π°ΠΌΠΏΡ‹, Ρ‚Π°ΠΊ ΠΆΠ΅ ΠΊΠ°ΠΊ ΠΈ ΠΏΠΎΠ»ΡƒΒ­ΠΏΡ€ΠΎΠ²ΠΎΠ΄Π½ΠΈΠΊΠΎΠ²Ρ‹Π΅ ΠΏΡ€ΠΈΠ±ΠΎΡ€Ρ‹, ΠΏΡ€Π΅Π΄Π½Π°Π·Π½Π°Ρ‡Π΅Π½Ρ‹ для Ρ€Π°Π·Π»ΠΈΡ‡Π½ΠΎΠ³ΠΎ Ρ€ΠΎΠ΄Π° ΠΏΡ€Π΅ΠΎΠ±Ρ€Π°Π·ΠΎΠ²Π°Π½ΠΈΠΉ элСктричСских сигналов.

Π’ настоящСС врСмя ΠΈΡ… ΠΏΡ€ΠΈΠΌΠ΅Π½ΡΡŽΡ‚ Π³Π»Π°Π²Π½Ρ‹ΠΌ ΠΎΠ±Ρ€Π°Π·ΠΎΠΌ Π½Π° свСрхвысоких частотах ΠΈ Π² ΠΌΠΎΡ‰Π½Ρ‹Ρ… усилитСлях ΠΈ Π³Π΅Π½Π΅Ρ€Π°Ρ‚ΠΎΡ€Π°Ρ….

Π’ элСктронно­лучСвых Ρ‚Ρ€ΡƒΠ±ΠΊΠ°Ρ… (Π­Π›Π’) Π΄Π²ΠΈΠΆΠ΅Π½ΠΈΠ΅ элСктронов происходит Π² Π²ΠΈΠ΄Π΅ ΡƒΠ·ΠΊΠΎΠ³ΠΎ Π»ΡƒΡ‡Π°, Ρ‡Ρ‚ΠΎ позволяСт ΠΏΡ€Π΅ΠΎΠ±Ρ€Π°Π·ΠΎΠ²Ρ‹Π²Π°Ρ‚ΡŒ элСктричСскиС сигналы Π² Π²ΠΈΠ΄ΠΈΠΌΡ‹Π΅ изобраТСния (кинСскопы Π² Ρ‚Π΅Π»Π΅Π²ΠΈΠ΄Π΅Π½ΠΈΠΈ, Π­Π›Π’ Π² осциллографах) ΠΈΠ»ΠΈ, Π½Π°ΠΎΠ±ΠΎΡ€ΠΎΡ‚, Π²ΠΈΠ΄ΠΈΠΌΡ‹Π΅ изоб­раТСния ΠΏΡ€Π΅ΠΎΠ±Ρ€Π°Π·ΠΎΠ²Ρ‹Π²Π°Ρ‚ΡŒ Π² элСктричСскиС сигналы (ΠΏΠ΅Ρ€Π΅Π΄Π°ΡŽΡ‰ΠΈΠ΅ Ρ‚Π΅Π»Π΅Π²ΠΈΠ·ΠΈΠΎΠ½Π½Ρ‹Π΅ Ρ‚Ρ€ΡƒΠ±ΠΊΠΈ). Π’ фотоэлСктронных ΠΏΡ€ΠΈΠ±ΠΎΡ€Π°Ρ… (ЀЭП) осущСствляСтся ΠΏΡ€Π΅ΠΎΠ±Ρ€Π°Π·ΠΎΠ²Π°Π½ΠΈΠ΅ элСктромагнитного излучСния Π² элСктричСскиС сигналы. ΠŸΡ€ΠΈΠ½Ρ†ΠΈΠΏ Ρ€Π°Π±ΠΎΡ‚Ρ‹ ЀЭП основан Π½Π° фотоэлСктронной эмиссии.

Π‘ΠΈΠ»Π΅Ρ‚ β„–7

ΠœΠ΅Ρ‚ΠΎΠ΄ΠΎΠ»ΠΎΠ³ΠΈΡ использования Π·Π°ΠΊΠΎΠ½ΠΎΠ² сохранСния Π² Π½Π°ΡƒΠΊΠ΅ ΠΈ Ρ‚Π΅Ρ…Π½ΠΈΠΊΠ΅.

БущСствуСт мноТСство Π·Π°ΠΊΠΎΠ½ΠΎΠ² сохранСния, Ρ‚Π°ΠΊΠΈΡ… ΠΊΠ°ΠΊ, Π·Π°ΠΊΠΎΠ½ сохранСния энСргии, ΠΈΠΌΠΏΡƒΠ»ΡŒΡΠ°, заряда, Π΄Π²ΠΈΠΆΠ΅Π½ΠΈΠΉ ΠΈ Ρ‚.Π΄. ВсС эти Π·Π°ΠΊΠΎΠ½Ρ‹ Π²Π°ΠΆΠ½Ρ‹ Π² мСтодологичСском ΠΏΠ»Π°Π½Π΅. Они ΠΏΠΎΠ·Π²ΠΎΠ»ΡΡŽΡ‚ с ΠΏΠΎΠΌΠΎΡ‰ΡŒΡŽ простых ΠΏΡ€ΠΈΠ΅ΠΌΠΎΠ² (Π°Π½Π°Π»ΠΈΠ· энСргСтичСского баланса) ΠΎΠΏΡ€Π΅Π΄Π΅Π»ΠΈΡ‚ΡŒ Π΄ΠΎΡΡ‚ΠΎΠ²Π΅Ρ€Π½ΠΎΡΡ‚ΡŒ ΠΏΠΎΠ»ΡƒΡ‡Π΅Π½Π½Ρ‹Ρ… Ρ„ΠΈΠ·ΠΈΠΊΠΎ-матСматичСских ΠΌΠΎΠ΄Π΅Π»Π΅ΠΉ.

Π‘ΠΎΠ²Ρ€Π΅ΠΌΠ΅Π½Π½Ρ‹Π΅ срСдства отобраТСния ΠΈΠ½Ρ„ΠΎΡ€ΠΌΠ°Ρ†ΠΈΠΈ.

ΠžΡ‚ΠΎΠ±Ρ€Π°ΠΆΠ΅Π½ΠΈΠ΅ ΠΈΠ½Ρ„ΠΎΡ€ΠΌΠ°Ρ†ΠΈΠΈ – это свойство тСхничСской систСмы Π²ΠΎΡΠΏΡ€ΠΎΠΈΠ·Π²ΠΎΠ΄ΠΈΡ‚ΡŒ Ρ‚Ρ€Π΅Π±ΡƒΠ΅ΠΌΡƒΡŽ ΠΈΠ½Ρ„ΠΎΡ€ΠΌΠ°Ρ†ΠΈΡŽ Π² Ρ„ΠΎΡ€ΠΌΠ΅, ΡƒΠ΄ΠΎΠ±Π½ΠΎΠΉ для нСпосрСдствСнного восприятия Ρ‡Π΅Π»ΠΎΠ²Π΅ΠΊΠΎΠΌ.

ВСхничСскиС срСдства, ΠΈΡΠΏΠΎΠ»ΡŒΠ·ΡƒΠ΅ΠΌΡ‹Π΅ для формирования ΠΈΠ½Ρ„ΠΎΡ€ΠΌΠ°Ρ†ΠΈΠΎΠ½Π½Ρ‹Ρ… ΠΌΠΎΠ΄Π΅Π»Π΅ΠΉ, Π½Π°Π·Ρ‹Π²Π°ΡŽΡ‚ΡΡ срСдствами отобраТСния ΠΈΠ½Ρ„ΠΎΡ€ΠΌΠ°Ρ†ΠΈΠΈ (БОИ). Π‘ ΠΏΠΎΠΌΠΎΡ‰ΡŒΡŽ БОИ получСнная ΠΎΡ‚ ΠΎΠ΄Π½ΠΎΠ³ΠΎ ΠΈΠ»ΠΈ Π½Π΅ΡΠΊΠΎΠ»ΡŒΠΊΠΈΡ… источников информация прСобразуСтся Π² ΠΈΠ½Ρ„ΠΎΡ€ΠΌΠ°Ρ†ΠΈΠΎΠ½Π½ΡƒΡŽ модСль, ΡƒΠ΄ΠΎΠ±Π½ΡƒΡŽ для нСпосрСдствСнного восприятия.

  1. БущСствуСт Ρ‚Ρ€ΠΈ способа отобраТСния ΠΈΠ½Ρ„ΠΎΡ€ΠΌΠ°Ρ†ΠΈΠΈ:
  2. 1)индикация – прСдставлСниС ΠΈΠ½Ρ„ΠΎΡ€ΠΌΠ°Ρ†ΠΈΠΈ Π² Ρ„ΠΎΡ€ΠΌΠ΅ изобраТСния (ΠΈΠ½Ρ„ΠΎΡ€ΠΌΠ°Ρ†ΠΈΠΎΠ½Π½ΠΎΠΉ ΠΌΠΎΠ΄Π΅Π»ΠΈ), ΠΏΠ°Ρ€Π°ΠΌΠ΅Ρ‚Ρ€Ρ‹ ΠΊΠΎΡ‚ΠΎΡ€ΠΎΠ³ΠΎ ΠΎΠ±Π΅ΡΠΏΠ΅Ρ‡ΠΈΠ²Π°ΡŽΡ‚ Ρ‚Ρ€Π΅Π±ΡƒΠ΅ΠΌΡƒΡŽ быстроту ΠΈ Ρ‚ΠΎΡ‡Π½ΠΎΡΡ‚ΡŒ восприятия, ΠΈΠ½Ρ„ΠΎΡ€ΠΌΠ°Ρ†ΠΈΠΎΠ½Π½ΡƒΡŽ Π΅ΠΌΠΊΠΎΡΡ‚ΡŒ ΠΈ ΡƒΠ΄ΠΎΠ²Π»Π΅Ρ‚Π²ΠΎΡ€ΡΡŽΡ‚ трСбованиям ΠΈΠ½ΠΆΠ΅Π½Π΅Ρ€Π½ΠΎΠΉ психологии (эргономики)
  3. 2)сигнализация – это ΠΎΡ‚ΠΎΠ±Ρ€Π°ΠΆΠ΅Π½ΠΈΠ΅ ΠΈΠ½Ρ„ΠΎΡ€ΠΌΠ°Ρ†ΠΈΠΈ для привлСчСния внимания ΠΊ измСнСнию состояния систСмы, Ρ…Π°Ρ€Π°ΠΊΡ‚Π΅Ρ€ΠΈΠ·ΡƒΠ΅ΠΌΠΎΠ΅ Ρ‡Π΅Ρ‚ΠΊΠΎ Ρ€Π°Π·Π»ΠΈΡ‡ΠΈΠΌΡ‹ΠΌΠΈ измСнСниями ΠΏΠ°Ρ€Π°ΠΌΠ΅Ρ‚Ρ€ΠΎΠ² ΠΈΠ½Ρ„ΠΎΡ€ΠΌΠ°Ρ†ΠΈΠΎΠ½Π½ΠΎΠΉ ΠΌΠΎΠ΄Π΅Π»ΠΈ;
  4. 3)рСгистрация– это прСдставлСниС ΠΈΠ½Ρ„ΠΎΡ€ΠΌΠ°Ρ†ΠΈΠΈ Π½Π° ΠΌΠ°Ρ‚Π΅Ρ€ΠΈΠ°Π»ΡŒΠ½ΠΎΠΌ носитСлС с Π²ΠΎΠ·ΠΌΠΎΠΆΠ½ΠΎΡΡ‚ΡŒΡŽ хранСния Π±Π΅Π· Π·Π°Ρ‚Ρ€Π°Ρ‚ энСргии.
  5. Π‘ΠΈΠ»Π΅Ρ‚ β„–8

1. ΠŸΠΎΠ½ΡΡ‚ΠΈΠ΅ «энСргия», «пространство» ΠΈ Ρ€Π΅Π·ΡƒΠ»ΡŒΡ‚Π°Ρ‚ ΠΈΡ… взаимодСйствия. Π Π°ΡΠΊΡ€Ρ‹Ρ‚ΡŒ эти понятия ΠΈ ΠΏΠΎΠΊΠ°Π·Π°Ρ‚ΡŒ ΠΈΡ… взаимодСйствиС Π½Π° практичСских ΠΏΡ€ΠΈΠΌΠ΅Ρ€Π°Ρ… взаимосвязи.

ЭнС́ргия β€” скалярная физичСская Π²Π΅Π»ΠΈΡ‡ΠΈΠ½Π°, ΡΠ²Π»ΡΡŽΡ‰Π°ΡΡΡ Π΅Π΄ΠΈΠ½ΠΎΠΉ ΠΌΠ΅Ρ€ΠΎΠΉ Ρ€Π°Π·Π»ΠΈΡ‡Π½Ρ‹Ρ… Ρ„ΠΎΡ€ΠΌ двиТСния ΠΈ взаимодСйствия ΠΌΠ°Ρ‚Π΅Ρ€ΠΈΠΈ, ΠΌΠ΅Ρ€ΠΎΠΉ ΠΏΠ΅Ρ€Π΅Ρ…ΠΎΠ΄Π° двиТСния ΠΌΠ°Ρ‚Π΅Ρ€ΠΈΠΈ ΠΈΠ· ΠΎΠ΄Π½ΠΈΡ… Ρ„ΠΎΡ€ΠΌ Π² Π΄Ρ€ΡƒΠ³ΠΈΠ΅.

Π’Π²Π΅Π΄Π΅Π½ΠΈΠ΅ понятия энСргии ΡƒΠ΄ΠΎΠ±Π½ΠΎ Ρ‚Π΅ΠΌ, Ρ‡Ρ‚ΠΎ Π² случаС, Ссли физичСская систСма являСтся Π·Π°ΠΌΠΊΠ½ΡƒΡ‚ΠΎΠΉ, Ρ‚ΠΎ Π΅Ρ‘ энСргия сохраняСтся Π² этой систСмС Π½Π° протяТСнии Π²Ρ€Π΅ΠΌΠ΅Π½ΠΈ, Π² Ρ‚Π΅Ρ‡Π΅Π½ΠΈΠ΅ ΠΊΠΎΡ‚ΠΎΡ€ΠΎΠ³ΠΎ систСма Π±ΡƒΠ΄Π΅Ρ‚ ΡΠ²Π»ΡΡ‚ΡŒΡΡ Π·Π°ΠΌΠΊΠ½ΡƒΡ‚ΠΎΠΉ.

Π­Ρ‚ΠΎ ΡƒΡ‚Π²Π΅Ρ€ΠΆΠ΄Π΅Π½ΠΈΠ΅ носит Π½Π°Π·Π²Π°Π½ΠΈΠ΅ Π·Π°ΠΊΠΎΠ½Π° сохранСния энСргии.

ВсС ΠΌΠ°Ρ‚Π΅Ρ€ΠΈΠ°Π»ΡŒΠ½Ρ‹Π΅ Ρ‚Π΅Π»Π° ΠΈΠΌΠ΅ΡŽΡ‚ ΠΏΡ€ΠΎΡ‚ΡΠΆΠ΅Π½Π½ΠΎΡΡ‚ΡŒ ΠΈ Π·Π°Π½ΠΈΠΌΠ°ΡŽΡ‚ ΠΎΠΏΡ€Π΅Π΄Π΅Π»Π΅Π½Π½ΠΎΠ΅ мСсто Π² пространствС. Π­Ρ‚Π° позиция Π΅Ρ‰Π΅ Π² Π΄Ρ€Π΅Π²Π½Π΅ΠΉΡˆΠΈΠ΅ Π²Ρ€Π΅ΠΌΠ΅Π½Π° Π±Ρ‹Π»Π° ΠΏΡ€ΠΎΠ²Π΅Ρ€Π΅Π½Π° ΡΠΊΡΠΏΠ΅Ρ€ΠΈΠΌΠ΅Π½Ρ‚Π°Π»ΡŒΠ½ΠΎ. И Ρ‚Π°ΠΊΠΈΠΌ ΠΎΠ±Ρ€Π°Π·ΠΎΠΌ ΠΏΠ΅Ρ€Π²Ρ‹Π΅ характСристики ΠΌΠ°Ρ‚Π΅Ρ€ΠΈΠ°Π»ΡŒΠ½Ρ‹Ρ… Ρ‚Π΅Π» Π²Ρ‹Ρ€Π°ΠΆΠ°Π»ΠΈΡΡŒ Π² Π²ΠΈΠ΄Π΅ гСомСтричСских Ρ€Π°Π·ΠΌΠ΅Ρ€ΠΎΠ².

Π­Ρ‚ΠΈΠΌΠΈ ΠΆΠ΅ характСристиками опрСдСляСтся пространство. ΠŸΠ΅Ρ€Π²Π°Ρ Ρ„ΠΎΡ€ΠΌΡƒΠ»ΠΈΡ€ΠΎΠ²ΠΊΠ° взаимосвязи гСомСтричСских Ρ€Π°Π·ΠΌΠ΅Ρ€ΠΎΠ² с Ρ„ΠΎΡ€ΠΌΠΎΠΉ Ρ‚Π΅Π»Π° Π±Ρ‹Π»Π° Π΄Π°Π½Π° Π•Π²ΠΊΠ»ΠΈΠ΄ΠΎΠΌ 2000 Π»Π΅Ρ‚ Π½Π°Π·Π°Π΄. Π’ Π³Π΅ΠΎΠΌΠ΅Ρ‚Ρ€ΠΈΠΈ Π•Π²ΠΊΠ»ΠΈΠ΄Π° понятиС Ρ€Π°Π·ΠΌΠ΅Ρ€Π° Ρ‚Π΅Π»Π° ΠΏΡ€Π΅Π΄ΡΡ‚Π°Π²Π»ΡΠ»ΠΎΡΡŒ Π½Π°Π΄ΠΌΠ΅Π½Π½ΠΎ, нСзависимо ΠΎΡ‚ скорости пСрСмСщСния Ρ‚Π΅Π»Π°.

Π‘ Ρ‚Π΅Ρ… ΠΏΠΎΡ€ ΡƒΡ‡Π΅Π½Ρ‹Π΅ подСлились Π½Π° Π΄Π²Π΅ части ( Π·Π°Π²ΠΈΡΠΈΠΌΠΎΡΡ‚ΡŒ ΠΈ Π½Π΅Π·Π°Π²ΠΈΡΠΈΠΌΠΎΡΡ‚ΡŒ с пространсвом).

E=mc2=mvc2=Ek+En=mc2/2+En

Π’ настоящСС врСмя ΠΌΠΎΠΆΠ½ΠΎ ΡΡ‡ΠΈΡ‚Π°Ρ‚ΡŒ, Ρ‡Ρ‚ΠΎ пространство Π½Π΅ ΠΌΠΎΠΆΠ΅Ρ‚ ΡΡƒΡ‰Π΅ΡΡ‚Π²ΠΎΠ²Π°Ρ‚ΡŒ само ΠΏΠΎ сСбС. Однако, для практичСского изучСния нас ΠΌΠΈΡ€Π°, ΠΎΠ±Π° ΠΏΠΎΠ΄Ρ…ΠΎΠ΄Π° ( Π½Π΅Π·Π°Π²ΠΈΡΠΈΠΌΠΎΡΡ‚ΡŒ ΠΈ Π·Π°Π²ΠΈΡΠΈΠΌΠΎΡΡ‚ΡŒ) оказались ΠΏΡ€ΠΈΠ΅ΠΌΠ»Π΅ΠΌΡ‹ ΠΈ высокоэффСктивны для Ρ€Π΅ΡˆΠ΅Π½ΠΈΡ своих Π·Π°Π΄Π°Ρ‡.

ΠŸΠ΅Ρ€Π²ΠΎΠΉ появилась Π”Π΅ΠΊΠ°Ρ€Ρ‚ΠΎΠ²Π° систСма ΠΊΠΎΠΎΡ€Π΄ΠΈΠ½Π°Ρ‚, которая Π²ΠΎΠ·Π½ΠΈΠΊΠ»Π° Π² Π²ΠΈΠ΄Π΅ Ρ‚Ρ€Π΅Ρ…ΠΌΠ΅Ρ€Π½ΠΎΠ³ΠΎ пространства. Π€ΠΎΡ€ΠΌΠΈΡ€ΠΎΠ²Π°Π½ΠΈΠ΅ Ρ‚Π°ΠΊΠΎΠΉ ΠΌΠ΅Ρ‚ΠΎΠ΄ΠΈΠΊΠΈ ΠΈΠ·ΡƒΡ‡Π°ΡŽΡ‰Π΅Π³ΠΎ пространства, ΠΏΡ€ΠΈΠ²Π΅Π»ΠΎ ΠΊ Ρ†Π΅Π»ΠΎΠΌΡƒ ряду понятий.

  • 1 ) ΠΌΠ°Ρ‚Π΅Ρ€ΠΈΠ°Π»ΡŒΠ½Π°Ρ Ρ‚ΠΎΡ‡ΠΊΠ° – это абстракция , которая позволяСт ввСсти Π² рассмотрСниС Π½Π΅ΠΊΠΎΡ‚ΠΎΡ€ΡƒΡŽ Π³Π΅ΠΏΠΎΡ‚Π΅Ρ‚ΠΈΡ‡Π΅ΡΠΊΡƒΡŽ пСрвочастицу, ΠΊΠ°ΠΊ Ρ‡Π°ΡΡ‚ΡŒ ΠΌΠ°Ρ‚Π΅Ρ€ΠΈΠ°Π»ΡŒΠ½ΠΎΠ³ΠΎ ΠΌΠΈΡ€Π°.
  • РассмотрСниС этой систСмы ΠΊΠΎΠΎΡ€Π΄ΠΈΠ½Π°Ρ‚ ΠΏΠΎΠ·Π²ΠΎΠ»ΠΈΠ»ΠΎ : 1) ΠΎΠΏΡ€Π΅Π΄Π΅Π»ΠΈΡ‚ΡŒ ΠΊΠΎΠΎΡ€Π΄ΠΈΠ½Π°Ρ‚Ρ‹ любого ΠΏΡ€Π΅Π΄ΠΌΠ΅Ρ‚Π°, 2) ΡΠ΄Π΅Π»Π°Ρ‚ΡŒ Π²Ρ‹Π²ΠΎΠ΄, Ρ‡Ρ‚ΠΎ гСомСтричСскиС Ρ€Π°Π·ΠΌΠ΅Ρ€Ρ‹ Π½Π΅ ΠΌΠ΅Π½ΡΡŽΡ‚ΡΡ ΠΎΡ‚ располоТСния Π² пространствС.
  • Π’ связи с Ρ‚Π΅ΠΌ, Ρ‡Ρ‚ΠΎ Ρ€Π°Π·ΠΌΠ΅Ρ€ ΠΎΠ±ΡŠΠ΅ΠΊΡ‚Π° Π½Π΅ мСняСтся Π² Π΄Π΅ΠΊΠ°Ρ€Ρ‚ΠΎΠ²ΠΎΠΉ систСмС, Π΅Π³ΠΎ Π΄Π»ΠΈΠ½Π½Π° являСтся ΠΈΠ½Π²Π°Ρ€ΠΈΠ°Π½Ρ‚ΠΎΠΉ ΠΈ ΠΌΠΎΠΆΠ΅Ρ‚ Π±Ρ‹Ρ‚ΡŒ Π²Ρ‹Ρ€Π°ΠΆΠ΅Π½Π° ΡΠ»Π΅Π΄ΡƒΡŽΡ‰ΠΈΠΌ ΡƒΡ€Π°Π²Π½Π΅Π½ΠΈΠ΅ΠΌ.
  • l2= Ξ”x2+ Ξ”y2+ Ξ”z2

ο»Ώ

Π˜ΡΡ‚ΠΎΡ‡Π½ΠΈΠΊ: https://infopedia.su/2Γ—5313.html

Π—Π°ΠΊΠΎΠ½ Ома для Π·Π°ΠΌΠΊΠ½ΡƒΡ‚ΠΎΠΉ Ρ†Π΅ΠΏΠΈ

Замкнутая
Ρ†Π΅ΠΏΡŒ содСрТит: источник Ρ‚ΠΎΠΊΠ°, сопротивлСния
(ΠΏΠΎΡ‚Ρ€Π΅Π±ΠΈ Ρ‚ΠΎΠΊΠ°),
ΠΏΡ€ΠΈΠ±ΠΎΡ€Ρ‹ для контроля характСристик
Ρ‚ΠΎΠΊΠ°, ΠΏΡ€ΠΎΠ²ΠΎΠ΄Π°, ΠΊΠ»ΡŽΡ‡. ΠŸΡ€ΠΈΠΌΠ΅ ΠΌΠΎΠΆΠ΅Ρ‚
ΡΠ»ΡƒΠΆΠΈΡ‚ΡŒ Ρ†Π΅ΠΏΡŒ, привСдСнная Π½Π° рис.5.

По
ΠΎΡ‚Π½ΠΎΡˆΠ΅Π½ΠΈΡŽ ΠΊ источнику ΠΌΠΎΠΆΠ½ΠΎ Π²Ρ‹Π΄Π΅Π»ΠΈΡ‚
внСшнюю Ρ†Π΅ΠΏΡŒ, ΡΠΎΠ΄Π΅Ρ€ΠΆΠ°Ρ‰ΡƒΡŽ элСмСнты,
находящиСся Π΄Π°Π½Π½ΠΎΠ³ΠΎ
источника, Ссли ΠΏΡ€ΠΎΡΠ»Π΅Π΄ΠΈΡ‚ΡŒ Π·Π° Ρ‚ΠΎΠΊΠΎΠΌ ΠΎΡ‚
ΠΎΠ΄Π½ΠΎΠΉ Π΅Π³ΠΎ ΠΊΠ»Π΅ΠΌΠΌΡ‹ Π΄Ρ€ΡƒΠ³ΠΎΠΉ, ΠΈ Π²Π½ΡƒΡ‚Ρ€Π΅Π½Π½ΡŽΡŽ,
ΠΊ ΠΊΠΎΡ‚ΠΎΡ€ΠΎΠΉ относят ΠΏΡ€ΠΎΠ²ΠΎΠ΄ΡΡ‰ΡƒΡŽ срСду
Π²Π½ΡƒΡ‚Ρ€ΠΈ источника
ΠΎΠ±ΠΎΠ·Π½Π°Ρ‡ΠΈΠΌ сопротивлСниС внСшнСй Ρ†Π΅ΠΏΠΈ
Ρ‡Π΅Ρ€Π΅Π· R,
Π²Π½ΡƒΡ‚Ρ€Π΅Π½Π½Π΅Π΅
сопротивлСниС
источника r.

Π’ΠΎΠ³Π΄Π°
Ρ‚ΠΎΠΊ Π² Ρ†Π΅ΠΏΠΈ опрСдСляСтся ΠΏΠΎ Π·Π°ΠΊΠΎΠ½Ρƒ для
Π·Π°ΠΌΠΊΠ½ΡƒΡ‚ΠΎΠΉ Ρ†Π΅ΠΏΠΈ, ΠΊΠΎΡ‚ΠΎΡ€Ρ‹ΠΉ гласит, Ρ‡Ρ‚ΠΎ Ρ‚ΠΎΠΊ
Π² Π·Π°ΠΌΠΊΠ½ΡƒΡ‚ΠΎΠΉ Ρ†Π΅ΠΏΠΈ прямопропорционалСн
Π²Π΅Π»ΠΈΡ‡ΠΈΠ½Π΅ Π­Π”Π‘ ΠΈ ΠΎΠ±Ρ€Π°Ρ‚Π½ΠΎ ΠΏΡ€ΠΎΠΏΠΎΡ€Ρ†ΠΈΠΎΠ½Π°Π»Π΅Π½ суммСвнутрСннСго
ΠΈ внСшнСго сопротивлСния Ρ†Π΅ΠΏΠΈ,
Ρ‚.Π΅.

Из
этого Π·Π°ΠΊΠΎΠ½Π° Π²Ρ‹Ρ‚Π΅ΠΊΠ°ΡŽΡ‚ ΡΠ»Π΅Π΄ΡƒΡŽΡ‰ΠΈΠ΅ частныС
случаи:

β€’ Если
RстрСмится
ΠΊ Π½ΡƒΠ»ΡŽ (Ρ‚.Π΅. R
>
r),
Ρ‚ΠΎΠΊ I
ΡƒΠΌΠ΅Π½ΡŒΡˆΠ°Π΅Ρ‚ΡΡ, ΠΈ ΠΏΠ°Π΄Π΅Π½ΠΈΠ΅ напряТСния Π²Π½ΡƒΡ‚Ρ€ΠΈ
источника Irстановится
намного мСньшС IR,
ΡΠ»Π΅Π΄ΠΎΠ²Π°Ρ‚Π΅Π»ΡŒΠ½ΠΎ

pngΒ»>IR.

Π—Π½Π°Ρ‡ΠΈΡ‚,
Π²Π΅Π»ΠΈΡ‡ΠΈΠ½Ρƒ
Π­Π”Π‘
источника ΠΌΠΎΠΆΠ½ΠΎ практичСски ΠΈΠ·ΠΌΠ΅Ρ€ΠΈΡ‚ΡŒ
с ΠΏΠΎΠΌΠΎΡ‰ΡŒΡŽ Π²ΠΎΠ»ΡŒΡ‚ΠΌΠ΅Ρ‚Ρ€Π°,
присоСдинСнного
ΠΊ ΠΊΠ»Π΅ΠΌΠΌΠ°ΠΌ источника ΠΏΡ€ΠΈ условии, Ρ‡Ρ‚ΠΎ
сопротивлСниС
Π²ΠΎΠ»ΡŒΡ‚ΠΌΠ΅Ρ‚Ρ€Π°
RV
>>
rΠΏΡ€ΠΈ
Ρ€Π°Π·ΠΎΠΌΠΊΠ½ΡƒΡ‚ΠΎΠΉ внСшнСй Ρ†Π΅ΠΏΠΈ.

РаспрСдСлСниС
энСргии ΠΏΡ€ΠΈ Ρ€Π°Π±ΠΎΡ‚Π΅ источника постоянного
Ρ‚ΠΎΠΊΠ°

  • сопротивлСниС
    rΠΈ
    Π·Π°ΠΌΠΊΠ½ΡƒΡ‚ Π½Π° сопротивлСниС внСшнСй
    Π½Π°Π³Ρ€ΡƒΠ·ΠΊΠΈ R.
  • ΠŸΡ€ΠΎΠ°Π½Π°Π»ΠΈΠ·ΠΈΡ€ΡƒΠ΅ΠΌ
    нСсколько Π²Π΅Π»ΠΈΡ‡ΠΈΠ½, Ρ…Π°Ρ€Π°ΠΊΡ‚Π΅Ρ€ΠΈΠ·ΡƒΡŽΡ‰ΠΈΡ…
    распрСдСлСниС энСргии
    ΠΏΡ€ΠΈ Ρ€Π°Π±ΠΎΡ‚Π΅ источника постоянного Ρ‚ΠΎΠΊΠ°.
  • Π°)
    ЗатрачСнная
    источником ΠΌΠΎΡ‰Π½ΠΎΡΡ‚ΡŒ Π .
  • Π Π°Π±ΠΎΡ‚Π°,
    ΡΠΎΠ²Π΅Ρ€ΡˆΠ°Π΅ΠΌΠ°Ρ сторонними силами Π² Π·Π°ΠΌΠΊΠ½ΡƒΡ‚ΠΎΠΉ
    Ρ†Π΅ΠΏΠΈ ΠΏΠΎ
  • ΠΏΠ΅Ρ€Π΅ΠΌΠ΅Ρ‰Π΅Π½ΠΈΡŽ
    заряда dq,
    Ρ€Π°Π²Π½Π°:
  • dA
    =
    dq(9)
  • Π˜ΡΡ…ΠΎΠ΄Ρ
    ΠΈΠ· опрСдСлСния, ΠΌΠΎΡ‰Π½ΠΎΡΡ‚ΡŒ, развиваСмая
    сторонними силами Π²
  • источникС,
    Ρ€Π°Π²Π½Π°:

Π­Ρ‚Π°
ΠΌΠΎΡ‰Π½ΠΎΡΡ‚ΡŒ расходуСтся источником Π²ΠΎ
внСшнСй ΠΈ Π²Π½ΡƒΡ‚Ρ€Π΅Π½Π½Π΅ΠΉ ΠΏΠΎ ΠΎΡ‚Π½ΠΎΡˆΠ΅Π½ΠΈΡŽ
ΠΊ источнику частях Ρ†Π΅ΠΏΠΈ. Π˜ΡΠΏΠΎΠ»ΡŒΠ·ΡƒΡ Π·Π°ΠΊΠΎΠ½
Ома для Π·Π°ΠΌΠΊΠ½ΡƒΡ‚ΠΎΠΉ Ρ†Π΅ΠΏΠΈ,
ΠΌΠΎΠΆΠ½ΠΎ Π·Π°Ρ‚Ρ€Π°Ρ‡Π΅Π½Π½ΡƒΡŽ ΠΌΠΎΡ‰Π½ΠΎΡΡ‚ΡŒ ΠΏΡ€Π΅Π΄ΡΡ‚Π°Π²ΠΈΡ‚ΡŒ
Π² Π²ΠΈΠ΄Π΅:

(11)

Если
сопротивлСниС Π½Π°Π³Ρ€ΡƒΠ·ΠΊΠΈ RΡƒΠΌΠ΅Π½ΡŒΡˆΠ°Π΅Ρ‚ΡΡ,
ΡΡ‚Ρ€Π΅ΠΌΡΡΡŒ ΠΊ Π½ΡƒΠ»ΡŽ, Ρ‚ΠΎ Π Π·Π°Ρ‚
Pmax=

pngΒ»>Если
RувСличиваСтся,
ΡΡ‚Ρ€Π΅ΠΌΡΡΡŒ Π² Π±Π΅ΡΠΊΠΎΠ½Π΅Ρ‡Π½ΠΎΡΡ‚ΡŒ, Ρ‚ΠΎ Π Π·Π°Ρ‚.

Π“Ρ€Π°Ρ„ΠΈΠΊ зависимости Π·Π°Ρ‚Ρ€Π°Ρ‡Π΅Π½Π½ΠΎΠΉ сторонними
силами мощности Π Π·Π°Ρ‚
ΠΎΡ‚
Π²Π΅Π»ΠΈΡ‡ΠΈΠ½Ρ‹ внСшнСго сопротивлСния RΠΏΠΎΠΊΠ°Π·Π°Π½
Π½Π° рисункС 5.

Π±)
ПолСзная
ΠΌΠΎΡ‰Π½ΠΎΡΡ‚ΡŒ Π ΠΏΠΎΠ΄:_

ПолСзной
ΠΏΠΎ ΠΎΡ‚Π½ΠΎΡˆΠ΅Π½ΠΈΡŽ ΠΊ источнику ΠΌΠΎΡ‰Π½ΠΎΡΡ‚ΡŒΡŽ Π ΠΏΠΎΠ΄
считаСтся ΠΌΠΎΡ‰Π½ΠΎΡΡ‚ΡŒ,
расходуСмая источником Π²ΠΎ внСшнСй Ρ†Π΅ΠΏΠΈ,
Ρ‚.Π΅. Π½Π° внСшнСй Π½Π°Π³Ρ€ΡƒΠ·ΠΊΠ΅.
Она Ρ€Π°Π²Π½Π°:

(12)

ΠŸΠΎΠ»ΡŒΠ·ΡƒΡΡΡŒ
Π·Π°ΠΊΠΎΠ½ΠΎΠΌ Ома для Π·Π°ΠΌΠΊΠ½ΡƒΡ‚ΠΎΠΉ Ρ†Π΅ΠΏΠΈ, ΠΈΠ»ΠΈ
Π·Π°ΠΌΠ΅Π½ΠΈΠ² Π² послСднСм Π²Ρ‹Ρ€Π°ΠΆΠ΅Π½ΠΈΠΈ I
Π½Π° /(R+r),
ΠΌΠΎΠΆΠ½ΠΎ
ΠΏΡ€Π΅Π΄ΡΡ‚Π°Π²ΠΈΡ‚ΡŒ Π² Π²ΠΈΠ΄Π΅

Если
Ρ‡ΠΈΡΠ»ΠΈΡ‚Π΅Π»ΡŒ ΠΈ Π·Π½Π°ΠΌΠ΅Π½Π°Ρ‚Π΅Π»ΡŒ этого выраТСния
Ρ€Π°Π·Π΄Π΅Π»ΠΈΡ‚ΡŒ Π½Π° R,
Ρ‚ΠΎ
получится Π²Ρ‹Ρ€Π°ΠΆΠ΅Π½ΠΈΠ΅

наглядно
Π΄Π΅ΠΌΠΎΠ½ΡΡ‚Ρ€ΠΈΡ€ΡƒΡŽΡ‰Π΅Π΅ Ρ‚ΠΎ, Ρ‡Ρ‚ΠΎ РполстрСмится
ΠΊ Π½ΡƒΠ»ΡŽ ΠΊΠ°ΠΊ ΠΏΡ€ΠΈ ΡƒΠΌΠ΅Π½ΡŒΡˆΠ΅Π½ΠΈΠΈ
RΠ΄ΠΎ
нуля, Ρ‚Π°ΠΊ ΠΈ ΠΏΡ€ΠΈ Π΅Π³ΠΎ бСсконСчном ΡƒΠ²Π΅Π»ΠΈΡ‡Π΅Π½ΠΈΠΈ,
Ρ‚.ΠΊ. Π² ΠΎΠ±ΠΎΠΈΡ… случаях
Π·Π½Π°ΠΌΠ΅Π½Π°Ρ‚Π΅Π»ΡŒ этого выраТСния стрСмится
ΠΊ бСсконСчности. Π­Ρ‚ΠΎ ΠΎΠ·Π½Π°Ρ‡Π°Π΅Ρ‚,
Ρ‡Ρ‚ΠΎ ΠΏΡ€ΠΈ Π½Π΅ΠΊΠΎΡ‚ΠΎΡ€ΠΎΠΌ ΠΎΠΏΡ‚ΠΈΠΌΠ°Π»ΡŒΠ½ΠΎΠΌ Π·Π½Π°Ρ‡Π΅Π½ΠΈΠΈ
RполСзная
ΠΌΠΎΡ‰Π½ΠΎΡΡ‚ΡŒ достигаСт
максимального значСния

  1. ΠžΠΏΡ€Π΅Π΄Π΅Π»ΠΈΡ‚ΡŒ
    ΠΎΠΏΡ‚ΠΈΠΌΠ°Π»ΡŒΠ½ΠΎΠ΅ Π·Π½Π°Ρ‡Π΅Π½ΠΈΠ΅ R,
    Π°
    Ρ‚Π°ΠΊΠΆΠ΅ ΠΈ Π·Π½Π°Ρ‡Π΅Π½ΠΈΠ΅ ,
    ΠΌΠΎΠΆΠ½ΠΎ,
    приравняв Π½ΡƒΠ»ΡŽ ΠΏΠ΅Ρ€Π²ΡƒΡŽ ΠΏΡ€ΠΎΠΈΠ·Π²ΠΎΠ΄Π½ΡƒΡŽ
    Ρ„ΡƒΠ½ΠΊΡ†ΠΈΠΈ Рпоя
    =
    f(R)
    ΠΏo
    R:
  2. (14)
  3. Как
    Π²ΠΈΠ΄Π½ΠΎ, ΠΏΠΎΠ»ΡƒΡ‡Π΅Π½Π½ΠΎΠ΅ равСнство ΡΠΎΠ±Π»ΡŽΠ΄Π°Π΅Ρ‚ΡΡ
    ΠΏΡ€ΠΈ условии
  4. (15)
  5. ΠΈΠ·
    Ρ‡Π΅Π³ΠΎ слСдуСт, Ρ‡Ρ‚ΠΎ R
    =
    r.
    Π’Π°ΠΊΠΈΠΌ
    ΠΎΠ±Ρ€Π°Π·ΠΎΠΌ, ΠΏΡ€ΠΈ сопротивлСнии внСшнСй Ρ†Π΅ΠΏΠΈ
    R,
    Ρ€Π°Π²Π½ΠΎΠΌ
    ΡΠΎΠΏΡ€ΠΎΡ‚ΠΈΠ²Π»Π΅Π½ΠΈΡŽ Π²Π½ΡƒΡ‚Ρ€Π΅Π½Π½Π΅ΠΉ Ρ†Π΅ΠΏΠΈ Π³,
    полСзная
    ΠΌΠΎΡ‰Π½ΠΎΡΡ‚ΡŒ источника Ρ‚ΠΎΠΊΠ°
    ΠΈΠΌΠ΅Π΅Ρ‚ максимальноС Π·Π½Π°Ρ‡Π΅Π½ΠΈΠ΅, ΠΊΠΎΡ‚ΠΎΡ€ΠΎΠ΅
    ΠΌΠΎΠΆΠ΅Ρ‚ Π±Ρ‹Ρ‚ΡŒ Π½Π°ΠΉΠ΄Π΅Π½ΠΎ ΠΏΠΎ Ρ„ΠΎΡ€ΠΌΡƒΠ»Π΅:

Π“Ρ€Π°Ρ„ΠΈΠΊ
зависимости PΠΏΠΎΠ»=f(R)
ΠΏΠΎΠΊΠ°Π·Π°Π½
Π½Π° рисункС 6.

  • Π²)
    ΠšΠΎΡΡ„Ρ„ΠΈΡ†ΠΈΠ΅Π½Ρ‚
    ΠΏΠΎΠ»Π΅Π·Π½ΠΎΠ³ΠΎ дСйствия.
  • Π’Π΅Π»ΠΈΡ‡ΠΈΠ½Π°
    коэффициСнта ΠΏΠΎΠ»Π΅Π·Π½ΠΎΠ³ΠΎ дСйствия Ρ†Π΅ΠΏΠΈ
    Π³| источника Ρ‚ΠΎΠΊΠ°, Π² соотвСтствии
    с ΠΎΠΏΡ€Π΅Π΄Π΅Π»Π΅Π½ΠΈΠ΅ΠΌ, составляСт:
  • (17)

ΠŸΡ€ΠΈ
R
Π²Π΅Π»ΠΈΡ‡ΠΈΠ½Π° 0, ΠΏΡ€ΠΈR

pngΒ»>
Π²Π΅Π»ΠΈΡ‡ΠΈΠ½Π° 100%. Π’ послСднСмслучаС
Π ΠΏΠΎΠ»
стрСмится ΠΊ Π½ΡƒΠ»ΡŽ, ΠΈ Ρ‚Π°ΠΊΠΈΠ΅ Ρ€Π΅ΠΆΠΈΠΌΡ‹ Ρ€Π°Π±ΠΎΡ‚Ρ‹
источника Π½Π΅ ΠΏΡ€Π΅Π΄ΡΡ‚Π°Π²Π»ΡΡŽΡ‚
практичСского интСрСса. Π“Ρ€Π°Ρ„ΠΈΠΊ зависимости
ΠšΠŸΠ”

pngΒ»>источника
Ρ‚ΠΎΠΊΠ° ΠΎΡ‚ Π²Π΅Π»ΠΈΡ‡ΠΈΠ½Ρ‹ Π½Π°Π³Ρ€ΡƒΠ·ΠΊΠΈ RΠΏΠΎΠΊΠ°Π·Π°Π½
Π½Π° рисункС 7.

Π˜ΡΡ‚ΠΎΡ‡Π½ΠΈΠΊ: https://studfile.net/preview/2147140/page:3/

Π˜ΡΠΏΠΎΠ»ΡŒΠ·ΡƒΠΉΡ‚Π΅ Π·Π°ΠΊΠΎΠ½ Ома — Π€ΠΈΠ·ΠΈΠΊΠ° срСднСй ΡˆΠΊΠΎΠ»Ρ‹

Если Π²Ρ‹ считаСтС, Ρ‡Ρ‚ΠΎ ΠΊΠΎΠ½Ρ‚Π΅Π½Ρ‚, доступный Ρ‡Π΅Ρ€Π΅Π· Π’Π΅Π±-сайт (ΠΊΠ°ΠΊ ΠΎΠΏΡ€Π΅Π΄Π΅Π»Π΅Π½ΠΎ Π² Π½Π°ΡˆΠΈΡ… Условиях обслуТивания), Π½Π°Ρ€ΡƒΡˆΠ°Π΅Ρ‚ ΠΈΠ»ΠΈ Π΄Ρ€ΡƒΠ³ΠΈΠ΅ ваши авторскиС ΠΏΡ€Π°Π²Π°, сообщитС Π½Π°ΠΌ, ΠΎΡ‚ΠΏΡ€Π°Π²ΠΈΠ² письмСнноС ΡƒΠ²Π΅Π΄ΠΎΠΌΠ»Π΅Π½ΠΈΠ΅ (Β«Π£Π²Π΅Π΄ΠΎΠΌΠ»Π΅Π½ΠΈΠ΅ ΠΎ Π½Π°Ρ€ΡƒΡˆΠ΅Π½ΠΈΠΈΒ»), содСрТащСС Π² ΠΈΠ½Ρ„ΠΎΡ€ΠΌΠ°Ρ†ΠΈΡŽ, ΠΎΠΏΠΈΡΠ°Π½Π½ΡƒΡŽ Π½ΠΈΠΆΠ΅, Π½Π°Π·Π½Π°Ρ‡Π΅Π½Π½ΠΎΠΌΡƒ Π½ΠΈΠΆΠ΅ Π°Π³Π΅Π½Ρ‚Ρƒ. Если Ρ€Π΅ΠΏΠ΅Ρ‚ΠΈΡ‚ΠΎΡ€Ρ‹ унивСрситСта ΠΏΡ€Π΅Π΄ΠΏΡ€ΠΈΠΌΡƒΡ‚ дСйствия Π² ΠΎΡ‚Π²Π΅Ρ‚ Π½Π° Π°Π½ Π£Π²Π΅Π΄ΠΎΠΌΠ»Π΅Π½ΠΈΠ΅ ΠΎ Π½Π°Ρ€ΡƒΡˆΠ΅Π½ΠΈΠΈ, ΠΎΠ½ΠΎ ΠΏΡ€Π΅Π΄ΠΏΡ€ΠΈΠΌΠ΅Ρ‚ Π΄ΠΎΠ±Ρ€ΠΎΡΠΎΠ²Π΅ΡΡ‚Π½ΡƒΡŽ ΠΏΠΎΠΏΡ‹Ρ‚ΠΊΡƒ ΡΠ²ΡΠ·Π°Ρ‚ΡŒΡΡ со стороной, которая прСдоставила Ρ‚Π°ΠΊΠΎΠΉ ΠΊΠΎΠ½Ρ‚Π΅Π½Ρ‚ срСдствами самого послСднСго адрСса элСктронной ΠΏΠΎΡ‡Ρ‚Ρ‹, Ссли Ρ‚Π°ΠΊΠΎΠ²ΠΎΠΉ имССтся, прСдоставлСнного Ρ‚Π°ΠΊΠΎΠΉ стороной Varsity Tutors.

Π’Π°ΡˆΠ΅ Π£Π²Π΅Π΄ΠΎΠΌΠ»Π΅Π½ΠΈΠ΅ ΠΎ Π½Π°Ρ€ΡƒΡˆΠ΅Π½ΠΈΠΈ ΠΏΡ€Π°Π² ΠΌΠΎΠΆΠ΅Ρ‚ Π±Ρ‹Ρ‚ΡŒ ΠΎΡ‚ΠΏΡ€Π°Π²Π»Π΅Π½ΠΎ сторонС, ΠΏΡ€Π΅Π΄ΠΎΡΡ‚Π°Π²ΠΈΠ²ΡˆΠ΅ΠΉ доступ ΠΊ ΠΊΠΎΠ½Ρ‚Π΅Π½Ρ‚Ρƒ, ΠΈΠ»ΠΈ Ρ‚Ρ€Π΅Ρ‚ΡŒΠΈΠΌ Π»ΠΈΡ†Π°ΠΌ, Ρ‚Π°ΠΊΠΈΠΌ ΠΊΠ°ΠΊ Π² качСствС ChillingEffects.org.

ΠžΠ±Ρ€Π°Ρ‚ΠΈΡ‚Π΅ Π²Π½ΠΈΠΌΠ°Π½ΠΈΠ΅, Ρ‡Ρ‚ΠΎ Π²Ρ‹ Π±ΡƒΠ΄Π΅Ρ‚Π΅ нСсти ΠΎΡ‚Π²Π΅Ρ‚ΡΡ‚Π²Π΅Π½Π½ΠΎΡΡ‚ΡŒ Π·Π° ΡƒΡ‰Π΅Ρ€Π± (Π²ΠΊΠ»ΡŽΡ‡Π°Ρ расходы ΠΈ Π³ΠΎΠ½ΠΎΡ€Π°Ρ€Ρ‹ Π°Π΄Π²ΠΎΠΊΠ°Ρ‚Π°ΠΌ), Ссли Π²Ρ‹ сущСствСнно ΠΈΡΠΊΠ°ΠΆΠ°Ρ‚ΡŒ ΠΈΠ½Ρ„ΠΎΡ€ΠΌΠ°Ρ†ΠΈΡŽ ΠΎ Ρ‚ΠΎΠΌ, Ρ‡Ρ‚ΠΎ ΠΏΡ€ΠΎΠ΄ΡƒΠΊΡ‚ ΠΈΠ»ΠΈ дСйствиС Π½Π°Ρ€ΡƒΡˆΠ°Π΅Ρ‚ ваши авторскиС ΠΏΡ€Π°Π²Π°. Π’Π°ΠΊΠΈΠΌ ΠΎΠ±Ρ€Π°Π·ΠΎΠΌ, Ссли Π²Ρ‹ Π½Π΅ ΡƒΠ²Π΅Ρ€Π΅Π½Ρ‹, Ρ‡Ρ‚ΠΎ ΠΊΠΎΠ½Ρ‚Π΅Π½Ρ‚ находится Π½Π° Π’Π΅Π±-сайтС ΠΈΠ»ΠΈ ΠΏΠΎ ссылкС с Π½Π΅Π³ΠΎ Π½Π°Ρ€ΡƒΡˆΠ°Π΅Ρ‚ ваши авторскиС ΠΏΡ€Π°Π²Π°, Π²Π°ΠΌ слСдуСт сначала ΠΎΠ±Ρ€Π°Ρ‚ΠΈΡ‚ΡŒΡΡ ΠΊ ΡŽΡ€ΠΈΡΡ‚Ρƒ.

Π§Ρ‚ΠΎΠ±Ρ‹ ΠΎΡ‚ΠΏΡ€Π°Π²ΠΈΡ‚ΡŒ ΡƒΠ²Π΅Π΄ΠΎΠΌΠ»Π΅Π½ΠΈΠ΅, Π²Ρ‹ΠΏΠΎΠ»Π½ΠΈΡ‚Π΅ ΡΠ»Π΅Π΄ΡƒΡŽΡ‰ΠΈΠ΅ дСйствия:

Π’Ρ‹ Π΄ΠΎΠ»ΠΆΠ½Ρ‹ Π²ΠΊΠ»ΡŽΡ‡ΠΈΡ‚ΡŒ ΡΠ»Π΅Π΄ΡƒΡŽΡ‰Π΅Π΅:

ЀизичСская ΠΈΠ»ΠΈ элСктронная подпись правообладатСля ΠΈΠ»ΠΈ Π»ΠΈΡ†Π°, ΡƒΠΏΠΎΠ»Π½ΠΎΠΌΠΎΡ‡Π΅Π½Π½ΠΎΠ³ΠΎ Π΄Π΅ΠΉΡΡ‚Π²ΠΎΠ²Π°Ρ‚ΡŒ ΠΎΡ‚ ΠΈΡ… ΠΈΠΌΠ΅Π½ΠΈ; Π˜Π΄Π΅Π½Ρ‚ΠΈΡ„ΠΈΠΊΠ°Ρ†ΠΈΡ авторских ΠΏΡ€Π°Π², ΠΊΠΎΡ‚ΠΎΡ€Ρ‹Π΅, ΠΊΠ°ΠΊ утвСрТдаСтся, Π±Ρ‹Π»ΠΈ Π½Π°Ρ€ΡƒΡˆΠ΅Π½Ρ‹; ОписаниС Ρ…Π°Ρ€Π°ΠΊΡ‚Π΅Ρ€Π° ΠΈ Ρ‚ΠΎΡ‡Π½ΠΎΠ³ΠΎ мСстонахоТдСния ΠΊΠΎΠ½Ρ‚Π΅Π½Ρ‚Π°, ΠΊΠΎΡ‚ΠΎΡ€Ρ‹ΠΉ, ΠΏΠΎ Π²Π°ΡˆΠ΅ΠΌΡƒ мнСнию, Π½Π°Ρ€ΡƒΡˆΠ°Π΅Ρ‚ ваши авторскиС ΠΏΡ€Π°Π²Π°, Π² \ достаточно подробностСй, Ρ‡Ρ‚ΠΎΠ±Ρ‹ ΠΏΠΎΠ·Π²ΠΎΠ»ΠΈΡ‚ΡŒ Ρ€Π΅ΠΏΠ΅Ρ‚ΠΈΡ‚ΠΎΡ€Π°ΠΌ унивСрситСтских школ Π½Π°ΠΉΡ‚ΠΈ ΠΈ Ρ‚ΠΎΡ‡Π½ΠΎ ΠΈΠ΄Π΅Π½Ρ‚ΠΈΡ„ΠΈΡ†ΠΈΡ€ΠΎΠ²Π°Ρ‚ΡŒ этот ΠΊΠΎΠ½Ρ‚Π΅Π½Ρ‚; Π½Π°ΠΏΡ€ΠΈΠΌΠ΅Ρ€ Π½Π°ΠΌ трСбуСтся Π° ссылка Π½Π° ΠΊΠΎΠ½ΠΊΡ€Π΅Ρ‚Π½Ρ‹ΠΉ вопрос (Π° Π½Π΅ Ρ‚ΠΎΠ»ΡŒΠΊΠΎ Π½Π° Π½Π°Π·Π²Π°Π½ΠΈΠ΅ вопроса), ΠΊΠΎΡ‚ΠΎΡ€Ρ‹ΠΉ содСрТит содСрТаниС ΠΈ описаниС ΠΊ ΠΊΠ°ΠΊΠΎΠΉ ΠΊΠΎΠ½ΠΊΡ€Π΅Ρ‚Π½ΠΎΠΉ части вопроса — ΠΈΠ·ΠΎΠ±Ρ€Π°ΠΆΠ΅Π½ΠΈΡŽ, ссылкС, тСксту ΠΈ Ρ‚. Π΄. — относится ваша ΠΆΠ°Π»ΠΎΠ±Π°; Π’Π°ΡˆΠ΅ имя, адрСс, Π½ΠΎΠΌΠ΅Ρ€ Ρ‚Π΅Π»Π΅Ρ„ΠΎΠ½Π° ΠΈ адрСс элСктронной ΠΏΠΎΡ‡Ρ‚Ρ‹; Π° Ρ‚Π°ΠΊΠΆΠ΅ Π’Π°ΡˆΠ΅ заявлСниС: (Π°) Π²Ρ‹ добросовСстно ΠΏΠΎΠ»Π°Π³Π°Π΅Ρ‚Π΅, Ρ‡Ρ‚ΠΎ использованиС ΠΊΠΎΠ½Ρ‚Π΅Π½Ρ‚Π°, ΠΊΠΎΡ‚ΠΎΡ€Ρ‹ΠΉ, ΠΏΠΎ Π²Π°ΡˆΠ΅ΠΌΡƒ мнСнию, Π½Π°Ρ€ΡƒΡˆΠ°Π΅Ρ‚ ваши авторскиС ΠΏΡ€Π°Π²Π° Π½Π΅ Ρ€Π°Π·Ρ€Π΅ΡˆΠ΅Π½Ρ‹ Π·Π°ΠΊΠΎΠ½ΠΎΠΌ, Π²Π»Π°Π΄Π΅Π»ΡŒΡ†Π΅ΠΌ авторских ΠΏΡ€Π°Π² ΠΈΠ»ΠΈ Π΅Π³ΠΎ Π°Π³Π΅Π½Ρ‚ΠΎΠΌ; (Π±) Ρ‡Ρ‚ΠΎ всС информация, содСрТащаяся Π² вашСм Π£Π²Π΅Π΄ΠΎΠΌΠ»Π΅Π½ΠΈΠΈ ΠΎ Π½Π°Ρ€ΡƒΡˆΠ΅Π½ΠΈΠΈ, являСтся Ρ‚ΠΎΡ‡Π½ΠΎΠΉ, ΠΈ (c) ΠΏΠΎΠ΄ страхом наказания Π·Π° Π»ΠΆΠ΅ΡΠ²ΠΈΠ΄Π΅Ρ‚Π΅Π»ΡŒΡΡ‚Π²ΠΎ, Ρ‡Ρ‚ΠΎ Π²Ρ‹ Π»ΠΈΠ±ΠΎ Π²Π»Π°Π΄Π΅Π»Π΅Ρ† авторских ΠΏΡ€Π°Π², Π»ΠΈΠ±ΠΎ Π»ΠΈΡ†ΠΎ, ΡƒΠΏΠΎΠ»Π½ΠΎΠΌΠΎΡ‡Π΅Π½Π½ΠΎΠ΅ Π΄Π΅ΠΉΡΡ‚Π²ΠΎΠ²Π°Ρ‚ΡŒ ΠΎΡ‚ ΠΈΡ… ΠΈΠΌΠ΅Π½ΠΈ.

ΠžΡ‚ΠΏΡ€Π°Π²ΡŒΡ‚Π΅ ΠΆΠ°Π»ΠΎΠ±Ρƒ Π½Π°ΡˆΠ΅ΠΌΡƒ ΡƒΠΏΠΎΠ»Π½ΠΎΠΌΠΎΡ‡Π΅Π½Π½ΠΎΠΌΡƒ Π°Π³Π΅Π½Ρ‚Ρƒ ΠΏΠΎ адрСсу:

Π§Π°Ρ€Π»ΡŒΠ· Кон Varsity Tutors LLC
101 S. Hanley Rd, Suite 300
St. Louis, MO 63105

Или Π·Π°ΠΏΠΎΠ»Π½ΠΈΡ‚Π΅ Ρ„ΠΎΡ€ΠΌΡƒ Π½ΠΈΠΆΠ΅:

Π¦Π΅ΠΏΠΈ сСрии

19.2 | Texas Gateway

ЭлСктричСскиС Ρ†Π΅ΠΏΠΈ ΠΈ рСзисторы

Π’Π΅ΠΏΠ΅Ρ€ΡŒ, ΠΊΠΎΠ³Π΄Π° ΠΌΡ‹ ΠΏΠΎΠ½ΠΈΠΌΠ°Π΅ΠΌ понятиС элСктричСского Ρ‚ΠΎΠΊΠ°, Π΄Π°Π²Π°ΠΉΡ‚Π΅ посмотрим, Ρ‡Ρ‚ΠΎ ΠΌΡ‹ ΠΌΠΎΠΆΠ΅ΠΌ с Π½ΠΈΠΌ ΡΠ΄Π΅Π»Π°Ρ‚ΡŒ.Как Π²Ρ‹, нСсомнСнно, Π·Π½Π°Π΅Ρ‚Π΅, соврСмСнный ΠΎΠ±Ρ€Π°Π· ΠΆΠΈΠ·Π½ΠΈ Π² Π·Π½Π°Ρ‡ΠΈΡ‚Π΅Π»ΡŒΠ½ΠΎΠΉ стСпСни зависит ΠΎΡ‚ элСктричСских устройств. Π­Ρ‚ΠΈ устройства содСрТат ΠΎΡ€ΠΈΠ³ΠΈΠ½Π°Π»ΡŒΠ½Ρ‹Π΅ элСктричСскиС Ρ†Π΅ΠΏΠΈ, ΠΏΡ€Π΅Π΄ΡΡ‚Π°Π²Π»ΡΡŽΡ‰ΠΈΠ΅ собой Π·Π°ΠΊΠΎΠ½Ρ‡Π΅Π½Π½Ρ‹Π΅ Π·Π°ΠΌΠΊΠ½ΡƒΡ‚Ρ‹Π΅ ΠΏΡƒΡ‚ΠΈ, ΠΏΠΎ ΠΊΠΎΡ‚ΠΎΡ€Ρ‹ΠΌ ΠΏΡ€ΠΎΡ‚Π΅ΠΊΠ°Π΅Ρ‚ элСктричСский Ρ‚ΠΎΠΊ. Π’ΠΎΠ·Π²Ρ€Π°Ρ‰Π°ΡΡΡŒ ΠΊ нашСй Π°Π½Π°Π»ΠΎΠ³ΠΈΠΈ с Π²ΠΎΠ΄ΠΎΠΉ, элСктричСская Ρ†Π΅ΠΏΡŒ ΠΏΡ€Π΅Π΄Π½Π°Π·Π½Π°Ρ‡Π΅Π½Π° для элСктричСского заряда, ΠΊΠ°ΠΊ ΡΠ΅Ρ‚ΡŒ Ρ‚Ρ€ΡƒΠ± — для Π²ΠΎΠ΄Ρ‹: элСктричСская Ρ†Π΅ΠΏΡŒ направляСт элСктричСский заряд ΠΎΡ‚ ΠΎΠ΄Π½ΠΎΠΉ Ρ‚ΠΎΡ‡ΠΊΠΈ ΠΊ Π΄Ρ€ΡƒΠ³ΠΎΠΉ, пропуская заряд Ρ‡Π΅Ρ€Π΅Π· Ρ€Π°Π·Π»ΠΈΡ‡Π½Ρ‹Π΅ устройства ΠΏΠΎ ΠΏΡƒΡ‚ΠΈ для извлСчСния Ρ€Π°Π±ΠΎΡ‚Ρ‹ ΠΈΠ»ΠΈ ΠΈΠ½Ρ„ΠΎΡ€ΠΌΠ°Ρ†ΠΈΠΈ.

ЭлСктричСскиС Ρ†Π΅ΠΏΠΈ ΠΈΠ·Π³ΠΎΡ‚Π°Π²Π»ΠΈΠ²Π°ΡŽΡ‚ΡΡ ΠΈΠ· ΠΌΠ½ΠΎΠ³ΠΈΡ… ΠΌΠ°Ρ‚Π΅Ρ€ΠΈΠ°Π»ΠΎΠ² ΠΈ ΠΎΡ…Π²Π°Ρ‚Ρ‹Π²Π°ΡŽΡ‚ ΠΎΠ³Ρ€ΠΎΠΌΠ½Ρ‹ΠΉ Π΄ΠΈΠ°ΠΏΠ°Π·ΠΎΠ½ Ρ€Π°Π·ΠΌΠ΅Ρ€ΠΎΠ², ΠΊΠ°ΠΊ ΠΏΠΎΠΊΠ°Π·Π°Π½ΠΎ Π½Π° рисункС 19.9. ΠšΠΎΠΌΠΏΡŒΡŽΡ‚Π΅Ρ€Ρ‹ ΠΈ сотовыС Ρ‚Π΅Π»Π΅Ρ„ΠΎΠ½Ρ‹ содСрТат элСктричСскиС Ρ†Π΅ΠΏΠΈ, Ρ€Π°Π·ΠΌΠ΅Ρ€Ρ‹ ΠΊΠΎΡ‚ΠΎΡ€Ρ‹Ρ… ΠΌΠΎΠ³ΡƒΡ‚ ΡΠΎΡΡ‚Π°Π²Π»ΡΡ‚ΡŒ ΠΏΡ€ΠΈΠΌΠ΅Ρ€Π½ΠΎ ΠΌΠΈΠ»Π»ΠΈΠ°Ρ€Π΄Π½ΡƒΡŽ долю ΠΌΠ΅Ρ‚Ρ€Π° (Π½Π°Π½ΠΎΠΌΠ΅Ρ‚Ρ€Π° ΠΈΠ»ΠΈ 10-9 ΠΌ Β· 10-9 ΠΌ). ΠŸΡƒΡ‚ΠΈ, ΠΊΠΎΡ‚ΠΎΡ€Ρ‹Π΅ Π½Π°ΠΏΡ€Π°Π²Π»ΡΡŽΡ‚ Ρ‚ΠΎΠΊ Π² этих устройствах, ΡΠΎΠ·Π΄Π°ΡŽΡ‚ΡΡ свСрхточной химичСской ΠΎΠ±Ρ€Π°Π±ΠΎΡ‚ΠΊΠΎΠΉ крСмния ΠΈΠ»ΠΈ Π΄Ρ€ΡƒΠ³ΠΈΡ… ΠΏΠΎΠ»ΡƒΠΏΡ€ΠΎΠ²ΠΎΠ΄Π½ΠΈΠΊΠΎΠ². Π‘ Π΄Ρ€ΡƒΠ³ΠΎΠΉ стороны, большиС энСргосистСмы содСрТат элСктричСскиС Ρ†Π΅ΠΏΠΈ, характСристики ΠΊΠΎΡ‚ΠΎΡ€Ρ‹Ρ… ΠΈΠ·ΠΌΠ΅Ρ€ΡΡŽΡ‚ΡΡ Π² ΠΌΠ°ΡΡˆΡ‚Π°Π±Π΅ ΠΌΠ΅Ρ‚Ρ€ΠΎΠ². Π­Ρ‚ΠΈ систСмы пСрСносят Ρ‚Π°ΠΊΠΈΠ΅ большиС элСктричСскиС Ρ‚ΠΎΠΊΠΈ, Ρ‡Ρ‚ΠΎ ΠΈΡ… физичСскиС Ρ€Π°Π·ΠΌΠ΅Ρ€Ρ‹ Π΄ΠΎΠ»ΠΆΠ½Ρ‹ Π±Ρ‹Ρ‚ΡŒ ΠΎΡ‚Π½ΠΎΡΠΈΡ‚Π΅Π»ΡŒΠ½ΠΎ большими.

Рисунок 19.9 На Ρ„ΠΎΡ‚ΠΎΠ³Ρ€Π°Ρ„ΠΈΠΈ слСва ΠΏΠΎΠΊΠ°Π·Π°Π½Π° микросхСма , содСрТащая ΡΠ»ΠΎΠΆΠ½ΡƒΡŽ ΠΈΠ½Ρ‚Π΅Π³Ρ€Π°Π»ΡŒΠ½ΡƒΡŽ ΡΠ»Π΅ΠΊΡ‚Ρ€ΠΈΡ‡Π΅ΡΠΊΡƒΡŽ схСму. Π’Π°ΠΊΠΈΠ΅ Ρ‡ΠΈΠΏΡ‹ Π»Π΅ΠΆΠ°Ρ‚ Π² основС Ρ‚Π°ΠΊΠΈΡ… устройств, ΠΊΠ°ΠΊ ΠΊΠΎΠΌΠΏΡŒΡŽΡ‚Π΅Ρ€Ρ‹ ΠΈ сотовыС Ρ‚Π΅Π»Π΅Ρ„ΠΎΠ½Ρ‹. На Ρ„ΠΎΡ‚ΠΎΠ³Ρ€Π°Ρ„ΠΈΠΈ справа ΠΏΠΎΠΊΠ°Π·Π°Π½Π° типовая элСктричСская схСма, нСобходимая для ΠΏΠ΅Ρ€Π΅Π΄Π°Ρ‡ΠΈ элСктроэнСргии большой мощности.

ΠŸΡƒΡ‚ΠΈ, ΠΎΠ±Ρ€Π°Π·ΡƒΡŽΡ‰ΠΈΠ΅ элСктричСскиС Ρ†Π΅ΠΏΠΈ, сдСланы ΠΈΠ· проводящСго ΠΌΠ°Ρ‚Π΅Ρ€ΠΈΠ°Π»Π°, ΠΎΠ±Ρ‹Ρ‡Π½ΠΎ ΠΈΠ· ΠΌΠ΅Ρ‚Π°Π»Π»Π° Π² макроскопичСских цСпях. НапримСр, ΠΌΠ΅Π΄Π½Ρ‹Π΅ ΠΏΡ€ΠΎΠ²ΠΎΠ΄Π° Π²Π½ΡƒΡ‚Ρ€ΠΈ здания ΡˆΠΊΠΎΠ»Ρ‹ ΠΎΠ±Ρ€Π°Π·ΡƒΡŽΡ‚ элСктричСскиС Ρ†Π΅ΠΏΠΈ, ΠΏΠΈΡ‚Π°ΡŽΡ‰ΠΈΠ΅ освСщСниС, ΠΏΡ€ΠΎΠ΅ΠΊΡ‚ΠΎΡ€Ρ‹, экраны, Π΄ΠΈΠ½Π°ΠΌΠΈΠΊΠΈ ΠΈ Ρ‚. Π”.Для прСдставлСния элСктричСской схСмы рисуСм ΠΏΡ€ΠΈΠ½Ρ†ΠΈΠΏΠΈΠ°Π»ΡŒΠ½Ρ‹Π΅ схСмы. ΠœΡ‹ ΠΈΡΠΏΠΎΠ»ΡŒΠ·ΡƒΠ΅ΠΌ Π»ΠΈΠ½ΠΈΠΈ ΠΈ символы для обозначСния элСмСнтов схСмы. ΠŸΡ€ΠΎΡΡ‚Π°Ρ элСктричСская схСма ΠΏΠΎΠΊΠ°Π·Π°Π½Π° Π² Π»Π΅Π²ΠΎΠΉ части рисунка 19.10. Π‘ΠΏΡ€Π°Π²Π° — Π°Π½Π°Π»ΠΎΠ³ΠΈΡ‡Π½Ρ‹ΠΉ водяной ΠΊΠΎΠ½Ρ‚ΡƒΡ€, ΠΊΠΎΡ‚ΠΎΡ€Ρ‹ΠΉ ΠΌΡ‹ обсудим Π½ΠΈΠΆΠ΅.

Рисунок 19.10 Π‘Π»Π΅Π²Π° ΠΏΡ€ΠΈΠ²Π΅Π΄Π΅Π½Π° ΠΏΡ€ΠΈΠ½Ρ†ΠΈΠΏΠΈΠ°Π»ΡŒΠ½Π°Ρ схСма, ΠΏΠΎΠΊΠ°Π·Ρ‹Π²Π°ΡŽΡ‰Π°Ρ Π±Π°Ρ‚Π°Ρ€Π΅ΡŽ (красным), рСзистор (Ρ‡Π΅Ρ€Π½Ρ‹ΠΉ Π·ΠΈΠ³Π·Π°Π³ΠΎΠΎΠ±Ρ€Π°Π·Π½Ρ‹ΠΉ элСмСнт) ΠΈ Ρ‚ΠΎΠΊ I . Π‘ΠΏΡ€Π°Π²Π° Π°Π½Π°Π»ΠΎΠ³ΠΈΡ‡Π½Ρ‹ΠΉ водяной ΠΊΠΎΠ½Ρ‚ΡƒΡ€. Насос ΠΏΠΎΠ΄ΠΎΠ±Π΅Π½ Π±Π°Ρ‚Π°Ρ€Π΅Π΅, пСсочный Ρ„ΠΈΠ»ΡŒΡ‚Ρ€ ΠΏΠΎΠ΄ΠΎΠ±Π΅Π½ рСзистору, Ρ‚ΠΎΠΊ Π²ΠΎΠ΄Ρ‹ ΠΏΠΎΠ΄ΠΎΠ±Π΅Π½ элСктричСскому Ρ‚ΠΎΠΊΡƒ, Π° Ρ€Π΅Π·Π΅Ρ€Π²ΡƒΠ°Ρ€ ΠΏΠΎΠ΄ΠΎΠ±Π΅Π½ Π·Π΅ΠΌΠ»Π΅.

Π•ΡΡ‚ΡŒ ΠΌΠ½ΠΎΠ³ΠΎ Ρ€Π°Π·Π½Ρ‹Ρ… символов, ΠΊΠΎΡ‚ΠΎΡ€Ρ‹Π΅ ΡƒΡ‡Π΅Π½Ρ‹Π΅ ΠΈ ΠΈΠ½ΠΆΠ΅Π½Π΅Ρ€Ρ‹ ΠΈΡΠΏΠΎΠ»ΡŒΠ·ΡƒΡŽΡ‚ Π² ΠΏΡ€ΠΈΠ½Ρ†ΠΈΠΏΠΈΠ°Π»ΡŒΠ½Ρ‹Ρ… схСмах, Π½ΠΎ ΠΌΡ‹ сосрСдоточимся Π½Π° Ρ‡Π΅Ρ‚Ρ‹Ρ€Π΅Ρ… основных символах: ΠΏΡ€ΠΎΠ²ΠΎΠ΄, батарСя ΠΈΠ»ΠΈ источник напряТСния, рСзисторы ΠΈ зСмля. Π’ΠΎΠ½ΠΊΠΈΠ΅ Ρ‡Π΅Ρ€Π½Ρ‹Π΅ Π»ΠΈΠ½ΠΈΠΈ Π½Π° элСктричСской схСмС ΠΎΠ±ΠΎΠ·Π½Π°Ρ‡Π°ΡŽΡ‚ ΠΏΡƒΡ‚ΡŒ, ΠΏΠΎ ΠΊΠΎΡ‚ΠΎΡ€ΠΎΠΌΡƒ Π΄ΠΎΠ»ΠΆΠ΅Π½ ΠΈΠ΄Ρ‚ΠΈ элСктричСский заряд. ΠŸΡ€Π΅Π΄ΠΏΠΎΠ»Π°Π³Π°Π΅Ρ‚ΡΡ, Ρ‡Ρ‚ΠΎ эти ΠΏΡƒΡ‚ΠΈ ΡΠ²Π»ΡΡŽΡ‚ΡΡ ΠΈΠ΄Π΅Π°Π»ΡŒΠ½Ρ‹ΠΌΠΈ ΠΏΡ€ΠΎΠ²ΠΎΠ΄Π½ΠΈΠΊΠ°ΠΌΠΈ, поэтому элСктричСский заряд ΠΌΠΎΠΆΠ΅Ρ‚ ΠΏΠ΅Ρ€Π΅ΠΌΠ΅Ρ‰Π°Ρ‚ΡŒΡΡ ΠΏΠΎ Π½ΠΈΠΌ Π±Π΅Π· ΠΏΠΎΡ‚Π΅Ρ€ΠΈ энСргии. На самом Π΄Π΅Π»Π΅ ΠΏΡ€ΠΎΠ²ΠΎΠ΄Π° Π² цСпях Π½Π΅ ΠΈΠ΄Π΅Π°Π»ΡŒΠ½Ρ‹, Π½ΠΎ ΠΎΠ½ΠΈ подходят для Π½Π°ΡˆΠΈΡ… Ρ†Π΅Π»Π΅ΠΉ достаточно Π±Π»ΠΈΠ·ΠΊΠΎ.

Π—ΠΈΠ³Π·Π°Π³ΠΎΠΎΠ±Ρ€Π°Π·Π½Ρ‹ΠΉ элСмСнт с ΠΌΠ°Ρ€ΠΊΠΈΡ€ΠΎΠ²ΠΊΠΎΠΉ R прСдставляСт собой рСзистор, ΠΊΠΎΡ‚ΠΎΡ€Ρ‹ΠΉ прСдставляСт собой элСмСнт схСмы, ΠΎΠ±Π΅ΡΠΏΠ΅Ρ‡ΠΈΠ²Π°ΡŽΡ‰ΠΈΠΉ извСстноС сопротивлСниС. ΠœΠ°ΠΊΡ€ΠΎΡΠΊΠΎΠΏΠΈΡ‡Π΅ΡΠΊΠΈΠ΅ рСзисторы часто ΠΈΠΌΠ΅ΡŽΡ‚ Ρ†Π²Π΅Ρ‚ΠΎΠ²ΡƒΡŽ ΠΊΠΎΠ΄ΠΈΡ€ΠΎΠ²ΠΊΡƒ для обозначСния ΠΈΡ… сопротивлСния, ΠΊΠ°ΠΊ ΠΏΠΎΠΊΠ°Π·Π°Π½ΠΎ Π½Π° рисункС 19.11.

ΠšΡ€Π°ΡΠ½Ρ‹ΠΉ элСмСнт Π½Π° рисункС 19.10 — это батарСя с ΠΎΠ±ΠΎΠ·Π½Π°Ρ‡Π΅Π½Π½Ρ‹ΠΌΠΈ ΠΏΠΎΠ»ΠΎΠΆΠΈΡ‚Π΅Π»ΡŒΠ½ΠΎΠΉ ΠΈ ΠΎΡ‚Ρ€ΠΈΡ†Π°Ρ‚Π΅Π»ΡŒΠ½ΠΎΠΉ ΠΊΠ»Π΅ΠΌΠΌΠ°ΠΌΠΈ; Π±ΠΎΠ»Π΅Π΅ длинная линия прСдставляСт собой ΠΏΠΎΠ»ΠΎΠΆΠΈΡ‚Π΅Π»ΡŒΠ½Ρ‹ΠΉ полюс Π±Π°Ρ‚Π°Ρ€Π΅ΠΈ, Π° Π±ΠΎΠ»Π΅Π΅ короткая линия — ΠΎΡ‚Ρ€ΠΈΡ†Π°Ρ‚Π΅Π»ΡŒΠ½Ρ‹ΠΉ полюс. ΠžΠ±Ρ€Π°Ρ‚ΠΈΡ‚Π΅ Π²Π½ΠΈΠΌΠ°Π½ΠΈΠ΅, Ρ‡Ρ‚ΠΎ Π·Π½Π°Ρ‡ΠΎΠΊ Π±Π°Ρ‚Π°Ρ€Π΅ΠΈ Π½Π΅ всСгда ΠΎΠΊΡ€Π°ΡˆΠ΅Π½ Π² красный Ρ†Π²Π΅Ρ‚; это сдСлано Π½Π° рисункС 19.10 просто для облСгчСния ΠΈΠ΄Π΅Π½Ρ‚ΠΈΡ„ΠΈΠΊΠ°Ρ†ΠΈΠΈ.

НаконСц, элСмСнт с ΠΌΠ°Ρ€ΠΊΠΈΡ€ΠΎΠ²ΠΊΠΎΠΉ зСмля Π² Π½ΠΈΠΆΠ½Π΅ΠΌ Π»Π΅Π²ΠΎΠΌ ΡƒΠ³Π»Ρƒ Ρ†Π΅ΠΏΠΈ ΡƒΠΊΠ°Π·Ρ‹Π²Π°Π΅Ρ‚, Ρ‡Ρ‚ΠΎ Ρ†Π΅ΠΏΡŒ ΠΏΠΎΠ΄ΠΊΠ»ΡŽΡ‡Π΅Π½Π° ΠΊ Π—Π΅ΠΌΠ»Π΅, которая прСдставляСт собой большой, практичСски Π½Π΅ΠΉΡ‚Ρ€Π°Π»ΡŒΠ½Ρ‹ΠΉ ΠΎΠ±ΡŠΠ΅ΠΊΡ‚, содСрТащий бСсконСчноС количСство заряда. Помимо ΠΏΡ€ΠΎΡ‡Π΅Π³ΠΎ, зСмля опрСдСляСт ΠΏΠΎΡ‚Π΅Π½Ρ†ΠΈΠ°Π» ΠΎΡ‚Ρ€ΠΈΡ†Π°Ρ‚Π΅Π»ΡŒΠ½ΠΎΠΉ ΠΊΠ»Π΅ΠΌΠΌΡ‹ аккумулятора. ΠžΠ±Ρ‹Ρ‡Π½ΠΎ ΠΏΠΎΡ‚Π΅Π½Ρ†ΠΈΠ°Π» Π·Π΅ΠΌΠ»ΠΈ опрСдСляСтся Ρ€Π°Π²Π½Ρ‹ΠΌ Π½ΡƒΠ»ΡŽ: Vground≑0Vground≑0. Π­Ρ‚ΠΎ ΠΎΠ·Π½Π°Ρ‡Π°Π΅Ρ‚, Ρ‡Ρ‚ΠΎ вСсь Π½ΠΈΠΆΠ½ΠΈΠΉ ΠΏΡ€ΠΎΠ²ΠΎΠ΄ Π½Π° рисункС 19.11 находится ΠΏΠΎΠ΄ напряТСниСм ноль Π²ΠΎΠ»ΡŒΡ‚.

Рисунок 19.11 НСкоторыС Ρ‚ΠΈΠΏΠΎΠ²Ρ‹Π΅ рСзисторы. Π¦Π²Π΅Ρ‚Π½Ρ‹Π΅ полосы ΡƒΠΊΠ°Π·Ρ‹Π²Π°ΡŽΡ‚ Π·Π½Π°Ρ‡Π΅Π½ΠΈΠ΅ сопротивлСния ΠΊΠ°ΠΆΠ΄ΠΎΠ³ΠΎ рСзистора.

ЭлСктричСский Ρ‚ΠΎΠΊ Π½Π° рисункС 19.10 ΠΎΠ±ΠΎΠ·Π½Π°Ρ‡Π΅Π½ синСй Π»ΠΈΠ½ΠΈΠ΅ΠΉ I . Π‘Ρ‚Ρ€Π΅Π»ΠΊΠ° ΡƒΠΊΠ°Π·Ρ‹Π²Π°Π΅Ρ‚ Π½Π°ΠΏΡ€Π°Π²Π»Π΅Π½ΠΈΠ΅, Π² ΠΊΠΎΡ‚ΠΎΡ€ΠΎΠΌ Π±ΡƒΠ΄Π΅Ρ‚ Ρ‚Π΅Ρ‡ΡŒ ΠΏΠΎΠ»ΠΎΠΆΠΈΡ‚Π΅Π»ΡŒΠ½Ρ‹ΠΉ заряд Π² этой Ρ†Π΅ΠΏΠΈ. Напомним, Ρ‡Ρ‚ΠΎ Π² ΠΌΠ΅Ρ‚Π°Π»Π»Π°Ρ… элСктроны ΡΠ²Π»ΡΡŽΡ‚ΡΡ ΠΌΠΎΠ±ΠΈΠ»ΡŒΠ½Ρ‹ΠΌΠΈ носитСлями заряда, поэтому ΠΎΡ‚Ρ€ΠΈΡ†Π°Ρ‚Π΅Π»ΡŒΠ½Ρ‹Π΅ заряды фактичСски Ρ‚Π΅ΠΊΡƒΡ‚ Π² ΠΏΡ€ΠΎΡ‚ΠΈΠ²ΠΎΠΏΠΎΠ»ΠΎΠΆΠ½ΠΎΠΌ Π½Π°ΠΏΡ€Π°Π²Π»Π΅Π½ΠΈΠΈ ΠΏΠΎ этой Ρ†Π΅ΠΏΠΈ (Ρ‚ΠΎ Π΅ΡΡ‚ΡŒ ΠΏΡ€ΠΎΡ‚ΠΈΠ² часовой стрСлки). Однако ΠΌΡ‹ ΠΏΡ€ΠΎΠ²ΠΎΠ΄ΠΈΠΌ Ρ‚ΠΎΠΊ, Ρ‡Ρ‚ΠΎΠ±Ρ‹ ΠΏΠΎΠΊΠ°Π·Π°Ρ‚ΡŒ Π½Π°ΠΏΡ€Π°Π²Π»Π΅Π½ΠΈΠ΅, Π² ΠΊΠΎΡ‚ΠΎΡ€ΠΎΠΌ Π±ΡƒΠ΄Π΅Ρ‚ Π΄Π²ΠΈΠ³Π°Ρ‚ΡŒΡΡ ΠΏΠΎΠ»ΠΎΠΆΠΈΡ‚Π΅Π»ΡŒΠ½Ρ‹ΠΉ заряд.

Π’ ΠΏΡ€Π°Π²ΠΎΠΉ части рисунка 19.10 ΠΈΠ·ΠΎΠ±Ρ€Π°ΠΆΠ΅Π½ Π°Π½Π°Π»ΠΎΠ³ΠΈΡ‡Π½Ρ‹ΠΉ водяной ΠΊΠΎΠ½Ρ‚ΡƒΡ€. Π’ΠΎΠ΄Π° ΠΏΠΎΠ΄ Π±ΠΎΠ»Π΅Π΅ высоким Π΄Π°Π²Π»Π΅Π½ΠΈΠ΅ΠΌ ΠΏΠΎΠΊΠΈΠ΄Π°Π΅Ρ‚ Π²Π΅Ρ€Ρ…Π½ΡŽΡŽ Ρ‡Π°ΡΡ‚ΡŒ насоса, Ρ‡Ρ‚ΠΎ ΠΏΠΎΠ΄ΠΎΠ±Π½ΠΎ зарядам, ΠΏΠΎΠΊΠΈΠ΄Π°ΡŽΡ‰ΠΈΠΌ ΠΏΠΎΠ»ΠΎΠΆΠΈΡ‚Π΅Π»ΡŒΠ½Ρ‹ΠΉ полюс Π±Π°Ρ‚Π°Ρ€Π΅ΠΈ. Π’ΠΎΠ΄Π° двиТСтся ΠΏΠΎ Ρ‚Ρ€ΡƒΠ±Π΅, ΠΊΠ°ΠΊ заряды ΠΏΠΎ ΠΏΡ€ΠΎΠ²ΠΎΠ»ΠΎΠΊΠ΅. Π—Π°Ρ‚Π΅ΠΌ Π²ΠΎΠ΄Π° ΠΏΡ€ΠΎΡ…ΠΎΠ΄ΠΈΡ‚ Ρ‡Π΅Ρ€Π΅Π· пСсочный Ρ„ΠΈΠ»ΡŒΡ‚Ρ€, ΠΊΠΎΡ‚ΠΎΡ€Ρ‹ΠΉ нагрСваСтся ΠΏΠΎ ΠΌΠ΅Ρ€Π΅ протСкания Π²ΠΎΠ΄Ρ‹. Π­Ρ‚ΠΎΡ‚ шаг ΠΏΠΎΠ΄ΠΎΠ±Π΅Π½ заряду, проходящСму Ρ‡Π΅Ρ€Π΅Π· рСзистор. Когда заряды проходят Ρ‡Π΅Ρ€Π΅Π· рСзистор, ΠΎΠ½ΠΈ Π½Π°Π³Ρ€Π΅Π²Π°ΡŽΡ‚ рСзистор.ΠŸΡ€ΠΎΠΉΠ΄Ρ Ρ‡Π΅Ρ€Π΅Π· пСсочный Ρ„ΠΈΠ»ΡŒΡ‚Ρ€, Π²ΠΎΠ΄Π° ΠΏΡ€Π΅ΠΎΠ±Ρ€Π°Π·ΡƒΠ΅Ρ‚ свою ΠΏΠΎΡ‚Π΅Π½Ρ†ΠΈΠ°Π»ΡŒΠ½ΡƒΡŽ ΡΠ½Π΅Ρ€Π³ΠΈΡŽ Π² Ρ‚Π΅ΠΏΠ»ΠΎ, поэтому Π΅Π΅ Π΄Π°Π²Π»Π΅Π½ΠΈΠ΅ Π½ΠΈΠΆΠ΅. Π’ΠΎΡ‡Π½ΠΎ Ρ‚Π°ΠΊ ΠΆΠ΅ заряды, выходящиС ΠΈΠ· рСзистора, ΠΏΡ€Π΅ΠΎΠ±Ρ€Π°Π·ΠΎΠ²Π°Π»ΠΈ свою ΠΏΠΎΡ‚Π΅Π½Ρ†ΠΈΠ°Π»ΡŒΠ½ΡƒΡŽ ΡΠ½Π΅Ρ€Π³ΠΈΡŽ Π² Ρ‚Π΅ΠΏΠ»ΠΎ, поэтому ΠΎΠ½ΠΈ ΠΈΠΌΠ΅ΡŽΡ‚ Π±ΠΎΠ»Π΅Π΅ Π½ΠΈΠ·ΠΊΠΎΠ΅ напряТСниС. Напомним, Ρ‡Ρ‚ΠΎ напряТСниС — это всСго лишь ΠΏΠΎΡ‚Π΅Π½Ρ†ΠΈΠ°Π»ΡŒΠ½Π°Ρ энСргия Π½Π° заряд. Π’Π°ΠΊΠΈΠΌ ΠΎΠ±Ρ€Π°Π·ΠΎΠΌ, Π΄Π°Π²Π»Π΅Π½ΠΈΠ΅ Π²ΠΎΠ΄Ρ‹ Π°Π½Π°Π»ΠΎΠ³ΠΈΡ‡Π½ΠΎ элСктричСской ΠΏΠΎΡ‚Π΅Π½Ρ†ΠΈΠ°Π»ΡŒΠ½ΠΎΠΉ энСргии (Ρ‚ΠΎ Π΅ΡΡ‚ΡŒ Π½Π°ΠΏΡ€ΡΠΆΠ΅Π½ΠΈΡŽ). Π’ΠΎΠ·Π²Ρ€Π°Ρ‰Π°ΡΡΡŒ снова ΠΊ водяному ΠΊΠΎΠ½Ρ‚ΡƒΡ€Ρƒ, ΠΌΡ‹ Π²ΠΈΠ΄ΠΈΠΌ, Ρ‡Ρ‚ΠΎ Π²ΠΎΠ΄Π° возвращаСтся Π² ниТнюю Ρ‡Π°ΡΡ‚ΡŒ насоса, Ρ‡Ρ‚ΠΎ ΠΏΠΎΠ΄ΠΎΠ±Π½ΠΎ заряду, Π²ΠΎΠ·Π²Ρ€Π°Ρ‰Π°ΡŽΡ‰Π΅ΠΌΡƒΡΡ Π½Π° ΠΎΡ‚Ρ€ΠΈΡ†Π°Ρ‚Π΅Π»ΡŒΠ½ΡƒΡŽ ΠΊΠ»Π΅ΠΌΠΌΡƒ аккумулятора.Водяной насос ΠΈΡΠΏΠΎΠ»ΡŒΠ·ΡƒΠ΅Ρ‚ источник энСргии, Ρ‡Ρ‚ΠΎΠ±Ρ‹ снова ΠΏΠ΅Ρ€Π΅ΠΊΠ°Ρ‡ΠΈΠ²Π°Ρ‚ΡŒ Π²ΠΎΠ΄Ρƒ Π΄ΠΎ высокого давлСния, создавая Π΄Π°Π²Π»Π΅Π½ΠΈΠ΅, Π½Π΅ΠΎΠ±Ρ…ΠΎΠ΄ΠΈΠΌΠΎΠ΅ для ΠΏΠΎΠ²Ρ‚ΠΎΡ€Π½ΠΎΠ³ΠΎ прохоТдСния Ρ‡Π΅Ρ€Π΅Π· ΠΊΠΎΠ½Ρ‚ΡƒΡ€. Водяной насос ΠΏΠΎΠ΄ΠΎΠ±Π΅Π½ Π±Π°Ρ‚Π°Ρ€Π΅Π΅, которая ΠΈΡΠΏΠΎΠ»ΡŒΠ·ΡƒΠ΅Ρ‚ Ρ…ΠΈΠΌΠΈΡ‡Π΅ΡΠΊΡƒΡŽ ΡΠ½Π΅Ρ€Π³ΠΈΡŽ для увСличСния напряТСния заряда Π΄ΠΎ уровня ΠΏΠΎΠ»ΠΎΠΆΠΈΡ‚Π΅Π»ΡŒΠ½ΠΎΠΉ ΠΊΠ»Π΅ΠΌΠΌΡ‹.

ΠŸΠΎΡ‚Π΅Π½Ρ†ΠΈΠ°Π»ΡŒΠ½Π°Ρ энСргия Π½Π° заряд Π½Π° ΠΏΠΎΠ»ΠΎΠΆΠΈΡ‚Π΅Π»ΡŒΠ½ΠΎΠΌ Π²Ρ‹Π²ΠΎΠ΄Π΅ Π±Π°Ρ‚Π°Ρ€Π΅ΠΈ — это номинальноС напряТСниС Π±Π°Ρ‚Π°Ρ€Π΅ΠΈ. Π­Ρ‚ΠΎ напряТСниС ΠΏΠΎΡ…ΠΎΠΆΠ΅ Π½Π° Π΄Π°Π²Π»Π΅Π½ΠΈΠ΅ Π²ΠΎΠ΄Ρ‹ Π² Π²Π΅Ρ€Ρ…Π½Π΅ΠΉ Ρ‚Ρ€ΡƒΠ±Π΅. Π’ΠΎΡ‡Π½ΠΎ Ρ‚Π°ΠΊ ΠΆΠ΅, ΠΊΠ°ΠΊ Π±ΠΎΠ»Π΅Π΅ высокоС Π΄Π°Π²Π»Π΅Π½ΠΈΠ΅ заставляСт Π²ΠΎΠ΄Ρƒ Π΄Π²ΠΈΠ³Π°Ρ‚ΡŒΡΡ ΠΊ Π±ΠΎΠ»Π΅Π΅ Π½ΠΈΠ·ΠΊΠΎΠΌΡƒ давлСнию, Π±ΠΎΠ»Π΅Π΅ высокоС напряТСниС заставляСт элСктричСский заряд Ρ‚Π΅Ρ‡ΡŒ ΠΊ Π±ΠΎΠ»Π΅Π΅ Π½ΠΈΠ·ΠΊΠΎΠΌΡƒ Π½Π°ΠΏΡ€ΡΠΆΠ΅Π½ΠΈΡŽ.Насос Π·Π°Π±ΠΈΡ€Π°Π΅Ρ‚ Π²ΠΎΠ΄Ρƒ ΠΏΠΎΠ΄ Π½ΠΈΠ·ΠΊΠΈΠΌ Π΄Π°Π²Π»Π΅Π½ΠΈΠ΅ΠΌ ΠΈ Ρ€Π°Π±ΠΎΡ‚Π°Π΅Ρ‚ Π½Π°Π΄ Π½Π΅ΠΉ, выбрасывая Π²ΠΎΠ΄Ρƒ ΠΏΠΎΠ΄ Π±ΠΎΠ»Π΅Π΅ высоким Π΄Π°Π²Π»Π΅Π½ΠΈΠ΅ΠΌ. Π’ΠΎΡ‡Π½ΠΎ Ρ‚Π°ΠΊ ΠΆΠ΅ аккумулятор заряТаСтся ΠΏΡ€ΠΈ Π½ΠΈΠ·ΠΊΠΎΠΌ напряТСнии, Ρ€Π°Π±ΠΎΡ‚Π°Π΅Ρ‚ Π½Π° Π½Π΅ΠΌ ΠΈ выбрасываСт заряд ΠΏΡ€ΠΈ Π±ΠΎΠ»Π΅Π΅ высоком напряТСнии.

ΠžΠ±Ρ€Π°Ρ‚ΠΈΡ‚Π΅ Π²Π½ΠΈΠΌΠ°Π½ΠΈΠ΅, Ρ‡Ρ‚ΠΎ Ρ‚ΠΎΠΊ Π² водяном ΠΊΠΎΠ½Ρ‚ΡƒΡ€Π΅, ΠΏΠΎΠΊΠ°Π·Π°Π½Π½ΠΎΠΌ Π½Π° РисункС 19.10, ΠΎΠ΄ΠΈΠ½Π°ΠΊΠΎΠ² Π²ΠΎ всСм ΠΊΠΎΠ½Ρ‚ΡƒΡ€Π΅. Π”Ρ€ΡƒΠ³ΠΈΠΌΠΈ словами, Ссли ΠΌΡ‹ ΠΈΠ·ΠΌΠ΅Ρ€ΠΈΠΌ количСство ΠΌΠΎΠ»Π΅ΠΊΡƒΠ» Π²ΠΎΠ΄Ρ‹, проходящих Ρ‡Π΅Ρ€Π΅Π· ΠΏΠΎΠΏΠ΅Ρ€Π΅Ρ‡Π½ΠΎΠ΅ сСчСниС Ρ‚Ρ€ΡƒΠ±Ρ‹ Π² Π΅Π΄ΠΈΠ½ΠΈΡ†Ρƒ Π²Ρ€Π΅ΠΌΠ΅Π½ΠΈ Π² любой Ρ‚ΠΎΡ‡ΠΊΠ΅ Ρ†Π΅ΠΏΠΈ, ΠΌΡ‹ ΠΏΠΎΠ»ΡƒΡ‡ΠΈΠΌ ΠΎΠ΄ΠΈΠ½ ΠΈ Ρ‚ΠΎΡ‚ ΠΆΠ΅ ΠΎΡ‚Π²Π΅Ρ‚ нСзависимо ΠΎΡ‚ Ρ‚ΠΎΠ³ΠΎ, Π³Π΄Π΅ Π² Ρ†Π΅ΠΏΠΈ ΠΌΡ‹ измСряСм.Π’ΠΎ ΠΆΠ΅ самоС ΠΈ с элСктричСской схСмой Π½Π° Ρ‚ΠΎΠΌ ΠΆΠ΅ рисункС. ЭлСктричСский Ρ‚ΠΎΠΊ ΠΎΠ΄ΠΈΠ½Π°ΠΊΠΎΠ² Π²ΠΎ всСх Ρ‚ΠΎΡ‡ΠΊΠ°Ρ… этой Ρ†Π΅ΠΏΠΈ, Π² Ρ‚ΠΎΠΌ числС Π²Π½ΡƒΡ‚Ρ€ΠΈ Π±Π°Ρ‚Π°Ρ€Π΅ΠΈ ΠΈ Π² рСзисторС. ЭлСктричСский Ρ‚ΠΎΠΊ Π½Π΅ ускоряСтся Π² ΠΏΡ€ΠΎΠ²ΠΎΠ΄Π°Ρ… ΠΈ Π½Π΅ замСдляСтся Π² рСзисторС. Π­Ρ‚ΠΎ создаст Ρ‚ΠΎΡ‡ΠΊΠΈ, Π² ΠΊΠΎΡ‚ΠΎΡ€Ρ‹Ρ… Π±ΡƒΠ΄Π΅Ρ‚ Π½Π°ΠΊΠ°ΠΏΠ»ΠΈΠ²Π°Ρ‚ΡŒΡΡ слишком ΠΌΠ½ΠΎΠ³ΠΎ ΠΈΠ»ΠΈ слишком ΠΌΠ°Π»ΠΎ заряда. Π’Π°ΠΊΠΈΠΌ ΠΎΠ±Ρ€Π°Π·ΠΎΠΌ, Ρ‚ΠΎΠΊ ΠΎΠ΄ΠΈΠ½Π°ΠΊΠΎΠ² Π²ΠΎ всСх Ρ‚ΠΎΡ‡ΠΊΠ°Ρ… Ρ†Π΅ΠΏΠΈ, ΠΏΠΎΠΊΠ°Π·Π°Π½Π½ΠΎΠΉ Π½Π° рисункС 19.10.

Π₯отя Ρ‚ΠΎΠΊ Π²Π΅Π·Π΄Π΅ ΠΎΠ΄ΠΈΠ½Π°ΠΊΠΎΠ²Ρ‹ΠΉ ΠΊΠ°ΠΊ Π² элСктричСском, Ρ‚Π°ΠΊ ΠΈ Π² водяном ΠΊΠΎΠ½Ρ‚ΡƒΡ€Π°Ρ…, напряТСниС ΠΈΠ»ΠΈ Π΄Π°Π²Π»Π΅Π½ΠΈΠ΅ Π²ΠΎΠ΄Ρ‹ измСняСтся ΠΏΠΎ ΠΌΠ΅Ρ€Π΅ Ρ‚ΠΎΠ³ΠΎ, ΠΊΠ°ΠΊ Π²Ρ‹ ΠΏΠ΅Ρ€Π΅ΠΌΠ΅Ρ‰Π°Π΅Ρ‚Π΅ΡΡŒ ΠΏΠΎ ΠΊΠΎΠ½Ρ‚ΡƒΡ€Π°ΠΌ.Π’ водяном ΠΊΠΎΠ½Ρ‚ΡƒΡ€Π΅ Π΄Π°Π²Π»Π΅Π½ΠΈΠ΅ Π²ΠΎΠ΄Ρ‹ Π½Π° Π²Ρ‹Ρ…ΠΎΠ΄Π΅ ΠΈΠ· насоса остаСтся Π½Π΅ΠΈΠ·ΠΌΠ΅Π½Π½Ρ‹ΠΌ Π΄ΠΎ Ρ‚Π΅Ρ… ΠΏΠΎΡ€, ΠΏΠΎΠΊΠ° Π²ΠΎΠ΄Π° Π½Π΅ ΠΏΡ€ΠΎΠΉΠ΄Π΅Ρ‚ Ρ‡Π΅Ρ€Π΅Π· пСсочный Ρ„ΠΈΠ»ΡŒΡ‚Ρ€, ΠΏΡ€ΠΈ условии отсутствия ΠΏΠΎΡ‚Π΅Ρ€ΡŒ энСргии Π² Ρ‚Ρ€ΡƒΠ±Π΅. Π’ΠΎΡ‡Π½ΠΎ Ρ‚Π°ΠΊ ΠΆΠ΅ напряТСниС Π² элСктричСской Ρ†Π΅ΠΏΠΈ ΠΎΠ΄ΠΈΠ½Π°ΠΊΠΎΠ²ΠΎ Π²ΠΎ всСх Ρ‚ΠΎΡ‡ΠΊΠ°Ρ… Π΄Π°Π½Π½ΠΎΠ³ΠΎ ΠΏΡ€ΠΎΠ²ΠΎΠ΄Π°, ΠΏΠΎΡ‚ΠΎΠΌΡƒ Ρ‡Ρ‚ΠΎ ΠΌΡ‹ ΠΏΡ€Π΅Π΄ΠΏΠΎΠ»ΠΎΠΆΠΈΠ»ΠΈ, Ρ‡Ρ‚ΠΎ ΠΏΡ€ΠΎΠ²ΠΎΠ΄Π° ΡΠ²Π»ΡΡŽΡ‚ΡΡ ΠΈΠ΄Π΅Π°Π»ΡŒΠ½Ρ‹ΠΌΠΈ ΠΏΡ€ΠΎΠ²ΠΎΠ΄Π½ΠΈΠΊΠ°ΠΌΠΈ. Π’Π°ΠΊΠΈΠΌ ΠΎΠ±Ρ€Π°Π·ΠΎΠΌ, ΠΊΠ°ΠΊ ΠΏΠΎΠΊΠ°Π·Ρ‹Π²Π°Π΅Ρ‚ постоянный красный Ρ†Π²Π΅Ρ‚ Π²Π΅Ρ€Ρ…Π½Π΅Π³ΠΎ ΠΏΡ€ΠΎΠ²ΠΎΠ΄Π° Π½Π° рис. 19.12, напряТСниС Π½Π° этом ΠΏΡ€ΠΎΠ²ΠΎΠ΄Π΅ постоянно ΠΈ Ρ€Π°Π²Π½ΠΎ V = VbatteryV = Vbattery. Π—Π°Ρ‚Π΅ΠΌ напряТСниС ΠΏΠ°Π΄Π°Π΅Ρ‚, ΠΊΠΎΠ³Π΄Π° Π²Ρ‹ ΠΏΡ€ΠΎΡ…ΠΎΠ΄ΠΈΡ‚Π΅ Ρ‡Π΅Ρ€Π΅Π· рСзистор, Π½ΠΎ ΠΊΠ°ΠΊ Ρ‚ΠΎΠ»ΡŒΠΊΠΎ Π²Ρ‹ Π΄ΠΎΠΉΠ΄Π΅Ρ‚Π΅ Π΄ΠΎ синСго ΠΏΡ€ΠΎΠ²ΠΎΠ΄Π°, напряТСниС останСтся Π½Π° Π½ΠΎΠ²ΠΎΠΌ ΡƒΡ€ΠΎΠ²Π½Π΅ V = 0V = 0 Π½Π° всСм ΠΏΡƒΡ‚ΠΈ Π΄ΠΎ ΠΎΡ‚Ρ€ΠΈΡ†Π°Ρ‚Π΅Π»ΡŒΠ½ΠΎΠΉ ΠΊΠ»Π΅ΠΌΠΌΡ‹ Π±Π°Ρ‚Π°Ρ€Π΅ΠΈ (Ρ‚.Π΅., синий Π²Ρ‹Π²ΠΎΠ΄ аккумуляторной Π±Π°Ρ‚Π°Ρ€Π΅ΠΈ).

Рисунок 19.12 НапряТСниС Π² красном ΠΏΡ€ΠΎΠ²ΠΎΠ΄Π΅ постоянно ΠΏΡ€ΠΈ V = VbatteryV = Vbattery ΠΎΡ‚ ΠΏΠΎΠ»ΠΎΠΆΠΈΡ‚Π΅Π»ΡŒΠ½ΠΎΠ³ΠΎ полюса Π±Π°Ρ‚Π°Ρ€Π΅ΠΈ Π΄ΠΎ Π²Π΅Ρ€Ρ…Π° рСзистора. НапряТСниС Π² синСм ΠΏΡ€ΠΎΠ²ΠΎΠ΄Π΅ постоянно ΠΈ Ρ€Π°Π²Π½ΠΎ V = Vground = 0V = Vground = 0 ΠΎΡ‚ Π½ΠΈΠΆΠ½Π΅ΠΉ части рСзистора Π΄ΠΎ ΠΎΡ‚Ρ€ΠΈΡ†Π°Ρ‚Π΅Π»ΡŒΠ½ΠΎΠΉ ΠΊΠ»Π΅ΠΌΠΌΡ‹ Π±Π°Ρ‚Π°Ρ€Π΅ΠΈ.

Если ΠΏΠ΅Ρ€Π΅ΠΉΡ‚ΠΈ ΠΎΡ‚ синСго ΠΏΡ€ΠΎΠ²ΠΎΠ΄Π° Ρ‡Π΅Ρ€Π΅Π· аккумулятор ΠΊ красному ΠΏΡ€ΠΎΠ²ΠΎΠ΄Ρƒ, Ρ‚ΠΎ напряТСниС возрастСт ΠΎΡ‚ V = 0V = 0 ΠΊ V = VbatteryV = Vbattery. Аналогичным ΠΎΠ±Ρ€Π°Π·ΠΎΠΌ, Ссли ΠΌΡ‹ ΠΏΠ΅Ρ€Π΅ΠΉΠ΄Π΅ΠΌ ΠΎΡ‚ синСго ΠΏΡ€ΠΎΠ²ΠΎΠ΄Π° Ρ‡Π΅Ρ€Π΅Π· рСзистор ΠΊ красному ΠΏΡ€ΠΎΠ²ΠΎΠ΄Ρƒ, напряТСниС Ρ‚Π°ΠΊΠΆΠ΅ измСнится с V = 0V = 0 Π½Π° V = VbatteryV = Vbattery.Π’Π°ΠΊΠΈΠΌ ΠΎΠ±Ρ€Π°Π·ΠΎΠΌ, ΠΈΡΠΏΠΎΠ»ΡŒΠ·ΡƒΡ Π·Π°ΠΊΠΎΠ½ Ома, ΠΌΡ‹ ΠΌΠΎΠΆΠ΅ΠΌ Π½Π°ΠΏΠΈΡΠ°Ρ‚ΡŒ

ВрСзистор = Vbattery = IR. Vresistor = Vbattery = IR.

ΠžΠ±Ρ€Π°Ρ‚ΠΈΡ‚Π΅ Π²Π½ΠΈΠΌΠ°Π½ΠΈΠ΅, Ρ‡Ρ‚ΠΎ VresistorVresistor измСряСтся ΠΎΡ‚ Π½ΠΈΠΆΠ½Π΅ΠΉ части рСзистора Π΄ΠΎ Π²Π΅Ρ€Ρ…Π½Π΅ΠΉ части, Ρ‡Ρ‚ΠΎ ΠΎΠ·Π½Π°Ρ‡Π°Π΅Ρ‚, Ρ‡Ρ‚ΠΎ вСрхняя Ρ‡Π°ΡΡ‚ΡŒ рСзистора находится ΠΏΠΎΠ΄ Π±ΠΎΠ»Π΅Π΅ высоким напряТСниСм, Ρ‡Π΅ΠΌ ниТняя Ρ‡Π°ΡΡ‚ΡŒ рСзистора. Π’Π°ΠΊΠΈΠΌ ΠΎΠ±Ρ€Π°Π·ΠΎΠΌ, Ρ‚ΠΎΠΊ Ρ‚Π΅Ρ‡Π΅Ρ‚ ΠΎΡ‚ Π²Π΅Ρ€Ρ…Π½Π΅ΠΉ части рСзистора ΠΈΠ»ΠΈ с Π±ΠΎΠ»Π΅Π΅ высоким напряТСниСм ΠΊ Π½ΠΈΠΆΠ½Π΅ΠΉ части рСзистора ΠΈΠ»ΠΈ с Π±ΠΎΠ»Π΅Π΅ Π½ΠΈΠ·ΠΊΠΈΠΌ напряТСниСм.

Virtual Physics

ЦСпь Π±Π°Ρ‚Π°Ρ€Π΅ΠΉΠ½ΠΎΠ³ΠΎ рСзистора

Π˜ΡΠΏΠΎΠ»ΡŒΠ·ΡƒΠΉΡ‚Π΅ это ΠΌΠΎΠ΄Π΅Π»ΠΈΡ€ΠΎΠ²Π°Π½ΠΈΠ΅, Ρ‡Ρ‚ΠΎΠ±Ρ‹ Π»ΡƒΡ‡ΡˆΠ΅ ΠΏΠΎΠ½ΡΡ‚ΡŒ, ΠΊΠ°ΠΊ связаны сопротивлСниС, напряТСниС ΠΈ Ρ‚ΠΎΠΊ.ΠœΠΎΠ΄Π΅Π»ΠΈΡ€ΠΎΠ²Π°Π½ΠΈΠ΅ ΠΏΠΎΠΊΠ°Π·Ρ‹Π²Π°Π΅Ρ‚ Π±Π°Ρ‚Π°Ρ€Π΅ΡŽ с рСзистором, ΠΏΠΎΠ΄ΠΊΠ»ΡŽΡ‡Π΅Π½Π½Ρ‹ΠΌ ΠΌΠ΅ΠΆΠ΄Ρƒ Π²Ρ‹Π²ΠΎΠ΄Π°ΠΌΠΈ Π±Π°Ρ‚Π°Ρ€Π΅ΠΈ, ΠΊΠ°ΠΊ Π½Π° ΠΏΡ€Π΅Π΄Ρ‹Π΄ΡƒΡ‰Π΅ΠΌ рисункС. Π’Ρ‹ ΠΌΠΎΠΆΠ΅Ρ‚Π΅ ΠΈΠ·ΠΌΠ΅Π½ΠΈΡ‚ΡŒ напряТСниС аккумулятора ΠΈ сопротивлСниС. ΠœΠΎΠ΄Π΅Π»ΠΈΡ€ΠΎΠ²Π°Π½ΠΈΠ΅ ΠΏΠΎΠΊΠ°Π·Ρ‹Π²Π°Π΅Ρ‚, ΠΊΠ°ΠΊ элСктроны Ρ€Π΅Π°Π³ΠΈΡ€ΡƒΡŽΡ‚ Π½Π° эти измСнСния. Он Ρ‚Π°ΠΊΠΆΠ΅ ΠΏΠΎΠΊΠ°Π·Ρ‹Π²Π°Π΅Ρ‚ Π°Ρ‚ΠΎΠΌΠ½Ρ‹Π΅ сСрдСчники Π² рСзисторС ΠΈ Ρ‚ΠΎ, ΠΊΠ°ΠΊ ΠΎΠ½ΠΈ Π²ΠΎΠ·Π±ΡƒΠΆΠ΄Π°ΡŽΡ‚ΡΡ ΠΈ Π½Π°Π³Ρ€Π΅Π²Π°ΡŽΡ‚ΡΡ ΠΏΠΎ ΠΌΠ΅Ρ€Π΅ прохоТдСния большСго Ρ‚ΠΎΠΊΠ° Ρ‡Π΅Ρ€Π΅Π· рСзистор.

НарисуйтС ΠΏΡ€ΠΈΠ½Ρ†ΠΈΠΏΠΈΠ°Π»ΡŒΠ½ΡƒΡŽ схСму Ρ†Π΅ΠΏΠΈ, ΠΎΠ±ΡΠ·Π°Ρ‚Π΅Π»ΡŒΠ½ΠΎ нарисовав стрСлку, ΡƒΠΊΠ°Π·Ρ‹Π²Π°ΡŽΡ‰ΡƒΡŽ Π½Π°ΠΏΡ€Π°Π²Π»Π΅Π½ΠΈΠ΅ Ρ‚ΠΎΠΊΠ°. Π’Π΅ΠΏΠ΅Ρ€ΡŒ ΠΎΡ‚ΠΌΠ΅Ρ‚ΡŒΡ‚Π΅ Ρ‚Ρ€ΠΈ Ρ‚ΠΎΡ‡ΠΊΠΈ вдоль ΠΏΡ€ΠΎΠ²ΠΎΠ»ΠΎΠΊΠΈ.НС мСняя настроСк, ΠΏΠΎΠ·Π²ΠΎΠ»ΡŒΡ‚Π΅ ΠΌΠΎΠ΄Π΅Π»ΠΈΡ€ΠΎΠ²Π°Π½ΠΈΡŽ Ρ€Π°Π±ΠΎΡ‚Π°Ρ‚ΡŒ Π² Ρ‚Π΅Ρ‡Π΅Π½ΠΈΠ΅ 20 сСкунд, ΠΏΠΎΠΊΠ° Π²Ρ‹ подсчитываСтС количСство элСктронов, проходящих Ρ‡Π΅Ρ€Π΅Π· это пятно. Π—Π°ΠΏΠΈΡˆΠΈΡ‚Π΅ Π½ΠΎΠΌΠ΅Ρ€ Π½Π° ΠΏΡ€ΠΈΠ½Ρ†ΠΈΠΏΠΈΠ°Π»ΡŒΠ½ΠΎΠΉ схСмС. Π’Π΅ΠΏΠ΅Ρ€ΡŒ ΠΏΡ€ΠΎΠ΄Π΅Π»Π°ΠΉΡ‚Π΅ Ρ‚ΠΎ ΠΆΠ΅ самоС с двумя Π΄Ρ€ΡƒΠ³ΠΈΠΌΠΈ Ρ‚ΠΎΡ‡ΠΊΠ°ΠΌΠΈ ΠΊΠΎΠ½Ρ‚ΡƒΡ€Π°. Π§Ρ‚ΠΎ Π²Ρ‹ Π·Π°ΠΌΠ΅Ρ‚ΠΈΠ»ΠΈ ΠΏΠΎ ΠΏΠΎΠ²ΠΎΠ΄Ρƒ количСства зарядов, проходящих Ρ‡Π΅Ρ€Π΅Π· ΠΊΠ°ΠΆΠ΄ΠΎΠ΅ пятно Π·Π° 20 с? ΠŸΠΎΠΌΠ½ΠΈΡ‚Π΅, Ρ‡Ρ‚ΠΎ этот Ρ‚ΠΎΠΊ опрСдСляСтся ΠΊΠ°ΠΊ ΡΠΊΠΎΡ€ΠΎΡΡ‚ΡŒ, с ΠΊΠΎΡ‚ΠΎΡ€ΠΎΠΉ заряды ΠΏΡ€ΠΎΡ‚Π΅ΠΊΠ°ΡŽΡ‚ ΠΏΠΎ Ρ†Π΅ΠΏΠΈ. Π§Ρ‚ΠΎ это Π·Π½Π°Ρ‡ΠΈΡ‚ для Ρ‚ΠΎΠΊΠ° Π²ΠΎ всСй Ρ†Π΅ΠΏΠΈ?

ΠŸΡ€ΠΎΠ²Π΅Ρ€ΠΊΠ° Π·Π°Ρ…Π²Π°Ρ‚Π°

Π‘ ΠΏΠΎΠΌΠΎΡ‰ΡŒΡŽ ΠΏΠΎΠ»Π·ΡƒΠ½ΠΊΠ° напряТСния ΠΏΠΎΠ΄Π°ΠΉΡ‚Π΅ Π½Π° аккумулятор ΠΏΠΎΠ»ΠΎΠΆΠΈΡ‚Π΅Π»ΡŒΠ½ΠΎΠ΅ напряТСниС.ΠžΠ±Ρ€Π°Ρ‚ΠΈΡ‚Π΅ Π²Π½ΠΈΠΌΠ°Π½ΠΈΠ΅, Ρ‡Ρ‚ΠΎ элСктроны Π² Π»Π΅Π²ΠΎΠΌ ΠΏΡ€ΠΎΠ²ΠΎΠ΄Π΅ располоТСны дальшС Π΄Ρ€ΡƒΠ³ ΠΎΡ‚ Π΄Ρ€ΡƒΠ³Π°, Ρ‡Π΅ΠΌ Π² ΠΏΡ€Π°Π²ΠΎΠΌ. Как это ΠΎΡ‚Ρ€Π°ΠΆΠ°Π΅Ρ‚ напряТСниС Π² Π΄Π²ΡƒΡ… ΠΏΡ€ΠΎΠ²ΠΎΠ΄Π°Ρ…?

  1. НапряТСниС ΠΌΠ΅ΠΆΠ΄Ρƒ статичСскими зарядами прямо ΠΏΡ€ΠΎΠΏΠΎΡ€Ρ†ΠΈΠΎΠ½Π°Π»ΡŒΠ½ΠΎ Ρ€Π°ΡΡΡ‚ΠΎΡΠ½ΠΈΡŽ ΠΌΠ΅ΠΆΠ΄Ρƒ Π½ΠΈΠΌΠΈ.
  2. НапряТСниС ΠΌΠ΅ΠΆΠ΄Ρƒ статичСскими зарядами прямо ΠΏΡ€ΠΎΠΏΠΎΡ€Ρ†ΠΈΠΎΠ½Π°Π»ΡŒΠ½ΠΎ ΠΊΠ²Π°Π΄Ρ€Π°Ρ‚Ρƒ расстояния ΠΌΠ΅ΠΆΠ΄Ρƒ Π½ΠΈΠΌΠΈ.
  3. НапряТСниС ΠΌΠ΅ΠΆΠ΄Ρƒ статичСскими зарядами ΠΎΠ±Ρ€Π°Ρ‚Π½ΠΎ ΠΏΡ€ΠΎΠΏΠΎΡ€Ρ†ΠΈΠΎΠ½Π°Π»ΡŒΠ½ΠΎ Ρ€Π°ΡΡΡ‚ΠΎΡΠ½ΠΈΡŽ ΠΌΠ΅ΠΆΠ΄Ρƒ Π½ΠΈΠΌΠΈ.
  4. НапряТСниС ΠΌΠ΅ΠΆΠ΄Ρƒ статичСскими зарядами ΠΎΠ±Ρ€Π°Ρ‚Π½ΠΎ ΠΏΡ€ΠΎΠΏΠΎΡ€Ρ†ΠΈΠΎΠ½Π°Π»ΡŒΠ½ΠΎ ΠΊΠ²Π°Π΄Ρ€Π°Ρ‚Ρƒ расстояния ΠΌΠ΅ΠΆΠ΄Ρƒ Π½ΠΈΠΌΠΈ.

Π”Ρ€ΡƒΠ³ΠΈΠ΅ Π²ΠΎΠ·ΠΌΠΎΠΆΠ½Ρ‹Π΅ элСмСнты схСмы Π²ΠΊΠ»ΡŽΡ‡Π°ΡŽΡ‚ кондСнсаторы ΠΈ ΠΏΠ΅Ρ€Π΅ΠΊΠ»ΡŽΡ‡Π°Ρ‚Π΅Π»ΠΈ. Они нарисованы, ΠΊΠ°ΠΊ ΠΏΠΎΠΊΠ°Π·Π°Π½ΠΎ Π² Π»Π΅Π²ΠΎΠΉ части рисунка 19.14. Π’Ρ‹ΠΊΠ»ΡŽΡ‡Π°Ρ‚Π΅Π»ΡŒ — это устройство, ΠΊΠΎΡ‚ΠΎΡ€ΠΎΠ΅ Ρ€Π°Π·ΠΌΡ‹ΠΊΠ°Π΅Ρ‚ ΠΈ Π·Π°ΠΌΡ‹ΠΊΠ°Π΅Ρ‚ Ρ†Π΅ΠΏΡŒ, ΠΊΠ°ΠΊ Π²Ρ‹ΠΊΠ»ΡŽΡ‡Π°Ρ‚Π΅Π»ΡŒ свСта. Он Π°Π½Π°Π»ΠΎΠ³ΠΈΡ‡Π΅Π½ ΠΊΠ»Π°ΠΏΠ°Π½Ρƒ Π² водяном ΠΊΠΎΠ½Ρ‚ΡƒΡ€Π΅, ΠΊΠ°ΠΊ ΠΏΠΎΠΊΠ°Π·Π°Π½ΠΎ Π² ΠΏΡ€Π°Π²ΠΎΠΉ части рисунка 19.14. ΠŸΡ€ΠΈ Ρ€Π°Π·ΠΎΠΌΠΊΠ½ΡƒΡ‚ΠΎΠΌ ΠΏΠ΅Ρ€Π΅ΠΊΠ»ΡŽΡ‡Π°Ρ‚Π΅Π»Π΅ Ρ‚ΠΎΠΊ Π² Ρ†Π΅ΠΏΠΈ Π½Π΅ ΠΏΡ€ΠΎΡ…ΠΎΠ΄ΠΈΡ‚.Когда ΠΏΠ΅Ρ€Π΅ΠΊΠ»ΡŽΡ‡Π°Ρ‚Π΅Π»ΡŒ Π·Π°ΠΌΠΊΠ½ΡƒΡ‚, ΠΎΠ½ становится Ρ‡Π°ΡΡ‚ΡŒΡŽ ΠΏΡ€ΠΎΠ²ΠΎΠ΄Π°, поэтому Ρ‚ΠΎΠΊ ΠΏΡ€ΠΎΡ…ΠΎΠ΄ΠΈΡ‚ Ρ‡Π΅Ρ€Π΅Π· Π½Π΅Π³ΠΎ Π±Π΅Π· ΠΏΠΎΡ‚Π΅Ρ€ΠΈ напряТСния.

ΠšΠΎΠ½Π΄Π΅Π½ΡΠ°Ρ‚ΠΎΡ€ ΠΎΠ±ΠΎΠ·Π½Π°Ρ‡Π΅Π½ Π±ΡƒΠΊΠ²ΠΎΠΉ C слСва Π½Π° РисункС 19.14. ΠšΠΎΠ½Π΄Π΅Π½ΡΠ°Ρ‚ΠΎΡ€ Π² элСктричСской Ρ†Π΅ΠΏΠΈ Π°Π½Π°Π»ΠΎΠ³ΠΈΡ‡Π΅Π½ Π³ΠΈΠ±ΠΊΠΎΠΉ ΠΌΠ΅ΠΌΠ±Ρ€Π°Π½Π΅ Π² водяном ΠΊΠΎΠ½Ρ‚ΡƒΡ€Π΅. Когда ΠΏΠ΅Ρ€Π΅ΠΊΠ»ΡŽΡ‡Π°Ρ‚Π΅Π»ΡŒ Π·Π°ΠΌΠΊΠ½ΡƒΡ‚ Π² Ρ†Π΅ΠΏΠΈ, ΠΏΠΎΠΊΠ°Π·Π°Π½Π½ΠΎΠΉ Π½Π° рисункС 19.14, батарСя заставляСт элСктричСский Ρ‚ΠΎΠΊ Ρ‚Π΅Ρ‡ΡŒ ΠΊ кондСнсатору, заряТая Π²Π΅Ρ€Ρ…Π½ΡŽΡŽ пластину кондСнсатора ΠΏΠΎΠ»ΠΎΠΆΠΈΡ‚Π΅Π»ΡŒΠ½Ρ‹ΠΌ зарядом. Когда это происходит, напряТСниС Π½Π° ΠΎΠ±ΠΊΠ»Π°Π΄ΠΊΠ°Ρ… кондСнсатора увСличиваСтся.Π­Ρ‚ΠΎ ΠΏΠΎΡ…ΠΎΠΆΠ΅ Π½Π° ΠΌΠ΅ΠΌΠ±Ρ€Π°Π½Ρƒ Π² водяном ΠΊΠΎΠ½Ρ‚ΡƒΡ€Π΅: ΠΊΠΎΠ³Π΄Π° ΠΊΠ»Π°ΠΏΠ°Π½ открываСтся, насос заставляСт Π²ΠΎΠ΄Ρƒ Ρ‚Π΅Ρ‡ΡŒ ΠΊ ΠΌΠ΅ΠΌΠ±Ρ€Π°Π½Π΅, заставляя Π΅Π΅ Ρ€Π°ΡΡ‚ΡΠ³ΠΈΠ²Π°Ρ‚ΡŒΡΡ для удСрТания излишков Π²ΠΎΠ΄Ρ‹. Когда это происходит, Π΄Π°Π²Π»Π΅Π½ΠΈΠ΅ Π·Π° ΠΌΠ΅ΠΌΠ±Ρ€Π°Π½ΠΎΠΉ увСличиваСтся.

Π’Π΅ΠΏΠ΅Ρ€ΡŒ, Ссли ΠΌΡ‹ Ρ€Π°Π·ΠΎΠΌΠΊΠ½Π΅ΠΌ Π²Ρ‹ΠΊΠ»ΡŽΡ‡Π°Ρ‚Π΅Π»ΡŒ, кондСнсатор Π±ΡƒΠ΄Π΅Ρ‚ ΡƒΠ΄Π΅Ρ€ΠΆΠΈΠ²Π°Ρ‚ΡŒ напряТСниС ΠΌΠ΅ΠΆΠ΄Ρƒ своими пластинами, ΠΏΠΎΡ‚ΠΎΠΌΡƒ Ρ‡Ρ‚ΠΎ зарядам Π½Π΅ΠΊΡƒΠ΄Π° Π΄Π΅Π²Π°Ρ‚ΡŒΡΡ. Π’ΠΎΡ‡Π½ΠΎ Ρ‚Π°ΠΊ ΠΆΠ΅, Ссли ΠΌΡ‹ Π·Π°ΠΊΡ€ΠΎΠ΅ΠΌ ΠΊΠ»Π°ΠΏΠ°Π½, Π²ΠΎΠ΄Π΅ Π½Π΅ΠΊΡƒΠ΄Π° Π±ΡƒΠ΄Π΅Ρ‚ ΠΈΠ΄Ρ‚ΠΈ, ΠΈ ΠΌΠ΅ΠΌΠ±Ρ€Π°Π½Π° Π±ΡƒΠ΄Π΅Ρ‚ ΠΏΠΎΠ΄Π΄Π΅Ρ€ΠΆΠΈΠ²Π°Ρ‚ΡŒ Π΄Π°Π²Π»Π΅Π½ΠΈΠ΅ Π²ΠΎΠ΄Ρ‹ Π² Ρ‚Ρ€ΡƒΠ±Π΅ ΠΌΠ΅ΠΆΠ΄Ρƒ собой ΠΈ ΠΊΠ»Π°ΠΏΠ°Π½ΠΎΠΌ.

Если ΠΏΠ΅Ρ€Π΅ΠΊΠ»ΡŽΡ‡Π°Ρ‚Π΅Π»ΡŒ Π² элСктричСской Ρ†Π΅ΠΏΠΈ Π·Π°ΠΌΠΊΠ½ΡƒΡ‚ Π² Ρ‚Π΅Ρ‡Π΅Π½ΠΈΠ΅ Π΄Π»ΠΈΡ‚Π΅Π»ΡŒΠ½ΠΎΠ³ΠΎ Π²Ρ€Π΅ΠΌΠ΅Π½ΠΈ ΠΈΠ»ΠΈ Ссли ΠΊΠ»Π°ΠΏΠ°Π½ Π² водяном ΠΊΠΎΠ½Ρ‚ΡƒΡ€Π΅ Π΄ΠΎΠ»Π³ΠΎΠ΅ врСмя ΠΎΡ‚ΠΊΡ€Ρ‹Ρ‚, Ρ‚ΠΎΠΊ Π² ΠΊΠΎΠ½Π΅Ρ‡Π½ΠΎΠΌ ΠΈΡ‚ΠΎΠ³Π΅ пСрСстанСт Ρ‚Π΅Ρ‡ΡŒ, ΠΏΠΎΡ‚ΠΎΠΌΡƒ Ρ‡Ρ‚ΠΎ кондСнсатор ΠΈΠ»ΠΈ ΠΌΠ΅ΠΌΠ±Ρ€Π°Π½Π° ΠΏΠΎΠ»Π½ΠΎΡΡ‚ΡŒΡŽ заряТСны. Π’Π΅ΠΏΠ΅Ρ€ΡŒ каТдая Ρ†Π΅ΠΏΡŒ находится Π² ΡƒΡΡ‚Π°Π½ΠΎΠ²ΠΈΠ²ΡˆΠ΅ΠΌΡΡ состоянии, Ρ‡Ρ‚ΠΎ ΠΎΠ·Π½Π°Ρ‡Π°Π΅Ρ‚, Ρ‡Ρ‚ΠΎ Π΅Π΅ характСристики Π½Π΅ ΠΌΠ΅Π½ΡΡŽΡ‚ΡΡ со Π²Ρ€Π΅ΠΌΠ΅Π½Π΅ΠΌ. Π’ этом случаС ΡƒΡΡ‚Π°Π½ΠΎΠ²ΠΈΠ²ΡˆΠ΅Π΅ΡΡ состояниС характСризуСтся Π½ΡƒΠ»Π΅Π²Ρ‹ΠΌ Ρ‚ΠΎΠΊΠΎΠΌ, ΠΈ ΠΎΠ½ Π½Π΅ мСняСтся, ΠΏΠΎΠΊΠ° ΠΏΠ΅Ρ€Π΅ΠΊΠ»ΡŽΡ‡Π°Ρ‚Π΅Π»ΡŒ ΠΈΠ»ΠΈ ΠΊΠ»Π°ΠΏΠ°Π½ ΠΎΡΡ‚Π°ΡŽΡ‚ΡΡ Π² Ρ‚ΠΎΠΌ ΠΆΠ΅ ΠΏΠΎΠ»ΠΎΠΆΠ΅Π½ΠΈΠΈ.Π’ ΡƒΡΡ‚Π°Π½ΠΎΠ²ΠΈΠ²ΡˆΠ΅ΠΌΡΡ Ρ€Π΅ΠΆΠΈΠΌΠ΅ Ρ‡Π΅Ρ€Π΅Π· кондСнсатор Π½Π΅ ΠΏΡ€ΠΎΡ…ΠΎΠ΄ΠΈΡ‚ элСктричСский Ρ‚ΠΎΠΊ ΠΈ Ρ‡Π΅Ρ€Π΅Π· ΠΌΠ΅ΠΌΠ±Ρ€Π°Π½Ρƒ Π½Π΅ ΠΏΡ€ΠΎΡ…ΠΎΠ΄ΠΈΡ‚ Ρ‚ΠΎΠΊ Π²ΠΎΠ΄Ρ‹. Π Π°Π·Π½ΠΈΡ†Π° напряТСний ΠΌΠ΅ΠΆΠ΄Ρƒ пластинами кондСнсатора Π±ΡƒΠ΄Π΅Ρ‚ Ρ‚Π°ΠΊΠΎΠΉ ΠΆΠ΅, ΠΊΠ°ΠΊ ΠΈ напряТСниС Π±Π°Ρ‚Π°Ρ€Π΅ΠΈ. Π’ водяном ΠΊΠΎΠ½Ρ‚ΡƒΡ€Π΅ Π΄Π°Π²Π»Π΅Π½ΠΈΠ΅ Π·Π° ΠΌΠ΅ΠΌΠ±Ρ€Π°Π½ΠΎΠΉ Π±ΡƒΠ΄Π΅Ρ‚ Ρ‚Π°ΠΊΠΈΠΌ ΠΆΠ΅, ΠΊΠ°ΠΊ Π΄Π°Π²Π»Π΅Π½ΠΈΠ΅, создаваСмоС насосом.

Π₯отя схСма Π½Π° рис. 19.14 ΠΌΠΎΠΆΠ΅Ρ‚ ΠΏΠΎΠΊΠ°Π·Π°Ρ‚ΡŒΡΡ Π½Π΅ΠΌΠ½ΠΎΠ³ΠΎ бСссмыслСнной, ΠΏΠΎΡ‚ΠΎΠΌΡƒ Ρ‡Ρ‚ΠΎ всС, Ρ‡Ρ‚ΠΎ происходит, ΠΊΠΎΠ³Π΄Π° ΠΏΠ΅Ρ€Π΅ΠΊΠ»ΡŽΡ‡Π°Ρ‚Π΅Π»ΡŒ Π·Π°ΠΌΠΊΠ½ΡƒΡ‚, — это зарядка кондСнсатора, это ΠΏΠΎΠΊΠ°Π·Ρ‹Π²Π°Π΅Ρ‚ ΡΠΏΠΎΡΠΎΠ±Π½ΠΎΡΡ‚ΡŒ кондСнсатора Π½Π°ΠΊΠ°ΠΏΠ»ΠΈΠ²Π°Ρ‚ΡŒ заряд.Π’Π°ΠΊΠΈΠΌ ΠΎΠ±Ρ€Π°Π·ΠΎΠΌ, кондСнсатор слуТит Ρ€Π΅Π·Π΅Ρ€Π²ΡƒΠ°Ρ€ΠΎΠΌ для заряда. Π­Ρ‚ΠΎ свойство кондСнсаторов ΠΈΡΠΏΠΎΠ»ΡŒΠ·ΡƒΠ΅Ρ‚ΡΡ Π² схСмах ΠΏΠΎ-Ρ€Π°Π·Π½ΠΎΠΌΡƒ. НапримСр, кондСнсаторы ΠΈΡΠΏΠΎΠ»ΡŒΠ·ΡƒΡŽΡ‚ΡΡ для питания Ρ†Π΅ΠΏΠ΅ΠΉ Π²ΠΎ врСмя зарядки аккумуляторов. ΠšΡ€ΠΎΠΌΠ΅ Ρ‚ΠΎΠ³ΠΎ, кондСнсаторы ΠΌΠΎΠ³ΡƒΡ‚ ΡΠ»ΡƒΠΆΠΈΡ‚ΡŒ Ρ„ΠΈΠ»ΡŒΡ‚Ρ€Π°ΠΌΠΈ. Π§Ρ‚ΠΎΠ±Ρ‹ ΠΏΠΎΠ½ΡΡ‚ΡŒ это, вСрнСмся ΠΊ Π°Π½Π°Π»ΠΎΠ³ΠΈΠΈ с Π²ΠΎΠ΄ΠΎΠΉ. ΠŸΡ€Π΅Π΄ΠΏΠΎΠ»ΠΎΠΆΠΈΠΌ, Ρƒ вас Π΅ΡΡ‚ΡŒ шланг для Π²ΠΎΠ΄Ρ‹ ΠΈ Π²Ρ‹ ΠΏΠΎΠ»ΠΈΠ²Π°Π΅Ρ‚Π΅ свой сад. Π’Π°Ρˆ Π΄Ρ€ΡƒΠ³ Π΄ΡƒΠΌΠ°Π΅Ρ‚, Ρ‡Ρ‚ΠΎ ΠΎΠ½ Π·Π°Π±Π°Π²Π½Ρ‹ΠΉ, ΠΈ ΠΏΠ΅Ρ€Π΅ΠΊΡ€ΡƒΡ‡ΠΈΠ²Π°Π΅Ρ‚ шланг. Пока шланг ΠΏΠ΅Ρ€Π΅ΠΊΡ€ΡƒΡ‡Π΅Π½, Π²ΠΎΠ΄Π° Π½Π΅ Ρ‚Π΅Ρ‡Π΅Ρ‚. Когда ΠΎΠ½ отпускаСт, Π²ΠΎΠ΄Π° снова Π½Π°Ρ‡ΠΈΠ½Π°Π΅Ρ‚ Ρ‚Π΅Ρ‡ΡŒ.Если ΠΎΠ½ сдСлаСт это ΠΎΡ‡Π΅Π½ΡŒ быстро, Π²Ρ‹ почувствуСтС, Ρ‡Ρ‚ΠΎ Π²ΠΎΠ΄Π° — Π½Π΅Ρ‚ Π²ΠΎΠ΄Ρ‹ — Π²ΠΎΠ΄Π° — Π½Π΅Ρ‚ Π²ΠΎΠ΄Ρ‹, ΠΈ это Π΄Π΅ΠΉΡΡ‚Π²ΠΈΡ‚Π΅Π»ΡŒΠ½ΠΎ Π½Π΅ способ ΠΏΠΎΠ»ΠΈΠ²Π°Ρ‚ΡŒ свой сад. Π’Π΅ΠΏΠ΅Ρ€ΡŒ ΠΏΡ€Π΅Π΄ΡΡ‚Π°Π²ΡŒΡ‚Π΅, Ρ‡Ρ‚ΠΎ шланг наполняСт большоС Π²Π΅Π΄Ρ€ΠΎ, ΠΈ Π²Ρ‹ ΠΏΠΎΠ»ΠΈΠ²Π°Π΅Ρ‚Π΅ Π΅Π³ΠΎ ΠΈΠ· Π½ΠΈΠΆΠ½Π΅ΠΉ части Π²Π΅Π΄Ρ€Π°. Если Ρƒ вас с самого Π½Π°Ρ‡Π°Π»Π° Π±Ρ‹Π»Π° Π²ΠΎΠ΄Π° Π² Π²Π΅Π΄Ρ€Π΅, Π° ваш Π΄Ρ€ΡƒΠ³ Π½Π΅ ΠΏΠ΅Ρ€Π΅Π³ΠΈΠ±Π°Π» шланг слишком Π΄ΠΎΠ»Π³ΠΎ, Π²Ρ‹ смоТСтС ΠΏΠΎΠ»ΠΈΠ²Π°Ρ‚ΡŒ свой сад Π±Π΅Π· ΠΏΠ΅Ρ€Π΅Ρ€Ρ‹Π²ΠΎΠ². Π’Π°Ρˆ Π΄Ρ€ΡƒΠ³, ΠΏΠ΅Ρ€Π΅Π³ΠΈΠ±Π°ΡŽΡ‰ΠΈΠΉ водяной шланг, ΠΎΡ‚Ρ„ΠΈΠ»ΡŒΡ‚Ρ€ΠΎΠ²Π°Π½ Π²ΠΎΠ΄ΠΎΠΉ ΠΈΠ· большого Π²Π΅Π΄Ρ€Π°, Ρ‚Π°ΠΊ Ρ‡Ρ‚ΠΎ это Π½Π΅ повлияСт Π½Π° Π²Π°ΡˆΡƒ ΡΠΏΠΎΡΠΎΠ±Π½ΠΎΡΡ‚ΡŒ ΠΏΠΎΠ»ΠΈΠ²Π°Ρ‚ΡŒ сад.ΠœΡ‹ ΠΌΠΎΠΆΠ΅ΠΌ Π΄ΡƒΠΌΠ°Ρ‚ΡŒ ΠΎ прСрываниях Ρ‚ΠΎΠΊΠ° (Π±ΡƒΠ΄ΡŒ Ρ‚ΠΎ Π²ΠΎΠ΄Π° ΠΈΠ»ΠΈ элСктричСский Ρ‚ΠΎΠΊ) ΠΊΠ°ΠΊ ΡˆΡƒΠΌ . ΠšΠΎΠ½Π΄Π΅Π½ΡΠ°Ρ‚ΠΎΡ€Ρ‹ Π΄Π΅ΠΉΡΡ‚Π²ΡƒΡŽΡ‚ Π°Π½Π°Π»ΠΎΠ³ΠΈΡ‡Π½ΠΎ Π²Π΅Π΄Ρ€Ρƒ с Π²ΠΎΠ΄ΠΎΠΉ, помогая ΠΎΡ‚Ρ„ΠΈΠ»ΡŒΡ‚Ρ€ΠΎΠ²Ρ‹Π²Π°Ρ‚ΡŒ ΡˆΡƒΠΌ. ΠšΠΎΠ½Π΄Π΅Π½ΡΠ°Ρ‚ΠΎΡ€Ρ‹ ΠΈΠΌΠ΅ΡŽΡ‚ Ρ‚Π°ΠΊ ΠΌΠ½ΠΎΠ³ΠΎ ΠΏΡ€ΠΈΠΌΠ΅Π½Π΅Π½ΠΈΠΉ, Ρ‡Ρ‚ΠΎ ΠΎΡ‡Π΅Π½ΡŒ Ρ€Π΅Π΄ΠΊΠΎ ΠΌΠΎΠΆΠ½ΠΎ Π½Π°ΠΉΡ‚ΠΈ ΡΠ»Π΅ΠΊΡ‚Ρ€ΠΎΠ½Π½ΡƒΡŽ схСму, Π² ΠΊΠΎΡ‚ΠΎΡ€ΠΎΠΉ Π½Π΅Ρ‚ кондСнсаторов.

Рис. 19.14 Π‘Π»Π΅Π²Π° прСдставлСна ​​элСктричСская Ρ†Π΅ΠΏΡŒ, содСрТащая Π±Π°Ρ‚Π°Ρ€Π΅ΡŽ, ΠΏΠ΅Ρ€Π΅ΠΊΠ»ΡŽΡ‡Π°Ρ‚Π΅Π»ΡŒ ΠΈ кондСнсатор. Π‘Π»Π΅Π²Π° — Π°Π½Π°Π»ΠΎΠ³ΠΈΡ‡Π½Ρ‹ΠΉ водяной ΠΊΠΎΠ½Ρ‚ΡƒΡ€ с насосом, ΠΊΠ»Π°ΠΏΠ°Π½ΠΎΠΌ ΠΈ Ρ€Π°ΡΡ‚ΡΠ³ΠΈΠ²Π°ΡŽΡ‰Π΅ΠΉΡΡ ΠΌΠ΅ΠΌΠ±Ρ€Π°Π½ΠΎΠΉ.Насос ΠΏΠΎΠ΄ΠΎΠ±Π΅Π½ Π±Π°Ρ‚Π°Ρ€Π΅Π΅, ΠΊΠ»Π°ΠΏΠ°Π½ — ΠΏΠ΅Ρ€Π΅ΠΊΠ»ΡŽΡ‡Π°Ρ‚Π΅Π»ΡŽ, Π° растяТимая ΠΌΠ΅ΠΌΠ±Ρ€Π°Π½Π° — кондСнсатору. Когда ΠΏΠ΅Ρ€Π΅ΠΊΠ»ΡŽΡ‡Π°Ρ‚Π΅Π»ΡŒ Π·Π°ΠΌΠΊΠ½ΡƒΡ‚, элСктричСский Ρ‚ΠΎΠΊ Ρ‚Π΅Ρ‡Π΅Ρ‚ ΠΏΠΎ ΠΌΠ΅Ρ€Π΅ зарядки кондСнсатора ΠΈ увСличСния Π΅Π³ΠΎ напряТСния. Π’ΠΎΡ‡Π½ΠΎ Ρ‚Π°ΠΊ ΠΆΠ΅ Π² водяном ΠΊΠΎΠ½Ρ‚ΡƒΡ€Π΅, ΠΊΠΎΠ³Π΄Π° ΠΊΠ»Π°ΠΏΠ°Π½ ΠΎΡ‚ΠΊΡ€Ρ‹Ρ‚, ΠΏΠΎΡ‚ΠΎΠΊ Π²ΠΎΠ΄Ρ‹ Ρ‚Π΅Ρ‡Π΅Ρ‚ ΠΏΠΎ ΠΌΠ΅Ρ€Π΅ Ρ‚ΠΎΠ³ΠΎ, ΠΊΠ°ΠΊ Ρ€Π°ΡΡ‚ΡΠ³ΠΈΠ²Π°ΡŽΡ‰Π°ΡΡΡ ΠΌΠ΅ΠΌΠ±Ρ€Π°Π½Π° растягиваСтся, ΠΈ Π΄Π°Π²Π»Π΅Π½ΠΈΠ΅ Π²ΠΎΠ΄Ρ‹ Π·Π° Π½Π΅ΠΉ увСличиваСтся.

Π Π°Π±ΠΎΡ‚Π° Π² области Ρ„ΠΈΠ·ΠΈΠΊΠΈ

Π§Ρ‚ΠΎ Π½ΡƒΠΆΠ½ΠΎ, Ρ‡Ρ‚ΠΎΠ±Ρ‹ ΡΡ‚Π°Ρ‚ΡŒ ΠΈΠ½ΠΆΠ΅Π½Π΅Ρ€ΠΎΠΌ-элСктриком

Π€ΠΈΠ·ΠΈΠΊΠ° ΠΈΡΠΏΠΎΠ»ΡŒΠ·ΡƒΠ΅Ρ‚ΡΡ Π² самых Ρ€Π°Π·Π½Ρ‹Ρ… областях.Одна ΠΈΠ· областСй, Ρ‚Ρ€Π΅Π±ΡƒΡŽΡ‰Π°Ρ ΠΎΡ‡Π΅Π½ΡŒ Π³Π»ΡƒΠ±ΠΎΠΊΠΈΡ… Π·Π½Π°Π½ΠΈΠΉ Ρ„ΠΈΠ·ΠΈΠΊΠΈ, — это элСктротСхника. Π˜Π½ΠΆΠ΅Π½Π΅Ρ€-элСктрик ΠΌΠΎΠΆΠ΅Ρ‚ Ρ€Π°Π±ΠΎΡ‚Π°Ρ‚ΡŒ Π½Π°Π΄ Ρ‡Π΅ΠΌ ΡƒΠ³ΠΎΠ΄Π½ΠΎ, ΠΎΡ‚ ΠΊΡ€ΡƒΠΏΠ½ΠΎΠΌΠ°ΡΡˆΡ‚Π°Π±Π½Ρ‹Ρ… энСргосистСм, ΠΎΠ±Π΅ΡΠΏΠ΅Ρ‡ΠΈΠ²Π°ΡŽΡ‰ΠΈΡ… энСргиСй большиС Π³ΠΎΡ€ΠΎΠ΄Π°, Π΄ΠΎ Π½Π°Π½ΠΎΡ€Π°Π·ΠΌΠ΅Ρ€Π½Ρ‹Ρ… элСктронных схСм, ΠΊΠΎΡ‚ΠΎΡ€Ρ‹Π΅ ΠΈΡΠΏΠΎΠ»ΡŒΠ·ΡƒΡŽΡ‚ΡΡ Π² ΠΊΠΎΠΌΠΏΡŒΡŽΡ‚Π΅Ρ€Π°Ρ… ΠΈ сотовых Ρ‚Π΅Π»Π΅Ρ„ΠΎΠ½Π°Ρ… (рис. 19.15).

Работая с энСргокомпаниями, Π²Ρ‹ ΠΌΠΎΠΆΠ΅Ρ‚Π΅ нСсти ΠΎΡ‚Π²Π΅Ρ‚ΡΡ‚Π²Π΅Π½Π½ΠΎΡΡ‚ΡŒ Π·Π° обслуТиваниС элСктросСти, которая поставляСт ΡΠ»Π΅ΠΊΡ‚Ρ€ΠΎΡΠ½Π΅Ρ€Π³ΠΈΡŽ Π½Π° большиС Ρ‚Π΅Ρ€Ρ€ΠΈΡ‚ΠΎΡ€ΠΈΠΈ. Π₯отя большая Ρ‡Π°ΡΡ‚ΡŒ этой Ρ€Π°Π±ΠΎΡ‚Ρ‹ выполняСтся ΠΈΠ· офиса, ΠΎΠ±Ρ‹Ρ‡Π½ΠΎ ΠΈΡ… Π²Ρ‹Π·Ρ‹Π²Π°ΡŽΡ‚ Π½Π° ΡΠ²Π΅Ρ€Ρ…ΡƒΡ€ΠΎΡ‡Π½ΡƒΡŽ Ρ€Π°Π±ΠΎΡ‚Ρƒ послС ΡˆΡ‚ΠΎΡ€ΠΌΠΎΠ² ΠΈΠ»ΠΈ Π΄Ρ€ΡƒΠ³ΠΈΡ… стихийных бСдствий.Многим ΠΈΠ½ΠΆΠ΅Π½Π΅Ρ€Π°ΠΌ-элСктрикам нравится эта Ρ‡Π°ΡΡ‚ΡŒ Ρ€Π°Π±ΠΎΡ‚Ρ‹, которая Ρ‚Ρ€Π΅Π±ΡƒΠ΅Ρ‚ ΠΎΡ‚ Π½ΠΈΡ… Π³ΠΎΠ½ΠΎΠΊ ΠΏΠΎ сСльской мСстности, Ρ€Π΅ΠΌΠΎΠ½Ρ‚ΠΈΡ€ΡƒΡŽΡ‰ΠΈΡ… Π²Ρ‹ΡΠΎΠΊΠΎΠ²ΠΎΠ»ΡŒΡ‚Π½Ρ‹Π΅ трансформаторы ΠΈ Π΄Ρ€ΡƒΠ³ΠΎΠ΅ ΠΎΠ±ΠΎΡ€ΡƒΠ΄ΠΎΠ²Π°Π½ΠΈΠ΅. Однако ΠΎΠ΄Π½ΠΈΠΌ ΠΈΠ· Π½Π°ΠΈΠ±ΠΎΠ»Π΅Π΅ нСприятных аспСктов этой Ρ€Π°Π±ΠΎΡ‚Ρ‹ являСтся ΡƒΠ΄Π°Π»Π΅Π½ΠΈΠ΅ Ρ‚Ρ€ΡƒΠΏΠΎΠ² нСсчастных Π±Π΅Π»ΠΎΠΊ ΠΈΠ»ΠΈ Π΄Ρ€ΡƒΠ³ΠΈΡ… ΠΆΠΈΠ²ΠΎΡ‚Π½Ρ‹Ρ…, Π·Π°Π±Ρ€Π΅Π΄ΡˆΠΈΡ… Π² трансформСры.

Другая ΠΊΠ°Ρ€ΡŒΠ΅Ρ€Π° Π² области элСктротСхники ΠΌΠΎΠΆΠ΅Ρ‚ Π²ΠΊΠ»ΡŽΡ‡Π°Ρ‚ΡŒ Ρ€Π°Π·Ρ€Π°Π±ΠΎΡ‚ΠΊΡƒ схСм для сотовых Ρ‚Π΅Π»Π΅Ρ„ΠΎΠ½ΠΎΠ², Ρ‡Ρ‚ΠΎ Ρ‚Ρ€Π΅Π±ΡƒΠ΅Ρ‚ Π²Ρ‚ΠΈΡΠ½ΡƒΡ‚ΡŒ ΠΎΠΊΠΎΠ»ΠΎ 10 ΠΌΠΈΠ»Π»ΠΈΠ°Ρ€Π΄ΠΎΠ² транзисторов Π² элСктронный Ρ‡ΠΈΠΏ Ρ€Π°Π·ΠΌΠ΅Ρ€ΠΎΠΌ с Π½ΠΎΠ³ΠΎΡ‚ΡŒ большого ΠΏΠ°Π»ΡŒΡ†Π°.Π­Ρ‚ΠΈ Ρ€Π°Π±ΠΎΡ‚Ρ‹ ΠΌΠΎΠ³ΡƒΡ‚ Π²ΠΊΠ»ΡŽΡ‡Π°Ρ‚ΡŒ ΠΌΠ½ΠΎΠ³ΠΎ Ρ€Π°Π±ΠΎΡ‚Ρ‹ с ΠΊΠΎΠΌΠΏΡŒΡŽΡ‚Π΅Ρ€Π½Ρ‹ΠΌ ΠΌΠΎΠ΄Π΅Π»ΠΈΡ€ΠΎΠ²Π°Π½ΠΈΠ΅ΠΌ, Π° Ρ‚Π°ΠΊΠΆΠ΅ ΠΌΠΎΠ³ΡƒΡ‚ Π²ΠΊΠ»ΡŽΡ‡Π°Ρ‚ΡŒ Π² сСбя Π΄Ρ€ΡƒΠ³ΠΈΠ΅ области, ΠΏΠΎΠΌΠΈΠΌΠΎ элСктроники. НапримСр, Π»ΠΈΠ½Π·Ρ‹ Π΄ΠΈΠ°ΠΌΠ΅Ρ‚Ρ€ΠΎΠΌ 1 ΠΌ, ΠΊΠΎΡ‚ΠΎΡ€Ρ‹Π΅ ΠΈΡΠΏΠΎΠ»ΡŒΠ·ΡƒΡŽΡ‚ΡΡ для изготовлСния этих схСм (ΠΏΠΎ ΡΠΎΡΡ‚ΠΎΡΠ½ΠΈΡŽ Π½Π° 2015 Π³ΠΎΠ΄), Π½Π°ΡΡ‚ΠΎΠ»ΡŒΠΊΠΎ Ρ‚ΠΎΡ‡Π½Ρ‹, Ρ‡Ρ‚ΠΎ ΠΈΡ… ΠΎΡ‚ΠΏΡ€Π°Π²Π»ΡΡŽΡ‚ с производства Π½Π° Π·Π°Π²ΠΎΠ΄ ΠΏΠΎ производству микросхСм Π½Π° Π³Ρ€ΡƒΠ·ΠΎΠ²ΠΈΠΊΠ°Ρ… с Ρ€Π΅Π³ΡƒΠ»ΠΈΡ€ΡƒΠ΅ΠΌΠΎΠΉ Ρ‚Π΅ΠΌΠΏΠ΅Ρ€Π°Ρ‚ΡƒΡ€ΠΎΠΉ, Ρ‡Ρ‚ΠΎΠ±Ρ‹ Π³Π°Ρ€Π°Π½Ρ‚ΠΈΡ€ΠΎΠ²Π°Ρ‚ΡŒ, Ρ‡Ρ‚ΠΎ ΠΎΠ½ΠΈ ΡƒΠ΄Π΅Ρ€ΠΆΠΈΠ²Π°ΡŽΡ‚ΡΡ Π² ΠΎΠΏΡ€Π΅Π΄Π΅Π»Π΅Π½Π½Ρ‹ΠΉ Ρ‚Π΅ΠΌΠΏΠ΅Ρ€Π°Ρ‚ΡƒΡ€Π½Ρ‹ΠΉ Π΄ΠΈΠ°ΠΏΠ°Π·ΠΎΠ½. Если ΠΎΠ½ΠΈ Π½Π°Π³Ρ€Π΅Π²Π°ΡŽΡ‚ΡΡ ΠΈΠ»ΠΈ ΠΎΡ…Π»Π°ΠΆΠ΄Π°ΡŽΡ‚ΡΡ слишком сильно, ΠΎΠ½ΠΈ слСгка Π΄Π΅Ρ„ΠΎΡ€ΠΌΠΈΡ€ΡƒΡŽΡ‚ΡΡ, дСлая ΠΈΡ… бСсполСзными для свСрхточной Ρ„ΠΎΡ‚ΠΎΠ»ΠΈΡ‚ΠΎΠ³Ρ€Π°Ρ„ΠΈΠΈ, Π½Π΅ΠΎΠ±Ρ…ΠΎΠ΄ΠΈΠΌΠΎΠΉ для производства этих Ρ‡ΠΈΠΏΠΎΠ².

Помимо Π³Π»ΡƒΠ±ΠΎΠΊΠΈΡ… ΠΏΠΎΠ·Π½Π°Π½ΠΈΠΉ Π² Ρ„ΠΈΠ·ΠΈΠΊΠ΅, ΠΈΠ½ΠΆΠ΅Π½Π΅Ρ€Ρ‹-элСктрики Π΄ΠΎΠ»ΠΆΠ½Ρ‹, ΠΏΡ€Π΅ΠΆΠ΄Π΅ всСго, Π±Ρ‹Ρ‚ΡŒ ΠΏΡ€Π°ΠΊΡ‚ΠΈΡ‡Π½Ρ‹ΠΌΠΈ. Рассмотрим, Π½Π°ΠΏΡ€ΠΈΠΌΠ΅Ρ€, ΠΊΠ°ΠΊ ΠΎΠ΄Π½ΠΎΠΉ ΠΊΠΎΡ€ΠΏΠΎΡ€Π°Ρ†ΠΈΠΈ ΡƒΠ΄Π°Π»ΠΎΡΡŒ Π·Π°ΠΏΡƒΡΡ‚ΠΈΡ‚ΡŒ нСсколько ΠΏΡ€ΠΎΡ‚ΠΈΠ²ΠΎΡ€Π°ΠΊΠ΅Ρ‚Π½Ρ‹Ρ… Ρ€Π°ΠΊΠ΅Ρ‚ Π½Π° Ρ€Π°ΠΊΠ΅Ρ‚Π½ΠΎΠΌ ΠΏΠΎΠ»ΠΈΠ³ΠΎΠ½Π΅ Π£Π°ΠΉΡ‚-Бэндс Π² Нью-МСксико Π² 1960-Ρ… Π³ΠΎΠ΄Π°Ρ…. ΠŸΠ΅Ρ€Π΅Π΄ запуском обшивка Ρ€Π°ΠΊΠ΅Ρ‚Ρ‹ Π΄ΠΎΠ»ΠΆΠ½Π° Π±Ρ‹Π»Π° Π½Π°Ρ…ΠΎΠ΄ΠΈΡ‚ΡŒΡΡ ΠΏΠΎΠ΄ Ρ‚Π΅ΠΌ ΠΆΠ΅ напряТСниСм, Ρ‡Ρ‚ΠΎ ΠΈ Ρ€Π΅Π»ΡŒΡ, с ΠΊΠΎΡ‚ΠΎΡ€ΠΎΠ³ΠΎ ΠΎΠ½Π° Π·Π°ΠΏΡƒΡΠΊΠ°Π»Π°ΡΡŒ. РСльс Π±Ρ‹Π» соСдинСн с Π·Π΅ΠΌΠ»Π΅ΠΉ большим ΠΌΠ΅Π΄Π½Ρ‹ΠΌ ΠΏΡ€ΠΎΠ²ΠΎΠ΄ΠΎΠΌ, соСдинСнным с ΠΊΠΎΠ»ΠΎΠΌ, Π²Π±ΠΈΡ‚Ρ‹ΠΌ Π² ΠΏΠ΅ΡΡ‡Π°Π½ΡƒΡŽ зСмлю. Однако Ρ€Π°ΠΊΠ΅Ρ‚Π° Π±Ρ‹Π»Π° соСдинСна с ΠΏΠΎΠΌΠΎΡ‰ΡŒΡŽ ΠΏΡƒΠΏΠΎΠ²ΠΈΠ½Ρ‹ с ΠΎΠ±ΠΎΡ€ΡƒΠ΄ΠΎΠ²Π°Π½ΠΈΠ΅ΠΌ Π² диспСтчСрской Π² β€‹β€‹Π½Π΅ΡΠΊΠΎΠ»ΡŒΠΊΠΈΡ… ΠΌΠ΅Ρ‚Ρ€Π°Ρ… ΠΎΡ‚ Π½Π΅Π΅, ΠΊΠΎΡ‚ΠΎΡ€ΠΎΠ΅ Π±Ρ‹Π»ΠΎ Π·Π°Π·Π΅ΠΌΠ»Π΅Π½ΠΎ Ρ‡Π΅Ρ€Π΅Π· Π΄Ρ€ΡƒΠ³ΡƒΡŽ Ρ†Π΅ΠΏΡŒ зазСмлСния.ΠŸΠ΅Ρ€Π΅Π΄ запуском Ρ€Π°ΠΊΠ΅Ρ‚Ρ‹ Ρ€Π°Π·Π½ΠΈΡ†Π° напряТСний ΠΌΠ΅ΠΆΠ΄Ρƒ обшивкой Ρ€Π°ΠΊΠ΅Ρ‚Ρ‹ ΠΈ Π½Π°ΠΏΡ€Π°Π²Π»ΡΡŽΡ‰Π΅ΠΉ Π΄ΠΎΠ»ΠΆΠ½Π° Π±Ρ‹Ρ‚ΡŒ ΠΌΠ΅Π½Π΅Π΅ 2,5 Π’. ПослС особСнно Π·Π°ΡΡƒΡˆΠ»ΠΈΠ²ΠΎΠΉ ΠΏΠΎΠ³ΠΎΠ΄Ρ‹ запуск Ρ€Π°ΠΊΠ΅Ρ‚Ρ‹ Π½Π΅Π²ΠΎΠ·ΠΌΠΎΠΆΠ΅Π½, Ρ‚Π°ΠΊ ΠΊΠ°ΠΊ Ρ€Π°Π·Π½ΠΈΡ†Π° напряТСний составляСт 5 Π’. Π“Ρ€ΡƒΠΏΠΏΠ° элСктричСских Ρ†Π΅ΠΏΠ΅ΠΉ. ΠΈΠ½ΠΆΠ΅Π½Π΅Ρ€Ρ‹, Π² Ρ‚ΠΎΠΌ числС ΠΎΡ‚Π΅Ρ† вашСго Π°Π²Ρ‚ΠΎΡ€Π°, стояли ΠΈ Ρ€Π°Π·ΠΌΡ‹ΡˆΠ»ΡΠ»ΠΈ, ΠΊΠ°ΠΊ ΡƒΠΌΠ΅Π½ΡŒΡˆΠΈΡ‚ΡŒ Ρ€Π°Π·Π½ΠΈΡ†Ρƒ напряТСний. Битуация Ρ€Π°Π·Ρ€Π΅ΡˆΠΈΠ»Π°ΡΡŒ, ΠΊΠΎΠ³Π΄Π° ΠΎΠ΄ΠΈΠ½ ΠΈΠ· ΠΈΠ½ΠΆΠ΅Π½Π΅Ρ€ΠΎΠ² понял, Ρ‡Ρ‚ΠΎ ΠΌΠΎΡ‡Π° содСрТит элСктролиты ΠΈ Π½Π΅ΠΏΠ»ΠΎΡ…ΠΎ ΠΏΡ€ΠΎΠ²ΠΎΠ΄ΠΈΡ‚ элСктричСство.ΠŸΡ€ΠΈ этом Ρ‡Π΅Ρ‚Ρ‹Ρ€Π΅ ΠΈΠ½ΠΆΠ΅Π½Π΅Ρ€Π° быстро Ρ€Π΅ΡˆΠΈΠ»ΠΈ ΠΏΡ€ΠΎΠ±Π»Π΅ΠΌΡƒ, ΠΏΠΎΠΌΠΎΡ‡ΠΈΠ²ΡˆΠΈΡΡŒ Π½Π° шип. Π Π°Π·Π½ΠΈΡ†Π° напряТСний сразу ΡƒΠΏΠ°Π»Π° Π½ΠΈΠΆΠ΅ 2,5 Π’, ΠΈ Ρ€Π°ΠΊΠ΅Ρ‚Π° Π±Ρ‹Π»Π° Π·Π°ΠΏΡƒΡ‰Π΅Π½Π° ΠΏΠΎ Π³Ρ€Π°Ρ„ΠΈΠΊΡƒ.

Рисунок 19.15 БистСмы, Π½Π°Π΄ ΠΊΠΎΡ‚ΠΎΡ€Ρ‹ΠΌΠΈ Ρ€Π°Π±ΠΎΡ‚Π°ΡŽΡ‚ ΠΈΠ½ΠΆΠ΅Π½Π΅Ρ€Ρ‹-элСктрики, Π²Π°Ρ€ΡŒΠΈΡ€ΡƒΡŽΡ‚ΡΡ ΠΎΡ‚ микропроцСссорных схСм (слСва) Π΄ΠΎ Ρ€Π°ΠΊΠ΅Ρ‚Π½Ρ‹Ρ… систСм (справа).

Virtual Physics

Π Π°Π·Π²Π»Π΅ΠΊΠ°ΠΉΡ‚Π΅ΡΡŒ, строя схСмы самых Ρ€Π°Π·Π½Ρ‹Ρ… Ρ„ΠΎΡ€ΠΌ ΠΈ Ρ€Π°Π·ΠΌΠ΅Ρ€ΠΎΠ². Π­Ρ‚ΠΎ ΠΌΠΎΠ΄Π΅Π»ΠΈΡ€ΠΎΠ²Π°Π½ΠΈΠ΅ прСдоставляСт Π²Π°ΠΌ Ρ€Π°Π·Π»ΠΈΡ‡Π½Ρ‹Π΅ стандартныС элСмСнты схСмы, Ρ‚Π°ΠΊΠΈΠ΅ ΠΊΠ°ΠΊ Π±Π°Ρ‚Π°Ρ€Π΅ΠΈ, источники ΠΏΠ΅Ρ€Π΅ΠΌΠ΅Π½Π½ΠΎΠ³ΠΎ напряТСния, рСзисторы, кондСнсаторы, Π»Π°ΠΌΠΏΠΎΡ‡ΠΊΠΈ, ΠΏΠ΅Ρ€Π΅ΠΊΠ»ΡŽΡ‡Π°Ρ‚Π΅Π»ΠΈ ΠΈ Ρ‚. Π”.Π’Ρ‹ ΠΌΠΎΠΆΠ΅Ρ‚Π΅ ΠΏΠΎΠ΄ΠΊΠ»ΡŽΡ‡ΠΈΡ‚ΡŒ ΠΈΡ… Π² любой ΠΊΠΎΠ½Ρ„ΠΈΠ³ΡƒΡ€Π°Ρ†ΠΈΠΈ, которая Π²Π°ΠΌ нравится, ΠΈ Π·Π°Ρ‚Π΅ΠΌ ΡƒΠ²ΠΈΠ΄Π΅Ρ‚ΡŒ Ρ€Π΅Π·ΡƒΠ»ΡŒΡ‚Π°Ρ‚.

Π‘ΠΎΠ·Π΄Π°ΠΉΡ‚Π΅ схСму, которая начинаСтся с рСзистора, ΠΏΠΎΠ΄ΠΊΠ»ΡŽΡ‡Π΅Π½Π½ΠΎΠ³ΠΎ ΠΊ кондСнсатору. ΠŸΠΎΠ΄ΠΊΠ»ΡŽΡ‡ΠΈΡ‚Π΅ ΡΠ²ΠΎΠ±ΠΎΠ΄Π½ΡƒΡŽ сторону рСзистора ΠΊ ΠΏΠΎΠ»ΠΎΠΆΠΈΡ‚Π΅Π»ΡŒΠ½ΠΎΠΉ ΠΊΠ»Π΅ΠΌΠΌΠ΅ Π±Π°Ρ‚Π°Ρ€Π΅ΠΈ, Π° ΡΠ²ΠΎΠ±ΠΎΠ΄Π½ΡƒΡŽ сторону кондСнсатора — ΠΊ ΠΎΡ‚Ρ€ΠΈΡ†Π°Ρ‚Π΅Π»ΡŒΠ½ΠΎΠΉ ΠΊΠ»Π΅ΠΌΠΌΠ΅ Π±Π°Ρ‚Π°Ρ€Π΅ΠΈ. НаТмитС ΠΊΠ½ΠΎΠΏΠΊΡƒ ΡΠ±Ρ€ΠΎΡΠΈΡ‚ΡŒ Π΄ΠΈΠ½Π°ΠΌΠΈΠΊΡƒ , Ρ‡Ρ‚ΠΎΠ±Ρ‹ ΡƒΠ²ΠΈΠ΄Π΅Ρ‚ΡŒ, ΠΊΠ°ΠΊ ΠΏΡ€ΠΎΡ‚Π΅ΠΊΠ°Π΅Ρ‚ Ρ‚ΠΎΠΊ, начиная с Π½ΡƒΠ»Π΅Π²ΠΎΠ³ΠΎ заряда кондСнсатора. Π’Π΅ΠΏΠ΅Ρ€ΡŒ Ρ‰Π΅Π»ΠΊΠ½ΠΈΡ‚Π΅ рСзистор ΠΏΡ€Π°Π²ΠΎΠΉ ΠΊΠ½ΠΎΠΏΠΊΠΎΠΉ ΠΌΡ‹ΡˆΠΈ, Ρ‡Ρ‚ΠΎΠ±Ρ‹ ΠΈΠ·ΠΌΠ΅Π½ΠΈΡ‚ΡŒ Π΅Π³ΠΎ Π·Π½Π°Ρ‡Π΅Π½ΠΈΠ΅. Когда Π²Ρ‹ ΡƒΠ²Π΅Π»ΠΈΡ‡ΠΈΠ²Π°Π΅Ρ‚Π΅ сопротивлСниС, схСма достигаСт ΡƒΡΡ‚Π°Π½ΠΎΠ²ΠΈΠ²ΡˆΠ΅Π³ΠΎΡΡ состояния быстрСС ΠΈΠ»ΠΈ ΠΌΠ΅Π΄Π»Π΅Π½Π½Π΅Π΅?

ΠŸΡ€ΠΎΠ²Π΅Ρ€ΠΊΠ° Π·Π°Ρ…Π²Π°Ρ‚Π°

Когда схСма достигла ΡƒΡΡ‚Π°Π½ΠΎΠ²ΠΈΠ²ΡˆΠ΅Π³ΠΎΡΡ состояния, ΠΊΠ°ΠΊ напряТСниС Π½Π° кондСнсаторС сравниваСтся с напряТСниСм Π±Π°Ρ‚Π°Ρ€Π΅ΠΈ? КакоС напряТСниС Π½Π° рСзисторС?

  1. НапряТСниС Π½Π° кондСнсаторС большС, Ρ‡Π΅ΠΌ напряТСниС аккумулятора.Π’ ΡƒΡΡ‚Π°Π½ΠΎΠ²ΠΈΠ²ΡˆΠ΅ΠΌΡΡ Ρ€Π΅ΠΆΠΈΠΌΠ΅ Ρ‡Π΅Ρ€Π΅Π· эту Ρ†Π΅ΠΏΡŒ Π½Π΅ ΠΏΡ€ΠΎΡ‚Π΅ΠΊΠ°Π΅Ρ‚ Ρ‚ΠΎΠΊ, поэтому напряТСниС Π½Π° рСзисторС Ρ€Π°Π²Π½ΠΎ Π½ΡƒΠ»ΡŽ.
  2. НапряТСниС Π½Π° кондСнсаторС мСньшС напряТСния Π±Π°Ρ‚Π°Ρ€Π΅ΠΈ. Π’ ΡƒΡΡ‚Π°Π½ΠΎΠ²ΠΈΠ²ΡˆΠ΅ΠΌΡΡ Ρ€Π΅ΠΆΠΈΠΌΠ΅ Ρ‡Π΅Ρ€Π΅Π· эту Ρ†Π΅ΠΏΡŒ ΠΏΡ€ΠΎΡ‚Π΅ΠΊΠ°Π΅Ρ‚ ΠΊΠΎΠ½Π΅Ρ‡Π½Ρ‹ΠΉ Ρ‚ΠΎΠΊ, поэтому напряТСниС Π½Π° рСзисторС ΠΊΠΎΠ½Π΅Ρ‡Π½ΠΎ.
  3. НапряТСниС Π½Π° кондСнсаторС Ρ‚Π°ΠΊΠΎΠ΅ ΠΆΠ΅, ΠΊΠ°ΠΊ напряТСниС аккумулятора. Π’ ΡƒΡΡ‚Π°Π½ΠΎΠ²ΠΈΠ²ΡˆΠ΅ΠΌΡΡ Ρ€Π΅ΠΆΠΈΠΌΠ΅ Ρ‡Π΅Ρ€Π΅Π· эту Ρ†Π΅ΠΏΡŒ Π½Π΅ ΠΏΡ€ΠΎΡ‚Π΅ΠΊΠ°Π΅Ρ‚ Ρ‚ΠΎΠΊ, поэтому напряТСниС Π½Π° рСзисторС Ρ€Π°Π²Π½ΠΎ Π½ΡƒΠ»ΡŽ.
  4. НапряТСниС Π½Π° кондСнсаторС Ρ‚Π°ΠΊΠΎΠ΅ ΠΆΠ΅, ΠΊΠ°ΠΊ напряТСниС аккумулятора. Π’ ΡƒΡΡ‚Π°Π½ΠΎΠ²ΠΈΠ²ΡˆΠ΅ΠΌΡΡ Ρ€Π΅ΠΆΠΈΠΌΠ΅ Ρ‡Π΅Ρ€Π΅Π· эту Ρ†Π΅ΠΏΡŒ ΠΏΡ€ΠΎΡ‚Π΅ΠΊΠ°Π΅Ρ‚ ΠΊΠΎΠ½Π΅Ρ‡Π½Ρ‹ΠΉ Ρ‚ΠΎΠΊ, поэтому напряТСниС Π½Π° рСзисторС ΠΊΠΎΠ½Π΅Ρ‡Π½ΠΎ.

ЭлСктричСскиС схСмы Π“Π»Π°Π²Π° 20 Π Π°Π·Π΄Π΅Π» Ρ‚Ρ€Π΅Ρ‚ΠΈΠΉ. Π‘Ρ‚Π°Ρ‚ΡŒΡ Π² Π½Π°ΡƒΡ‡Π½ΠΎΠΌ ΠΆΡƒΡ€Π½Π°Π»Π΅ β„– 42, ΠΈΠ·Π»Π°Π³Π°ΡŽΡ‰Π°Ρ Π·Π°ΠΊΠΎΠ½ Ома ΠΈ Π΅Π³ΠΎ связь с Ρ‚ΠΎΠΊΠΎΠΌ, сопротивлСниСм ΠΈ напряТСниСм.

ΠŸΡ€Π΅Π·Π΅Π½Ρ‚Π°Ρ†ΠΈΡ Π½Π° Ρ‚Π΅ΠΌΡƒ: «ЭлСктричСскиС схСмы Π“Π»Π°Π²Π° 20 Π Π°Π·Π΄Π΅Π» Ρ‚Ρ€Π΅Ρ‚ΠΈΠΉ.Π‘Ρ‚Π°Ρ‚ΡŒΡ Π² Научном ΠΆΡƒΡ€Π½Π°Π»Π΅ β„– 42, ΠΈΠ·Π»Π°Π³Π°ΡŽΡ‰Π°Ρ Π·Π°ΠΊΠΎΠ½ Ома ΠΈ Π΅Π³ΠΎ связь с Ρ‚ΠΎΠΊΠΎΠΌ, сопротивлСниСм ΠΈ напряТСниСм Β»- стСнограмма ΠΏΡ€Π΅Π·Π΅Π½Ρ‚Π°Ρ†ΠΈΠΈ:

ins [data-ad-slot = «4502451947»] {display: none! important;}} @media (max-width: 800px) {# place_14> ins: not ([data-ad-slot = «4502451947»]) {display: none! important;}} @media (max-width: 800px) {# place_14 {width: 250px;}} @media (max-width: 500 пиксСлСй) {# place_14 {width: 120px;}} ]]>

1 ЭлСктричСскиС схСмы Π“Π»Π°Π²Π° 20 Π Π°Π·Π΄Π΅Π» 3

2 Π‘Ρ‚Π°Ρ‚ΡŒΡ Π² Π½Π°ΡƒΡ‡Π½ΠΎΠΌ ΠΆΡƒΡ€Π½Π°Π»Π΅ β„– 42, ΠΈΠ·Π»Π°Π³Π°ΡŽΡ‰Π°Ρ Π·Π°ΠΊΠΎΠ½ Ома ΠΈ Π΅Π³ΠΎ связь с Ρ‚ΠΎΠΊΠΎΠΌ, сопротивлСниСм ΠΈ напряТСниСм.

3 Π‘Ρ…Π΅ΠΌΡ‹ элСктричСских Ρ†Π΅ΠΏΠ΅ΠΉ ЭлСктричСская Ρ†Π΅ΠΏΡŒ — это ΠΏΠΎΠ»Π½Ρ‹ΠΉ ΠΏΡƒΡ‚ΡŒ, ΠΏΠΎ ΠΊΠΎΡ‚ΠΎΡ€ΠΎΠΌΡƒ ΠΌΠΎΠΆΠ΅Ρ‚ Ρ‚Π΅Ρ‡ΡŒ заряд. ЭлСктричСская Ρ†Π΅ΠΏΡŒ — это ΠΏΠΎΠ»Π½Ρ‹ΠΉ ΠΏΡƒΡ‚ΡŒ, ΠΏΠΎ ΠΊΠΎΡ‚ΠΎΡ€ΠΎΠΌΡƒ ΠΌΠΎΠΆΠ΅Ρ‚ Ρ‚Π΅Ρ‡ΡŒ заряд. На ΠΏΡ€ΠΈΠ½Ρ†ΠΈΠΏΠΈΠ°Π»ΡŒΠ½Ρ‹Ρ… схСмах ΠΈΡΠΏΠΎΠ»ΡŒΠ·ΡƒΡŽΡ‚ΡΡ символы для обозначСния частСй Ρ†Π΅ΠΏΠΈ, Π²ΠΊΠ»ΡŽΡ‡Π°Ρ источник элСктричСской энСргии ΠΈ устройства, Ρ€Π°Π±ΠΎΡ‚Π°ΡŽΡ‰ΠΈΠ΅ ΠΎΡ‚ элСктричСской энСргии. ΠŸΡ€ΠΈΠ½Ρ†ΠΈΠΏΠΈΠ°Π»ΡŒΠ½Π°Ρ схСма ΠΏΠΎΠΊΠ°Π·Ρ‹Π²Π°Π΅Ρ‚ ΠΎΠ΄ΠΈΠ½ ΠΈΠ»ΠΈ нСсколько ΠΏΠΎΠ»Π½Ρ‹Ρ… ΠΏΡƒΡ‚Π΅ΠΉ, ΠΏΠΎ ΠΊΠΎΡ‚ΠΎΡ€Ρ‹ΠΌ ΠΌΠΎΠΆΠ΅Ρ‚ Ρ‚Π΅Ρ‡ΡŒ заряд. На ΠΏΡ€ΠΈΠ½Ρ†ΠΈΠΏΠΈΠ°Π»ΡŒΠ½Ρ‹Ρ… схСмах ΠΈΡΠΏΠΎΠ»ΡŒΠ·ΡƒΡŽΡ‚ΡΡ символы для обозначСния частСй Ρ†Π΅ΠΏΠΈ, Π²ΠΊΠ»ΡŽΡ‡Π°Ρ источник элСктричСской энСргии ΠΈ устройства, Ρ€Π°Π±ΠΎΡ‚Π°ΡŽΡ‰ΠΈΠ΅ ΠΎΡ‚ элСктричСской энСргии.ΠŸΡ€ΠΈΠ½Ρ†ΠΈΠΏΠΈΠ°Π»ΡŒΠ½Π°Ρ схСма ΠΏΠΎΠΊΠ°Π·Ρ‹Π²Π°Π΅Ρ‚ ΠΎΠ΄ΠΈΠ½ ΠΈΠ»ΠΈ нСсколько ΠΏΠΎΠ»Π½Ρ‹Ρ… ΠΏΡƒΡ‚Π΅ΠΉ, ΠΏΠΎ ΠΊΠΎΡ‚ΠΎΡ€Ρ‹ΠΌ ΠΌΠΎΠΆΠ΅Ρ‚ Ρ‚Π΅Ρ‡ΡŒ заряд.

4 ΠŸΠ΅Ρ€Π΅ΠΊΠ»ΡŽΡ‡Π°Ρ‚Π΅Π»ΠΈ Ρ€Π°Π·ΠΎΠΌΠΊΠ½ΡƒΡ‚ΠΎΠΉ ΠΈΠ»ΠΈ Π·Π°ΠΌΠΊΠ½ΡƒΡ‚ΠΎΠΉ Ρ†Π΅ΠΏΠΈ ΠΏΠΎΠΊΠ°Π·Ρ‹Π²Π°ΡŽΡ‚ мСста, Π³Π΄Π΅ Ρ†Π΅ΠΏΡŒ ΠΌΠΎΠΆΠ΅Ρ‚ Π±Ρ‹Ρ‚ΡŒ Ρ€Π°Π·ΠΎΠΌΠΊΠ½ΡƒΡ‚Π°. Если ΠΏΠ΅Ρ€Π΅ΠΊΠ»ΡŽΡ‡Π°Ρ‚Π΅Π»ΡŒ Ρ€Π°Π·ΠΎΠΌΠΊΠ½ΡƒΡ‚, Ρ†Π΅ΠΏΡŒ ΠΈΠ»ΠΈ ΠΏΡƒΡ‚ΡŒ ΠΏΡ€Π΅Ρ€Π²Π°Π½Ρ‹, ΠΈ Ρ‚ΠΎΠΊ прСкращаСтся. Когда ΠΏΠ΅Ρ€Π΅ΠΊΠ»ΡŽΡ‡Π°Ρ‚Π΅Π»ΡŒ Π·Π°ΠΌΠΊΠ½ΡƒΡ‚, Ρ†Π΅ΠΏΡŒ ΠΈΠ»ΠΈ ΠΏΡƒΡ‚ΡŒ Π·Π°ΠΌΡ‹ΠΊΠ°ΡŽΡ‚ΡΡ, ΠΈ заряд ΠΌΠΎΠΆΠ΅Ρ‚ Ρ‚Π΅Ρ‡ΡŒ. Π’Ρ‹ΠΊΠ»ΡŽΡ‡Π°Ρ‚Π΅Π»ΠΈ ΠΏΠΎΠΊΠ°Π·Ρ‹Π²Π°ΡŽΡ‚ мСста, Π³Π΄Π΅ Ρ†Π΅ΠΏΡŒ ΠΌΠΎΠΆΠ΅Ρ‚ Π±Ρ‹Ρ‚ΡŒ Ρ€Π°Π·ΠΎΠΌΠΊΠ½ΡƒΡ‚Π°. Если ΠΏΠ΅Ρ€Π΅ΠΊΠ»ΡŽΡ‡Π°Ρ‚Π΅Π»ΡŒ Ρ€Π°Π·ΠΎΠΌΠΊΠ½ΡƒΡ‚, Ρ†Π΅ΠΏΡŒ ΠΈΠ»ΠΈ ΠΏΡƒΡ‚ΡŒ ΠΏΡ€Π΅Ρ€Π²Π°Π½Ρ‹, ΠΈ Ρ‚ΠΎΠΊ прСкращаСтся.Когда ΠΏΠ΅Ρ€Π΅ΠΊΠ»ΡŽΡ‡Π°Ρ‚Π΅Π»ΡŒ Π·Π°ΠΌΠΊΠ½ΡƒΡ‚, Ρ†Π΅ΠΏΡŒ ΠΈΠ»ΠΈ ΠΏΡƒΡ‚ΡŒ Π·Π°ΠΌΡ‹ΠΊΠ°ΡŽΡ‚ΡΡ, ΠΈ заряд ΠΌΠΎΠΆΠ΅Ρ‚ Ρ‚Π΅Ρ‡ΡŒ. ΠŸΠ΅Ρ€Π΅ΠΊΠ»ΡŽΡ‡Π°Ρ‚Π΅Π»ΡŒ Π½Π° этой ΠΊΠ°Ρ€Ρ‚ΠΈΠ½ΠΊΠ΅ ΠΎΡ‚ΠΊΡ€Ρ‹Ρ‚ ΠΈΠ»ΠΈ Π·Π°ΠΊΡ€Ρ‹Ρ‚? ЦСпь Ρ€Π°Π·ΠΎΡ€Π²Π°Π½Π°? ΠŸΠ΅Ρ€Π΅ΠΊΠ»ΡŽΡ‡Π°Ρ‚Π΅Π»ΡŒ Π½Π° этой ΠΊΠ°Ρ€Ρ‚ΠΈΠ½ΠΊΠ΅ ΠΎΡ‚ΠΊΡ€Ρ‹Ρ‚ ΠΈΠ»ΠΈ Π·Π°ΠΊΡ€Ρ‹Ρ‚? ЦСпь Ρ€Π°Π·ΠΎΡ€Π²Π°Π½Π°?

5 Π’ΠΈΠΏΡ‹ Ρ†Π΅ΠΏΠ΅ΠΉ Π•ΡΡ‚ΡŒ Π΄Π²Π° Ρ‚ΠΈΠΏΠ° Ρ†Π΅ΠΏΠ΅ΠΉ: ΠΏΠΎΡΠ»Π΅Π΄ΠΎΠ²Π°Ρ‚Π΅Π»ΡŒΠ½Ρ‹Π΅ Ρ†Π΅ΠΏΠΈ ΠΈ ΠΏΠ°Ρ€Π°Π»Π»Π΅Π»ΡŒΠ½Ρ‹Π΅ Ρ†Π΅ΠΏΠΈ. Π•ΡΡ‚ΡŒ Π΄Π²Π° Ρ‚ΠΈΠΏΠ° Ρ†Π΅ΠΏΠ΅ΠΉ: ΠΏΠΎΡΠ»Π΅Π΄ΠΎΠ²Π°Ρ‚Π΅Π»ΡŒΠ½Ρ‹Π΅ Ρ†Π΅ΠΏΠΈ ΠΈ ΠΏΠ°Ρ€Π°Π»Π»Π΅Π»ΡŒΠ½Ρ‹Π΅ Ρ†Π΅ΠΏΠΈ. Π’ ΠΏΠΎΡΠ»Π΅Π΄ΠΎΠ²Π°Ρ‚Π΅Π»ΡŒΠ½ΠΎΠΉ Ρ†Π΅ΠΏΠΈ заряд ΠΈΠΌΠ΅Π΅Ρ‚ Ρ‚ΠΎΠ»ΡŒΠΊΠΎ ΠΎΠ΄ΠΈΠ½ ΠΏΡƒΡ‚ΡŒ, ΠΏΠΎ ΠΊΠΎΡ‚ΠΎΡ€ΠΎΠΌΡƒ ΠΎΠ½ ΠΌΠΎΠΆΠ΅Ρ‚ Ρ‚Π΅Ρ‡ΡŒ.Если ΠΎΠ΄Π½Π° Π»Π°ΠΌΠΏΠΎΡ‡ΠΊΠ° ΠΏΠ΅Ρ€Π΅Π³ΠΎΡ€Π°Π΅Ρ‚ Π² ΠΏΠΎΡΠ»Π΅Π΄ΠΎΠ²Π°Ρ‚Π΅Π»ΡŒΠ½ΠΎΠΉ Ρ†Π΅ΠΏΠΈ, ΠΎΠ½Π° становится Ρ€Π°Π·ΠΎΠΌΠΊΠ½ΡƒΡ‚ΠΎΠΉ. Π’Π°ΠΊΠΈΠΌ ΠΎΠ±Ρ€Π°Π·ΠΎΠΌ, Ссли ΠΎΠ΄ΠΈΠ½ элСмСнт пСрСстаСт Ρ€Π°Π±ΠΎΡ‚Π°Ρ‚ΡŒ Π² ΠΏΠΎΡΠ»Π΅Π΄ΠΎΠ²Π°Ρ‚Π΅Π»ΡŒΠ½ΠΎΠΉ Ρ†Π΅ΠΏΠΈ, Π½ΠΈ ΠΎΠ΄ΠΈΠ½ ΠΈΠ· элСмСнтов Π½Π΅ ΠΌΠΎΠΆΠ΅Ρ‚ Ρ€Π°Π±ΠΎΡ‚Π°Ρ‚ΡŒ. Π’ ΠΏΠΎΡΠ»Π΅Π΄ΠΎΠ²Π°Ρ‚Π΅Π»ΡŒΠ½ΠΎΠΉ Ρ†Π΅ΠΏΠΈ заряд ΠΈΠΌΠ΅Π΅Ρ‚ Ρ‚ΠΎΠ»ΡŒΠΊΠΎ ΠΎΠ΄ΠΈΠ½ ΠΏΡƒΡ‚ΡŒ, ΠΏΠΎ ΠΊΠΎΡ‚ΠΎΡ€ΠΎΠΌΡƒ ΠΎΠ½ ΠΌΠΎΠΆΠ΅Ρ‚ Ρ‚Π΅Ρ‡ΡŒ. Если ΠΎΠ΄Π½Π° Π»Π°ΠΌΠΏΠΎΡ‡ΠΊΠ° ΠΏΠ΅Ρ€Π΅Π³ΠΎΡ€Π°Π΅Ρ‚ Π² ΠΏΠΎΡΠ»Π΅Π΄ΠΎΠ²Π°Ρ‚Π΅Π»ΡŒΠ½ΠΎΠΉ Ρ†Π΅ΠΏΠΈ, ΠΎΠ½Π° становится Ρ€Π°Π·ΠΎΠΌΠΊΠ½ΡƒΡ‚ΠΎΠΉ. Π’Π°ΠΊΠΈΠΌ ΠΎΠ±Ρ€Π°Π·ΠΎΠΌ, Ссли ΠΎΠ΄ΠΈΠ½ элСмСнт пСрСстаСт Ρ€Π°Π±ΠΎΡ‚Π°Ρ‚ΡŒ Π² ΠΏΠΎΡΠ»Π΅Π΄ΠΎΠ²Π°Ρ‚Π΅Π»ΡŒΠ½ΠΎΠΉ Ρ†Π΅ΠΏΠΈ, Π½ΠΈ ΠΎΠ΄ΠΈΠ½ ΠΈΠ· элСмСнтов Π½Π΅ ΠΌΠΎΠΆΠ΅Ρ‚ Ρ€Π°Π±ΠΎΡ‚Π°Ρ‚ΡŒ. ΠŸΠ°Ρ€Π°Π»Π»Π΅Π»ΡŒΠ½Π°Ρ Ρ†Π΅ΠΏΡŒ — это элСктричСская Ρ†Π΅ΠΏΡŒ с двумя ΠΈΠ»ΠΈ Π±ΠΎΠ»Π΅Π΅ путями, ΠΏΠΎ ΠΊΠΎΡ‚ΠΎΡ€Ρ‹ΠΌ ΠΌΠΎΠ³ΡƒΡ‚ Ρ‚Π΅Ρ‡ΡŒ заряды.Если ΠΎΠ΄ΠΈΠ½ элСмСнт пСрСстаСт Ρ€Π°Π±ΠΎΡ‚Π°Ρ‚ΡŒ Π² ΠΏΠ°Ρ€Π°Π»Π»Π΅Π»ΡŒΠ½ΠΎΠΉ Ρ†Π΅ΠΏΠΈ, ΠΎΡΡ‚Π°Π»ΡŒΠ½Ρ‹Π΅ элСмСнты всС Π΅Ρ‰Π΅ ΠΌΠΎΠ³ΡƒΡ‚ Ρ€Π°Π±ΠΎΡ‚Π°Ρ‚ΡŒ. ΠŸΠ°Ρ€Π°Π»Π»Π΅Π»ΡŒΠ½Π°Ρ Ρ†Π΅ΠΏΡŒ — это элСктричСская Ρ†Π΅ΠΏΡŒ с двумя ΠΈΠ»ΠΈ Π±ΠΎΠ»Π΅Π΅ путями, ΠΏΠΎ ΠΊΠΎΡ‚ΠΎΡ€Ρ‹ΠΌ ΠΌΠΎΠ³ΡƒΡ‚ Ρ‚Π΅Ρ‡ΡŒ заряды. Если ΠΎΠ΄ΠΈΠ½ элСмСнт пСрСстаСт Ρ€Π°Π±ΠΎΡ‚Π°Ρ‚ΡŒ Π² ΠΏΠ°Ρ€Π°Π»Π»Π΅Π»ΡŒΠ½ΠΎΠΉ Ρ†Π΅ΠΏΠΈ, ΠΎΡΡ‚Π°Π»ΡŒΠ½Ρ‹Π΅ элСмСнты всС Π΅Ρ‰Π΅ ΠΌΠΎΠ³ΡƒΡ‚ Ρ€Π°Π±ΠΎΡ‚Π°Ρ‚ΡŒ.

6 ΠŸΠΎΡΠ»Π΅Π΄ΠΎΠ²Π°Ρ‚Π΅Π»ΡŒΠ½Ρ‹Π΅ ΠΈ ΠΏΠ°Ρ€Π°Π»Π»Π΅Π»ΡŒΠ½Ρ‹Π΅ схСмы ΠΈ ΡΠΎΠΎΡ‚Π²Π΅Ρ‚ΡΡ‚Π²ΡƒΡŽΡ‰ΠΈΠ΅ схСмы

7 НаправлСниС Ρ‚ΠΎΠΊΠ° ΠΏΠΎ ΡΡ€Π°Π²Π½Π΅Π½ΠΈΡŽ с Π½Π°ΠΏΡ€Π°Π²Π»Π΅Π½ΠΈΠ΅ΠΌ ΠΏΠΎΡ‚ΠΎΠΊΠ° элСктронов НаправлСниС Ρ‚ΠΎΠΊΠ° — это Π½Π°ΠΏΡ€Π°Π²Π»Π΅Π½ΠΈΠ΅, Π² ΠΊΠΎΡ‚ΠΎΡ€ΠΎΠΌ Ρ‚Π΅ΠΊΡƒΡ‚ ΠΏΠΎΠ»ΠΎΠΆΠΈΡ‚Π΅Π»ΡŒΠ½Ρ‹Π΅ заряды.НаправлСниС Ρ‚ΠΎΠΊΠ° — это Π½Π°ΠΏΡ€Π°Π²Π»Π΅Π½ΠΈΠ΅, Π² ΠΊΠΎΡ‚ΠΎΡ€ΠΎΠΌ Ρ‚Π΅ΠΊΡƒΡ‚ ΠΏΠΎΠ»ΠΎΠΆΠΈΡ‚Π΅Π»ΡŒΠ½Ρ‹Π΅ заряды. Когда источником энСргии являСтся аккумулятор, Ρ‚ΠΎΠΊ Ρ‚Π΅Ρ‡Π΅Ρ‚ ΠΎΡ‚ ΠΏΠΎΠ»ΠΎΠΆΠΈΡ‚Π΅Π»ΡŒΠ½ΠΎΠΉ ΠΊΠ»Π΅ΠΌΠΌΡ‹ ΠΊ ΠΎΡ‚Ρ€ΠΈΡ†Π°Ρ‚Π΅Π»ΡŒΠ½ΠΎΠΉ. Когда источником энСргии являСтся аккумулятор, Ρ‚ΠΎΠΊ Ρ‚Π΅Ρ‡Π΅Ρ‚ ΠΎΡ‚ ΠΏΠΎΠ»ΠΎΠΆΠΈΡ‚Π΅Π»ΡŒΠ½ΠΎΠΉ ΠΊΠ»Π΅ΠΌΠΌΡ‹ ΠΊ ΠΎΡ‚Ρ€ΠΈΡ†Π°Ρ‚Π΅Π»ΡŒΠ½ΠΎΠΉ. Π­Π»Π΅ΠΊΡ‚Ρ€ΠΎΠ½Ρ‹ двиТутся Π² ΠΎΠ±Ρ€Π°Ρ‚Π½ΠΎΠΌ Π½Π°ΠΏΡ€Π°Π²Π»Π΅Π½ΠΈΠΈ. Π­Π»Π΅ΠΊΡ‚Ρ€ΠΎΠ½Ρ‹ двиТутся Π² ΠΎΠ±Ρ€Π°Ρ‚Π½ΠΎΠΌ Π½Π°ΠΏΡ€Π°Π²Π»Π΅Π½ΠΈΠΈ.

8 Π˜ΡΡ‚ΠΎΡ‡Π½ΠΈΠΊΠΈ сопротивлСния Π‘ΠΎΠΏΡ€ΠΎΡ‚ΠΈΠ²Π»Π΅Π½ΠΈΠ΅ — это противодСйствиС ΠΏΠΎΡ‚ΠΎΠΊΡƒ зарядов Π² ΠΌΠ°Ρ‚Π΅Ρ€ΠΈΠ°Π»Π΅.Π‘ΠΎΠΏΡ€ΠΎΡ‚ΠΈΠ²Π»Π΅Π½ΠΈΠ΅ — это противостояниС ΠΏΠΎΡ‚ΠΎΠΊΡƒ зарядов Π² ΠΌΠ°Ρ‚Π΅Ρ€ΠΈΠ°Π»Π΅. Π›Π°ΠΌΠΏΠΎΡ‡ΠΊΠΈ Π² Ρ†Π΅ΠΏΠΈ ΡΠ²Π»ΡΡŽΡ‚ΡΡ источниками сопротивлСния. Π›Π°ΠΌΠΏΠΎΡ‡ΠΊΠΈ Π² Ρ†Π΅ΠΏΠΈ ΡΠ²Π»ΡΡŽΡ‚ΡΡ источниками сопротивлСния. ΠŸΠΎΡΠ»Π΅Π΄ΠΎΠ²Π°Ρ‚Π΅Π»ΡŒΠ½Ρ‹ΠΉ Ρ‚ΠΎΠΊ ΡƒΠΌΠ΅Π½ΡŒΡˆΠ°Π΅Ρ‚ΡΡ Π² Ρ€Π΅Π·ΡƒΠ»ΡŒΡ‚Π°Ρ‚Π΅ этого сопротивлСния, ΠΈ Π»Π°ΠΌΠΏΠΎΡ‡ΠΊΠΈ свСтятся ΠΌΠ΅Π½Π΅Π΅ ярко. ΠŸΠΎΡΠ»Π΅Π΄ΠΎΠ²Π°Ρ‚Π΅Π»ΡŒΠ½Ρ‹ΠΉ Ρ‚ΠΎΠΊ ΡƒΠΌΠ΅Π½ΡŒΡˆΠ°Π΅Ρ‚ΡΡ Π² Ρ€Π΅Π·ΡƒΠ»ΡŒΡ‚Π°Ρ‚Π΅ этого сопротивлСния, ΠΈ Π»Π°ΠΌΠΏΠΎΡ‡ΠΊΠΈ свСтятся ΠΌΠ΅Π½Π΅Π΅ ярко.

9 ΠœΠΎΡ‰Π½ΠΎΡΡ‚ΡŒ ΠΈ энСргия ΠŸΠΎΡΠΊΠΎΠ»ΡŒΠΊΡƒ ΠΌΠΎΡ‰Π½ΠΎΡΡ‚ΡŒ — это ΡΠΊΠΎΡ€ΠΎΡΡ‚ΡŒ выполнСния Ρ€Π°Π±ΠΎΡ‚Ρ‹, элСктроэнСргия — это ΡΠΊΠΎΡ€ΠΎΡΡ‚ΡŒ, с ΠΊΠΎΡ‚ΠΎΡ€ΠΎΠΉ элСктричСская энСргия прСобразуСтся Π² Π΄Ρ€ΡƒΠ³ΡƒΡŽ Ρ„ΠΎΡ€ΠΌΡƒ энСргии.ΠŸΠΎΡΠΊΠΎΠ»ΡŒΠΊΡƒ ΠΌΠΎΡ‰Π½ΠΎΡΡ‚ΡŒ — это ΡΠΊΠΎΡ€ΠΎΡΡ‚ΡŒ выполнСния Ρ€Π°Π±ΠΎΡ‚Ρ‹, элСктроэнСргия — это ΡΠΊΠΎΡ€ΠΎΡΡ‚ΡŒ, с ΠΊΠΎΡ‚ΠΎΡ€ΠΎΠΉ элСктричСская энСргия прСобразуСтся Π² Π΄Ρ€ΡƒΠ³ΡƒΡŽ Ρ„ΠΎΡ€ΠΌΡƒ энСргии. Π•Π΄ΠΈΠ½ΠΈΡ†Π° измСрСния элСктричСской мощности — Π΄ΠΆΠΎΡƒΠ»ΡŒ Π² сСкунду ΠΈΠ»ΠΈ Π²Π°Ρ‚Ρ‚ (Π’Ρ‚). Π•Π΄ΠΈΠ½ΠΈΡ†Π° измСрСния элСктричСской мощности — Π΄ΠΆΠΎΡƒΠ»ΡŒ Π² сСкунду ΠΈΠ»ΠΈ Π²Π°Ρ‚Ρ‚ (Π’Ρ‚). ЭлСктричСская ΠΌΠΎΡ‰Π½ΠΎΡΡ‚ΡŒ ΠΎΠ±Ρ‹Ρ‡Π½ΠΎ измСряСтся Π² тысячах Π²Π°Ρ‚Ρ‚, Π° Π΅Π΄ΠΈΠ½ΠΈΡ†Π° измСрСния — ΠΊΠΈΠ»ΠΎΠ²Π°Ρ‚Ρ‚ (ΠΊΠ’Ρ‚). ЭлСктричСская ΠΌΠΎΡ‰Π½ΠΎΡΡ‚ΡŒ ΠΎΠ±Ρ‹Ρ‡Π½ΠΎ измСряСтся Π² тысячах Π²Π°Ρ‚Ρ‚, Π° Π΅Π΄ΠΈΠ½ΠΈΡ†Π° измСрСния — ΠΊΠΈΠ»ΠΎΠ²Π°Ρ‚Ρ‚ (ΠΊΠ’Ρ‚). Π­Π»Π΅ΠΊΡ‚Ρ€ΠΈΡ‡Π΅ΡΠΊΡƒΡŽ ΠΌΠΎΡ‰Π½ΠΎΡΡ‚ΡŒ ΠΌΠΎΠΆΠ½ΠΎ Ρ€Π°ΡΡΡ‡ΠΈΡ‚Π°Ρ‚ΡŒ, ΡƒΠΌΠ½ΠΎΠΆΠΈΠ² напряТСниС Π½Π° Ρ‚ΠΎΠΊ.Π­Π»Π΅ΠΊΡ‚Ρ€ΠΈΡ‡Π΅ΡΠΊΡƒΡŽ ΠΌΠΎΡ‰Π½ΠΎΡΡ‚ΡŒ ΠΌΠΎΠΆΠ½ΠΎ Ρ€Π°ΡΡΡ‡ΠΈΡ‚Π°Ρ‚ΡŒ, ΡƒΠΌΠ½ΠΎΠΆΠΈΠ² напряТСниС Π½Π° Ρ‚ΠΎΠΊ. P (Π²Π°Ρ‚Ρ‚Ρ‹) = I (Π°ΠΌΠΏΠ΅Ρ€Ρ‹) x V (Π²ΠΎΠ»ΡŒΡ‚Ρ‹) https://youtu.be/5QfxTSGh9Yc

10 ЭлСктроэнСргия Номинальная ΠΌΠΎΡ‰Π½ΠΎΡΡ‚ΡŒ ΠΏΡ€ΠΈΠ±ΠΎΡ€Π° ΠΈΠ½Ρ„ΠΎΡ€ΠΌΠΈΡ€ΡƒΠ΅Ρ‚ потрСбитСля ΠΎ Ρ‚ΠΎΠΌ, сколько энСргии ΠΎΠ½ΠΎ потрСбляСт Π² Π½ΠΎΡ€ΠΌΠ°Π»ΡŒΠ½Ρ‹Ρ… условиях. Π§Ρ‚ΠΎΠ±Ρ‹ Π½Π°ΠΉΡ‚ΠΈ количСство элСктроэнСргии, потрСбляСмой ΠΏΡ€ΠΈΠ±ΠΎΡ€Π°ΠΌΠΈ, ΡƒΠΌΠ½ΠΎΠΆΡŒΡ‚Π΅ ΠΌΠΎΡ‰Π½ΠΎΡΡ‚ΡŒ Π½Π° врСмя. Если ΡΡƒΡˆΠΈΠ»ΠΊΠ° для бСлья потрСбляСт 5400 Π’Ρ‚, сколько это Π±ΡƒΠ΄Π΅Ρ‚ Π² ΠΊΠΈΠ»ΠΎΠ²Π°Ρ‚Ρ‚Π°Ρ…? Если ΡΡƒΡˆΠΈΠ»ΠΊΠ° ΠΈΡΠΏΠΎΠ»ΡŒΠ·ΡƒΠ΅Ρ‚ΡΡ Π² Ρ‚Π΅Ρ‡Π΅Π½ΠΈΠ΅ 2 часов, сколько энСргии ΠΎΠ½Π° потрСбляСт Π² ΠΊΠΈΠ»ΠΎΠ²Π°Ρ‚Ρ‚-часах? ΠšΠΈΠ»ΠΎΠ²Π°Ρ‚Ρ‚-час Ρ€Π°Π²Π΅Π½ 3 600 000 Π΄ΠΆΠΎΡƒΠ»Π΅ΠΉ.Π˜Ρ‚Π°ΠΊ, сколько Π΄ΠΆΠΎΡƒΠ»Π΅ΠΉ Π±ΡƒΠ΄Π΅Ρ‚ израсходовано этой ΡΡƒΡˆΠΈΠ»ΠΊΠΎΠΉ Π·Π° 2 часа?

11 ΠΠ΅ΠΈΡΠΏΡ€Π°Π²Π½ΠΎΡΡ‚ΡŒ Ρ†Π΅ΠΏΠΈ ΠŸΡ€Π°Π²ΠΈΠ»ΡŒΠ½Π°Ρ ΠΏΡ€ΠΎΠ²ΠΎΠ΄ΠΊΠ°, ΠΏΡ€Π΅Π΄ΠΎΡ…Ρ€Π°Π½ΠΈΡ‚Π΅Π»ΠΈ, автоматичСскиС Π²Ρ‹ΠΊΠ»ΡŽΡ‡Π°Ρ‚Π΅Π»ΠΈ, изоляция ΠΈ Π·Π°Π·Π΅ΠΌΠ»Π΅Π½Π½Ρ‹Π΅ Π²ΠΈΠ»ΠΊΠΈ ΠΏΠΎΠΌΠΎΠ³Π°ΡŽΡ‚ ΡΠ΄Π΅Π»Π°Ρ‚ΡŒ использованиС элСктроэнСргии бСзопасным. Π‘Ρ€Π΅Π΄Π½Π΅Π΅ напряТСниС Π² Ρ†Π΅ΠΏΠΈ срСднСго Π΄ΠΎΠΌΠΎΡ‡Π°Π΄Ρ†Π° — 120 Π²ΠΎΠ»ΡŒΡ‚. Π‘ΠΈΠ»Π° Ρ‚ΠΎΠΊΠ° Π² Ρ†Π΅ΠΏΠΈ ΠΌΠΎΠΆΠ΅Ρ‚ Π±Ρ‹Ρ‚ΡŒ Ρ€Π°Π·Π½ΠΎΠΉ. Когда Π΄Π²Π° устройства Π²ΠΊΠ»ΡŽΡ‡Π΅Π½Ρ‹ Π² ΠΏΠ°Ρ€Π°Π»Π»Π΅Π»ΡŒΠ½ΡƒΡŽ Ρ†Π΅ΠΏΡŒ, ΡƒΠ²Π΅Π»ΠΈΡ‡Π΅Π½ΠΈΠ΅ Ρ‚ΠΎΠΊΠ° ΠΌΠΎΠΆΠ΅Ρ‚ ΠΏΡ€Π΅Π²Ρ‹ΡΠΈΡ‚ΡŒ бСзопасный ΠΏΡ€Π΅Π΄Π΅Π» Ρ†Π΅ΠΏΠΈ.Π’ этом случаС ΠΏΡ€ΠΎΠ²ΠΎΠ΄ ΠΌΠΎΠΆΠ΅Ρ‚ ΠΏΠ΅Ρ€Π΅Π³Ρ€Π΅Ρ‚ΡŒΡΡ ΠΈ Π²Ρ‹Π·Π²Π°Ρ‚ΡŒ Π²ΠΎΠ·Π³ΠΎΡ€Π°Π½ΠΈΠ΅.

12 Π Π°Π·ΠΌΡ‹ΠΊΠ°Π½ΠΈΠ΅ Ρ†Π΅ΠΏΠΈ ΠŸΡ€Π΅Π΄ΠΎΡ…Ρ€Π°Π½ΠΈΡ‚Π΅Π»ΠΈ ΠΈ автоматичСскиС Π²Ρ‹ΠΊΠ»ΡŽΡ‡Π°Ρ‚Π΅Π»ΠΈ ΠΎΠ±Π΅ΡΠΏΠ΅Ρ‡ΠΈΠ²Π°ΡŽΡ‚ Π±Π΅Π·ΠΎΠΏΠ°ΡΠ½ΠΎΡΡ‚ΡŒ Π΄ΠΎΠΌΠ°, обСспСчивая Ρ€Π°Π·Ρ€Ρ‹Π² Ρ†Π΅ΠΏΠΈ, ΠΊΠΎΠ³Π΄Π° Ρ‚ΠΎΠΊ становится слишком высоким. Π­Ρ‚ΠΎ ΠΏΠΎΡ…ΠΎΠΆΠ΅ Π½Π° ΠΎΡ‚ΠΊΡ€Ρ‹Ρ‚Ρ‹ΠΉ ΠΏΠ΅Ρ€Π΅ΠΊΠ»ΡŽΡ‡Π°Ρ‚Π΅Π»ΡŒ, ΠΎ ΠΊΠΎΡ‚ΠΎΡ€ΠΎΠΌ ΠΌΡ‹ Π³ΠΎΠ²ΠΎΡ€ΠΈΠ»ΠΈ Ρ€Π°Π½Π΅Π΅. Разомкнутая Ρ†Π΅ΠΏΡŒ, такая ΠΊΠ°ΠΊ Ρ€Π°Π·ΠΎΠΌΠΊΠ½ΡƒΡ‚Ρ‹ΠΉ ΠΏΠ΅Ρ€Π΅ΠΊΠ»ΡŽΡ‡Π°Ρ‚Π΅Π»ΡŒ, ΠΏΡ€Π΅Ρ€Ρ‹Π²Π°Π΅Ρ‚ ΠΏΡƒΡ‚ΡŒ Ρ‚ΠΎΠΊΠ°, Ρ‡Ρ‚ΠΎ ΠΎΠ·Π½Π°Ρ‡Π°Π΅Ρ‚, Ρ‡Ρ‚ΠΎ Ρ‚ΠΎΠΊ большС Π½Π΅ ΠΌΠΎΠΆΠ΅Ρ‚ Ρ‚Π΅Ρ‡ΡŒ Π΄ΠΎ Ρ‚Π΅Ρ… ΠΏΠΎΡ€, ΠΏΠΎΠΊΠ° Π½Π΅ Π±ΡƒΠ΄Π΅Ρ‚ Π·Π°ΠΌΠ΅Π½Π΅Π½ ΠΏΡ€Π΅Π΄ΠΎΡ…Ρ€Π°Π½ΠΈΡ‚Π΅Π»ΡŒ ΠΈΠ»ΠΈ Π½Π΅ Π±ΡƒΠ΄Π΅Ρ‚ ΡΠ±Ρ€ΠΎΡˆΠ΅Π½ автоматичСский Π²Ρ‹ΠΊΠ»ΡŽΡ‡Π°Ρ‚Π΅Π»ΡŒ.

13 ΠŸΡ€Π΅Π΄ΠΎΡ…Ρ€Π°Π½ΠΈΡ‚Π΅Π»ΠΈ, автоматичСскиС Π²Ρ‹ΠΊΠ»ΡŽΡ‡Π°Ρ‚Π΅Π»ΠΈ ΠΈ изолированная ΠΏΡ€ΠΎΠ²ΠΎΠ΄ΠΊΠ° ΠŸΡ€Π΅Π΄ΠΎΡ…Ρ€Π°Π½ΠΈΡ‚Π΅Π»ΡŒ ΠΏΡ€Π΅Π΄ΠΎΡ‚Π²Ρ€Π°Ρ‰Π°Π΅Ρ‚ ΠΏΠ΅Ρ€Π΅Π³Ρ€ΡƒΠ·ΠΊΡƒ, ΠΊΠΎΠ³Π΄Π° ΠΏΡ€ΠΎΠ²ΠΎΠ΄ Π² Ρ†Π΅Π½Ρ‚Ρ€Π΅ прСдохранитСля плавится, Ссли Ρ‡Π΅Ρ€Π΅Π· Π½Π΅Π³ΠΎ ΠΏΡ€ΠΎΡ…ΠΎΠ΄ΠΈΡ‚ слишком большой Ρ‚ΠΎΠΊ. ΠŸΡ€Π΅Π΄ΠΎΡ…Ρ€Π°Π½ΠΈΡ‚Π΅Π»ΡŒ Π½Π΅ΠΎΠ±Ρ…ΠΎΠ΄ΠΈΠΌΠΎ Π·Π°ΠΌΠ΅Π½ΠΈΡ‚ΡŒ Π½ΠΎΠ²Ρ‹ΠΌ, ΠΏΡ€Π΅ΠΆΠ΄Π΅ Ρ‡Π΅ΠΌ Ρ†Π΅ΠΏΡŒ снова смоТСт ΠΏΡ€ΠΎΠΏΡƒΡΠΊΠ°Ρ‚ΡŒ Ρ‚ΠΎΠΊ. Π’ Π±ΠΎΠ»ΡŒΡˆΠΈΠ½ΡΡ‚Π²Π΅ соврСмСнных Π΄ΠΎΠΌΠΎΠ² Π΅ΡΡ‚ΡŒ автоматичСскиС Π²Ρ‹ΠΊΠ»ΡŽΡ‡Π°Ρ‚Π΅Π»ΠΈ, Π° Π½Π΅ ΠΏΡ€Π΅Π΄ΠΎΡ…Ρ€Π°Π½ΠΈΡ‚Π΅Π»ΠΈ. АвтоматичСский Π²Ρ‹ΠΊΠ»ΡŽΡ‡Π°Ρ‚Π΅Π»ΡŒ — это Π²Ρ‹ΠΊΠ»ΡŽΡ‡Π°Ρ‚Π΅Π»ΡŒ, ΠΊΠΎΡ‚ΠΎΡ€Ρ‹ΠΉ размыкаСтся ΠΏΡ€ΠΈ слишком высоком Ρ‚ΠΎΠΊΠ΅ Π² Ρ†Π΅ΠΏΠΈ. ЭлСктричСство Π½Π΅ Π±ΡƒΠ΄Π΅Ρ‚ доступно для Ρ†Π΅ΠΏΠΈ, ΠΏΠΎΠΊΠ° Π²Ρ‹ΠΊΠ»ΡŽΡ‡Π°Ρ‚Π΅Π»ΡŒ Π½Π΅ Π±ΡƒΠ΄Π΅Ρ‚ ΠΏΠ΅Ρ€Π΅Π·Π°Π³Ρ€ΡƒΠΆΠ΅Π½.Π­Π»Π΅ΠΊΡ‚Ρ€ΠΎΠΏΡ€ΠΎΠ²ΠΎΠ΄ΠΊΠ° Π² Π΄ΠΎΠΌΠ΅ ΠΈΠ·ΠΎΠ»ΠΈΡ€ΠΎΠ²Π°Π½Π° для Π·Π°Ρ‰ΠΈΡ‚Ρ‹ людСй, Π½ΠΎ Ссли изоляция ΠΏΠΎΠ²Ρ€Π΅ΠΆΠ΄Π΅Π½Π°, ΠΌΠΎΠΆΠ΅Ρ‚ ΠΏΡ€ΠΎΠΈΠ·ΠΎΠΉΡ‚ΠΈ ΠΏΠΎΡ€Π°ΠΆΠ΅Π½ΠΈΠ΅ элСктричСским Ρ‚ΠΎΠΊΠΎΠΌ Ρ‡Π΅Ρ€Π΅Π· ΠΎΠ³ΠΎΠ»Π΅Π½Π½Ρ‹Π΅ участки.

14 Личная Π±Π΅Π·ΠΎΠΏΠ°ΡΠ½ΠΎΡΡ‚ΡŒ Π’ΠΎ ΠΈΠ·Π±Π΅ΠΆΠ°Π½ΠΈΠ΅ пораТСния элСктричСским Ρ‚ΠΎΠΊΠΎΠΌ Π½ΠΈΠΊΠΎΠ³Π΄Π° Π½Π΅ ΠΏΡ€ΠΈΠΊΠ°ΡΠ°ΠΉΡ‚Π΅ΡΡŒ ΠΊ элСктричСским устройствам ΠΌΠΎΠΊΡ€Ρ‹ΠΌΠΈ Ρ€ΡƒΠΊΠ°ΠΌΠΈ. ΠŸΠΎΠ²Ρ€Π΅ΠΆΠ΄Π΅Π½Π½Π°Ρ изоляция ΠΌΠΎΠΆΠ΅Ρ‚ Π²Ρ‹Π·Π²Π°Ρ‚ΡŒ ΠΊΠΎΡ€ΠΎΡ‚ΠΊΠΎΠ΅ Π·Π°ΠΌΡ‹ΠΊΠ°Π½ΠΈΠ΅. ΠšΠΎΡ€ΠΎΡ‚ΠΊΠΎΠ΅ Π·Π°ΠΌΡ‹ΠΊΠ°Π½ΠΈΠ΅ происходит, ΠΊΠΎΠ³Π΄Π° Ρ‚ΠΎΠΊ Π½Π°Ρ…ΠΎΠ΄ΠΈΡ‚ ΠΊΠΎΡ€ΠΎΡ‚ΠΊΠΈΠΉ ΠΏΡƒΡ‚ΡŒ Ρ‡Π΅Ρ€Π΅Π· Ρ†Π΅ΠΏΡŒ с мСньшим сопротивлСниСм, Ρ‡Π΅ΠΌ ΠΏΠΎΠ»Π½Ρ‹ΠΉ ΠΏΡƒΡ‚ΡŒ Ρ‡Π΅Ρ€Π΅Π· Ρ†Π΅ΠΏΡŒ.Π’ΠΈΠ»ΠΊΠ° с трСмя ΠΊΠΎΠ½Ρ‚Π°ΠΊΡ‚Π°ΠΌΠΈ ΠΌΠΎΠΆΠ΅Ρ‚ ΠΏΡ€Π΅Π΄ΠΎΡ‚Π²Ρ€Π°Ρ‚ΠΈΡ‚ΡŒ ΡƒΠ΄Π°Ρ€Ρ‹, Π²Ρ‹Π·Π²Π°Π½Π½Ρ‹Π΅ ΠΊΠΎΡ€ΠΎΡ‚ΠΊΠΈΠΌ Π·Π°ΠΌΡ‹ΠΊΠ°Π½ΠΈΠ΅ΠΌ, ΠΏΠΎΡ‚ΠΎΠΌΡƒ Ρ‡Ρ‚ΠΎ Ρ‚Ρ€Π΅Ρ‚ΠΈΠΉ ΠΊΠΎΠ½Ρ‚Π°ΠΊΡ‚ ΠΏΠΎΠ΄ΠΊΠ»ΡŽΡ‡Π°Π΅Ρ‚ΡΡ ΠΊ Π·Π΅ΠΌΠ»Π΅, обСспСчивая Π±ΠΎΠ»Π΅Π΅ Π»Π΅Π³ΠΊΠΈΠΉ ΠΏΡƒΡ‚ΡŒ ΠΊ зазСмлСнию Ρ‡Π΅Ρ€Π΅Π· Π·Π°Π·Π΅ΠΌΠ»ΡΡŽΡ‰ΠΈΠΉ ΠΏΡ€ΠΎΠ²ΠΎΠ΄, Π° Π½Π΅ Ρ‡Π΅Ρ€Π΅Π· вашС Ρ‚Π΅Π»ΠΎ. ΠŸΠ΅Ρ€Π΅Π΄Π°Ρ‡Π° ΠΈΠ·Π±Ρ‹Ρ‚ΠΎΡ‡Π½ΠΎΠ³ΠΎ заряда Ρ‡Π΅Ρ€Π΅Π· ΠΏΡ€ΠΎΠ²ΠΎΠ΄Π½ΠΈΠΊ Π½Π° Π—Π΅ΠΌΠ»ΡŽ называСтся Π·Π°Π·Π΅ΠΌΠ»Π΅Π½ΠΈΠ΅ΠΌ.


ΠŸΠ°Ρ€Π°Π»Π»Π΅Π»ΡŒΠ½Π°Ρ схСма ΠΈ Π·Π°ΠΊΠΎΠ½ Ома: мноТСство ΠΏΡƒΡ‚Π΅ΠΉ для элСктричСства — Π£Ρ€ΠΎΠΊ

(0 Π Π΅ΠΉΡ‚ΠΈΠ½Π³ΠΈ)

Быстрый просмотр

Π£Ρ€ΠΎΠ²Π΅Π½ΡŒ ΠΎΡ†Π΅Π½ΠΊΠΈ: 4 (3-5)

Π’Ρ€Π΅Π±ΡƒΠ΅ΠΌΠΎΠ΅ врСмя: 45 ΠΌΠΈΠ½ΡƒΡ‚

Π—Π°Π²ΠΈΡΠΈΠΌΠΎΡΡ‚ΡŒ ΡƒΡ€ΠΎΠΊΠ°:

ВСматичСскиС области: АлгСбра, физичСскиС Π½Π°ΡƒΠΊΠΈ

ΠžΠΆΠΈΠ΄Π°Π΅ΠΌΡ‹Π΅ характСристики NGSS:


ΠŸΠΎΠ΄Π΅Π»ΠΈΡ‚ΡŒΡΡ:

РСзюмС

Π‘Ρ‚ΡƒΠ΄Π΅Π½Ρ‚Ρ‹ ΠΈΠ·ΡƒΡ‡Π°ΡŽΡ‚ состав ΠΈ практичСскоС ΠΏΡ€ΠΈΠΌΠ΅Π½Π΅Π½ΠΈΠ΅ ΠΏΠ°Ρ€Π°Π»Π»Π΅Π»ΡŒΠ½ΠΎΠΉ схСмы ΠΏΠΎ ΡΡ€Π°Π²Π½Π΅Π½ΠΈΡŽ с ΠΏΠΎΡΠ»Π΅Π΄ΠΎΠ²Π°Ρ‚Π΅Π»ΡŒΠ½ΠΎΠΉ схСмой.Π‘Ρ‚ΡƒΠ΄Π΅Π½Ρ‚Ρ‹ ΠΏΡ€ΠΎΠ΅ΠΊΡ‚ΠΈΡ€ΡƒΡŽΡ‚ ΠΈ строят ΠΏΠ°Ρ€Π°Π»Π»Π΅Π»ΡŒΠ½Ρ‹Π΅ схСмы, ΠΈΡΡΠ»Π΅Π΄ΡƒΡŽΡ‚ ΠΈΡ… характСристики ΠΈ ΠΏΡ€ΠΈΠΌΠ΅Π½ΡΡŽΡ‚ Π·Π°ΠΊΠΎΠ½ Ома. Π­Ρ‚Π° инТСнСрная ΠΏΡ€ΠΎΠ³Ρ€Π°ΠΌΠΌΠ° соотвСтствуСт Π½Π°ΡƒΡ‡Π½Ρ‹ΠΌ стандартам Π½ΠΎΠ²ΠΎΠ³ΠΎ поколСния (NGSS).

Π˜Π½ΠΆΠ΅Π½Π΅Ρ€Π½ΠΎΠ΅ соСдинСниС

Π˜Π½ΠΆΠ΅Π½Π΅Ρ€Ρ‹

Ρ€Π°Π·Ρ€Π°Π±ΠΎΡ‚Π°Π»ΠΈ ΠΎΡ‡Π΅Π½ΡŒ ΡΠ»ΠΎΠΆΠ½ΡƒΡŽ схСму, Π½Π°Π·Ρ‹Π²Π°Π΅ΠΌΡƒΡŽ ΠΈΠ½Ρ‚Π΅Π³Ρ€Π°Π»ΡŒΠ½ΠΎΠΉ схСмой, которая ΠΎΠ±ΡŠΠ΅Π΄ΠΈΠ½ΡΠ΅Ρ‚ тысячи ΠΈ ΠΌΠΈΠ»Π»ΠΈΠΎΠ½Ρ‹ ΠΏΠ°Ρ€Π°Π»Π»Π΅Π»ΡŒΠ½Ρ‹Ρ… ΠΈ ΠΏΠΎΡΠ»Π΅Π΄ΠΎΠ²Π°Ρ‚Π΅Π»ΡŒΠ½Ρ‹Ρ… схСм, Ρ€Π°Π±ΠΎΡ‚Π°ΡŽΡ‰ΠΈΡ… вмСстС. Одним ΠΈΠ· Ρ‚ΠΈΠΏΠΎΠ² ΠΈΠ½Ρ‚Π΅Π³Ρ€Π°Π»ΡŒΠ½Ρ‹Ρ… схСм, ΠΊΠΎΡ‚ΠΎΡ€Ρ‹Π΅ Ρ€Π°Π±ΠΎΡ‚Π°ΡŽΡ‚ ΠΊΠ°ΠΊ ΠΏΠΎΠ»Π½ΠΎΡ†Π΅Π½Π½Ρ‹ΠΉ Π²Ρ‹Ρ‡ΠΈΡΠ»ΠΈΡ‚Π΅Π»ΡŒΠ½Ρ‹ΠΉ ΠΌΠ΅Ρ…Π°Π½ΠΈΠ·ΠΌ, являСтся микропроцСссор, извСстный ΠΊΠ°ΠΊ Ρ†Π΅Π½Ρ‚Ρ€Π°Π»ΡŒΠ½Ρ‹ΠΉ процСссор ΠΈΠ»ΠΈ ЦП.ΠœΠΈΠΊΡ€ΠΎΠΏΡ€ΠΎΡ†Π΅ΡΡΠΎΡ€Ρ‹ Π½Π΅ΠΎΠ±Ρ…ΠΎΠ΄ΠΈΠΌΡ‹ Π² автомобилях, Π²ΠΈΠ΄Π΅ΠΎΠΈΠ³Ρ€Π°Ρ…, Π΄Π΅Ρ‚Π΅ΠΊΡ‚ΠΎΡ€Π°Ρ… Π΄Ρ‹ΠΌΠ°, DVD-ΠΏΠ»Π΅Π΅Ρ€Π°Ρ…, ΠΌΠ΅Ρ…Π°Π½ΠΈΠ·ΠΌΠ°Ρ… открывания Π³Π°Ρ€Π°ΠΆΠ½Ρ‹Ρ… Π²ΠΎΡ€ΠΎΡ‚, бСспроводных Ρ‚Π΅Π»Π΅Ρ„ΠΎΠ½Π°Ρ…, часах ΠΈ ΠΊΠ°Π»ΡŒΠΊΡƒΠ»ΡΡ‚ΠΎΡ€Π°Ρ…. Π˜Π½ΠΆΠ΅Π½Π΅Ρ€Ρ‹ постоянно Ρ€Π°Π·Ρ€Π°Π±Π°Ρ‚Ρ‹Π²Π°ΡŽΡ‚ Π½ΠΎΠ²Ρ‹Π΅ Ρ‚Π΅Ρ…Π½ΠΎΠ»ΠΎΠ³ΠΈΠΈ, Ρ‡Ρ‚ΠΎΠ±Ρ‹ ΠΎΠ½ΠΈ ΠΌΠΎΠ³Π»ΠΈ ΠΈΡΠΏΠΎΠ»ΡŒΠ·ΠΎΠ²Π°Ρ‚ΡŒ элСктричСство для Ρ€Π΅ΡˆΠ΅Π½ΠΈΡ повсСднСвных ΠΏΡ€ΠΎΠ±Π»Π΅ΠΌ — усилий, ΠΊΠΎΡ‚ΠΎΡ€Ρ‹Π΅ ΡΠΏΠΎΡΠΎΠ±ΡΡ‚Π²ΡƒΡŽΡ‚ созданию Π±ΠΎΠ»Π΅Π΅ Π·Π΄ΠΎΡ€ΠΎΠ²ΠΎΠΉ, счастливой ΠΈ бСзопасной ΠΎΠΊΡ€ΡƒΠΆΠ°ΡŽΡ‰Π΅ΠΉ срСды.

Π¦Π΅Π»ΠΈ обучСния

ПослС этого ΡƒΡ€ΠΎΠΊΠ° учащиСся Π΄ΠΎΠ»ΠΆΠ½Ρ‹ ΡƒΠΌΠ΅Ρ‚ΡŒ:

  • Π Π°Π·Π»ΠΈΡ‡Π°ΡŽΡ‚ ΠΏΠΎΡΠ»Π΅Π΄ΠΎΠ²Π°Ρ‚Π΅Π»ΡŒΠ½Ρ‹Π΅ ΠΈ ΠΏΠ°Ρ€Π°Π»Π»Π΅Π»ΡŒΠ½Ρ‹Π΅ части Ρ†Π΅ΠΏΠΈ.
  • ΠžΠΏΠΈΡˆΠΈΡ‚Π΅, ΠΊΠ°ΠΊ измСняСтся Ρ‚ΠΎΠΊ Π² ΠΏΠ°Ρ€Π°Π»Π»Π΅Π»ΡŒΠ½ΠΎΠΉ Ρ†Π΅ΠΏΠΈ, ΠΊΠΎΠ³Π΄Π° Π»Π°ΠΌΠΏΠΎΡ‡ΠΊΠ° удаляСтся ΠΈΠ· Ρ†Π΅ΠΏΠΈ ΠΈΠ»ΠΈ добавляСтся ΠΊ Π½Π΅ΠΉ.
  • ΠžΠΏΠΈΡˆΠΈΡ‚Π΅ связи ΠΌΠ΅ΠΆΠ΄Ρƒ изобраТСниями условных ΠΎΠ±ΠΎΠ·Π½Π°Ρ‡Π΅Π½ΠΈΠΉ Ρ†Π΅ΠΏΠ΅ΠΉ
  • ΠŸΡ€ΠΈΠ·Π½Π°Ρ‚ΡŒ, Ρ‡Ρ‚ΠΎ ΠΈΠ½ΠΆΠ΅Π½Π΅Ρ€Ρ‹-элСктрики, ΠΌΠ°Ρ‚Π΅Ρ€ΠΈΠ°Π»ΠΎΠ²Π΅Π΄Ρ‹ / ΠΈΠ½ΠΆΠ΅Π½Π΅Ρ€Ρ‹, ΠΈΠ½ΠΆΠ΅Π½Π΅Ρ€Ρ‹-ΠΌΠ΅Ρ…Π°Π½ΠΈΠΊΠΈ ΠΈ Ρ„ΠΈΠ·ΠΈΠΊΠΈ вносят свой Π²ΠΊΠ»Π°Π΄ Π² Ρ€Π°Π·Π²ΠΈΡ‚ΠΈΠ΅ элСктронных Ρ‚Π΅Ρ…Π½ΠΎΠ»ΠΎΠ³ΠΈΠΉ.

ΠžΠ±Ρ€Π°Π·ΠΎΠ²Π°Ρ‚Π΅Π»ΡŒΠ½Ρ‹Π΅ стандарты

ΠšΠ°ΠΆΠ΄Ρ‹ΠΉ ΡƒΡ€ΠΎΠΊ ΠΈΠ»ΠΈ Π·Π°Π΄Π°Π½ΠΈΠ΅ TeachEngineering соотносится с ΠΎΠ΄Π½ΠΈΠΌ ΠΈΠ»ΠΈ нСсколькими Π½Π°ΡƒΡ‡Π½Ρ‹ΠΌΠΈ ΠΏΡ€Π΅Π΄ΠΌΠ΅Ρ‚Π°ΠΌΠΈ K-12, ΠΎΠ±Ρ€Π°Π·ΠΎΠ²Π°Ρ‚Π΅Π»ΡŒΠ½Ρ‹Π΅ стандарты Π² области Ρ‚Π΅Ρ…Π½ΠΎΠ»ΠΎΠ³ΠΈΠΉ, ΠΈΠ½ΠΆΠ΅Π½Π΅Ρ€ΠΈΠΈ ΠΈΠ»ΠΈ ΠΌΠ°Ρ‚Π΅ΠΌΠ°Ρ‚ΠΈΠΊΠΈ (STEM).

ВсС 100000+ стандартов K-12 STEM, ΠΎΡ…Π²Π°Ρ‚Ρ‹Π²Π°Π΅ΠΌΡ‹Ρ… TeachEngineering , ΡΠΎΠ±ΠΈΡ€Π°ΡŽΡ‚ΡΡ, ΠΎΠ±ΡΠ»ΡƒΠΆΠΈΠ²Π°ΡŽΡ‚ΡΡ ΠΈ ΡƒΠΏΠ°ΠΊΠΎΠ²Ρ‹Π²Π°ΡŽΡ‚ΡΡ ΡΠ΅Ρ‚ΡŒΡŽ стандартов достиТСний (ASN) , ΠΏΡ€ΠΎΠ΅ΠΊΡ‚ D2L (www.achievementstandards.org).

Π’ ASN стандарты иСрархичСски структурированы: сначала ΠΏΠΎ источникам; Π½Π°ΠΏΡ€ΠΈΠΌΠ΅Ρ€ , ΠΏΠΎ ΡˆΡ‚Π°Ρ‚Π°ΠΌ; Π²Π½ΡƒΡ‚Ρ€ΠΈ источника ΠΏΠΎ Ρ‚ΠΈΠΏΡƒ; Π½Π°ΠΏΡ€ΠΈΠΌΠ΅Ρ€ , СстСствСнныС Π½Π°ΡƒΠΊΠΈ ΠΈΠ»ΠΈ ΠΌΠ°Ρ‚Π΅ΠΌΠ°Ρ‚ΠΈΠΊΠ°; Π²Π½ΡƒΡ‚Ρ€ΠΈ Ρ‚ΠΈΠΏΠ° ΠΏΠΎ ΠΏΠΎΠ΄Ρ‚ΠΈΠΏΡƒ, Π·Π°Ρ‚Π΅ΠΌ ΠΏΠΎ классу, ΠΈ Ρ‚. Π΄. .

NGSS: Π½Π°ΡƒΡ‡Π½Ρ‹Π΅ стандарты Π½ΠΎΠ²ΠΎΠ³ΠΎ поколСния — Π½Π°ΡƒΠΊΠ°
ΠžΠΆΠΈΠ΄Π°Π΅ΠΌΡ‹Π΅ характСристики NGSS

4-ПБ3-2. ΠŸΡ€ΠΎΠ²Π΅Π΄ΠΈΡ‚Π΅ наблюдСния, Ρ‡Ρ‚ΠΎΠ±Ρ‹ Π΄ΠΎΠΊΠ°Π·Π°Ρ‚ΡŒ, Ρ‡Ρ‚ΠΎ энСргия ΠΌΠΎΠΆΠ΅Ρ‚ ΠΏΠ΅Ρ€Π΅Π΄Π°Π²Π°Ρ‚ΡŒΡΡ с мСста Π½Π° мСсто с ΠΏΠΎΠΌΠΎΡ‰ΡŒΡŽ Π·Π²ΡƒΠΊΠ°, свСта, Ρ‚Π΅ΠΏΠ»Π° ΠΈ элСктричСского Ρ‚ΠΎΠΊΠ°.(4 класс)

Π’Ρ‹ согласны с Ρ‚Π°ΠΊΠΈΠΌ раскладом? Бпасибо Π·Π° ваш ΠΎΡ‚Π·Ρ‹Π²!

НаТмитС, Ρ‡Ρ‚ΠΎΠ±Ρ‹ ΠΏΡ€ΠΎΡΠΌΠΎΡ‚Ρ€Π΅Ρ‚ΡŒ Π΄Ρ€ΡƒΠ³ΠΈΠ΅ ΡƒΡ‡Π΅Π±Π½Ρ‹Π΅ ΠΏΡ€ΠΎΠ³Ρ€Π°ΠΌΠΌΡ‹, ΡΠΎΠΎΡ‚Π²Π΅Ρ‚ΡΡ‚Π²ΡƒΡŽΡ‰ΠΈΠ΅ этим оТиданиям ΠΎΡ‚ Ρ€Π΅Π·ΡƒΠ»ΡŒΡ‚Π°Ρ‚ΠΎΠ².
Π­Ρ‚ΠΎΡ‚ ΡƒΡ€ΠΎΠΊ посвящСн ΡΠ»Π΅Π΄ΡƒΡŽΡ‰ΠΈΠΌ аспСктам Ρ‚Ρ€Π΅Ρ…ΠΌΠ΅Ρ€Π½ΠΎΠ³ΠΎ обучСния NGSS:
Наука ΠΈ инТСнСрная ΠΏΡ€Π°ΠΊΡ‚ΠΈΠΊΠ° ΠžΡΠ½ΠΎΠ²Π½Ρ‹Π΅ дисциплинарныС ΠΈΠ΄Π΅ΠΈ Π‘ΠΊΠ²ΠΎΠ·Π½Ρ‹Π΅ ΠΊΠΎΠ½Ρ†Π΅ΠΏΡ†ΠΈΠΈ
ΠŸΡ€ΠΎΠ²Π΅Π΄ΠΈΡ‚Π΅ наблюдСния, Ρ‡Ρ‚ΠΎΠ±Ρ‹ ΠΏΠΎΠ»ΡƒΡ‡ΠΈΡ‚ΡŒ Π΄Π°Π½Π½Ρ‹Π΅, ΠΊΠΎΡ‚ΠΎΡ€Ρ‹Π΅ послуТат основой для ΡΠ²ΠΈΠ΄Π΅Ρ‚Π΅Π»ΡŒΡΡ‚Π² для объяснСния явлСния ΠΈΠ»ΠΈ ΠΏΡ€ΠΎΠ²Π΅Ρ€ΠΊΠΈ ΠΏΡ€ΠΎΠ΅ΠΊΡ‚Π½ΠΎΠ³ΠΎ Ρ€Π΅ΡˆΠ΅Π½ΠΈΡ.

БоглашСниС ΠΎ Π²Ρ‹Ρ€Π°Π²Π½ΠΈΠ²Π°Π½ΠΈΠΈ: Бпасибо Π·Π° ваш ΠΎΡ‚Π·Ρ‹Π²!

ЭнСргия ΠΌΠΎΠΆΠ΅Ρ‚ ΠΏΠ΅Ρ€Π΅Π΄Π°Π²Π°Ρ‚ΡŒΡΡ с мСста Π½Π° мСсто с ΠΏΠΎΠΌΠΎΡ‰ΡŒΡŽ двиТущихся ΠΎΠ±ΡŠΠ΅ΠΊΡ‚ΠΎΠ², Π·Π²ΡƒΠΊΠ°, свСта ΠΈΠ»ΠΈ элСктричСского Ρ‚ΠΎΠΊΠ°.

БоглашСниС ΠΎ Π²Ρ‹Ρ€Π°Π²Π½ΠΈΠ²Π°Π½ΠΈΠΈ: Бпасибо Π·Π° ваш ΠΎΡ‚Π·Ρ‹Π²!

ЭнСргия присутствуСт всякий Ρ€Π°Π·, ΠΊΠΎΠ³Π΄Π° Π΅ΡΡ‚ΡŒ двиТущиСся ΠΎΠ±ΡŠΠ΅ΠΊΡ‚Ρ‹, Π·Π²ΡƒΠΊ, свСт ΠΈΠ»ΠΈ Ρ‚Π΅ΠΏΠ»ΠΎ. Когда ΠΎΠ±ΡŠΠ΅ΠΊΡ‚Ρ‹ ΡΡ‚Π°Π»ΠΊΠΈΠ²Π°ΡŽΡ‚ΡΡ, энСргия ΠΌΠΎΠΆΠ΅Ρ‚ ΠΏΠ΅Ρ€Π΅Π΄Π°Π²Π°Ρ‚ΡŒΡΡ ΠΎΡ‚ ΠΎΠ΄Π½ΠΎΠ³ΠΎ ΠΎΠ±ΡŠΠ΅ΠΊΡ‚Π° ΠΊ Π΄Ρ€ΡƒΠ³ΠΎΠΌΡƒ, Ρ‚Π΅ΠΌ самым измСняя ΠΈΡ… Π΄Π²ΠΈΠΆΠ΅Π½ΠΈΠ΅.ΠŸΡ€ΠΈ Ρ‚Π°ΠΊΠΈΡ… столкновСниях нСкоторая энСргия ΠΎΠ±Ρ‹Ρ‡Π½ΠΎ Ρ‚Π°ΠΊΠΆΠ΅ пСрСдаСтся ΠΎΠΊΡ€ΡƒΠΆΠ°ΡŽΡ‰Π΅ΠΌΡƒ Π²ΠΎΠ·Π΄ΡƒΡ…Ρƒ; Π² Ρ€Π΅Π·ΡƒΠ»ΡŒΡ‚Π°Ρ‚Π΅ Π²ΠΎΠ·Π΄ΡƒΡ… нагрСваСтся ΠΈ раздаСтся Π·Π²ΡƒΠΊ.

БоглашСниС ΠΎ Π²Ρ‹Ρ€Π°Π²Π½ΠΈΠ²Π°Π½ΠΈΠΈ: Бпасибо Π·Π° ваш ΠΎΡ‚Π·Ρ‹Π²!

Π‘Π²Π΅Ρ‚ Ρ‚Π°ΠΊΠΆΠ΅ ΠΏΠ΅Ρ€Π΅Π΄Π°Π΅Ρ‚ ΡΠ½Π΅Ρ€Π³ΠΈΡŽ с мСста Π½Π° мСсто.

БоглашСниС ΠΎ Π²Ρ‹Ρ€Π°Π²Π½ΠΈΠ²Π°Π½ΠΈΠΈ: Бпасибо Π·Π° ваш ΠΎΡ‚Π·Ρ‹Π²!

ЭнСргия Ρ‚Π°ΠΊΠΆΠ΅ ΠΌΠΎΠΆΠ΅Ρ‚ ΠΏΠ΅Ρ€Π΅Π΄Π°Π²Π°Ρ‚ΡŒΡΡ с мСста Π½Π° мСсто с ΠΏΠΎΠΌΠΎΡ‰ΡŒΡŽ элСктричСского Ρ‚ΠΎΠΊΠ°, ΠΊΠΎΡ‚ΠΎΡ€Ρ‹ΠΉ Π·Π°Ρ‚Π΅ΠΌ ΠΌΠΎΠΆΠ΅Ρ‚ ΠΈΡΠΏΠΎΠ»ΡŒΠ·ΠΎΠ²Π°Ρ‚ΡŒΡΡ локально для создания двиТСния, Π·Π²ΡƒΠΊΠ°, Ρ‚Π΅ΠΏΠ»Π° ΠΈΠ»ΠΈ свСта.Π‘ самого Π½Π°Ρ‡Π°Π»Π° Ρ‚ΠΎΠΊΠΈ ΠΌΠΎΠ³Π»ΠΈ Π±Ρ‹Ρ‚ΡŒ созданы ΠΏΡƒΡ‚Π΅ΠΌ прСобразования энСргии двиТСния Π² ΡΠ»Π΅ΠΊΡ‚Ρ€ΠΈΡ‡Π΅ΡΠΊΡƒΡŽ.

БоглашСниС ΠΎ Π²Ρ‹Ρ€Π°Π²Π½ΠΈΠ²Π°Π½ΠΈΠΈ: Бпасибо Π·Π° ваш ΠΎΡ‚Π·Ρ‹Π²!

ЭнСргия ΠΌΠΎΠΆΠ΅Ρ‚ ΠΏΠ΅Ρ€Π΅Π΄Π°Π²Π°Ρ‚ΡŒΡΡ Ρ€Π°Π·Π»ΠΈΡ‡Π½Ρ‹ΠΌΠΈ способами ΠΈ ΠΌΠ΅ΠΆΠ΄Ρƒ ΠΎΠ±ΡŠΠ΅ΠΊΡ‚Π°ΠΌΠΈ.

БоглашСниС ΠΎ Π²Ρ‹Ρ€Π°Π²Π½ΠΈΠ²Π°Π½ΠΈΠΈ: Бпасибо Π·Π° ваш ΠΎΡ‚Π·Ρ‹Π²!

ΠžΠ±Ρ‰ΠΈΠ΅ основныС государствСнныС стандарты — ΠΌΠ°Ρ‚Π΅ΠΌΠ°Ρ‚ΠΈΠΊΠ°
  • Π£ΠΌΠ½ΠΎΠΆΡŒΡ‚Π΅ Ρ†Π΅Π»ΠΎΠ΅ число Π΄ΠΎ Ρ‡Π΅Ρ‚Ρ‹Ρ€Π΅Ρ… Ρ†ΠΈΡ„Ρ€ Π½Π° ΠΎΠ΄Π½ΠΎΠ·Π½Π°Ρ‡Π½ΠΎΠ΅ Ρ†Π΅Π»ΠΎΠ΅ число ΠΈ ΡƒΠΌΠ½ΠΎΠΆΡŒΡ‚Π΅ Π΄Π²Π° Π΄Π²ΡƒΠ·Π½Π°Ρ‡Π½Ρ‹Ρ… числа, ΠΈΡΠΏΠΎΠ»ΡŒΠ·ΡƒΡ стратСгии, основанныС Π½Π° разрядах ΠΈ свойствах ΠΎΠΏΠ΅Ρ€Π°Ρ†ΠΈΠΉ.ΠŸΡ€ΠΎΠΈΠ»Π»ΡŽΡΡ‚Ρ€ΠΈΡ€ΡƒΠΉΡ‚Π΅ ΠΈ ΠΎΠ±ΡŠΡΡΠ½ΠΈΡ‚Π΅ расчСт с ΠΏΠΎΠΌΠΎΡ‰ΡŒΡŽ ΡƒΡ€Π°Π²Π½Π΅Π½ΠΈΠΉ, ΠΏΡ€ΡΠΌΠΎΡƒΠ³ΠΎΠ»ΡŒΠ½Ρ‹Ρ… массивов ΠΈ / ΠΈΠ»ΠΈ ΠΌΠΎΠ΄Π΅Π»Π΅ΠΉ ΠΏΠ»ΠΎΡ‰Π°Π΄Π΅ΠΉ. (ΠžΡ†Π΅Π½ΠΊΠ° 4) ΠŸΠΎΠ΄Ρ€ΠΎΠ±Π½Π΅Π΅

    ΠŸΠΎΡΠΌΠΎΡ‚Ρ€Π΅Ρ‚ΡŒ ΡΠΎΠ³Π»Π°ΡΠΎΠ²Π°Π½Π½ΡƒΡŽ ΡƒΡ‡Π΅Π±Π½ΡƒΡŽ ΠΏΡ€ΠΎΠ³Ρ€Π°ΠΌΠΌΡƒ

    Π’Ρ‹ согласны с Ρ‚Π°ΠΊΠΈΠΌ раскладом? Бпасибо Π·Π° ваш ΠΎΡ‚Π·Ρ‹Π²!

  • Π Π΅ΡˆΠ°ΠΉΡ‚Π΅ Ρ€Π΅Π°Π»ΡŒΠ½Ρ‹Π΅ ΠΈ матСматичСскиС ΠΏΡ€ΠΎΠ±Π»Π΅ΠΌΡ‹, записывая ΠΈ Ρ€Π΅ΡˆΠ°Ρ уравнСния Π²ΠΈΠ΄Π° x + p = q ΠΈ px = q для случаСв, ΠΊΠΎΠ³Π΄Π° p, q ΠΈ x ΡΠ²Π»ΡΡŽΡ‚ΡΡ Π½Π΅ΠΎΡ‚Ρ€ΠΈΡ†Π°Ρ‚Π΅Π»ΡŒΠ½Ρ‹ΠΌΠΈ Ρ€Π°Ρ†ΠΈΠΎΠ½Π°Π»ΡŒΠ½Ρ‹ΠΌΠΈ числами.(ΠžΡ†Π΅Π½ΠΊΠ° 6) ΠŸΠΎΠ΄Ρ€ΠΎΠ±Π½Π΅Π΅

    ΠŸΠΎΡΠΌΠΎΡ‚Ρ€Π΅Ρ‚ΡŒ ΡΠΎΠ³Π»Π°ΡΠΎΠ²Π°Π½Π½ΡƒΡŽ ΡƒΡ‡Π΅Π±Π½ΡƒΡŽ ΠΏΡ€ΠΎΠ³Ρ€Π°ΠΌΠΌΡƒ

    Π’Ρ‹ согласны с Ρ‚Π°ΠΊΠΈΠΌ раскладом? Бпасибо Π·Π° ваш ΠΎΡ‚Π·Ρ‹Π²!

ΠœΠ΅ΠΆΠ΄ΡƒΠ½Π°Ρ€ΠΎΠ΄Π½Π°Ρ ассоциация ΠΏΡ€Π΅ΠΏΠΎΠ΄Π°Π²Π°Ρ‚Π΅Π»Π΅ΠΉ Ρ‚Π΅Ρ…Π½ΠΎΠ»ΠΎΠ³ΠΈΠΉ ΠΈ ΠΈΠ½ΠΆΠ΅Π½Π΅Ρ€ΠΈΠΈ — ВСхнология
  • ΠœΠ΅ΠΆΠ΄Ρƒ тСхнологиями ΠΈ Π΄Ρ€ΡƒΠ³ΠΈΠΌΠΈ областями обучСния ΡΡƒΡ‰Π΅ΡΡ‚Π²ΡƒΡŽΡ‚ Ρ€Π°Π·Π»ΠΈΡ‡Π½Ρ‹Π΅ ΠΎΡ‚Π½ΠΎΡˆΠ΅Π½ΠΈΡ.(ΠžΡ†Π΅Π½ΠΊΠΈ 3 — 5) ΠŸΠΎΠ΄Ρ€ΠΎΠ±Π½Π΅Π΅

    ΠŸΠΎΡΠΌΠΎΡ‚Ρ€Π΅Ρ‚ΡŒ ΡΠΎΠ³Π»Π°ΡΠΎΠ²Π°Π½Π½ΡƒΡŽ ΡƒΡ‡Π΅Π±Π½ΡƒΡŽ ΠΏΡ€ΠΎΠ³Ρ€Π°ΠΌΠΌΡƒ

    Π’Ρ‹ согласны с Ρ‚Π°ΠΊΠΈΠΌ раскладом? Бпасибо Π·Π° ваш ΠΎΡ‚Π·Ρ‹Π²!

  • Π˜Π½ΡΡ‚Ρ€ΡƒΠΌΠ΅Π½Ρ‚Ρ‹, ΠΌΠ°ΡˆΠΈΠ½Ρ‹, ΠΏΡ€ΠΎΠ΄ΡƒΠΊΡ‚Ρ‹ ΠΈ систСмы ΠΈΡΠΏΠΎΠ»ΡŒΠ·ΡƒΡŽΡ‚ ΡΠ½Π΅Ρ€Π³ΠΈΡŽ для Ρ€Π°Π±ΠΎΡ‚Ρ‹.(ΠžΡ†Π΅Π½ΠΊΠΈ 3 — 5) ΠŸΠΎΠ΄Ρ€ΠΎΠ±Π½Π΅Π΅

    ΠŸΠΎΡΠΌΠΎΡ‚Ρ€Π΅Ρ‚ΡŒ ΡΠΎΠ³Π»Π°ΡΠΎΠ²Π°Π½Π½ΡƒΡŽ ΡƒΡ‡Π΅Π±Π½ΡƒΡŽ ΠΏΡ€ΠΎΠ³Ρ€Π°ΠΌΠΌΡƒ

    Π’Ρ‹ согласны с Ρ‚Π°ΠΊΠΈΠΌ раскладом? Бпасибо Π·Π° ваш ΠΎΡ‚Π·Ρ‹Π²!

Π“ΠžΠ‘Π’ ΠŸΡ€Π΅Π΄Π»ΠΎΠΆΠΈΡ‚Π΅ Π²Ρ‹Ρ€Π°Π²Π½ΠΈΠ²Π°Π½ΠΈΠ΅, Π½Π΅ ΡƒΠΊΠ°Π·Π°Π½Π½ΠΎΠ΅ Π²Ρ‹ΡˆΠ΅

КакоС Π°Π»ΡŒΡ‚Π΅Ρ€Π½Π°Ρ‚ΠΈΠ²Π½ΠΎΠ΅ Π²Ρ‹Ρ€Π°Π²Π½ΠΈΠ²Π°Π½ΠΈΠ΅ Π²Ρ‹ ΠΏΡ€Π΅Π΄Π»Π°Π³Π°Π΅Ρ‚Π΅ для этого ΠΊΠΎΠ½Ρ‚Π΅Π½Ρ‚Π°?

Π‘ΠΎΠ»ΡŒΡˆΠ΅ ΠΏΠΎΠ΄ΠΎΠ±Π½ΠΎΠΉ ΠΏΡ€ΠΎΠ³Ρ€Π°ΠΌΠΌΡ‹

ΠŸΡ€Π΅Π΄Π²Π°Ρ€ΠΈΡ‚Π΅Π»ΡŒΠ½Ρ‹Π΅ знания

Π¦Π΅ΠΏΠΈ сСрии

Π’Π²Π΅Π΄Π΅Π½ΠΈΠ΅ / ΠœΠΎΡ‚ΠΈΠ²Π°Ρ†ΠΈΡ

ΠŸΠΎΠΏΡ€ΠΎΡΠΈΡ‚Π΅ учащихся ΠΏΠΎΠ΄Π½ΡΡ‚ΡŒ Ρ€ΡƒΠΊΠΈ, Ссли ΠΎΠ½ΠΈ ΠΊΠΎΠ³Π΄Π°-Π»ΠΈΠ±ΠΎ использовали Π²ΠΈΠ΄Π΅ΠΎΠΈΠ³Ρ€Ρ‹, ΠΏΡƒΠ»ΡŒΡ‚ дистанционного управлСния (для Ρ‚Π΅Π»Π΅Π²ΠΈΠ·ΠΎΡ€Π° ΠΈΠ»ΠΈ Π΄Ρ€ΡƒΠ³ΠΎΠ³ΠΎ элСктронного устройства) ΠΈΠ»ΠΈ ΠΊΠ»Π°Π²ΠΈΠ°Ρ‚ΡƒΡ€Ρƒ.БпроситС, Ρƒ ΠΊΠΎΠ³ΠΎ-Π½ΠΈΠ±ΡƒΠ΄ΡŒ ΠΈΠ· Π½ΠΈΡ… ΠΊΠΎΠ³Π΄Π°-Π»ΠΈΠ±ΠΎ пСрСстала Ρ€Π°Π±ΠΎΡ‚Π°Ρ‚ΡŒ Ρ‚ΠΎΠ»ΡŒΠΊΠΎ ΠΎΠ΄Π½Π° ΠΊΠ½ΠΎΠΏΠΊΠ° ΠΈΠ»ΠΈ клавиша, Π² Ρ‚ΠΎ врСмя ΠΊΠ°ΠΊ ΠΎΡΡ‚Π°Π»ΡŒΠ½Π°Ρ Ρ‡Π°ΡΡ‚ΡŒ ΠΊΠΎΠ½Ρ‚Ρ€ΠΎΠ»Π»Π΅Ρ€Π° Π²ΠΈΠ΄Π΅ΠΎΠΈΠ³Ρ€Ρ‹, ΠΏΡƒΠ»ΡŒΡ‚Π° дистанционного управлСния ΠΈΠ»ΠΈ ΠΊΠ»Π°Π²ΠΈΠ°Ρ‚ΡƒΡ€Ρ‹ ΠΏΡ€ΠΎΠ΄ΠΎΠ»ΠΆΠ°Π»Π° Ρ€Π°Π±ΠΎΡ‚Π°Ρ‚ΡŒ. Π§Ρ‚ΠΎ происходит Π² элСктронном Π²ΠΈΠ΄Π΅, Ρ‡Ρ‚ΠΎ Π²Ρ‹Π·Ρ‹Π²Π°Π΅Ρ‚ это? Как ΠΌΠΎΠΆΠ΅Ρ‚ Π±Ρ‹Ρ‚ΡŒ сломана Ρ‚ΠΎΠ»ΡŒΠΊΠΎ ΠΎΠ΄Π½Π° ΠΊΠ½ΠΎΠΏΠΊΠ°, Π° ΠΎΡΡ‚Π°Π»ΡŒΠ½Π°Ρ Ρ‡Π°ΡΡ‚ΡŒ ΠΊΠΎΠ½Ρ‚Ρ€ΠΎΠ»Π»Π΅Ρ€Π° всС Π΅Ρ‰Π΅ Ρ€Π°Π±ΠΎΡ‚Π°Π΅Ρ‚?

БпроситС студСнтов, Π·Π°Ρ…ΠΎΠ΄ΠΈΠ»ΠΈ Π»ΠΈ ΠΎΠ½ΠΈ ΠΊΠΎΠ³Π΄Π°-Π½ΠΈΠ±ΡƒΠ΄ΡŒ Π² ΠΊΠΎΠΌΠ½Π°Ρ‚Ρƒ с нСсколькими Π»Π°ΠΌΠΏΠ°ΠΌΠΈ ΠΈ Π²ΠΊΠ»ΡŽΡ‡Π°Π»ΠΈ Π»ΠΈ ΠΎΠ½ΠΈ Ρ‚ΠΎΠ»ΡŒΠΊΠΎ ΠΎΠ΄ΠΈΠ½. НапомнитС учащимся ΠΎ ΠΏΠΎΡΠ»Π΅Π΄ΠΎΠ²Π°Ρ‚Π΅Π»ΡŒΠ½ΠΎ построСнных цСпях, ΠΊΠΎΡ‚ΠΎΡ€Ρ‹Π΅ ΠΎΠ½ΠΈ построили Ρ€Π°Π½Π΅Π΅. Когда ΠΎΠ΄Π½Π° Π»Π°ΠΌΠΏΠΎΡ‡ΠΊΠ° Π±Ρ‹Π»Π° Π²Ρ‹Π½ΡƒΡ‚Π° ΠΈΠ· Ρ†Π΅ΠΏΠΈ, ΠΎΠ±Ρ€Π°Π·ΠΎΠ²Π°Π»Π°ΡΡŒ разомкнутая Ρ†Π΅ΠΏΡŒ, ΠΈ элСктроны Π½Π΅ ΠΌΠΎΠ³Π»ΠΈ Ρ‚Π΅Ρ‡ΡŒ, Ρ‡Ρ‚ΠΎΠ±Ρ‹ Π·Π°ΠΆΠ΅Ρ‡ΡŒ Π΄Ρ€ΡƒΠ³ΠΈΠ΅ Π»Π°ΠΌΠΏΠΎΡ‡ΠΊΠΈ.Π’Π΅ΠΏΠ΅Ρ€ΡŒ спроситС ΠΈΡ…, ΠΊΠ°ΠΊ это Π²ΠΎΠ·ΠΌΠΎΠΆΠ½ΠΎ, Ρ‡Ρ‚ΠΎ Π²Ρ‹ ΠΌΠΎΠΆΠ΅Ρ‚Π΅ Π²ΠΊΠ»ΡŽΡ‡ΠΈΡ‚ΡŒ ΠΎΠ΄ΠΈΠ½ свСт Π² ΠΊΠΎΠΌΠ½Π°Ρ‚Π΅, ΠΈ ΠΎΠ½ Π±ΡƒΠ΄Π΅Ρ‚ Ρ€Π°Π±ΠΎΡ‚Π°Ρ‚ΡŒ, Π½ΠΎ Π²Π°ΠΌ Π½Π΅ Π½ΡƒΠΆΠ½ΠΎ Π±Ρ‹Π»ΠΎ Π²ΠΊΠ»ΡŽΡ‡Π°Ρ‚ΡŒ всС ΠΎΡΡ‚Π°Π»ΡŒΠ½Ρ‹Π΅ ΡΠ²Π΅Ρ‚ΠΈΠ»ΡŒΠ½ΠΈΠΊΠΈ?

ΠžΠ±ΡŠΡΡΠ½ΠΈΡ‚Π΅ учащимся, Ρ‡Ρ‚ΠΎ Π² этих Π΄Π²ΡƒΡ… ΠΏΡ€ΠΈΠΌΠ΅Ρ€Π°Ρ… ΠΈΡΠΏΠΎΠ»ΡŒΠ·ΡƒΡŽΡ‚ΡΡ ΠΏΠ°Ρ€Π°Π»Π»Π΅Π»ΡŒΠ½Ρ‹Π΅ схСмы. Π˜Π½ΠΆΠ΅Π½Π΅Ρ€Ρ‹ ΠΏΠΎΠ΄ΠΊΠ»ΡŽΡ‡Π°ΡŽΡ‚ элСмСнты ΠΏΠ°Ρ€Π°Π»Π»Π΅Π»ΡŒΠ½ΠΎ , поэтому, Ссли ΠΎΠ΄Π½Π° Ρ‡Π°ΡΡ‚ΡŒ Ρ†Π΅ΠΏΠΈ Π²Ρ‹Ρ…ΠΎΠ΄ΠΈΡ‚ ΠΈΠ· строя, ΠΎΡΡ‚Π°Π»ΡŒΠ½Π°Ρ Ρ‡Π°ΡΡ‚ΡŒ Ρ†Π΅ΠΏΠΈ всС Π΅Ρ‰Π΅ Ρ€Π°Π±ΠΎΡ‚Π°Π΅Ρ‚.

ΠŸΠΎΠΏΡ€ΠΎΡΠΈΡ‚Π΅ Ρ‚Ρ€Π΅Ρ… Π΄ΠΎΠ±Ρ€ΠΎΠ²ΠΎΠ»ΡŒΡ†Π΅Π². ΠΠ°Π·Π½Π°Ρ‡ΡŒΡ‚Π΅ ΠΎΠ΄Π½ΠΎΠ³ΠΎ Π΄ΠΎΠ±Ρ€ΠΎΠ²ΠΎΠ»ΡŒΡ†Π° Β«Π±Π°Ρ‚Π°Ρ€Π΅Π΅ΠΉΒ», Π° Π΄Π²ΡƒΡ… — Β«Π»Π°ΠΌΠΏΠΎΡ‡ΠΊΠ°ΠΌΠΈΒ». (Π­Ρ‚ΠΎ ΠΌΠΎΠΆΠ΅Ρ‚ ΠΏΠΎΠΌΠΎΡ‡ΡŒ Π½Π°Ρ€ΠΈΡΠΎΠ²Π°Ρ‚ΡŒ ΡΠΎΠΎΡ‚Π²Π΅Ρ‚ΡΡ‚Π²ΡƒΡŽΡ‰ΠΈΠ΅ символы Π½Π° листах Π±ΡƒΠΌΠ°Π³ΠΈ ΠΈ ΠΏΡ€ΠΈΠΊΠ»Π΅ΠΈΡ‚ΡŒ ΠΈΡ… ΠΊ Ρ€ΡƒΠ±Π°ΡˆΠΊΠ°ΠΌ.) ΠŸΠΎΠΏΡ€ΠΎΡΠΈΡ‚Π΅ учащихся физичСски ΠΈΠ·ΠΎΠ±Ρ€Π°Π·ΠΈΡ‚ΡŒ ΠΏΠΎΡΠ»Π΅Π΄ΠΎΠ²Π°Ρ‚Π΅Π»ΡŒΠ½ΡƒΡŽ Ρ†Π΅ΠΏΡŒ, взявшись Π·Π° Ρ€ΡƒΠΊΠΈ Π² ΠΊΡ€ΡƒΠ³Π΅. Π—Π°Ρ‚Π΅ΠΌ попроситС учащихся ΠΈΠ·ΠΎΠ±Ρ€Π°Π·ΠΈΡ‚ΡŒ ΠΏΠ°Ρ€Π°Π»Π»Π΅Π»ΡŒΠ½ΡƒΡŽ Ρ†Π΅ΠΏΡŒ, ΠΏΠΎΠ²Π΅Ρ€Π½ΡƒΠ² Π»Π°ΠΌΠΏΠΎΡ‡ΠΊΠΈ ΠΈ подставку для Π±Π°Ρ‚Π°Ρ€Π΅Π΅ΠΊ Π² ΠΎΠ΄Π½ΠΎΠΌ Π½Π°ΠΏΡ€Π°Π²Π»Π΅Π½ΠΈΠΈ, Π° ΠΈΡ… Ρ€ΡƒΠΊΠΈ ΠΊΠ°ΡΠ°ΡŽΡ‚ΡΡ Π»ΠΎΠΊΡ‚Π΅ΠΉ Ρ‡Π΅Π»ΠΎΠ²Π΅ΠΊΠ° ΠΏΠ΅Ρ€Π΅Π΄ Π½ΠΈΠΌΠΈ.

ΠžΡ‡Π΅Π½ΡŒ слоТная схСма, ΠΎΠ±ΡŠΠ΅Π΄ΠΈΠ½ΡΡŽΡ‰Π°Ρ тысячи ΠΈ ΠΌΠΈΠ»Π»ΠΈΠΎΠ½Ρ‹ ΠΏΠ°Ρ€Π°Π»Π»Π΅Π»ΡŒΠ½Ρ‹Ρ… ΠΈ ΠΏΠΎΡΠ»Π΅Π΄ΠΎΠ²Π°Ρ‚Π΅Π»ΡŒΠ½Ρ‹Ρ… схСм, Ρ€Π°Π±ΠΎΡ‚Π°ΡŽΡ‰ΠΈΡ… вмСстС, называСтся ΠΈΠ½Ρ‚Π΅Π³Ρ€Π°Π»ΡŒΠ½ΠΎΠΉ схСмой (см. Рисунок 1). ΠœΠΈΠΊΡ€ΠΎΠΏΡ€ΠΎΡ†Π΅ΡΡΠΎΡ€, извСстный ΠΊΠ°ΠΊ Ρ†Π΅Π½Ρ‚Ρ€Π°Π»ΡŒΠ½Ρ‹ΠΉ процСссор ΠΈΠ»ΠΈ ЦП, прСдставляСт собой Ρ‚ΠΈΠΏ ΠΈΠ½Ρ‚Π΅Π³Ρ€Π°Π»ΡŒΠ½ΠΎΠΉ схСмы, которая Ρ€Π°Π±ΠΎΡ‚Π°Π΅Ρ‚ ΠΊΠ°ΠΊ ΠΏΠΎΠ»Π½ΠΎΡ†Π΅Π½Π½Ρ‹ΠΉ Π²Ρ‹Ρ‡ΠΈΡΠ»ΠΈΡ‚Π΅Π»ΡŒΠ½Ρ‹ΠΉ ΠΌΠ΅Ρ…Π°Π½ΠΈΠ·ΠΌ.Π’ наши Π΄Π½ΠΈ Π² срСднСм Π΄ΠΎΠΌΠ΅ Π² БША Π΅ΡΡ‚ΡŒ ΠΎΠΊΠΎΠ»ΠΎ 40 Ρ‚Π°ΠΊΠΈΡ… микропроцСссоров Π² Π΄ΠΎΠΏΠΎΠ»Π½Π΅Π½ΠΈΠ΅ ΠΊ 10 ΠΈΠ»ΠΈ ΠΎΠΊΠΎΠ»ΠΎ Ρ‚ΠΎΠ³ΠΎ Π² ΠΎΠ΄Π½ΠΎΠΌ ΠΎΠ±Ρ‹Ρ‡Π½ΠΎΠΌ ΠΏΠ΅Ρ€ΡΠΎΠ½Π°Π»ΡŒΠ½ΠΎΠΌ ΠΊΠΎΠΌΠΏΡŒΡŽΡ‚Π΅Ρ€Π΅. ΠœΠΈΠΊΡ€ΠΎΠΏΡ€ΠΎΡ†Π΅ΡΡΠΎΡ€Ρ‹ ΠΈΡΠΏΠΎΠ»ΡŒΠ·ΡƒΡŽΡ‚ΡΡ Π² автомобилях, Π²ΠΈΠ΄Π΅ΠΎΠΈΠ³Ρ€Π°Ρ…, Π΄Π΅Ρ‚Π΅ΠΊΡ‚ΠΎΡ€Π°Ρ… Π΄Ρ‹ΠΌΠ°, DVD-ΠΏΠ»Π΅Π΅Ρ€Π°Ρ…, ΠΌΠ΅Ρ…Π°Π½ΠΈΠ·ΠΌΠ°Ρ… открывания Π³Π°Ρ€Π°ΠΆΠ½Ρ‹Ρ… Π²ΠΎΡ€ΠΎΡ‚, бСспроводных Ρ‚Π΅Π»Π΅Ρ„ΠΎΠ½Π°Ρ…, часах ΠΈ ΠΊΠ°Π»ΡŒΠΊΡƒΠ»ΡΡ‚ΠΎΡ€Π°Ρ…. Π˜Ρ… Π΄Π°ΠΆΠ΅ Π²ΠΆΠΈΠ²Π»ΡΡŽΡ‚ ΠΆΠΈΠ²ΠΎΡ‚Π½Ρ‹ΠΌ Π² качСствС элСктронных ΠΈΠ΄Π΅Π½Ρ‚ΠΈΡ„ΠΈΠΊΠ°Ρ†ΠΈΠΎΠ½Π½Ρ‹Ρ… ΠΌΠ΅Ρ‚ΠΎΠΊ.

Рис. 1. ΠŸΡ€ΠΈΠ½Ρ†ΠΈΠΏΠΈΠ°Π»ΡŒΠ½Π°Ρ схСма простой ΠΏΠ°Ρ€Π°Π»Π»Π΅Π»ΡŒΠ½ΠΎΠΉ схСмы (слСва) ΠΈ ΠΈΠ½Ρ‚Π΅Π³Ρ€Π°Π»ΡŒΠ½ΠΎΠΉ схСмы (справа). Copyright

Copyright Β© http: // whyfiles.larc.nasa.gov/text/kids/Problem_Board/problems/electricity/images/circuits05.gif (слСва) ΠΈ http://www.lbl.gov/Education/HGP-images/integrated-circuit-small.gif (справа )

ΠŸΡ€Π΅Π΄ΠΏΠΎΡΡ‹Π»ΠΊΠΈ ΠΈ ΠΊΠΎΠ½Ρ†Π΅ΠΏΡ†ΠΈΠΈ ΡƒΡ€ΠΎΠΊΠ° для ΡƒΡ‡ΠΈΡ‚Π΅Π»Π΅ΠΉ

ΠŸΠ°Ρ€Π°Π»Π»Π΅Π»ΡŒΠ½Ρ‹Π΅ схСмы

ΠŸΠ°Ρ€Π°Π»Π»Π΅Π»ΡŒΠ½Π°Ρ Ρ†Π΅ΠΏΡŒ ΠΈ ΡΠΎΠΎΡ‚Π²Π΅Ρ‚ΡΡ‚Π²ΡƒΡŽΡ‰Π°Ρ Π΅ΠΉ ΠΏΡ€ΠΈΠ½Ρ†ΠΈΠΏΠΈΠ°Π»ΡŒΠ½Π°Ρ схСма ΠΏΠΎΠΊΠ°Π·Π°Π½Ρ‹ Π½Π° рисункС 2. ΠŸΠΎΡΠΊΠΎΠ»ΡŒΠΊΡƒ сущСствуСт Π±ΠΎΠ»Π΅Π΅ ΠΎΠ΄Π½ΠΎΠ³ΠΎ ΠΏΡƒΡ‚ΠΈ для прохоТдСния заряда ΠΏΠΎ Ρ†Π΅ΠΏΠΈ, Ρ‚ΠΎΠΊ дСлится ΠΌΠ΅ΠΆΠ΄Ρƒ двумя Π»Π°ΠΌΠΏΠΎΡ‡ΠΊΠ°ΠΌΠΈ.Π‘Π»Π΅Π΄ΠΎΠ²Π°Ρ‚Π΅Π»ΡŒΠ½ΠΎ, Ρ‚ΠΎΠΊ ΠΏΠ΅Ρ€Π΅Π΄ Π»Π°ΠΌΠΏΠΎΡ‡ΠΊΠ°ΠΌΠΈ (Π² ΡƒΠ·Π»Π΅; пСрСсСчСниС Π΄Π²ΡƒΡ… ΠΏΡ€ΠΎΠ²ΠΎΠ΄ΠΎΠ²) ΠΈ послС Π»Π°ΠΌΠΏΠΎΡ‡Π΅ΠΊ (Π² ΡƒΠ·Π»Π΅; пСрСсСчСниС Π΄Π²ΡƒΡ… ΠΏΡ€ΠΎΠ²ΠΎΠ΄ΠΎΠ²) ΠΎΠ΄ΠΈΠ½Π°ΠΊΠΎΠ², Π½ΠΎ Ρ€Π°Π·Π΄Π΅Π»Π΅Π½ Π² Π»Π°ΠΌΠΏΠΎΡ‡ΠΊΠ°Ρ…. Π”Ρ€ΡƒΠ³ΠΈΠΌΠΈ словами, ΠΏΠΎΠ»Π½Ρ‹ΠΉ Ρ‚ΠΎΠΊ Π² Ρ†Π΅ΠΏΠΈ Ρ€Π°Π²Π΅Π½ суммС Ρ‚ΠΎΠΊΠΎΠ² Π½Π° ΠΏΠ°Ρ€Π°Π»Π»Π΅Π»ΡŒΠ½Ρ‹Ρ… участках. ΠžΠ±Ρ€Π°Ρ‚ΠΈΡ‚Π΅ Π²Π½ΠΈΠΌΠ°Π½ΠΈΠ΅: Ссли Ρƒ Π»Π°ΠΌΠΏΠΎΡ‡Π΅ΠΊ ΠΎΠ΄ΠΈΠ½Π°ΠΊΠΎΠ²ΠΎΠ΅ сопротивлСниС, Ρ‚ΠΎΠΊ дСлится ΠΌΠ΅ΠΆΠ΄Ρƒ Π½ΠΈΠΌΠΈ ΠΏΠΎΡ€ΠΎΠ²Π½Ρƒ. Π‘ Π΄Ρ€ΡƒΠ³ΠΎΠΉ стороны, Ссли Π»Π°ΠΌΠΏΡ‹ ΠΈΠΌΠ΅ΡŽΡ‚ Ρ€Π°Π·Π½ΠΎΠ΅ сопротивлСниС, Π»Π°ΠΌΠΏΠ° с большим сопротивлСниСм Π±ΡƒΠ΄Π΅Ρ‚ ΠΈΠΌΠ΅Ρ‚ΡŒ мСньший Ρ‚ΠΎΠΊ. ΠžΠ±Ρ‰Π΅Π΅ сопротивлСниС Ρ†Π΅ΠΏΠΈ ΡƒΠΌΠ΅Π½ΡŒΡˆΠ°Π΅Ρ‚ΡΡ ΠΏΡ€ΠΈ ΡƒΠ²Π΅Π»ΠΈΡ‡Π΅Π½ΠΈΠΈ количСства ΠΏΠ°Ρ€Π°Π»Π»Π΅Π»ΡŒΠ½Ρ‹Ρ… Ρ†Π΅ΠΏΠ΅ΠΉ.ПадСниС напряТСния Π½Π° ΠΊΠ°ΠΆΠ΄ΠΎΠΉ части ΠΏΠ°Ρ€Π°Π»Π»Π΅Π»ΡŒΠ½ΠΎΠΉ Ρ†Π΅ΠΏΠΈ ΠΎΠ΄ΠΈΠ½Π°ΠΊΠΎΠ²ΠΎ, ΠΏΠΎΡ‚ΠΎΠΌΡƒ Ρ‡Ρ‚ΠΎ каТдая Ρ‡Π°ΡΡ‚ΡŒ ΠΏΠΎΠ΄ΠΊΠ»ΡŽΡ‡Π΅Π½Π° ΠΊ ΠΎΠ΄Π½ΠΈΠΌ ΠΈ Ρ‚Π΅ΠΌ ΠΆΠ΅ Π΄Π²ΡƒΠΌ Ρ‚ΠΎΡ‡ΠΊΠ°ΠΌ. Π‘Ρ‚ΡƒΠ΄Π΅Π½Ρ‚Ρ‹ ΠΌΠΎΠ³ΡƒΡ‚ ΠΏΠΎΠΏΡ€Π°ΠΊΡ‚ΠΈΠΊΠΎΠ²Π°Ρ‚ΡŒΡΡ Π² построСнии собствСнных ΠΏΠ°Ρ€Π°Π»Π»Π΅Π»ΡŒΠ½Ρ‹Ρ… Ρ†Π΅ΠΏΠ΅ΠΉ с ΠΏΠΎΠΌΠΎΡ‰ΡŒΡŽ связанного с Π½ΠΈΠΌΠΈ задания Β«Π›Π°ΠΌΠΏΠΎΡ‡ΠΊΠΈ ΠΈ Π±Π°Ρ‚Π°Ρ€Π΅ΠΈ Π±ΠΎΠΊ ΠΎ Π±ΠΎΠΊΒ».

Рис. 2. ΠŸΠΎΡΠ»Π΅Π΄ΠΎΠ²Π°Ρ‚Π΅Π»ΡŒΠ½Π°Ρ схСма (слСва) ΠΈ ΡΠΎΠΎΡ‚Π²Π΅Ρ‚ΡΡ‚Π²ΡƒΡŽΡ‰Π°Ρ ΠΏΡ€ΠΈΠ½Ρ†ΠΈΠΏΠΈΠ°Π»ΡŒΠ½Π°Ρ схСма (справа). АвторскоС ΠΏΡ€Π°Π²ΠΎ

АвторскиС ΠΏΡ€Π°Π²Π° Β© Π”ΠΆΠΎ ЀридрихсСн, ΠŸΡ€ΠΎΠ³Ρ€Π°ΠΌΠΌΠ° ΠΈ лаборатория ITL, УнивСрситСт ΠšΠΎΠ»ΠΎΡ€Π°Π΄ΠΎ Π² Π‘ΠΎΡƒΠ»Π΄Π΅Ρ€Π΅, 2003.

ΠŸΡ€ΠΈ ΠΏΠ°Ρ€Π°Π»Π»Π΅Π»ΡŒΠ½ΠΎΠΌ соСдинСнии Π±Π°Ρ‚Π°Ρ€Π΅ΠΉ ΠΎΠ±Ρ‰ΠΈΠΉ ΠΏΡ€ΠΎΠΈΠ·Π²ΠΎΠ΄ΠΈΠΌΡ‹ΠΉ Ρ‚ΠΎΠΊ увСличиваСтся.НапримСр, Ссли ΠΌΡ‹ сдСлаСм схСму с использованиСм Ρ‚Ρ€Π΅Ρ… ΠΏΠ°Ρ€Π°Π»Π»Π΅Π»ΡŒΠ½Ρ‹Ρ… Π±Π°Ρ‚Π°Ρ€Π΅ΠΉ 1,5 Π’ Π² качСствС источника напряТСния, ΠΎΠ±Ρ‰Π΅Π΅ напряТСниС, обСспСчиваСмоС Π±Π°Ρ‚Π°Ρ€Π΅Π΅ΠΉ, всС Ρ€Π°Π²Π½ΠΎ Π±ΡƒΠ΄Π΅Ρ‚ 1,5 Π’. Однако Ρ‚ΠΎΠΊ Π±ΡƒΠ΄Π΅Ρ‚ Π² Ρ‚Ρ€ΠΈ Ρ€Π°Π·Π° большС, Ρ‡Π΅ΠΌ Ρƒ ΠΎΠ΄Π½ΠΎΠΉ Π±Π°Ρ‚Π°Ρ€Π΅ΠΈ 1,5 Π’. ΠŸΠΎΠΌΠ½ΠΈΡ‚Π΅, Ρ‡Ρ‚ΠΎ Π²Π΅Π»ΠΈΡ‡ΠΈΠ½Π° Ρ‚ΠΎΠΊΠ° Π² Ρ†Π΅ΠΏΠΈ зависит ΠΎΡ‚ сопротивлСний устройств Π² Ρ†Π΅ΠΏΠΈ. Когда ΠΈΠ½ΠΆΠ΅Π½Π΅Ρ€ ΠΏΡ€ΠΎΠ΅ΠΊΡ‚ΠΈΡ€ΡƒΠ΅Ρ‚ устройство, Π½Π°ΠΏΡ€ΠΈΠΌΠ΅Ρ€ ΠΏΠΎΡ€Ρ‚Π°Ρ‚ΠΈΠ²Π½Ρ‹ΠΉ ΠΏΡ€ΠΎΠΈΠ³Ρ€Ρ‹Π²Π°Ρ‚Π΅Π»ΡŒ ΠΊΠΎΠΌΠΏΠ°ΠΊΡ‚-дисков, ΠΎΠ½ / ΠΎΠ½Π° Ρ€Π΅ΡˆΠ°Π΅Ρ‚, сколько Π±Π°Ρ‚Π°Ρ€Π΅ΠΉ Π½Π΅ΠΎΠ±Ρ…ΠΎΠ΄ΠΈΠΌΠΎ ΠΏΠ°Ρ€Π°Π»Π»Π΅Π»ΡŒΠ½ΠΎ, Ρ‡Ρ‚ΠΎΠ±Ρ‹ ΠΎΠ±Π΅ΡΠΏΠ΅Ρ‡ΠΈΡ‚ΡŒ достаточный Ρ‚ΠΎΠΊ. Как Π²ΠΈΠ΄ΠΈΡ‚Π΅, ΠΈΠ½ΠΆΠ΅Π½Π΅Ρ€Ρ‹-элСктрики Π΄ΠΎΠ»ΠΆΠ½Ρ‹ Ρ…ΠΎΡ€ΠΎΡˆΠΎ Ρ€Π°Π·Π±ΠΈΡ€Π°Ρ‚ΡŒΡΡ Π² элСктричСствС, Π½ΠΎ ΠΏΡ€ΠΈ этом Π±Ρ‹Ρ‚ΡŒ ΠΎΡ‡Π΅Π½ΡŒ ΠΊΡ€Π΅Π°Ρ‚ΠΈΠ²Π½Ρ‹ΠΌΠΈ Π² своСй Ρ€Π°Π±ΠΎΡ‚Π΅!

ЭлСктричСство Π² Π΄ΠΎΠΌΠ΅

Когда Π²Ρ‹ ΠΏΠΎΠ΄ΠΊΠ»ΡŽΡ‡Π°Π΅Ρ‚Π΅ ΠΏΡ€ΠΈΠ±ΠΎΡ€ ΠΊ Ρ€ΠΎΠ·Π΅Ρ‚ΠΊΠ΅ Π΄ΠΎΠΌΠ°, Π²Ρ‹ добавляСтС ΠΏΠ°Ρ€Π°Π»Π»Π΅Π»ΡŒΠ½ΡƒΡŽ Π²Π΅Ρ‚Π²ΡŒ ΠΊ Ρ†Π΅ΠΏΠΈ, которая ΠΈΠ΄Π΅Ρ‚ Π΄ΠΎ вашСй мСстной элСктростанции.К Ρ€ΠΎΠ·Π΅Ρ‚ΠΊΠ°ΠΌ ΠΏΠΎΠ΄ΠΊΠ»ΡŽΡ‡Π΅Π½Ρ‹ Π΄Π²Π° ΠΏΡ€ΠΎΠ²ΠΎΠ΄Π°, Π½Π°Π·Ρ‹Π²Π°Π΅ΠΌΡ‹Π΅ линиями; ΠΎΠ΄Π½Π° линия называСтся ΠΏΡ€ΠΎΠ²ΠΎΠ΄ΠΎΠΌ ΠΏΠΎΠ΄ напряТСниСм, Π° другая — Π½ΡƒΠ»Π΅Π²Ρ‹ΠΌ ΠΏΡ€ΠΎΠ²ΠΎΠ΄ΠΎΠΌ. Π­Ρ‚ΠΈ Π»ΠΈΠ½ΠΈΠΈ ΠΏΠΎΠ΄Π°ΡŽΡ‚ ΠΏΠ΅Ρ€Π΅ΠΌΠ΅Π½Π½Ρ‹ΠΉ Ρ‚ΠΎΠΊ (AC) напряТСниСм 110–120 Π’. Часто Ρ‚Ρ€Π΅Ρ‚ΠΈΠΉ ΠΊΠΎΠ½Ρ‚Π°ΠΊΡ‚ Π² Ρ€ΠΎΠ·Π΅Ρ‚ΠΊΠ΅ являСтся Π·Π°Π·Π΅ΠΌΠ»ΡΡŽΡ‰ΠΈΠΌ ΠΏΡ€ΠΎΠ²ΠΎΠ΄ΠΎΠΌ. Π—Π°Π·Π΅ΠΌΠ»ΡΡŽΡ‰ΠΈΠΉ ΠΏΡ€ΠΎΠ²ΠΎΠ΄ ΠΏΠΎΠ΄ΠΊΠ»ΡŽΡ‡Π°Π΅Ρ‚ΡΡ нСпосрСдствСнно ΠΊ Π·Π΅ΠΌΠ»Π΅, Ρ‡Ρ‚ΠΎΠ±Ρ‹ Π½Π°ΠΏΡ€Π°Π²ΠΈΡ‚ΡŒ Ρ‚ΠΎΠΊ Π² зСмлю, Ссли ΠΏΡ€ΠΎΠ²ΠΎΠ΄ ΠΏΠΎΠ΄ напряТСниСм случайно касаСтся ΠΌΠ΅Ρ‚Π°Π»Π»Π° Π½Π° ΠΏΡ€ΠΈΠ±ΠΎΡ€Π΅. Π­Ρ‚ΠΎ ΠΏΡ€Π΅Π΄ΠΎΡ‚Π²Ρ€Π°Ρ‚ΠΈΡ‚ ΠΏΠΎΡ€Π°ΠΆΠ΅Π½ΠΈΠ΅ элСктричСским Ρ‚ΠΎΠΊΠΎΠΌ любого, ΠΊΡ‚ΠΎ прикоснСтся ΠΊ ΠΏΡ€ΠΈΠ±ΠΎΡ€Ρƒ. ΠšΠΎΠ½Π΅Ρ‡Π½ΠΎ, ΠΏΡ€ΠΈΠ±ΠΎΡ€ Π΄ΠΎΠ»ΠΆΠ΅Π½ Π±Ρ‹Ρ‚ΡŒ ΠΏΠΎΠ΄ΠΊΠ»ΡŽΡ‡Π΅Π½ ΠΊ Π·Π°Π·Π΅ΠΌΠ»ΡΡŽΡ‰Π΅ΠΌΡƒ ΠΏΡ€ΠΎΠ²ΠΎΠ΄Ρƒ с ΠΏΠΎΠΌΠΎΡ‰ΡŒΡŽ Π°Π΄Π°ΠΏΡ‚Π΅Ρ€Π° ΠΈΠ»ΠΈ Ρ‚Ρ€Π΅Ρ…ΠΊΠΎΠ½Ρ‚Π°ΠΊΡ‚Π½ΠΎΠΉ Π²ΠΈΠ»ΠΊΠΈ.Π˜Π½ΠΆΠ΅Π½Π΅Ρ€Ρ‹ нСсут ΠΎΡ‚Π²Π΅Ρ‚ΡΡ‚Π²Π΅Π½Π½ΠΎΡΡ‚ΡŒ Π·Π° Π±Π΅Π·ΠΎΠΏΠ°ΡΠ½ΠΎΡΡ‚ΡŒ использования Π±Ρ‹Ρ‚ΠΎΠ²ΠΎΠΉ Ρ‚Π΅Ρ…Π½ΠΈΠΊΠΈ; Π½Π°Π΄Π»Π΅ΠΆΠ°Ρ‰Π΅Π΅ Π·Π°Π·Π΅ΠΌΠ»Π΅Π½ΠΈΠ΅ являСтся Π²Π°ΠΆΠ½Ρ‹ΠΌ аспСктом ΠΏΡ€ΠΈ ΠΏΡ€ΠΎΠ΅ΠΊΡ‚ΠΈΡ€ΠΎΠ²Π°Π½ΠΈΠΈ, ΠΈ ΠΎΠ½ΠΈ всСгда заботятся ΠΎΠ± общСствСнной бСзопасности.

ЭлСктроэнСргСтика

Всякий Ρ€Π°Π·, ΠΊΠΎΠ³Π΄Π° Π² Ρ†Π΅ΠΏΠΈ Π΅ΡΡ‚ΡŒ Ρ‚ΠΎΠΊ, элСктричСская энСргия ΠΈΡΠΏΠΎΠ»ΡŒΠ·ΡƒΠ΅Ρ‚ΡΡ для выполнСния ΠΎΠΏΡ€Π΅Π΄Π΅Π»Π΅Π½Π½ΠΎΠ³ΠΎ Π²ΠΈΠ΄Π° Ρ€Π°Π±ΠΎΡ‚Ρ‹, Π° элСктричСская энСргия прСобразуСтся Π² Π΄Ρ€ΡƒΠ³ΠΎΠΉ Ρ‚ΠΈΠΏ энСргии. Π­Ρ‚ΠΎ ΠΌΠΎΠΆΠ΅Ρ‚ Π±Ρ‹Ρ‚ΡŒ Π²Ρ€Π°Ρ‰Π΅Π½ΠΈΠ΅ лопастСй вСнтилятора, освСщСниС ΠΊΠΎΠΌΠ½Π°Ρ‚Ρ‹ ΠΈΠ»ΠΈ Π½Π°Π³Ρ€Π΅Π²Π°Π½ΠΈΠ΅ ΠΏΠΈΡ‰ΠΈ. Π‘ΠΊΠΎΡ€ΠΎΡΡ‚ΡŒ, с ΠΊΠΎΡ‚ΠΎΡ€ΠΎΠΉ эта Ρ€Π°Π±ΠΎΡ‚Π° выполняСтся зарядом Π² Ρ†Π΅ΠΏΠΈ, — это элСктричСская энСргия.ЭлСктроэнСргия — это Ρ‚Π°ΠΊΠΆΠ΅ ΡΠΊΠΎΡ€ΠΎΡΡ‚ΡŒ использования элСктричСской энСргии, ΡΠ»Π΅Π΄ΠΎΠ²Π°Ρ‚Π΅Π»ΡŒΠ½ΠΎ, ΠΌΠΎΡ‰Π½ΠΎΡΡ‚ΡŒ = энСргия / врСмя. ЭлСктричСская ΠΌΠΎΡ‰Π½ΠΎΡΡ‚ΡŒ, потрСбляСмая ΠΏΡ€ΠΈΠ±ΠΎΡ€ΠΎΠΌ, Ρ€Π°Π²Π½Π° P = I * V, Π³Π΄Π΅ P — элСктричСская ΠΌΠΎΡ‰Π½ΠΎΡΡ‚ΡŒ, I — Ρ‚ΠΎΠΊ Π² ΠΏΡ€ΠΈΠ±ΠΎΡ€Π΅ Π² Π°ΠΌΠΏΠ΅Ρ€Π°Ρ… [A], Π° V — напряТСниС ΠΏΡ€ΠΈΠ±ΠΎΡ€Π° Π² Π²ΠΎΠ»ΡŒΡ‚Π°Ρ… [V]. Π‘Π»Π΅Π΄ΠΎΠ²Π°Ρ‚Π΅Π»ΡŒΠ½ΠΎ, элСктричСская ΠΌΠΎΡ‰Π½ΠΎΡΡ‚ΡŒ выраТаСтся Π² Π²Π°Ρ‚Ρ‚Π°Ρ… (Π’Ρ‚), Π³Π΄Π΅ 1 Π’Ρ‚ = 1 А * Π’. Π‘Ρ‚ΠΎΠΈΠΌΠΎΡΡ‚ΡŒ элСктроэнСргии указываСтся Π² Ρ†Π΅Π½Ρ‚Π°Ρ… Π·Π° ΠΊΠΈΠ»ΠΎΠ²Π°Ρ‚Ρ‚-час (ΠΊΠ’Ρ‚Ρ‡), Π³Π΄Π΅ 1 ΠΊΠ’Ρ‚Ρ‡ = 1000 Π’Ρ‚Ρ‡ (Π’Π°Ρ‚Ρ‚-час). ΠšΠΈΠ»ΠΎΠ²Π°Ρ‚Ρ‚-час — это количСство элСктроэнСргии, потрСбляСмой Π² час ΠΈΠ· расчСта 1 ΠΊΠ’Ρ‚.Π Π°Π·Ρ€Π°Π±ΠΎΡ‚ΠΊΠ° устройств, эффСктивно ΠΏΠΎΡ‚Ρ€Π΅Π±Π»ΡΡŽΡ‰ΠΈΡ… ΡΠ»Π΅ΠΊΡ‚Ρ€ΠΎΡΠ½Π΅Ρ€Π³ΠΈΡŽ, являСтся Π²Π°ΠΆΠ½ΠΎΠΉ Π·Π°Π΄Π°Ρ‡Π΅ΠΉ для ΠΈΠ½ΠΆΠ΅Π½Π΅Ρ€ΠΎΠ², которая Π² ΠΊΠΎΠ½Π΅Ρ‡Π½ΠΎΠΌ ΠΈΡ‚ΠΎΠ³Π΅ ΠΏΠΎΠΌΠΎΠ³Π°Π΅Ρ‚ ΡƒΠ»ΡƒΡ‡ΡˆΠΈΡ‚ΡŒ общСство.

Π‘ΠΎΠΏΡƒΡ‚ΡΡ‚Π²ΡƒΡŽΡ‰ΠΈΠ΅ мСроприятия

Π—Π°ΠΊΡ€Ρ‹Ρ‚ΠΈΠ΅ ΡƒΡ€ΠΎΠΊΠ°

ΠŸΡ€Π΅Π΄Π»ΠΎΠΆΠΈΡ‚Π΅ учащимся ΠΏΡ€Π΅Π΄Π»ΠΎΠΆΠΈΡ‚ΡŒ ΠΏΡ€ΠΈΠΌΠ΅Ρ€Ρ‹ устройств, содСрТащих ΠΊΠΎΠΌΠΏΡŒΡŽΡ‚Π΅Ρ€Π½Ρ‹Π΅ микросхСмы; Π½Π°ΠΏΠΈΡˆΠΈΡ‚Π΅ названия ΠΏΡ€Π΅Π΄ΠΌΠ΅Ρ‚ΠΎΠ² Π½Π° доскС. (Π’ΠΎΠ·ΠΌΠΎΠΆΠ½Ρ‹Π΅ ΠΎΡ‚Π²Π΅Ρ‚Ρ‹: микроволновая ΠΏΠ΅Ρ‡ΡŒ, Π°Π²Ρ‚ΠΎΠΎΡ‚Π²Π΅Ρ‚Ρ‡ΠΈΠΊ, машина, DVD-ΠΏΠ»Π΅Π΅Ρ€ ΠΈ Ρ‚. Π”.Π—Π°Ρ‚Π΅ΠΌ нарисуйтС Π½Π° ΠΏΠ»Π°Ρ‚Π΅ схСму с нСсколькими ΠΊΠΎΠΌΠΏΠΎΠ½Π΅Π½Ρ‚Π°ΠΌΠΈ (ΠΏΡ€ΠΈΠΌΠ΅Ρ€ эскиза см. На рис. 3). ΠŸΠΎΠΏΡ€ΠΎΡΠΈΡ‚Π΅ класс ΠΎΠΏΡ€Π΅Π΄Π΅Π»ΠΈΡ‚ΡŒ, ΠΊΠ°ΠΊΠΈΠ΅ ΠΊΠΎΠΌΠΏΠΎΠ½Π΅Π½Ρ‚Ρ‹ схСмы ΠΏΠΎΠ΄ΠΊΠ»ΡŽΡ‡Π΅Π½Ρ‹ ΠΏΠΎΡΠ»Π΅Π΄ΠΎΠ²Π°Ρ‚Π΅Π»ΡŒΠ½ΠΎ, Π° ΠΊΠ°ΠΊΠΈΠ΅ — ΠΏΠ°Ρ€Π°Π»Π»Π΅Π»ΡŒΠ½ΠΎ.

Рис. 3. ΠŸΡ€ΠΈΠ½Ρ†ΠΈΠΏΠΈΠ°Π»ΡŒΠ½Π°Ρ схСма, состоящая ΠΈΠ· Π±Π°Ρ‚Π°Ρ€Π΅ΠΈ ΠΈ Ρ‚Ρ€Π΅Ρ… рСзисторов, Π΄Π΅ΠΌΠΎΠ½ΡΡ‚Ρ€ΠΈΡ€ΡƒΡŽΡ‰Π°Ρ ΠΊΠΎΠΌΠΏΠΎΠ½Π΅Π½Ρ‚Ρ‹ ΠΏΠΎΡΠ»Π΅Π΄ΠΎΠ²Π°Ρ‚Π΅Π»ΡŒΠ½ΠΎΠΉ ΠΈ ΠΏΠ°Ρ€Π°Π»Π»Π΅Π»ΡŒΠ½ΠΎΠΉ Ρ†Π΅ΠΏΠΈ. АвторскоС ΠΏΡ€Π°Π²ΠΎ

АвторскиС ΠΏΡ€Π°Π²Π° Β© 2012 ΠšΠ°Ρ€Π»ΠΈ Бамсон, УнивСрситСт ΠšΠΎΠ»ΠΎΡ€Π°Π΄ΠΎ Π² Π‘ΠΎΡƒΠ»Π΄Π΅Ρ€Π΅

Π—Π°Ρ‚Π΅ΠΌ нарисуйтС Π½Π° ΠΏΠ»Π°Ρ‚Π΅ ΠΏΡ€ΠΈΠ½Ρ†ΠΈΠΏΠΈΠ°Π»ΡŒΠ½ΡƒΡŽ схСму, ΠΊΠ°ΠΊ ΠΏΠΎΠΊΠ°Π·Π°Π½ΠΎ Π½Π° рисункС 4.Π˜ΡΠΏΠΎΠ»ΡŒΠ·ΡƒΠΉΡ‚Π΅ Π·Π°ΠΊΠΎΠ½ Ома (I = V / R), Ρ‡Ρ‚ΠΎΠ±Ρ‹ ΡΡ€Π°Π²Π½ΠΈΡ‚ΡŒ Ρ‚ΠΎΠΊ Π² Ρ‚Ρ€Π΅Ρ… Π»Π°ΠΌΠΏΠ°Ρ…, каТдая с ΡƒΠ²Π΅Π»ΠΈΡ‡ΠΈΠ²Π°ΡŽΡ‰ΠΈΠΌΡΡ сопротивлСниСм, ΠΏΠΎΠ΄ΠΊΠ»ΡŽΡ‡Π΅Π½Π½Ρ‹Ρ… ΠΏΠ°Ρ€Π°Π»Π»Π΅Π»ΡŒΠ½ΠΎ. (ΠžΡ‚Π²Π΅Ρ‚: см. РасчСты Π½Π° рис. 4. ΠœΠ°ΠΊΡΠΈΠΌΠ°Π»ΡŒΠ½Ρ‹ΠΉ Ρ‚ΠΎΠΊ Π² Π»Π°ΠΌΠΏΠΎΡ‡ΠΊΠ΅ с наимСньшим сопротивлСниСм ΠΈ наимСньший — Π² Π»Π°ΠΌΠΏΠ΅ с наибольшим сопротивлСниСм.) БпроситС, Ρ‡Ρ‚ΠΎ происходит с напряТСниСм ΠΏΡ€ΠΈ ΠΏΠ°Ρ€Π°Π»Π»Π΅Π»ΡŒΠ½ΠΎΠΌ ΠΏΠΎΠ΄ΠΊΠ»ΡŽΡ‡Π΅Π½ΠΈΠΈ Π±Π°Ρ‚Π°Ρ€Π΅ΠΉ? (ΠžΡ‚Π²Π΅Ρ‚: напряТСниС Π½Π° ΠΊΠ»Π΅ΠΌΠΌΠ°Ρ… остаСтся ΠΏΡ€Π΅ΠΆΠ½ΠΈΠΌ.)

Рис. 4. ΠŸΠ°Ρ€Π°Π»Π»Π΅Π»ΡŒΠ½Π°Ρ схСма, состоящая ΠΈΠ· Ρ‚Ρ€Π΅Ρ… Π»Π°ΠΌΠΏΠΎΡ‡Π΅ΠΊ с ΡƒΠ²Π΅Π»ΠΈΡ‡ΠΈΠ²Π°ΡŽΡ‰ΠΈΠΌΡΡ сопротивлСниСм (слСва), ΠΈ расчСт Π·Π°ΠΊΠΎΠ½Π° Ома для опрСдСлСния Ρ‚ΠΎΠΊΠ° ΠΊΠ°ΠΆΠ΄ΠΎΠΉ Π»Π°ΠΌΠΏΡ‹ (справа).Copyright

Copyright Β© Π”Π°Ρ€ΡŒΡ ΠšΠΎΡ‚ΠΈΡ-Π¨Π²Π°Ρ€Ρ†, ΠŸΡ€ΠΎΠ³Ρ€Π°ΠΌΠΌΠ° ΠΈ лаборатория ITL, УнивСрситСт ΠšΠΎΠ»ΠΎΡ€Π°Π΄ΠΎ Π² Π‘ΠΎΡƒΠ»Π΄Π΅Ρ€Π΅, 2004.

Π‘Π»ΠΎΠ²Π°Ρ€ΡŒ / ΠžΠΏΡ€Π΅Π΄Π΅Π»Π΅Π½ΠΈΡ

Π˜Π½Ρ‚Π΅Π³Ρ€Π°Π»ΡŒΠ½Π°Ρ схСма: микроэлСктронная схСма, вытравлСнная ΠΈΠ»ΠΈ отпСчатанная Π½Π° ΠΏΠΎΠ»ΡƒΠΏΡ€ΠΎΠ²ΠΎΠ΄Π½ΠΈΠΊΠΎΠ²ΠΎΠΌ кристаллС.

ΠΏΠ°Ρ€Π°Π»Π»Π΅Π»ΡŒΠ½Π°Ρ Ρ†Π΅ΠΏΡŒ: элСктричСская Ρ†Π΅ΠΏΡŒ, ΠΎΠ±Π΅ΡΠΏΠ΅Ρ‡ΠΈΠ²Π°ΡŽΡ‰Π°Ρ Π±ΠΎΠ»Π΅Π΅ ΠΎΠ΄Π½ΠΎΠ³ΠΎ проводящСго ΠΏΡƒΡ‚ΠΈ.

ΠžΡ†Π΅Π½ΠΊΠ°

ΠžΡ†Π΅Π½ΠΊΠ° ΠΏΠ΅Ρ€Π΅Π΄ ΡƒΡ€ΠΎΠΊΠΎΠΌ

Вопросы для обсуТдСния: Π—Π°Π΄Π°ΠΉΡ‚Π΅ ΡƒΡ‡Π΅Π½ΠΈΠΊΠ°ΠΌ ΠΈ обсудитС Π² классС:

  • К ΠΊΠ°ΠΊΠΎΠΉ схСмС Π²Ρ‹ Ρ…ΠΎΡ‚ΠΈΡ‚Π΅ ΠΏΠΎΠ΄ΠΊΠ»ΡŽΡ‡ΠΈΡ‚ΡŒ свой Π΄ΠΎΠΌ ΠΈΠ»ΠΈ Π²ΠΈΠ΄Π΅ΠΎΠΈΠ³Ρ€Ρƒ ΠΈ ΠΏΠΎΡ‡Π΅ΠΌΡƒ? (ΠžΡ‚Π²Π΅Ρ‚: Π‘Ρ‚ΡƒΠ΄Π΅Π½Ρ‚Ρ‹, вСроятно, вспомнят ΡƒΡ€ΠΎΠΊ ΠΎ ΠΏΠΎΡΠ»Π΅Π΄ΠΎΠ²Π°Ρ‚Π΅Π»ΡŒΠ½ΠΎΠΉ схСмС ΠΈ ΠΎΠ±ΡŠΡΡΠ½ΡΡ‚, ΠΊΠ°ΠΊ Ρ€Π°Π±ΠΎΡ‚Π°Π΅Ρ‚ этот Ρ‚ΠΈΠΏ схСмы.ΠžΠ±ΡΡƒΠ΄ΠΈΡ‚Π΅ ΠΏΠ»ΡŽΡΡ‹ ΠΈ минусы ΠΏΠΎΡΠ»Π΅Π΄ΠΎΠ²Π°Ρ‚Π΅Π»ΡŒΠ½Ρ‹Ρ… схСм.)
  • Если Π²Ρ‹ ΡƒΠ΄Π°Π»ΠΈΡ‚Π΅ ΠΎΠ΄Π½Ρƒ Π»Π°ΠΌΠΏΠΎΡ‡ΠΊΡƒ ΠΈΠ· ΠΏΠΎΡΠ»Π΅Π΄ΠΎΠ²Π°Ρ‚Π΅Π»ΡŒΠ½ΠΎΠΉ Ρ†Π΅ΠΏΠΈ с трСмя Π»Π°ΠΌΠΏΠΎΡ‡ΠΊΠ°ΠΌΠΈ, Ρ†Π΅ΠΏΡŒ Π±ΡƒΠ΄Π΅Ρ‚ (n) ____________ Ρ†Π΅ΠΏΡŒΡŽ. ΠžΡ‚ΠΊΡ€Ρ‹Ρ‚Ρ‹ΠΉ ΠΈΠ»ΠΈ Π·Π°ΠΊΡ€Ρ‹Ρ‚Ρ‹ΠΉ? (ΠžΡ‚Π²Π΅Ρ‚: ΠžΡ‚ΠΊΡ€Ρ‹Ρ‚Ρ‹ΠΉ.)
  • Π§Ρ‚ΠΎ ΠΏΡ€ΠΎΠΈΠ·ΠΎΠΉΠ΄Π΅Ρ‚ с Π΄Ρ€ΡƒΠ³ΠΈΠΌΠΈ Π»Π°ΠΌΠΏΠΎΡ‡ΠΊΠ°ΠΌΠΈ Π² ΠΏΠΎΡΠ»Π΅Π΄ΠΎΠ²Π°Ρ‚Π΅Π»ΡŒΠ½ΠΎΠΉ Ρ†Π΅ΠΏΠΈ, Ссли ΠΎΠ΄Π½Π° Π»Π°ΠΌΠΏΠΎΡ‡ΠΊΠ° ΠΏΠ΅Ρ€Π΅Π³ΠΎΡ€ΠΈΡ‚? (ΠžΡ‚Π²Π΅Ρ‚: ВсС гаснут.)
  • ΠŸΡ€ΠΈ ΠΏΠΎΡΠ»Π΅Π΄ΠΎΠ²Π°Ρ‚Π΅Π»ΡŒΠ½ΠΎΠΌ соСдинСнии аккумуляторов напряТСниС Π½Π° Π½ΠΈΡ… ____________. УвСличиваСтся, ΡƒΠΌΠ΅Π½ΡŒΡˆΠ°Π΅Ρ‚ΡΡ ΠΈΠ»ΠΈ остаСтся Π½Π΅ΠΈΠ·ΠΌΠ΅Π½Π½Ρ‹ΠΌ? (ΠžΡ‚Π²Π΅Ρ‚: УвСличиваСтся Π΄ΠΎ ΠΎΠ±Ρ‰Π΅Π³ΠΎ суммарного значСния напряТСния аккумуляторной Π±Π°Ρ‚Π°Ρ€Π΅ΠΈ.)

ΠžΡ†Π΅Π½ΠΊΠ° послС ввСдСния

Вопрос / ΠΎΡ‚Π²Π΅Ρ‚: Π—Π°Π΄Π°ΠΉΡ‚Π΅ студСнтам вопросы ΠΈ попроситС ΠΈΡ… ΠΏΠΎΠ΄Π½ΡΡ‚ΡŒ Ρ€ΡƒΠΊΠΈ, Ρ‡Ρ‚ΠΎΠ±Ρ‹ ΠΎΡ‚Π²Π΅Ρ‚ΠΈΡ‚ΡŒ. ΠΠ°ΠΏΠΈΡˆΠΈΡ‚Π΅ ΠΎΡ‚Π²Π΅Ρ‚Ρ‹ Π½Π° доскС

  • Как это Π²ΠΎΠ·ΠΌΠΎΠΆΠ½ΠΎ, Ρ‡Ρ‚ΠΎ Π²Ρ‹ ΠΌΠΎΠΆΠ΅Ρ‚Π΅ Π²ΠΊΠ»ΡŽΡ‡ΠΈΡ‚ΡŒ ΠΎΠ΄ΠΈΠ½ свСт Π² ΠΊΠΎΠΌΠ½Π°Ρ‚Π΅, ΠΈ ΠΎΠ½ Π±ΡƒΠ΄Π΅Ρ‚ Ρ€Π°Π±ΠΎΡ‚Π°Ρ‚ΡŒ, Π±Π΅Π· нСобходимости Π²ΠΊΠ»ΡŽΡ‡Π°Ρ‚ΡŒ всС ΠΎΡΡ‚Π°Π»ΡŒΠ½Ρ‹Π΅ ΡΠ²Π΅Ρ‚ΠΈΠ»ΡŒΠ½ΠΈΠΊΠΈ? (ΠžΡ‚Π²Π΅Ρ‚: Π­Π»Π΅ΠΊΡ‚Ρ€ΠΎΠΏΡ€ΠΎΠ²ΠΎΠ΄ΠΊΠ° Π² Π΄ΠΎΠΌΠ΅ — ΠΏΠ°Ρ€Π°Π»Π»Π΅Π»ΡŒΠ½Π°Ρ.)
  • Как называСтся ΠΎΡ‡Π΅Π½ΡŒ слоТная схСма, ΠΎΠ±ΡŠΠ΅Π΄ΠΈΠ½ΡΡŽΡ‰Π°Ρ тысячи ΠΈ ΠΌΠΈΠ»Π»ΠΈΠΎΠ½Ρ‹ ΠΏΠ°Ρ€Π°Π»Π»Π΅Π»ΡŒΠ½Ρ‹Ρ… ΠΈ ΠΏΠΎΡΠ»Π΅Π΄ΠΎΠ²Π°Ρ‚Π΅Π»ΡŒΠ½Ρ‹Ρ… Ρ†Π΅ΠΏΠ΅ΠΉ, Ρ€Π°Π±ΠΎΡ‚Π°ΡŽΡ‰ΠΈΡ… вмСстС? (ΠžΡ‚Π²Π΅Ρ‚: ΠΈΠ½Ρ‚Π΅Π³Ρ€Π°Π»ΡŒΠ½Π°Ρ схСма ΠΈΠ»ΠΈ микропроцСссор.)

Π˜Ρ‚ΠΎΠ³ΠΈ ΡƒΡ€ΠΎΠΊΠ° ΠžΡ†Π΅Π½ΠΊΠ°

ΠΏΡ€ΠΎΠ½ΡƒΠΌΠ΅Ρ€ΠΎΠ²Π°Π½Π½Ρ‹Ρ… Ρ€ΡƒΠΊΠΎΠ²ΠΎΠ΄ΠΈΡ‚Π΅Π»Π΅ΠΉ: Π Π°Π·Π΄Π΅Π»ΠΈΡ‚Π΅ класс Π½Π° ΠΊΠΎΠΌΠ°Π½Π΄Ρ‹ ΠΎΡ‚ Ρ‚Ρ€Π΅Ρ… Π΄ΠΎ пяти Ρ‡Π΅Π»ΠΎΠ²Π΅ΠΊ. ΠŸΠΎΠΏΡ€ΠΎΡΠΈΡ‚Π΅ ΡƒΡ‡Π΅Π½ΠΈΠΊΠΎΠ² Π² ΠΊΠ°ΠΆΠ΄ΠΎΠΉ ΠΊΠΎΠΌΠ°Π½Π΄Π΅ Π½ΡƒΠΌΠ΅Ρ€ΠΎΠ²Π°Ρ‚ΡŒ, Ρ‡Ρ‚ΠΎΠ±Ρ‹ Ρƒ ΠΊΠ°ΠΆΠ΄ΠΎΠ³ΠΎ Ρ‡Π»Π΅Π½Π° Π±Ρ‹Π» свой Π½ΠΎΠΌΠ΅Ρ€. Π—Π°Π΄Π°ΠΉΡ‚Π΅ студСнтам вопросы (ΠΏΡ€ΠΈ ΠΆΠ΅Π»Π°Π½ΠΈΠΈ Π΄Π°ΠΉΡ‚Π΅ ΠΈΠΌ Π²Ρ€Π΅ΠΌΠ΅Π½Π½Ρ‹Π΅ Ρ€Π°ΠΌΠΊΠΈ для Ρ€Π΅ΡˆΠ΅Π½ΠΈΡ ΠΊΠ°ΠΆΠ΄ΠΎΠ³ΠΎ ΠΈΠ· Π½ΠΈΡ…). Π§Π»Π΅Π½Ρ‹ ΠΊΠ°ΠΆΠ΄ΠΎΠΉ ΠΊΠΎΠΌΠ°Π½Π΄Ρ‹ Π΄ΠΎΠ»ΠΆΠ½Ρ‹ Ρ€Π°Π±ΠΎΡ‚Π°Ρ‚ΡŒ вмСстС, Ρ‡Ρ‚ΠΎΠ±Ρ‹ ΠΎΡ‚Π²Π΅Ρ‚ΠΈΡ‚ΡŒ Π½Π° вопросы. ВсС Π² ΠΊΠΎΠΌΠ°Π½Π΄Π΅ Π΄ΠΎΠ»ΠΆΠ½Ρ‹ Π·Π½Π°Ρ‚ΡŒ ΠΎΡ‚Π²Π΅Ρ‚. НабСритС ΠΏΡ€ΠΎΠΈΠ·Π²ΠΎΠ»ΡŒΠ½Ρ‹ΠΉ Π½ΠΎΠΌΠ΅Ρ€. Π‘Ρ‚ΡƒΠ΄Π΅Π½Ρ‚Ρ‹ с этим Π½ΠΎΠΌΠ΅Ρ€ΠΎΠΌ Π΄ΠΎΠ»ΠΆΠ½Ρ‹ ΠΏΠΎΠ΄Π½ΡΡ‚ΡŒ Ρ€ΡƒΠΊΠΈ, Ρ‡Ρ‚ΠΎΠ±Ρ‹ Π΄Π°Ρ‚ΡŒ ΠΎΡ‚Π²Π΅Ρ‚.Если Π½Π΅ всС учащиСся с этим Π½ΠΎΠΌΠ΅Ρ€ΠΎΠΌ ΠΏΠΎΠ΄Π½ΠΈΠΌΠ°ΡŽΡ‚ Ρ€ΡƒΠΊΠΈ, Π΄Π°ΠΉΡ‚Π΅ ΠΊΠΎΠΌΠ°Π½Π΄Π°ΠΌ ΠΏΠΎΡ€Π°Π±ΠΎΡ‚Π°Ρ‚ΡŒ Π΅Ρ‰Π΅ Π½Π΅ΠΌΠ½ΠΎΠ³ΠΎ. БпроситС Ρƒ студСнтов:

  • Если Π²Ρ‹ ΡƒΠ΄Π°Π»ΠΈΡ‚Π΅ ΠΎΠ΄Π½Ρƒ Π»Π°ΠΌΠΏΠΎΡ‡ΠΊΡƒ ΠΈΠ· ΠΏΠ°Ρ€Π°Π»Π»Π΅Π»ΡŒΠ½ΠΎΠΉ Ρ†Π΅ΠΏΠΈ с трСмя Π»Π°ΠΌΠΏΠ°ΠΌΠΈ, Π²ΠΊΠ»ΡŽΡ‡Π΅Π½Π½Ρ‹ΠΌΠΈ ΠΏΠ°Ρ€Π°Π»Π»Π΅Π»ΡŒΠ½ΠΎ, Ρ†Π΅ΠΏΡŒ станСт (n) ____________ Ρ†Π΅ΠΏΡŒΡŽ. ΠžΡ‚ΠΊΡ€Ρ‹Ρ‚Ρ‹ΠΉ ΠΈΠ»ΠΈ Π·Π°ΠΊΡ€Ρ‹Ρ‚Ρ‹ΠΉ? (ΠžΡ‚Π²Π΅Ρ‚: Π·Π°ΠΊΡ€Ρ‹Ρ‚ΠΎ.)
  • Π§Ρ‚ΠΎ ΠΏΡ€ΠΎΠΈΠ·ΠΎΠΉΠ΄Π΅Ρ‚ с Π΄Ρ€ΡƒΠ³ΠΈΠΌΠΈ Π»Π°ΠΌΠΏΠ°ΠΌΠΈ Π² ΠΏΠ°Ρ€Π°Π»Π»Π΅Π»ΡŒΠ½ΠΎΠΉ Ρ†Π΅ΠΏΠΈ, Ссли ΠΎΠ΄Π½Π° Π»Π°ΠΌΠΏΠΎΡ‡ΠΊΠ° ΠΏΠ΅Ρ€Π΅Π³ΠΎΡ€ΠΈΡ‚? (ΠžΡ‚Π²Π΅Ρ‚: горят.)
  • ΠŸΡ€ΠΈ ΠΏΠ°Ρ€Π°Π»Π»Π΅Π»ΡŒΠ½ΠΎΠΌ ΠΏΠΎΠ΄ΠΊΠ»ΡŽΡ‡Π΅Π½ΠΈΠΈ Π»Π°ΠΌΠΏΠΎΡ‡Π΅ΠΊ ΠΎΠ±Ρ‰Π΅Π΅ сопротивлСниС Ρ€Π°Π²Π½ΠΎ ____________ ΡΠΎΠΏΡ€ΠΎΡ‚ΠΈΠ²Π»Π΅Π½ΠΈΡŽ ΠΎΠ΄Π½ΠΎΠΉ Π»Π°ΠΌΠΏΠΎΡ‡ΠΊΠΈ.МСньшС, большС ΠΈΠ»ΠΈ Ρ‚Π°ΠΊΠΎΠ΅ ΠΆΠ΅, ΠΊΠ°ΠΊ? (ΠžΡ‚Π²Π΅Ρ‚: МСнСС.)
  • ΠŸΡ€ΠΈ ΠΏΠ°Ρ€Π°Π»Π»Π΅Π»ΡŒΠ½ΠΎΠΌ ΠΏΠΎΠ΄ΠΊΠ»ΡŽΡ‡Π΅Π½ΠΈΠΈ аккумуляторов напряТСниС Π½Π° Π½ΠΈΡ… ____________. УвСличиваСтся, ΡƒΠΌΠ΅Π½ΡŒΡˆΠ°Π΅Ρ‚ΡΡ ΠΈΠ»ΠΈ остаСтся Π½Π΅ΠΈΠ·ΠΌΠ΅Π½Π½Ρ‹ΠΌ? (ΠžΡ‚Π²Π΅Ρ‚: остаСтся ΠΏΡ€Π΅ΠΆΠ½ΠΈΠΌ.)
  • НарисуйтС ΠΏΡ€ΠΈΠ½Ρ†ΠΈΠΏΠΈΠ°Π»ΡŒΠ½ΡƒΡŽ схСму ΠΏΠ°Ρ€Π°Π»Π»Π΅Π»ΡŒΠ½ΠΎΠΉ Ρ†Π΅ΠΏΠΈ с двумя батарСями, Π²ΠΊΠ»ΡŽΡ‡Π΅Π½Π½Ρ‹ΠΌΠΈ ΠΏΠ°Ρ€Π°Π»Π»Π΅Π»ΡŒΠ½ΠΎ, ΠΈ двумя Π»Π°ΠΌΠΏΠΎΡ‡ΠΊΠ°ΠΌΠΈ, Π²ΠΊΠ»ΡŽΡ‡Π΅Π½Π½Ρ‹ΠΌΠΈ ΠΏΠ°Ρ€Π°Π»Π»Π΅Π»ΡŒΠ½ΠΎ.

Рисунок Race: ΠΠ°ΠΏΠΈΡˆΠΈΡ‚Π΅ символы схСм Π½Π° ΠΏΠ»Π°Ρ‚Π΅ (см. Рисунок 5). Π Π°Π·Π΄Π΅Π»ΠΈΡ‚Π΅ класс Π½Π° ΠΊΠΎΠΌΠ°Π½Π΄Ρ‹ ΠΏΠΎ Ρ‡Π΅Ρ‚Ρ‹Ρ€Π΅ Ρ‡Π΅Π»ΠΎΠ²Π΅ΠΊΠ° Ρ‚Π°ΠΊ, Ρ‡Ρ‚ΠΎΠ±Ρ‹ Ρƒ ΠΊΠ°ΠΆΠ΄ΠΎΠ³ΠΎ Ρ‡Π»Π΅Π½Π° ΠΊΠΎΠΌΠ°Π½Π΄Ρ‹ Π±Ρ‹Π» Π΄Ρ€ΡƒΠ³ΠΎΠΉ Π½ΠΎΠΌΠ΅Ρ€, ΠΎΡ‚ ΠΎΠ΄Π½ΠΎΠ³ΠΎ Π΄ΠΎ Ρ‡Π΅Ρ‚Ρ‹Ρ€Π΅Ρ….ΠŸΠΎΠ·Π²ΠΎΠ½ΠΈΡ‚Π΅ ΠΏΠΎ Π½ΠΎΠΌΠ΅Ρ€Ρƒ ΠΈ попроситС учащихся с этим Π½ΠΎΠΌΠ΅Ρ€ΠΎΠΌ ΠΏΠΎΡΠΏΠ΅ΡˆΠΈΡ‚ΡŒ ΠΊ доскС, Ρ‡Ρ‚ΠΎΠ±Ρ‹ Π½Π°Ρ€ΠΈΡΠΎΠ²Π°Ρ‚ΡŒ ΠΏΡ€Π°Π²ΠΈΠ»ΡŒΠ½ΡƒΡŽ ΠΏΡ€ΠΈΠ½Ρ†ΠΈΠΏΠΈΠ°Π»ΡŒΠ½ΡƒΡŽ схСму. Π”Π°ΠΉΡ‚Π΅ ΠΎΡ‡ΠΊΠΎ ΠΊΠΎΠΌΠ°Π½Π΄Π΅, Ρ‡Π΅ΠΉ Ρ‚ΠΎΠ²Π°Ρ€ΠΈΡ‰ ΠΏΠΎ ΠΊΠΎΠΌΠ°Π½Π΄Π΅ ΠΏΠ΅Ρ€Π²Ρ‹ΠΌ Π·Π°ΠΊΠΎΠ½Ρ‡ΠΈΡ‚ Ρ€ΠΎΠ·Ρ‹Π³Ρ€Ρ‹Ρˆ. ΠŸΠΎΠΏΡ€ΠΎΡΠΈΡ‚Π΅ учащихся Π½Π°Ρ€ΠΈΡΠΎΠ²Π°Ρ‚ΡŒ ΠΏΡ€ΠΈΠ½Ρ†ΠΈΠΏΠΈΠ°Π»ΡŒΠ½Ρ‹Π΅ схСмы ΡΠ»Π΅Π΄ΡƒΡŽΡ‰Π΅Π³ΠΎ:

  • ЦСпь с ΠΎΠ΄Π½ΠΎΠΉ Π±Π°Ρ‚Π°Ρ€Π΅Π΅ΠΉ ΠΈ двумя ΠΏΠ°Ρ€Π°Π»Π»Π΅Π»ΡŒΠ½Ρ‹ΠΌΠΈ Π»Π°ΠΌΠΏΠΎΡ‡ΠΊΠ°ΠΌΠΈ.
  • ЦСпь с трСмя ΠΏΠ°Ρ€Π°Π»Π»Π΅Π»ΡŒΠ½ΠΎ Π²ΠΊΠ»ΡŽΡ‡Π΅Π½Π½Ρ‹ΠΌΠΈ батарСями ΠΈ двумя ΠΏΠ°Ρ€Π°Π»Π»Π΅Π»ΡŒΠ½Ρ‹ΠΌΠΈ Π»Π°ΠΌΠΏΠΎΡ‡ΠΊΠ°ΠΌΠΈ.
  • ЦСпь с двумя батарСями, Π²ΠΊΠ»ΡŽΡ‡Π΅Π½Π½Ρ‹ΠΌΠΈ ΠΏΠ°Ρ€Π°Π»Π»Π΅Π»ΡŒΠ½ΠΎ, ΠΎΠ΄Π½ΠΈΠΌ рСзистором ΠΈ ΠΎΠ΄Π½ΠΎΠΉ Π»Π°ΠΌΠΏΠΎΡ‡ΠΊΠΎΠΉ.
  • ЦСпь с ΠΎΠ΄Π½ΠΎΠΉ Π±Π°Ρ‚Π°Ρ€Π΅Π΅ΠΉ, ΠΎΠ΄Π½ΠΈΠΌ ΠΏΠ΅Ρ€Π΅ΠΊΠ»ΡŽΡ‡Π°Ρ‚Π΅Π»Π΅ΠΌ ΠΈ трСмя ΠΏΠ°Ρ€Π°Π»Π»Π΅Π»ΡŒΠ½Ρ‹ΠΌΠΈ Π»Π°ΠΌΠΏΠΎΡ‡ΠΊΠ°ΠΌΠΈ.
  • Π‘Ρ…Π΅ΠΌΠ° с ΠΎΠ΄Π½ΠΎΠΉ Π±Π°Ρ‚Π°Ρ€Π΅Π΅ΠΉ, ΠΎΠ΄Π½ΠΈΠΌ ΠΏΠ΅Ρ€Π΅ΠΊΠ»ΡŽΡ‡Π°Ρ‚Π΅Π»Π΅ΠΌ ΠΈ двумя ΠΏΠ°Ρ€Π°Π»Π»Π΅Π»ΡŒΠ½Ρ‹ΠΌΠΈ рСзисторами.
  • ЦСпь с ΠΎΠ΄Π½ΠΎΠΉ Π±Π°Ρ‚Π°Ρ€Π΅Π΅ΠΉ, ΠΎΠ΄Π½ΠΈΠΌ ΠΏΠ΅Ρ€Π΅ΠΊΠ»ΡŽΡ‡Π°Ρ‚Π΅Π»Π΅ΠΌ, ΠΎΠ΄Π½ΠΎΠΉ Π»Π°ΠΌΠΏΠΎΡ‡ΠΊΠΎΠΉ ΠΈ рСзистором, Π²ΠΊΠ»ΡŽΡ‡Π΅Π½Π½Ρ‹ΠΌΠΈ ΠΏΠ°Ρ€Π°Π»Π»Π΅Π»ΡŒΠ½ΠΎ.
  • ЦСпь с двумя батарСями, Π²ΠΊΠ»ΡŽΡ‡Π΅Π½Π½Ρ‹ΠΌΠΈ ΠΏΠ°Ρ€Π°Π»Π»Π΅Π»ΡŒΠ½ΠΎ, ΠΈ ΠΎΠ΄Π½ΠΎΠΉ Π»Π°ΠΌΠΏΠΎΡ‡ΠΊΠΎΠΉ, Π²ΠΊΠ»ΡŽΡ‡Π΅Π½Π½ΠΎΠΉ ΠΏΠ°Ρ€Π°Π»Π»Π΅Π»ΡŒΠ½ΠΎ, с Π»Π°ΠΌΠΏΠΎΡ‡ΠΊΠΎΠΉ ΠΈ рСзистором.

Рис. 5. Π’Ρ‹Π±ΠΎΡ€ графичСских символов ΠΏΡ€ΠΈΠ½Ρ†ΠΈΠΏΠΈΠ°Π»ΡŒΠ½ΠΎΠΉ схСмы. АвторскоС ΠΏΡ€Π°Π²ΠΎ

Copyright Β© Π”Π°Ρ€ΡŒΡ ΠšΠΎΡ‚ΠΈΡ-Π¨Π²Π°Ρ€Ρ†, Лаборатория ITL, УнивСрситСт ΠšΠΎΠ»ΠΎΡ€Π°Π΄ΠΎ Π² Π‘ΠΎΡƒΠ»Π΄Π΅Ρ€Π΅, 2004.

ΠŸΡ€Π΅Π·Π΅Π½Ρ‚Π°Ρ†ΠΈΡ Π² классС: Работая Π² Π³Ρ€ΡƒΠΏΠΏΠ°Ρ… ΠΎΡ‚ Π΄Π²ΡƒΡ… Π΄ΠΎ Ρ‡Π΅Ρ‚Ρ‹Ρ€Π΅Ρ… Ρ‡Π΅Π»ΠΎΠ²Π΅ΠΊ, попроситС учащихся ΠΏΡ€Π΅Π΄ΡΡ‚Π°Π²ΠΈΡ‚ΡŒ ΠΊΠ»Π°ΡΡΠ½ΡƒΡŽ ΠΏΡ€Π΅Π·Π΅Π½Ρ‚Π°Ρ†ΠΈΡŽ, Π² ΠΊΠΎΡ‚ΠΎΡ€ΠΎΠΉ ΠΎΠ½ΠΈ динамичСски Ρ€Π°Π·Ρ‹Π³Ρ€Ρ‹Π²Π°ΡŽΡ‚ ΠΊΠΎΠ½Ρ†Π΅ΠΏΡ†ΠΈΠΈ, ΠΊΠΎΡ‚ΠΎΡ€Ρ‹Π΅ ΠΎΠ½ΠΈ ΠΈΠ·ΡƒΡ‡ΠΈΠ»ΠΈ Π² этом ΠΌΠΎΠ΄ΡƒΠ»Π΅. ΠŸΠΎΠΎΡ‰Ρ€ΡΠΉΡ‚Π΅ Ρ€ΠΎΠ»Π΅Π²Ρ‹Π΅ ΠΈΠ³Ρ€Ρ‹ ΠΈ творчСство.

  • ΠŸΡ€Π΅Π΄Π»ΠΎΠΆΠΈΡ‚Π΅ ΡƒΡ‡Π΅Π½ΠΈΠΊΠ°ΠΌ Ρ€Π°Π·Ρ‹Π³Ρ€Π°Ρ‚ΡŒ сцСнарий ΠΈΠ½ΠΆΠ΅Π½Π΅Ρ€Π°-элСктрика, ΠΊΠΎΡ‚ΠΎΡ€Ρ‹ΠΉ Ρ‚ΠΎΠ»ΡŒΠΊΠΎ Ρ‡Ρ‚ΠΎ ΠΈΠ·ΠΎΠ±Ρ€Π΅Π» Π½ΠΎΠ²ΡƒΡŽ ΠΈΠ³Ρ€ΡƒΡˆΠΊΡƒ, ΠΈΡΠΏΠΎΠ»ΡŒΠ·ΡƒΡ ΠΏΠΎΡΠ»Π΅Π΄ΠΎΠ²Π°Ρ‚Π΅Π»ΡŒΠ½Ρ‹Π΅ ΠΈΠ»ΠΈ ΠΏΠ°Ρ€Π°Π»Π»Π΅Π»ΡŒΠ½Ρ‹Π΅ (ΠΈΠ»ΠΈ ΠΊΠΎΠΌΠ±ΠΈΠ½Π°Ρ†ΠΈΡŽ ΠΎΠ±ΠΎΠΈΡ…) Ρ†Π΅ΠΏΠ΅ΠΉ. Π”Ρ€ΡƒΠ³ΠΈΠΌΠΈ ΠΈΠ³Ρ€ΠΎΠΊΠ°ΠΌΠΈ ΠΌΠΎΠ³ΡƒΡ‚ Π±Ρ‹Ρ‚ΡŒ ΠΏΠΎΡ‚Ρ€Π΅Π±ΠΈΡ‚Π΅Π»ΠΈ, ΠΏΠ°Ρ‚Π΅Π½Ρ‚Π½Ρ‹Π΅ слуТащиС, сосСди, Π΄Ρ€ΡƒΠ³ΠΈΠ΅ ΠΈΠ½ΠΆΠ΅Π½Π΅Ρ€Ρ‹ ΠΈ Ρ‚. Π”.ΠšΠ°ΠΆΠ΄Ρ‹ΠΉ сцСнарий Π΄ΠΎΠ»ΠΆΠ΅Π½ Π²ΠΊΠ»ΡŽΡ‡Π°Ρ‚ΡŒ описаниС схСмы ΠΈ Ρ‚ΠΎΠ³ΠΎ, ΠΊΠ°ΠΊ ΠΎΠ½Π° Ρ€Π°Π±ΠΎΡ‚Π°Π΅Ρ‚, Π° Ρ‚Π°ΠΊΠΆΠ΅ рисунок схСмы Π½Π° ΠΏΠ»Π°Ρ‚Π΅.

ΠœΠ΅Ρ€ΠΎΠΏΡ€ΠΈΡΡ‚ΠΈΡ ΠΏΠΎ ΠΏΡ€ΠΎΠ΄Π»Π΅Π½ΠΈΡŽ ΡƒΡ€ΠΎΠΊΠ°

ΠŸΡ€Π΅Π΄Π»ΠΎΠΆΠΈΡ‚Π΅ студСнтам ΠΈΠ·ΡƒΡ‡ΠΈΡ‚ΡŒ ΠΈΡΡ‚ΠΎΡ€ΠΈΡŽ ΠΊΠΎΠΌΠΏΡŒΡŽΡ‚Π΅Ρ€Π½ΠΎΠΉ индустрии ΠΈ ΠΈΠ½Ρ‚Π΅Π³Ρ€Π°Π»ΡŒΠ½Ρ‹Ρ… схСм. Они ΠΌΠΎΠ³ΡƒΡ‚ ΠΏΠΎΠ΄Π³ΠΎΡ‚ΠΎΠ²ΠΈΡ‚ΡŒ ΠΏΠ»Π°ΠΊΠ°Ρ‚Ρ‹ ΠΈ ΠΏΡ€Π΅Π·Π΅Π½Ρ‚Π°Ρ†ΠΈΠΈ ΠΎ ΠΊΠ»ΡŽΡ‡Π΅Π²Ρ‹Ρ… изобрСтСниях, Π° Ρ‚Π°ΠΊΠΆΠ΅ ΠΎΠ± ΠΈΠ½ΠΆΠ΅Π½Π΅Ρ€Π°Ρ… ΠΈ исслСдоватСлях, ΡΡ‹Π³Ρ€Π°Π²ΡˆΠΈΡ… Π²Π°ΠΆΠ½ΡƒΡŽ Ρ€ΠΎΠ»ΡŒ Π² Ρ€Π°Π·Ρ€Π°Π±ΠΎΡ‚ΠΊΠ΅ ΠΌΠΈΠΊΡ€ΠΎΡ‡ΠΈΠΏΠΎΠ² ΠΈ микропроцСссоров.

ΠœΠΈΠΊΡ€ΠΎΡ‡ΠΈΠΏΡ‹

всС Ρ‡Π°Ρ‰Π΅ ΠΈΡΠΏΠΎΠ»ΡŒΠ·ΡƒΡŽΡ‚ΡΡ Π² устройствах, Π½Π°ΠΏΡ€ΠΈΠΌΠ΅Ρ€, Π² ΡƒΡ‚ΡŽΠΆΠΊΠ°Ρ… для ΠΎΠ΄Π΅ΠΆΠ΄Ρ‹, ΠΊΠΎΡ‚ΠΎΡ€Ρ‹Π΅ автоматичСски ΠΎΡ‚ΠΊΠ»ΡŽΡ‡Π°ΡŽΡ‚ΡΡ, ΠΈ Π² тостСрах, ΠΎΠ±Π½Π°Ρ€ΡƒΠΆΠΈΠ²Π°ΡŽΡ‰ΠΈΡ… идСально подрумянСнныС тосты. ΠŸΠΎΠΏΡ€ΠΎΡΠΈΡ‚Π΅ учащихся всякиС Π±Ρ‹Ρ‚ΠΎΠ²Ρ‹Π΅ ΠΏΡ€ΠΈΠ±ΠΎΡ€Ρ‹, ΠΊΠΎΡ‚ΠΎΡ€Ρ‹Π΅ ΠΎΠ½ΠΈ ΠΌΠΎΠ³ΡƒΡ‚ ΠΏΡ€ΠΈΠ΄ΡƒΠΌΠ°Ρ‚ΡŒ, с ΠΌΠΈΠΊΡ€ΠΎΡ‡ΠΈΠΏΠΎΠΌ. ΠœΠΈΠΊΡ€ΠΎΡ‡ΠΈΠΏΡ‹ ΠΈΡΠΏΠΎΠ»ΡŒΠ·ΡƒΡŽΡ‚ΡΡ Π² посудомоСчных ΠΌΠ°ΡˆΠΈΠ½Π°Ρ…, ΡΡ‚ΠΈΡ€Π°Π»ΡŒΠ½Ρ‹Ρ… ΠΈ ΡΡƒΡˆΠΈΠ»ΡŒΠ½Ρ‹Ρ… ΠΌΠ°ΡˆΠΈΠ½Π°Ρ…, Ρ‚Π΅Π»Π΅Π²ΠΈΠ·ΠΎΡ€Π°Ρ…, ΠΌΠΈΠΊΡ€ΠΎΠ²ΠΎΠ»Π½ΠΎΠ²Ρ‹Ρ… ΠΏΠ΅Ρ‡Π°Ρ…, автомобилях, Π²ΠΈΠ΄Π΅ΠΎΠΌΠ°Π³Π½ΠΈΡ‚ΠΎΡ„ΠΎΠ½Π°Ρ…, DVD-ΠΏΠ»Π΅Π΅Ρ€Π°Ρ…, ΠΏΡ€ΠΈΠ΅ΠΌΠ½ΠΈΠΊΠ°Ρ… спутниковых Ρ‚Π°Ρ€Π΅Π»ΠΎΠΊ, ΠΏΡƒΠ»ΡŒΡ‚Π°Ρ… дистанционного управлСния, Π²ΠΈΠ΄Π΅ΠΎΠΈΠ³Ρ€Π°Ρ…, ΠΊΠ°ΠΌΠ΅Ρ€Π°Ρ…, Π²ΠΈΠ΄Π΅ΠΎΠΊΠ°ΠΌΠ΅Ρ€Π°Ρ…, Π΄Π΅Ρ‚Π΅ΠΊΡ‚ΠΎΡ€Π°Ρ… Π΄Ρ‹ΠΌΠ°, ΠΌΠ΅Ρ…Π°Π½ΠΈΠ·ΠΌΠ°Ρ… открывания Π³Π°Ρ€Π°ΠΆΠ½Ρ‹Ρ… Π²ΠΎΡ€ΠΎΡ‚, бСспроводных Ρ‚Π΅Π»Π΅Ρ„ΠΎΠ½Π°Ρ…, ΠΌΠΎΠ±ΠΈΠ»ΡŒΠ½Ρ‹Ρ… Ρ‚Π΅Π»Π΅Ρ„ΠΎΠ½Π°Ρ…, факсы, тСлСскопы, ΠΏΡ€ΠΈΠ΅ΠΌΠ½ΠΈΠΊΠΈ GPS, Ρ€Π°Π΄ΠΈΠΎ, ΠΊΠ»Π°Π²ΠΈΠ°Ρ‚ΡƒΡ€Ρ‹, MP3-ΠΏΠ»Π΅Π΅Ρ€Ρ‹, ΠΌΠ°Π³Π½ΠΈΡ‚ΠΎΡ„ΠΎΠ½Ρ‹, стСрСосистСмы, часы, ΠΊΠ°Π»ΡŒΠΊΡƒΠ»ΡΡ‚ΠΎΡ€Ρ‹, ΠΏΡ€ΠΈΠ½Ρ‚Π΅Ρ€Ρ‹, сканСры, КПК ΠΈ Π±ΠΈΡ€ΠΊΠΈ для ΠΈΠ΄Π΅Π½Ρ‚ΠΈΡ„ΠΈΠΊΠ°Ρ†ΠΈΠΈ ΠΆΠΈΠ²ΠΎΡ‚Π½Ρ‹Ρ….

ВыраТСния ΠΈ уравнСния: ΠŸΠΎΠΏΡ€ΠΎΡΠΈΡ‚Π΅ учащихся Ρ€Π΅ΡˆΠΈΡ‚ΡŒ Π·Π°ΠΊΠΎΠ½ Ома (I = V / R) Π² ΠΊΠΎΠ½Ρ†Π΅ ΡƒΡ€ΠΎΠΊΠ° для Ρ€Π°Π·Π»ΠΈΡ‡Π½Ρ‹Ρ… ΠΏΠ΅Ρ€Π΅ΠΌΠ΅Π½Π½Ρ‹Ρ…, Π²ΠΊΠ»ΡŽΡ‡Π°Ρ напряТСниС, Ρ‚ΠΎΠΊ ΠΈ сопротивлСниС, Π° Π½Π΅ Ρ‚ΠΎΠ»ΡŒΠΊΠΎ Ρ‚ΠΎΠΊ.

использованная Π»ΠΈΡ‚Π΅Ρ€Π°Ρ‚ΡƒΡ€Π°

Π₯ΡŒΡŽΠΈΡ‚Ρ‚, Пол Π“. ΠšΠΎΠ½Ρ†Π΅ΠΏΡ‚ΡƒΠ°Π»ΡŒΠ½Π°Ρ Ρ„ΠΈΠ·ΠΈΠΊΠ°. 8-Π΅ ΠΈΠ·Π΄Π°Π½ΠΈΠ΅. Нью-Π™ΠΎΡ€ΠΊ, ΡˆΡ‚Π°Ρ‚ Нью-Π™ΠΎΡ€ΠΊ: ΠΈΠ·Π΄Π°Ρ‚Π΅Π»ΡŒΡΡ‚Π²ΠΎ Addison Publishing Company, 1998.

Каган, БпСнсСр. БовмСстноС ΠΎΠ±ΡƒΡ‡Π΅Π½ΠΈΠ΅. ΠšΠ°ΠΏΠΈΡΡ‚Ρ€Π°Π½ΠΎ, ΠšΠ°Π»ΠΈΡ„ΠΎΡ€Π½ΠΈΡ: ΠšΠΎΠΎΠΏΠ΅Ρ€Π°Ρ‚ΠΈΠ²Π½ΠΎΠ΅ ΠΎΠ±ΡƒΡ‡Π΅Π½ΠΈΠ΅ Кагана, 1994.(Π˜ΡΡ‚ΠΎΡ‡Π½ΠΈΠΊ для Π΄Π΅ΡΡ‚Π΅Π»ΡŒΠ½ΠΎΡΡ‚ΠΈ ΠΏΠΎ ΠΎΡ†Π΅Π½ΠΊΠ΅ ΠΏΡ€ΠΎΠ½ΡƒΠΌΠ΅Ρ€ΠΎΠ²Π°Π½Π½Ρ‹Ρ… Π³Π»Π°Π².)

Π ΠΈΠ΄, Π’. Π§ΠΈΠΏ: ΠΊΠ°ΠΊ Π΄Π²Π° Π°ΠΌΠ΅Ρ€ΠΈΠΊΠ°Π½Ρ†Π° ΠΈΠ·ΠΎΠ±Ρ€Π΅Π»ΠΈ ΠΌΠΈΠΊΡ€ΠΎΡ‡ΠΈΠΏ ΠΈ ΡΠΎΠ²Π΅Ρ€ΡˆΠΈΠ»ΠΈ Ρ€Π΅Π²ΠΎΠ»ΡŽΡ†ΠΈΡŽ. Нью-Π™ΠΎΡ€ΠΊ, ΡˆΡ‚Π°Ρ‚ Нью-Π™ΠΎΡ€ΠΊ: Random House, 2001, стр. 309.

авторскоС ΠΏΡ€Π°Π²ΠΎ

Β© 2004 Π Π΅Π³Π΅Π½Ρ‚Ρ‹ УнивСрситСта ΠšΠΎΠ»ΠΎΡ€Π°Π΄ΠΎ.

Авторы

ΠšΡΠΎΡ‡ΠΈΡ‚Π» Π—Π°ΠΌΠΎΡ€Π° Вомпсон; Π‘Π°Π±Π΅Ρ€ Π”ΡƒΡ€Π΅Π½; Π”Π°Ρ€ΡŒΡ ΠšΠΎΡ‚Ρ‹Ρ-Π¨Π²Π°Ρ€Ρ†; Малинда Π¨Π΅Ρ„Π΅Ρ€ ЗарскС; Π”Π΅Π½ΠΈΠ· ΠšΠ°Ρ€Π»ΡΠΎΠ½; Π”ΠΆΠ°Π½Π΅Ρ‚ Йоуэлл

ΠŸΡ€ΠΎΠ³Ρ€Π°ΠΌΠΌΠ° ΠΏΠΎΠ΄Π΄Π΅Ρ€ΠΆΠΊΠΈ

КомплСксная ΠΏΡ€ΠΎΠ³Ρ€Π°ΠΌΠΌΠ° прСподавания ΠΈ обучСния, Π˜Π½ΠΆΠ΅Π½Π΅Ρ€Π½Ρ‹ΠΉ ΠΊΠΎΠ»Π»Π΅Π΄ΠΆ, УнивСрситСт ΠšΠΎΠ»ΠΎΡ€Π°Π΄ΠΎ Π² Π‘ΠΎΡƒΠ»Π΄Π΅Ρ€Π΅

Благодарности

Π‘ΠΎΠ΄Π΅Ρ€ΠΆΠ°Π½ΠΈΠ΅ этой ΡƒΡ‡Π΅Π±Π½ΠΎΠΉ ΠΏΡ€ΠΎΠ³Ρ€Π°ΠΌΠΌΡ‹ ΠΏΠΎ Ρ†ΠΈΡ„Ρ€ΠΎΠ²ΠΎΠΉ Π±ΠΈΠ±Π»ΠΈΠΎΡ‚Π΅ΠΊΠ΅ Π±Ρ‹Π»ΠΎ Ρ€Π°Π·Ρ€Π°Π±ΠΎΡ‚Π°Π½ΠΎ Π·Π° счСт Π³Ρ€Π°Π½Ρ‚Π° Π€ΠΎΠ½Π΄Π° ΡƒΠ»ΡƒΡ‡ΡˆΠ΅Π½ΠΈΡ послСсрСднСго образования (FIPSE), U.S. ΠœΠΈΠ½ΠΈΡΡ‚Π΅Ρ€ΡΡ‚Π²ΠΎ образования ΠΈ ΠΠ°Ρ†ΠΈΠΎΠ½Π°Π»ΡŒΠ½Ρ‹ΠΉ Π½Π°ΡƒΡ‡Π½Ρ‹ΠΉ Ρ„ΠΎΠ½Π΄ Π“Πš-12, Π³Ρ€Π°Π½Ρ‚ No. 0338326. Однако это содСрТаниС Π½Π΅ ΠΎΠ±ΡΠ·Π°Ρ‚Π΅Π»ΡŒΠ½ΠΎ ΠΎΡ‚Ρ€Π°ΠΆΠ°Π΅Ρ‚ ΠΏΠΎΠ»ΠΈΡ‚ΠΈΠΊΡƒ ΠœΠΈΠ½ΠΈΡΡ‚Π΅Ρ€ΡΡ‚Π²Π° образования ΠΈΠ»ΠΈ ΠΠ°Ρ†ΠΈΠΎΠ½Π°Π»ΡŒΠ½ΠΎΠ³ΠΎ Π½Π°ΡƒΡ‡Π½ΠΎΠ³ΠΎ Ρ„ΠΎΠ½Π΄Π°, ΠΈ Π²Π°ΠΌ Π½Π΅ слСдуСт ΠΏΡ€Π΅Π΄ΠΏΠΎΠ»Π°Π³Π°Ρ‚ΡŒ, Ρ‡Ρ‚ΠΎ ΠΎΠ½ΠΎ ΠΎΠ΄ΠΎΠ±Ρ€Π΅Π½ΠΎ Ρ„Π΅Π΄Π΅Ρ€Π°Π»ΡŒΠ½Ρ‹ΠΌ ΠΏΡ€Π°Π²ΠΈΡ‚Π΅Π»ΡŒΡΡ‚Π²ΠΎΠΌ.

ПослСднСС ΠΈΠ·ΠΌΠ΅Π½Π΅Π½ΠΈΠ΅: 16 октября 2021 Π³.

ΠŸΡ€ΠΈΠΌΠ΅Ρ€ Π·Π°ΠΊΠΎΠ½Π°

Ома с Ρ€Π΅ΡˆΠ΅Π½ΠΈΡΠΌΠΈ для срСднСй ΡˆΠΊΠΎΠ»Ρ‹

Π­ΠΊΡΠΏΠ΅Ρ€ΠΈΠΌΠ΅Π½Ρ‚Π°Π»ΡŒΠ½ΠΎ установлСно, Ρ‡Ρ‚ΠΎ ΠΊΠΎΠ³Π΄Π° напряТСниС ΠΈΠ»ΠΈ Ρ€Π°Π·Π½ΠΎΡΡ‚ΡŒ ΠΏΠΎΡ‚Π΅Π½Ρ†ΠΈΠ°Π»ΠΎΠ² $ \ Delta V $ прикладываСтся ΠΊ ΠΊΠΎΠ½Ρ†Π°ΠΌ ΠΎΠΏΡ€Π΅Π΄Π΅Π»Π΅Π½Π½Ρ‹Ρ… ΠΏΡ€ΠΎΠ²ΠΎΠ΄Π½ΠΈΠΊΠΎΠ², Ρ‚ΠΎΠΊ Ρ‡Π΅Ρ€Π΅Π· Π½ΠΈΡ… ΠΏΡ€ΠΎΠΏΠΎΡ€Ρ†ΠΈΠΎΠ½Π°Π»Π΅Π½ ΠΏΡ€ΠΈΠ»ΠΎΠΆΠ΅Π½Π½ΠΎΠΌΡƒ Π½Π°ΠΏΡ€ΡΠΆΠ΅Π½ΠΈΡŽ, Ρ‚ΠΎ Π΅ΡΡ‚ΡŒ $ I \ propto \ Delta V $.

ΠšΠΎΠ½ΡΡ‚Π°Π½Ρ‚Π° ΠΏΡ€ΠΎΠΏΠΎΡ€Ρ†ΠΈΠΎΠ½Π°Π»ΡŒΠ½ΠΎΡΡ‚ΠΈ называСтся сопротивлСниСм этого ΠΏΡ€ΠΎΠ²ΠΎΠ΄Π½ΠΈΠΊΠ° .

Π”Ρ€ΡƒΠ³ΠΈΠΌΠΈ словами, сопротивлСниС опрСдСляСтся ΠΊΠ°ΠΊ ΠΎΡ‚Π½ΠΎΡˆΠ΅Π½ΠΈΠ΅ напряТСния Π² ΠΏΡ€ΠΎΠ²ΠΎΠ΄Π½ΠΈΠΊΠ΅ ΠΊ Ρ‚ΠΎΠΊΡƒ, ΠΏΡ€ΠΎΡ‚Π΅ΠΊΠ°ΡŽΡ‰Π΅ΠΌΡƒ ΠΏΠΎ Π½Π΅ΠΌΡƒ. \ [R \ Equiv \ frac {\ Delta V} {I} \] Π­Ρ‚ΠΎ простоС ΡΠΎΠΎΡ‚Π½ΠΎΡˆΠ΅Π½ΠΈΠ΅ ΠΌΠ΅ΠΆΠ΄Ρƒ Ρ€Π°Π·Π½ΠΎΡΡ‚ΡŒΡŽ ΠΏΠΎΡ‚Π΅Π½Ρ†ΠΈΠ°Π»ΠΎΠ² ΠΈ Ρ‚ΠΎΠΊΠΎΠΌ извСстный ΠΊΠ°ΠΊ Π·Π°ΠΊΠΎΠ½ Ома .

Π•Π΄ΠΈΠ½ΠΈΡ†Ρ‹ сопротивлСния Π² систСмС БИ: Π²ΠΎΠ»ΡŒΡ‚ Π½Π° Π°ΠΌΠΏΠ΅Ρ€ , ΠΊΠΎΡ‚ΠΎΡ€Ρ‹Π΅ Π½Π°Π·Ρ‹Π²Π°ΡŽΡ‚ΡΡ Ом ($ \ Omega $).

ΠŸΡ€ΠΎΠ²ΠΎΠ΄Π½ΠΈΠΊ, ΠΊΠΎΡ‚ΠΎΡ€Ρ‹ΠΉ обСспСчиваСт ΠΎΠΏΡ€Π΅Π΄Π΅Π»Π΅Π½Π½ΠΎΠ΅ сопротивлСниС Π² элСктричСской Ρ†Π΅ΠΏΠΈ, называСтся рСзистором .

НапримСр, Ссли рСзистор 10 Ом ΠΏΠΎΠ΄ΠΊΠ»ΡŽΡ‡Π΅Π½ ΠΊ ΠΊΠ»Π΅ΠΌΠΌΠ°ΠΌ аккумулятора с напряТСниСм 240 Π’, Ρ‚ΠΎ Ρ‡Π΅Ρ€Π΅Π· Π½Π΅Π³ΠΎ ΠΏΡ€ΠΎΡ…ΠΎΠ΄ΠΈΡ‚ Ρ‚ΠΎΠΊ $ \ frac {240} {10} = 24 \, {\ rm A} $.

Напротив, Π² элСктроникС Ρ‚Π°ΠΊΠΆΠ΅ Π΅ΡΡ‚ΡŒ ΠΏΡ€ΠΎΠ²ΠΎΠ΄Π½ΠΈΠΊΠΈ ΠΈΠ»ΠΈ ΠΌΠ°Ρ‚Π΅Ρ€ΠΈΠ°Π»Ρ‹, ΠΊΠΎΡ‚ΠΎΡ€Ρ‹Π΅ Π²Ρ‹ΡˆΠ΅ простой Π»ΠΈΠ½Π΅ΠΉΠ½ΠΎΠΉ зависимости ΠΌΠ΅ΠΆΠ΄Ρƒ напряТСниСм ΠΈ Ρ‚ΠΎΠΊΠΎΠΌ Π½Π΅ ΡΠΎΠ±Π»ΡŽΠ΄Π°ΡŽΡ‚ΡΡ, Ρ‚Π°ΠΊΠΈΠ΅ ΠΊΠ°ΠΊ Π΄ΠΈΠΎΠ΄Ρ‹, транзисторы ΠΈΠ»ΠΈ Π»ΡŽΠΌΠΈΠ½Π΅ΡΡ†Π΅Π½Ρ‚Π½Ρ‹Π΅ Π»Π°ΠΌΠΏΡ‹.

Π’ Ρ‚Π°ΠΊΠΈΡ… ΠΌΠ°Ρ‚Π΅Ρ€ΠΈΠ°Π»Π°Ρ… сущСствуСт нСлинСйная Π·Π°Π²ΠΈΡΠΈΠΌΠΎΡΡ‚ΡŒ напряТСния ΠΎΡ‚ Ρ‚ΠΎΠΊΠ°. Π­Ρ‚ΠΈ ΠΏΡ€ΠΎΠ²ΠΎΠ΄Π½ΠΈΠΊΠΈ Π½Π°Π·Ρ‹Π²Π°ΡŽΡ‚ΡΡ нСомичСскими ΠΌΠ°Ρ‚Π΅Ρ€ΠΈΠ°Π»Π°ΠΌΠΈ .


Π“ΠΎΡ‚ΠΎΠ²ΠΈΡ‚Π΅ΡΡŒ ΠΊ экзамСну AP Physics? ΠŸΡ€ΠΎΡ‡Ρ‚ΠΈΡ‚Π΅ это:
ΠŸΡ€Π°ΠΊΡ‚ΠΈΡ‡Π΅ΡΠΊΠΈΠ΅ Π·Π°Π΄Π°Ρ‡ΠΈ ΠΏΠΎ схСмам для экзамСна AP


НиТС ΠΏΡ€ΠΈΠ²Π΅Π΄Π΅Π½Ρ‹ Π½Π΅ΠΊΠΎΡ‚ΠΎΡ€Ρ‹Π΅ простыС вопросы ΠΈ ΠΎΡ‚Π²Π΅Ρ‚Ρ‹ ΠΎ Π·Π°ΠΊΠΎΠ½Π΅ Ома с ΠΏΠΎΠ΄Ρ€ΠΎΠ±Π½Ρ‹ΠΌΠΈ объяснСниями. ВсС Π·Π°Π΄Π°Ρ‡ΠΈ подходят ΡΡ‚Π°Ρ€ΡˆΠ΅ΠΊΠ»Π°ΡΡΠ½ΠΈΠΊΡƒ.

ΠŸΡ€ΠΈΠΌΠ΅Ρ€Ρ‹ Π·Π°ΠΊΠΎΠ½Π° Ома

ΠŸΡ€ΠΈΠΌΠ΅Ρ€ (1): Π­Π»Π΅ΠΊΡ‚Ρ€ΠΎΠ½Π½ΠΎΠ΅ устройство ΠΈΠΌΠ΅Π΅Ρ‚ сопротивлСниС 20 Ом ΠΈ Ρ‚ΠΎΠΊ 15 А. КакоС напряТСниС Π½Π° устройствС?

РСшСниС : сопротивлСниС, Ρ‚ΠΎΠΊ ΠΈ напряТСниС связаны Π·Π°ΠΊΠΎΠ½ΠΎΠΌ Ома ΠΊΠ°ΠΊ $ V = IR $.Π’Π°ΠΊΠΈΠΌ ΠΎΠ±Ρ€Π°Π·ΠΎΠΌ, напряТСниС устройства получаСтся ΠΊΠ°ΠΊ \ begin {align *} V & = IR \\ & = 15 \ times 20 \\ & = 300 \ quad {\ rm V} \ end {align *}


ΠŸΡ€ΠΈΠΌΠ΅Ρ€ (2): Ρ€Π°Π·Π½ΠΎΡΡ‚ΡŒ ΠΏΠΎΡ‚Π΅Π½Ρ†ΠΈΠ°Π»ΠΎΠ² $ 3 — {\ rm V} $ ΠΏΡ€ΠΈΠ»ΠΎΠΆΠ΅Π½Π° ΠΊ рСзистору $ 6 \, {\ rm \ Omega} $. Какой Ρ‚ΠΎΠΊ ΠΏΡ€ΠΎΡ‚Π΅ΠΊΠ°Π΅Ρ‚ Ρ‡Π΅Ρ€Π΅Π· рСзистор?

РСшСниС : Π—Π°ΠΊΠΎΠ½ Ома гласит, Ρ‡Ρ‚ΠΎ Ρ€Π°Π·Π½ΠΎΡΡ‚ΡŒ ΠΏΠΎΡ‚Π΅Π½Ρ†ΠΈΠ°Π»ΠΎΠ² Π½Π° рСзисторС Ρ€Π°Π²Π½Π° ΡΠΎΠΏΡ€ΠΎΡ‚ΠΈΠ²Π»Π΅Π½ΠΈΡŽ, ΡƒΠΌΠ½ΠΎΠΆΠ΅Π½Π½ΠΎΠΌΡƒ Π½Π° Ρ‚ΠΎΠΊ, поэтому ΠΌΡ‹ ΠΏΠΎΠ»ΡƒΡ‡Π°Π΅ΠΌ \ begin {align *} I & = \ frac VR \\ & = \ frac {3} {6} \\ & = 0.5 \ quad {\ rm A} \ end {align *}


ΠŸΡ€ΠΈΠΌΠ΅Ρ€ (3): Π’ΠΎΠΊ Π²Π΅Π»ΠΈΡ‡ΠΈΠ½ΠΎΠΉ $ 0,2 \, {\ rm A} $ ΠΏΡ€ΠΎΡ…ΠΎΠ΄ΠΈΡ‚ Ρ‡Π΅Ρ€Π΅Π· рСзистор $ 1,4 \, {\ rm k \ Omega} $. КакоС напряТСниС Π½Π° Π½Π΅ΠΌ?

РСшСниС : ΠΈΡΠΏΠΎΠ»ΡŒΠ·ΡƒΡ Π·Π°ΠΊΠΎΠ½ Ома, $ V = IR $, ΠΌΡ‹ ΠΈΠΌΠ΅Π΅ΠΌ \ begin {align *} V & = IR \\ & = (0.2 \, {\ rm A}) (1.4 \ times 1000 \, {\ rm \ Omega}) \\ & = 280 \ quad {\ rm V} \ end {align *}



ΠŸΡ€ΠΈΠΌΠ΅Ρ€ (4): Π’ схСмС, ΠΏΠΎΠΊΠ°Π·Π°Π½Π½ΠΎΠΉ Π½ΠΈΠΆΠ΅, ΠΊΠ°ΠΊΠΎΠΉ Ρ‚ΠΎΠΊ ΠΏΠΎΠΊΠ°Π·Ρ‹Π²Π°Π΅Ρ‚ Π°ΠΌΠΏΠ΅Ρ€ΠΌΠ΅Ρ‚Ρ€?

РСшСниС : Π»Π°ΠΌΠΏΠ° прСдставляСт собой элСктронный ΠΊΠΎΠΌΠΏΠΎΠ½Π΅Π½Ρ‚ с высоким сопротивлСниСм.На рисункС напряТСниС Π½Π° Π½Π΅ΠΌ Ρ‚Π°ΠΊΠΎΠ΅ ΠΆΠ΅, ΠΊΠ°ΠΊ Ρƒ Π±Π°Ρ‚Π°Ρ€Π΅ΠΈ $ V = 20 \, {\ rm V} $. Π’ΠΎΠΊ, проходящий Ρ‡Π΅Ρ€Π΅Π· Π½Π΅Π³ΠΎ, связан с сопротивлСниСм ΠΈ ΠΏΠ°Π΄Π΅Π½ΠΈΠ΅ΠΌ напряТСния ΠΏΠΎ Π·Π°ΠΊΠΎΠ½Ρƒ Ома \ begin {align *} I & = \ frac VR \\ & = \ frac {20} {8} \\ & = 1.25 \ quad {\ rm A} \ end {align *}


НуТно большС? Π—Π°Π³Ρ€ΡƒΠ·ΠΈΡ‚Π΅ эти Π΄ΠΎΠΏΠΎΠ»Π½ΠΈΡ‚Π΅Π»ΡŒΠ½Ρ‹Π΅ вопросы для экзамСнов здСсь.



ΠŸΡ€ΠΈΠΌΠ΅Ρ€ (5): Π’ Ρ†Π΅ΠΏΠΈ ΠΏΠ°Π΄Π΅Π½ΠΈΠ΅ ΠΏΠΎΡ‚Π΅Π½Ρ†ΠΈΠ°Π»Π° Π½Π° рСзисторС 10 кОм составляСт 100 Π’. Каков Ρ‚ΠΎΠΊ Ρ‡Π΅Ρ€Π΅Π· рСзистор?

РСшСниС : ΠΏΠΎΠ΄ΡΡ‚Π°Π²ΡŒΡ‚Π΅ всС извСстныС числовыС значСния Π² ΡƒΡ€Π°Π²Π½Π΅Π½ΠΈΠ΅ Ома, $ V = IR $.\ begin {align *} I & = \ frac VR \\\\ & = \ frac {100 \, {\ rm V}} {10000 \, {\ rm \ Omega}} \\\\ & = 0.01 \ quad { \ rm A} \ end {align *}


ΠŸΡ€ΠΈΠΌΠ΅Ρ€ (6): Π² ΡΠ»Π΅Π΄ΡƒΡŽΡ‰ΠΈΡ… схСмах Π½Π°ΠΉΠ΄ΠΈΡ‚Π΅ нСизвСстныС.

РСшСниС : Π’ ΠΊΠ°ΠΆΠ΄ΠΎΠΉ ΠΈΠ· схСм ΠΈΡΠΏΠΎΠ»ΡŒΠ·ΡƒΠΉΡ‚Π΅ Π·Π°ΠΊΠΎΠ½ Ома $ V = IR $ ΠΈ Π½Π°ΠΉΠ΄ΠΈΡ‚Π΅ нСизвСстноС. Π’ Π»Π΅Π²ΠΎΠΉ Ρ†Π΅ΠΏΠΈ Ρ‚ΠΎΠΊ Ρ‡Π΅Ρ€Π΅Π· рСзистор Π·Π°ΠΏΡ€Π°ΡˆΠΈΠ²Π°Π΅Ρ‚ΡΡ Π² ΠΌΠΈΠ»Π»ΠΈΠ°ΠΌΠΏΠ΅Ρ€Π°Ρ…. Π’Π°ΠΊΠΈΠΌ ΠΎΠ±Ρ€Π°Π·ΠΎΠΌ, \ begin {align *} I & = \ frac VR \\\\ & = \ frac {120} {100} \\\\ & = 1.2 \ quad {\ rm A} \ end {align *} Π§Ρ‚ΠΎΠ±Ρ‹ ΠΏΡ€Π΅ΠΎΠ±Ρ€Π°Π·ΠΎΠ²Π°Ρ‚ΡŒ Π΅Π³ΠΎ Π² ΠΌΠΈΠ»Π»ΠΈΠ°ΠΌΠΏΠ΅Ρ€Ρ‹, ΡƒΠΌΠ½ΠΎΠΆΡŒΡ‚Π΅ Π΅Π³ΠΎ Π½Π° 1000, Ρ‡Ρ‚ΠΎΠ±Ρ‹ ΠΏΠΎΠ»ΡƒΡ‡ΠΈΡ‚ΡŒ $ I = 1200 \, {\ rm mA} $.{-3}} \\\\ & = 40 \ quad {\ rm \ Omega} \ end {align *}




ΠŸΡ€ΠΈΠΌΠ΅Ρ€ (7): ΠšΡ€ΠΈΠ²Π°Ρ напряТСниС-Ρ‚ΠΎΠΊ для омичСского ΠΏΡ€ΠΎΠ²ΠΎΠ΄Π½ΠΈΠΊΠ° построСна, ΠΊΠ°ΠΊ ΠΏΠΎΠΊΠ°Π·Π°Π½ΠΎ Π½Π° рисункС Π½ΠΈΠΆΠ΅. КакоС сопротивлСниС рСзисторов 1 ΠΈ 2?

РСшСниС: Π—Π°ΠΊΠΎΠ½ Ома Π³ΠΎΠ²ΠΎΡ€ΠΈΡ‚ Π½Π°ΠΌ, Ρ‡Ρ‚ΠΎ сопротивлСниС — это Π½Π°ΠΊΠ»ΠΎΠ½ ΠΊΡ€ΠΈΠ²ΠΎΠΉ зависимости напряТСния ΠΎΡ‚ Ρ‚ΠΎΠΊΠ° $ R = \ frac {\ Delta V} {I} $. Напомним, Ρ‡Ρ‚ΠΎ Π½Π°ΠΊΠ»ΠΎΠ½ $ m $ прямой ΠΌΠ΅ΠΆΠ΄Ρƒ двумя Ρ‚ΠΎΡ‡ΠΊΠ°ΠΌΠΈ $ A (x_1, y_1) $ ΠΈ $ B (x_2, y_2) $ опрСдСляСтся ΠΊΠ°ΠΊ \ [m = \ frac {\ Delta y} {\ Delta x} = \ frac {y_2-y_1} {x_2-x_1} \] Π’Π°ΠΊΠΈΠΌ ΠΎΠ±Ρ€Π°Π·ΠΎΠΌ, Π½Π°ΠΊΠ»ΠΎΠ½ ΠΊΡ€ΠΈΠ²ΠΎΠΉ напряТСниС-Ρ‚ΠΎΠΊ, ΠΊΠΎΡ‚ΠΎΡ€Ρ‹ΠΉ являСтся сопротивлСниСм, получаСтся ΡΠ»Π΅Π΄ΡƒΡŽΡ‰ΠΈΠΌ ΠΎΠ±Ρ€Π°Π·ΠΎΠΌ:
Π’ΠΎΡ‡ΠΊΠΈ $ A (0,0) $ ΠΈ $ B (2,20) $ находятся Π½Π° Π»ΠΈΠ½ΠΈΠΈ (1): \ [R_1 = \ frac {20-0} {2-0} = 10 \ quad {\ rm \ Omega} \]
Π’ΠΎΡ‡ΠΊΠΈ $ A (0, 0) $ ΠΈ $ B (4,10) $ находятся Π² строкС (2): \ [R_2 = \ frac {10-0} {4-0} = 2.5 \ quad {\ rm \ Omega} \]



ΠŸΡ€ΠΈΠΌΠ΅Ρ€ (8): ΠŸΠΎΠΌΠ΅Π½ΡΠΉΡ‚Π΅ мСстами ΠΏΠ°Π΄Π΅Π½ΠΈΠ΅ ΠΏΠΎΡ‚Π΅Π½Ρ†ΠΈΠ°Π»Π° Π½Π° ΠΏΡ€ΠΎΠ²ΠΎΠ΄Π½ΠΈΠΊΠ΅ ΠΈ Ρ‚ΠΎΠΊ, проходящий Ρ‡Π΅Ρ€Π΅Π· Π½Π΅Π³ΠΎ Π² ΠΏΡ€Π΅Π΄Ρ‹Π΄ΡƒΡ‰Π΅ΠΉ Π·Π°Π΄Π°Ρ‡Π΅, Ρ‡Ρ‚ΠΎΠ±Ρ‹ ΠΏΠΎΠ»ΡƒΡ‡ΠΈΡ‚ΡŒ Π²ΠΎΠ»ΡŒΡ‚-Π°ΠΌΠΏΠ΅Ρ€Π½ΡƒΡŽ ΠΊΡ€ΠΈΠ²ΡƒΡŽ. Π’Π΅ΠΏΠ΅Ρ€ΡŒ Π½Π°ΠΉΠ΄ΠΈΡ‚Π΅ сопротивлСниС рСзисторов 1 ΠΈ 2?

РСшСниС: Ссли ΠΌΡ‹ ΠΈΠ·ΠΌΠ΅Π½ΠΈΠΌ Π·Π°ΠΊΠΎΠ½ Ома ΠΊΠ°ΠΊ $ I = \ frac {1} {R} \ Delta V $, ΠΌΡ‹ ΡƒΠ²ΠΈΠ΄ΠΈΠΌ, Ρ‡Ρ‚ΠΎ Π½Π°ΠΊΠ»ΠΎΠ½ ΠΊΡ€ΠΈΠ²ΠΎΠΉ Π²ΠΎΠ»ΡŒΡ‚-Π°ΠΌΠΏΠ΅Ρ€Π½ΠΎΠΉ характСристики Π² этом случаС Π΄Π°Π΅Ρ‚ Π²Π΅Π»ΠΈΡ‡ΠΈΠ½Ρƒ, ΠΎΠ±Ρ€Π°Ρ‚Π½ΡƒΡŽ ΡΠΎΠΏΡ€ΠΎΡ‚ΠΈΠ²Π»Π΅Π½ΠΈΡŽ. . Π‘Π»Π΅Π΄ΠΎΠ²Π°Ρ‚Π΅Π»ΡŒΠ½ΠΎ, ΠΊΠ°ΠΊ ΠΈ Π² ΠΏΡ€Π΅Π΄Ρ‹Π΄ΡƒΡ‰Π΅ΠΉ Π·Π°Π΄Π°Ρ‡Π΅, Π½Π°ΠΊΠ»ΠΎΠ½
Π»ΠΈΠ½ΠΈΠΈ (1) Ρ€Π°Π²Π΅Π½ \ [\ frac {1} {R_1} = \ frac {20-0} {2-0} = 10 \], Ρ‡Ρ‚ΠΎ Π΄Π°Π΅Ρ‚ $ R_1 = 0.1 \, {\ rm \ Omega} $, Π° Π½Π°ΠΊΠ»ΠΎΠ½ Π»ΠΈΠ½ΠΈΠΈ (2) Ρ€Π°Π²Π΅Π½ \ [\ frac {1} {R_2} = \ frac {10-0} {4-0} = 2,5 \], Ρ‡Ρ‚ΠΎ Π΄Π°Π΅Ρ‚ $ R_2 = 0,4 \, {\ rm \ Omega} $.


ΠŸΡ€ΠΈΠΌΠ΅Ρ€ (9): Π‘Ρ‚ΡƒΠ΄Π΅Π½Ρ‚ ΠΏΡ€ΠΎΠ²ΠΎΠ΄ΠΈΡ‚ экспСримСнт ΠΈ измСряСт Ρ‚ΠΎΠΊ ΠΈ напряТСниС Π½Π° Π΄Π²ΡƒΡ… нСизвСстных рСзисторах. Π—Π°Ρ‚Π΅ΠΌ ΠΎΠ½Π° строит своС ΠΎΡ‚ΠΊΡ€Ρ‹Ρ‚ΠΈΠ΅ Π² ΠΊΠΎΠΎΡ€Π΄ΠΈΠ½Π°Ρ‚Π°Ρ… «Ρ‚ΠΎΠΊ-напряТСниС», ΠΊΠ°ΠΊ ΠΏΠΎΠΊΠ°Π·Π°Π½ΠΎ Π½Π° рисункС. Π§Ρ‚ΠΎ ΠΌΠΎΠΆΠ½ΠΎ ΡΠΊΠ°Π·Π°Ρ‚ΡŒ ΠΎ рСзисторах А ΠΈ Π’?

РСшСниС : омичСскиС ΠΌΠ°Ρ‚Π΅Ρ€ΠΈΠ°Π»Ρ‹ — это ΠΌΠ°Ρ‚Π΅Ρ€ΠΈΠ°Π»Ρ‹, ΠΊΠΎΡ‚ΠΎΡ€Ρ‹Π΅ ΠΈΠΌΠ΅ΡŽΡ‚ постоянноС сопротивлСниС Π² ΡˆΠΈΡ€ΠΎΠΊΠΎΠΌ Π΄ΠΈΠ°ΠΏΠ°Π·ΠΎΠ½Π΅ ΠΏΡ€ΠΈΠ»ΠΎΠΆΠ΅Π½Π½Ρ‹Ρ… напряТСний.Π”Ρ€ΡƒΠ³ΠΈΠΌΠΈ словами, Π² омичСском ΠΏΡ€ΠΎΠ²ΠΎΠ΄Π½ΠΈΠΊΠ΅ ΠΎΡ‚Π½ΠΎΡˆΠ΅Π½ΠΈΠ΅ напряТСния Π½Π° Π½Π΅ΠΌ ΠΊ Ρ‚ΠΎΠΊΡƒ Ρ‡Π΅Ρ€Π΅Π· Π½Π΅Π³ΠΎ, ΠΊΠΎΡ‚ΠΎΡ€ΠΎΠ΅ опрСдСляСтся ΠΊΠ°ΠΊ сопротивлСниС, всСгда являСтся постоянным.

Π’Π°ΠΊΠΈΠΌ ΠΎΠ±Ρ€Π°Π·ΠΎΠΌ, омичСскиС ΠΌΠ°Ρ‚Π΅Ρ€ΠΈΠ°Π»Ρ‹ ΠΈΠΌΠ΅ΡŽΡ‚ Π»ΠΈΠ½Π΅ΠΉΠ½ΡƒΡŽ Π·Π°Π²ΠΈΡΠΈΠΌΠΎΡΡ‚ΡŒ Ρ‚ΠΎΠΊΠ° ΠΎΡ‚ напряТСния, ΠΈ Π΅Π΅ кривая ΠΏΡ€ΠΎΡ…ΠΎΠ΄ΠΈΡ‚ Ρ‡Π΅Ρ€Π΅Π· Π½Π°Ρ‡Π°Π»ΠΎ ΠΊΠΎΠΎΡ€Π΄ΠΈΠ½Π°Ρ‚. Напротив, ΠΌΠ°Ρ‚Π΅Ρ€ΠΈΠ°Π»Ρ‹, ΠΈΠΌΠ΅ΡŽΡ‰ΠΈΠ΅ сопротивлСниС, ΠΊΠΎΡ‚ΠΎΡ€ΠΎΠ΅ измСняСтся ΠΏΡ€ΠΈ ΠΏΠ°Π΄Π΅Π½ΠΈΠΈ ΠΏΠΎΡ‚Π΅Π½Ρ†ΠΈΠ°Π»Π° ΠΈΠ»ΠΈ Ρ‚ΠΎΠΊΠ΅, Π½Π°Π·Ρ‹Π²Π°ΡŽΡ‚ΡΡ нСомичСскими.

ΠšΡ€ΠΈΠ²Π°Ρ нСомичСского ΠΌΠ°Ρ‚Π΅Ρ€ΠΈΠ°Π»Π° Π½Π΅ являСтся Π»ΠΈΠ½Π΅ΠΉΠ½ΠΎΠΉ. ΠŸΡ€ΠΈΠΌΠ΅Ρ€Π°ΠΌΠΈ нСомичСских ΠΌΠ°Ρ‚Π΅Ρ€ΠΈΠ°Π»ΠΎΠ², Π½Π°Ρ€ΡƒΡˆΠ°ΡŽΡ‰ΠΈΡ… Π·Π°ΠΊΠΎΠ½ Ома, ΡΠ²Π»ΡΡŽΡ‚ΡΡ Π΄ΠΈΠΎΠ΄Ρ‹ ΠΈ транзисторы.

Π‘ этими пояснСниями, ΠΏΠΎΡΠΊΠΎΠ»ΡŒΠΊΡƒ кривая (A) Π»ΠΈΠ½Π΅ΠΉΠ½Π° ΠΈ ΠΏΡ€ΠΎΡ…ΠΎΠ΄ΠΈΡ‚ Ρ‡Π΅Ρ€Π΅Π· Π½Π°Ρ‡Π°Π»ΠΎ ΠΊΠΎΠΎΡ€Π΄ΠΈΠ½Π°Ρ‚, Π·Π½Π°Ρ‡ΠΈΡ‚, это омичСский ΠΏΡ€ΠΎΠ²ΠΎΠ΄Π½ΠΈΠΊ, Π½Π°ΠΊΠ»ΠΎΠ½ ΠΊΠΎΡ‚ΠΎΡ€ΠΎΠ³ΠΎ Π΄Π°Π΅Ρ‚ ΠΎΠ±Ρ€Π°Ρ‚Π½ΠΎΠ΅ сопротивлСниС. Как ΠΈ Π² ΠΏΡ€Π΅Π΄Ρ‹Π΄ΡƒΡ‰Π΅ΠΉ Π·Π°Π΄Π°Ρ‡Π΅, Π΅Π³ΠΎ сопротивлСниС рассчитываСтся ΠΊΠ°ΠΊ $ R_A = 5 \, {\ rm \ Omega} $.

РСзистор (B) ΠΈΠΌΠ΅Π΅Ρ‚ Π½Π΅Π»ΠΈΠ½Π΅ΠΉΠ½ΡƒΡŽ Π·Π°Π²ΠΈΡΠΈΠΌΠΎΡΡ‚ΡŒ ΠΌΠ΅ΠΆΠ΄Ρƒ напряТСниСм Π½Π° Π½Π΅ΠΌ ΠΈ Ρ‚ΠΎΠΊΠΎΠΌ, поэтому это нСомичСский ΠΏΡ€ΠΎΠ²ΠΎΠ΄Π½ΠΈΠΊ с ΠΏΠ΅Ρ€Π΅ΠΌΠ΅Π½Π½Ρ‹ΠΌ сопротивлСниСм.


Π—Π°ΠΊΠΎΠ½ Ома: практичСскиС ΠΏΡ€ΠΎΠ±Π»Π΅ΠΌΡ‹ с Ρ€Π΅ΡˆΠ΅Π½ΠΈΠ΅ΠΌ

Π’Π΅ΠΏΠ΅Ρ€ΡŒ ΠΌΡ‹ Ρ…ΠΎΡ‚ΠΈΠΌ Ρ€Π΅ΡˆΠΈΡ‚ΡŒ Π½Π΅ΠΊΠΎΡ‚ΠΎΡ€Ρ‹Π΅ практичСскиС Π·Π°Π΄Π°Ρ‡ΠΈ, Ρ‡Ρ‚ΠΎΠ±Ρ‹ ΠΏΠΎΠΊΠ°Π·Π°Ρ‚ΡŒ Π²Π°ΠΌ, ΠΊΠ°ΠΊ ΠΈΡΠΏΠΎΠ»ΡŒΠ·ΠΎΠ²Π°Ρ‚ΡŒ Π·Π°ΠΊΠΎΠ½ Ома для Ρ€Π΅ΡˆΠ΅Π½ΠΈΡ ΠΏΡ€ΠΎΠ±Π»Π΅ΠΌ с элСктричСством.

ΠŸΡ€Π°ΠΊΡ‚ΠΈΡ‡Π΅ΡΠΊΠ°Ρ Π·Π°Π΄Π°Ρ‡Π° (1): Π±ΡƒΠ΄ΠΈΠ»ΡŒΠ½ΠΈΠΊ потрСбляСт Ρ‚ΠΎΠΊ 0,5 А ΠΏΡ€ΠΈ ΠΏΠΎΠ΄ΠΊΠ»ΡŽΡ‡Π΅Π½ΠΈΠΈ ΠΊ Ρ†Π΅ΠΏΠΈ 120 Π’. НайдитС Π΅Π³ΠΎ сопротивлСниС.

РСшСниС : Π·Π°Π΄Π°Π½Ρ‹ Ρ‚ΠΎΠΊ $ I = 0,5 \, {\ rm A} $ ΠΈ ΠΏΠ°Π΄Π΅Π½ΠΈΠ΅ напряТСния $ V = 120 \, {\ rm V} $. Π Π΅ΡˆΠΈΡ‚Π΅ Π·Π°ΠΊΠΎΠ½ Ома для нСизвСстного $ R $ ΠΊΠ°ΠΊ \ begin {align *} R & = \ frac VI \\ \\ & = \ frac {120} {0.5} \\ \\ & = 240 \ quad {\ rm \ Omega} \ ΠΊΠΎΠ½Π΅Ρ† {align *}


ΠŸΡ€Π°ΠΊΡ‚ΠΈΡ‡Π΅ΡΠΊΠ°Ρ ΠΏΡ€ΠΎΠ±Π»Π΅ΠΌΠ° (2): сабвуфСру трСбуСтся домашнСС напряТСниС 110 Π’, Ρ‡Ρ‚ΠΎΠ±Ρ‹ ΠΏΡ€ΠΎΡ‚ΠΎΠ»ΠΊΠ½ΡƒΡ‚ΡŒ Ρ‚ΠΎΠΊ 5.5 А Ρ‡Π΅Ρ€Π΅Π· ΠΊΠ°Ρ‚ΡƒΡˆΠΊΡƒ. КакоС сопротивлСниС сабвуфСра?

РСшСниС : Π˜Π·Π²Π΅ΡΡ‚Π½Ρ‹ Ρ€Π°Π·Π½ΠΎΡΡ‚ΡŒ напряТСний $ V = 110 \, {\ rm V} $ ΠΈ Ρ‚ΠΎΠΊ $ I = 5.5 \, {\ rm A} $. Π—Π°ΠΊΠΎΠ½ Ома связываСт ΠΈΡ… ΡΠ»Π΅Π΄ΡƒΡŽΡ‰ΠΈΠΌ ΠΎΠ±Ρ€Π°Π·ΠΎΠΌ: \ begin {align *} R & = \ frac VI \\ \\ & = \ frac {110} {5.5} \\ \\ & = 20 \ quad {\ rm \ Omega} \ end {align *}

ΠŸΡ€ΠΎΠ±Π»Π΅ΠΌΠ° (3): Бколько Ρ‚ΠΎΠΊΠ° потрСбляСт Ρ†Π΅ΠΏΡŒ с рСзистором Π½Π° 1000 Ом ΠΏΡ€ΠΈ ΠΏΠΈΡ‚Π°Π½ΠΈΠΈ ΠΎΡ‚ Π±Π°Ρ‚Π°Ρ€Π΅ΠΈ с напряТСниСм 1,5 Π’.

РСшСниС : Π‘ΠΎΠΏΡ€ΠΎΡ‚ΠΈΠ²Π»Π΅Π½ΠΈΠ΅ $ R = 1000 \, {\ rm \ Omega} $ ΠΈ напряТСниС $ V = 1.5 \, {\ rm V} $ извСстны, поэтому Ρƒ нас Π΅ΡΡ‚ΡŒ \ begin {align *} I & = \ frac VR \\\\ & = \ frac {1.5} {1000} \\\\ & = 1.5 \ quad { \ rm mA} \ end {align *}


ΠŸΡ€ΠΎΠ±Π»Π΅ΠΌΠ° (4): ЭлСктричСский Π½Π°Π³Ρ€Π΅Π²Π°Ρ‚Π΅Π»ΡŒ ΠΈΠΌΠ΅Π΅Ρ‚ ΡΠΏΠΈΡ€Π°Π»ΡŒΠ½Ρ‹ΠΉ мСталличСский ΠΏΡ€ΠΎΠ²ΠΎΠ΄, ΠΊΠΎΡ‚ΠΎΡ€Ρ‹ΠΉ потрСбляСт Ρ‚ΠΎΠΊ 100 А. Π‘ΠΎΠΏΡ€ΠΎΡ‚ΠΈΠ²Π»Π΅Π½ΠΈΠ΅ ΠΏΡ€ΠΎΠ²ΠΎΠ΄Π° составляСт 1,1 Ом. РассчитайтС напряТСниС, ΠΊΠΎΡ‚ΠΎΡ€ΠΎΠ΅ Π½Π΅ΠΎΠ±Ρ…ΠΎΠ΄ΠΈΠΌΠΎ Π½Π° Π½Π΅ΠΌ ΡƒΡΡ‚Π°Π½ΠΎΠ²ΠΈΡ‚ΡŒ.

РСшСниС : Π’ΠΎΠΊ $ I = 100 \, {\ rm A} $ ΠΈ сопротивлСниС $ R = 1.1 \, {\ rm \ Omega} $ связаны ΠΊΠ°ΠΊ \ begin {align *} V & = IR \\ & = 100 \ Ρ€Π°Π· 1.1 \\ & = 110 \ quad {\ rm V} \ end {align *}


ΠŸΡ€ΠΎΠ±Π»Π΅ΠΌΠ° (5): ΠœΠ°ΠΊΡΠΈΠΌΠ°Π»ΡŒΠ½Ρ‹ΠΉ Ρ‚ΠΎΠΊ, ΠΊΠΎΡ‚ΠΎΡ€Ρ‹ΠΉ ΠΏΡ€ΠΎΡ…ΠΎΠ΄ΠΈΡ‚ Ρ‡Π΅Ρ€Π΅Π· Π»Π°ΠΌΠΏΠΎΡ‡ΠΊΡƒ с сопротивлСниСм 5 Ом, составляСт 10 А. КакоС напряТСниС Π΄ΠΎΠ»ΠΆΠ½ΠΎ Π±Ρ‹Ρ‚ΡŒ ΠΏΡ€ΠΈΠ»ΠΎΠΆΠ΅Π½ΠΎ ΠΊ Π΅Π΅ ΠΊΠΎΠ½Ρ†Π°ΠΌ, ΠΏΡ€Π΅ΠΆΠ΄Π΅ Ρ‡Π΅ΠΌ Π»Π°ΠΌΠΏΠ° сломаСтся?

РСшСниС : МаксимальноС напряТСниС ΠΌΠΎΠΆΠ½ΠΎ Π½Π°ΠΉΡ‚ΠΈ с ΠΏΠΎΠΌΠΎΡ‰ΡŒΡŽ Π·Π°ΠΊΠΎΠ½Π° Ома, ΠΊΠ°ΠΊ ΠΏΠΎΠΊΠ°Π·Π°Π½ΠΎ Π½ΠΈΠΆΠ΅ \ begin {align *} V & = IR \\ & = 10 \ times 5 \\ & = 50 \ quad {\ rm V} \ end {align *} Если Π½Π° схСму ΠΏΠΎΠ΄Π°Ρ‚ΡŒ напряТСниС Π²Ρ‹ΡˆΠ΅ этого значСния, Π»Π°ΠΌΠΏΠ° ΠΏΠ΅Ρ€Π΅Π³ΠΎΡ€ΠΈΡ‚.


ΠŸΡ€ΠΎΠ±Π»Π΅ΠΌΠ° (6): Π’ Ρ†Π΅ΠΏΠΈ ΠΌΡ‹ замСняСм ΠΏΡ€Π΅Π΄Ρ‹Π΄ΡƒΡ‰ΡƒΡŽ Π±Π°Ρ‚Π°Ρ€Π΅ΡŽ Π½Π° 1,5 Π’ Π½Π° Π½ΠΎΠ²ΡƒΡŽ Π±Π°Ρ‚Π°Ρ€Π΅ΡŽ Π½Π° 3 Π’. Π§Ρ‚ΠΎ происходит с этой схСмой?

РСшСниС : Π—Π°ΠΊΠΎΠ½ Ома Π³ΠΎΠ²ΠΎΡ€ΠΈΡ‚ Π½Π°ΠΌ, Ρ‡Ρ‚ΠΎ, ΠΊΠΎΠ³Π΄Π° большСС напряТСниС устанавливаСтся Π² Ρ†Π΅ΠΏΠΈ, Π±ΠΎΠ»Π΅Π΅ высокий Ρ‚ΠΎΠΊ Π±ΡƒΠ΄Π΅Ρ‚ Ρ‚Π΅Ρ‡ΡŒ Ρ‡Π΅Ρ€Π΅Π· рСзисторы Π² Ρ†Π΅ΠΏΠΈ, Ρ‚Π°ΠΊΠΎΠΉ ΠΊΠ°ΠΊ элСктричСскиС Π½Π°Π³Ρ€Π΅Π²Π°Ρ‚Π΅Π»ΠΈ, Π»Π°ΠΌΠΏΠΎΡ‡ΠΊΠΈ ΠΈ Ρ‚. Π”.

Π‘ΠΎΠ»Π΅Π΅ высокий Ρ‚ΠΎΠΊ ΠΌΠΎΠΆΠ΅Ρ‚ Π²Ρ‹Π·Π²Π°Ρ‚ΡŒ ΠΏΠΎΠ²Ρ€Π΅ΠΆΠ΄Π΅Π½ΠΈΠ΅ ΠΈΠ»ΠΈ Π²Ρ‹Ρ…ΠΎΠ΄ ΠΈΠ· строя Π±Ρ‹Ρ‚ΠΎΠ²ΠΎΠΉ Ρ‚Π΅Ρ…Π½ΠΈΠΊΠΈ. НапримСр, Π»Π°ΠΌΠΏΠΎΡ‡ΠΊΠ° с сопротивлСниСм $ R = 1.5 \, {\ rm \ Omega} $ потрСбляСт Ρ‚ΠΎΠΊ $ I = \ frac {1.5} {1.5} = 1 \, {\ rm A} $ с Π±Π°Ρ‚Π°Ρ€Π΅Π΅ΠΉ $ 1.5 $ Π²ΠΎΠ»ΡŒΡ‚ ΠΈ Ρ‚ΠΎΠΊΠΎΠΌ $ I = \ frac { 3} {1.5} = 2 \, {\ rm A} $ с Π·Π°ΠΌΠ΅Π½ΠΎΠΉ Π½ΠΎΠ²Ρ‹ΠΌ. Π’ этих случаях Π»Π°ΠΌΠΏΠΎΡ‡ΠΊΠ°, скорСС всСго, ΠΏΠ΅Ρ€Π΅Π³ΠΎΡ€ΠΈΡ‚.


ΠŸΡ€ΠΎΠ±Π»Π΅ΠΌΠ° (7): Π’ схСмС ΡƒΠ΄Π°Π»Π΅Π½ рСзистор $ 10 \, {\ rm \ Omega} $ ΠΈ Π·Π°ΠΌΠ΅Π½Π΅Π½ рСзистором $ 20 \, {\ rm \ Omega} $. Π§Ρ‚ΠΎ происходит с Ρ‚ΠΎΠΊΠΎΠΌ Π² Ρ†Π΅ΠΏΠΈ.

РСшСниС : ΠŸΠΎΡΠΊΠΎΠ»ΡŒΠΊΡƒ Π½ΠΈΡ‡Π΅Π³ΠΎ Π½Π΅ сказано ΠΎ ΠΏΠ°Π΄Π΅Π½ΠΈΠΈ напряТСния Π² Ρ†Π΅ΠΏΠΈ, ΠΌΡ‹ ΠΏΡ€Π΅Π΄ΠΏΠΎΠ»Π°Π³Π°Π΅ΠΌ, Ρ‡Ρ‚ΠΎ ΠΎΠ½ΠΎ постоянноС, скаТСм, $ V = 120 \, {\ rm V} $.Π‘Π»Π΅Π΄ΠΎΠ²Π°Ρ‚Π΅Π»ΡŒΠ½ΠΎ, ΠΈΡΠΏΠΎΠ»ΡŒΠ·ΡƒΡ Ρ„ΠΎΡ€ΠΌΡƒΠ»Ρƒ Π·Π°ΠΊΠΎΠ½Π° Ома, $ I = \ frac VR $, Ρ‚ΠΎΠΊ $ I = \ frac {120} {10} = 12 \, {\ rm A} $ Ρ‚Π΅Ρ‡Π΅Ρ‚ Ρ‡Π΅Ρ€Π΅Π· $ 10 \, {\ rm \ Omega} $ рСзистор ΠΈ $ I = \ frac {120} {20} = 6 \, {\ rm A} $ Ρ‡Π΅Ρ€Π΅Π· рСзистор $ 20 \, {\ rm \ Omega} $.

ΠœΡ‹ Π²ΠΈΠ΄ΠΈΠΌ, Ρ‡Ρ‚ΠΎ для Ρ‚ΠΎΠ³ΠΎ ΠΆΠ΅ напряТСния ΡƒΠ΄Π²ΠΎΠ΅Π½ΠΈΠ΅ сопротивлСний ΠΏΡ€ΠΈΠ²ΠΎΠ΄ΠΈΡ‚ ΠΊ ΡƒΠΌΠ΅Π½ΡŒΡˆΠ΅Π½ΠΈΡŽ, Ρ‚ΠΎΡ‡Π½Π΅Π΅, ΠΊ ΡƒΠΌΠ΅Π½ΡŒΡˆΠ΅Π½ΠΈΡŽ Π²Π΄Π²ΠΎΠ΅ Ρ‚ΠΎΠΊΠΎΠ².

alexxlab

Π”ΠΎΠ±Π°Π²ΠΈΡ‚ΡŒ ΠΊΠΎΠΌΠΌΠ΅Π½Ρ‚Π°Ρ€ΠΈΠΉ

Π’Π°Ρˆ адрСс email Π½Π΅ Π±ΡƒΠ΄Π΅Ρ‚ ΠΎΠΏΡƒΠ±Π»ΠΈΠΊΠΎΠ²Π°Π½. ΠžΠ±ΡΠ·Π°Ρ‚Π΅Π»ΡŒΠ½Ρ‹Π΅ поля ΠΏΠΎΠΌΠ΅Ρ‡Π΅Π½Ρ‹ *